CPA A2.2 STRATEGIC PERFORMANCE MANAGEMENT Study Manual

User Manual:

Open the PDF directly: View PDF PDF.
Page Count: 877

DownloadCPA A2.2 - STRATEGIC PERFORMANCE MANAGEMENT Study Manual
Open PDF In BrowserView PDF
INSIDE COVER – BLANK

INSTITUTE OF

CERTIFIED PUBLIC ACCOUNTANTS
OF

RWANDA
Foundation A2.2
A2.2 STRATEGIC PERFORMANCE
MANAGEMENT
First Edition 2012
This study manual has been fully revised and updated
in accordance with the current syllabus.
It has been developed in consultation with experienced lecturers.

© iCPAR

All rights reserved.
The text of this publication, or any part thereof, may not be reproduced or transmitted in any form
or by any means, electronic or mechanical, including photocopying, recording, storage in an
information retrieval system, or otherwise, without prior permission of the publisher.

Whilst every effort has been made to ensure that the contents of this book are accurate, no
responsibility for loss occasioned to any person acting or refraining from action as a result of any
material in this publication can be accepted by the publisher or authors. In addition to this, the
authors and publishers accept no legal responsibility or liability for any errors or omissions in
relation to the contents of this book.

Page 1

CONTENTS
Study
Unit

Title

Page

1

Activity Based Costing
Activity Based Costing
Absorption Costing Versus ABC
Merits And Criticisms Of ABC
Implications Of Switching To ABC
Customer Profitability Analysis

13
15
19
25
27
33

2

Pricing Decisions
Demand
Other Issues That Influence Pricing Decisions
Profit Maximisation In Imperfect Markets
Deriving The Demand Curve
The Profit-Maximising Price / Output Level

43
45
57
63
65
69

3

Pricing Approaches And Strategies
Full Cost-plus Pricing
Marginal Cost-plus Or Mark-up Pricing
Economists’ Versus Accountants’ Views On Pricing Decisions
Pricing Based on Mark-up Per Unit Of Limiting Factor
Pricing Strategies For Special Orders
Pricing Strategies For New Products
Other Pricing Strategies

83
85
95
97
101
103
105
109

4

Target Costing
What Is Target Costing?
Implementing Target Costing
Deriving A Target Cost
Implications Of Using Target Costing
Closing A Target Cost Gap
Target Costing In Service Industries

115
117
119
121
123
125
127

5

Lifecycle Costing
What Are Life Cycle Costs?
The Product Life Cycle
The Implications Of Life Cycle Costing

131
133
135
137

6

Backflush Accounting
Backflush Costing
Costing Systems And Manufacturing Philosophy

145
147
151

Page 2

Study
Unit

Title

Page

7

Throughput Accounting
Theory Of Constraints
Throughput Accounting
Performance Measures In Throughput Accounting
Throughput And Decision Making

157
159
161
165
167

8

Limiting Factor Analysis
Limiting Factors
Limiting Factor Analysis And Restricted Freedom Of Action
Make Or Buy Decisions And Scarce Resources
Limiting Factors And Shadow Prices
Using Limiting Factor Analysis

175
177
185
191
195
197

9

Linear Programming: The Graphical Method
The Graphical Method
The Graphical Method Using Simultaneous Equations
Sensitivity Analysis

201
203
223
229

10

Linear Programming: The Simplex Method
The Principles Of The Simplex Method
Sensitivity Analysis
Using Computer Packages
Using Linear Programming

241
243
257
265
267

11

Risk And Uncertainty
Risk And Uncertainty
Allowing For Uncertainty
Probabilities And Expected Values
Decision Rules
Decision Trees
The Value Of Information
Sensitivity Analysis
Simulation Methods

273
275
277
281
285
293
303
315
317

12

Objectives Of Budgetary Control
Objectives
The Planning And Control Cycle
Objectives Of Budgetary Systems
Behavioural Implications Of Budgeting
Setting The Difficulty Level Of A Budget
Participation In Budgeting
Mutually Exclusive Projects With Unequal Lives

321
323
327
333
335
339
341
345

Page 3

Study
Unit

Title

Page

13

Budgetary Systems
Traditional Budgetary Systems
Fixed And Flexible Budgets
Zero Based Budgeting Systems
Activity Based Budgeting
Rolling Budgets
Beyond Budgeting
Information Used In Budget Systems
Changing Budgetary Systems
Budget Systems And Uncertainty

355
357
361
363
369
373
377
381
383
385

14

Quantitative Analysis In Budgeting
Analysing Fixed And Variable Costs
Forecasting Techniques
Time Series
Learning Curves
Applying Expected Values
Using Spreadsheets In Budgeting

387
389
399
401
421
435
439

15

Budgeting And Standard Costing
The Use Of Standard Costs
Deriving Standards
Budgets And Standards Compared
Allowing For Waste And Idle Time
Flexible Budgets
The Principle Of Controllability

445
447
451
457
459
463
475

16

Variance Analysis
Basic Variances
The Reasons For Variances
Labour Variances And The Learning Curve
Idle Time And Waste
Operating Statements
ABC And Variance Analysis
Investigating Variances
Materials Mix And Yield Variances

481
483
489
491
495
503
511
515
523

17

Behavioural Aspects Of Standard Costing
Planning And Operational Variances
Behavioural Aspects Of Standard Costing

535
537
555

18

Divisional Performance And Transfer Pricing Issues
Divisional Structure And Performance Measures
Measuring Performance
When Transfer Pricing Is Required

563
567
577
579

Page 4

Study
Unit

Title

Page

The ‘General Rule’
The Use Of Market Price
Cost-Based Approaches To Transfer Pricing
Fixed Costs And Transfer Pricing
Standard Cost Versus Actual Cost
Cost-Based Approaches With No External Market
Opportunity Costs And Transfer Prices
Transfer Pricing When Intermediate Products Are In Short Supply
Transfer Pricing And A Range Of Limiting Factors
Shadow Price And Transfer Prices
Negotiated Transfer Prices
Multinational Transfer Pricing

583
585
589
595
597
599
603
605
617
621
623
625

19

Value Based Management
Value-based Management Approaches

633
635

20

Performance Hierarchy
Mission Statements And Vision
Goals And Objectives: An Introduction
Corporate Objectives
Subsidiary Or Secondary Objectives
Social And Ethical Obligations
The Short Term And Long Term
The Planning Gap And Strategies To Fill It
Operational Performance
Planning And Control At Different Levels In The Performance Hierarchy

645
647
655
659
663
669
683
685
689
691

21

Competitive Strategies
Strategic Models Used In Planning And Assessing Business Performance
Criticisms Of Performance Indicators

697
699
713

22

Performance Measurement Models
The Balanced Scoreboard
The Performance Pyramid
Building Blocks

717
719
723
725

23

Financial Performance Measurement
The Private Sector: Shareholder Benefits
Survival And Growth
Profitability
Gearing
Liquidity
Short-Run And Long-Run Financial Performance
Profits And Share Value
Comparisons Of Accounting Figures

735
737
745
749
771
779
785
789
793

Page 5

Study
Unit

Title

Page

24

Non-Financial Performance Indicators
Disadvantages Of Financial Performance Indicators
Growing Emphasis On NFPIs
The Value Of NFPIs
NFPIs In Relation To Employees
NFPIs In Relation To Product / Service Quality
Qualitative Issues

797
799
801
805
807
809
813

25

Impact Of Developments In Information Technology & E-Commerce
Information Needs Of Manufacturing And Service Businesses
Instant Access To Data
Remote Input Of Data
Developing Management Accounting Systems
Game Theory

817
819
833
841
843
849

26

Benchmarking
Benchmarking Benefits And Difficulties

857
859

27

Business Process Re-Engineering
Business Process Re-Engineering

865
867

Page 6

Stage: Advanced Level 2
Subject Title: A2.2 Strategic Performance Management

Aim
The aim of this subject is to ensure that students have the ability to conduct a critical,
strategic analysis of unfamiliar business situations. This includes the ability to select,
integrate and apply the appropriate techniques and approaches in order to identify problems,
opportunities, and recommended strategies in specific situations.

Strategic Performance Management as an Integral Part of the Syllabus
This subject builds on the technical knowledge of Managerial Finance, Management
Accounting, and other disciplines acquired in the earlier examination stages. Strategic
Performance Management requires students to integrate and expand that knowledge so as to
provide a framework for strategic analysis of business issues.

Learning Outcomes
On successful completion of this subject students should be able to:
•

Select, integrate, and apply strategic performance management techniques which are
appropriate to the particular context of specific (but novel and possibly unstructured)
business situations described in a case study, through the application of critical
strategic thinking, appropriate professional scepticism and ethical judgement.

•

Justify a portfolio of strategic performance management techniques selected by
reference to the particular illustrative context of a case study situation (and not merely
by repetition of abstract or textbook knowledge) and:
− Predict the strategic advantages for the organisation of applying the proposed
techniques in the particular situation;
− Predict any possible adverse side-effects and propose means of minimising
such effects.

Page 7

•

Critically evaluate existing and proposed strategic performance management
structures as described in a particular case study, and:
− Construct an improved set of strategic performance management
structures
which builds on any identified strengths of existing structures while
implementing any new structures identified as necessary;
− Justify the improved set of strategic performance management structures in its
totality (as opposed to justifying only specific elements in a disconnected
fashion).

•

Assess the likely effects of existing and proposed strategic performance management
structures on intra-organisational behaviours, work practices, and group norms (in the
context of the business situation of an entity described in a case study) and:
− Propose means of ameliorating any adverse effects.
− Propose means of maximising the extent and strategic advantage of any
favourable effects.

•

Evaluate the implications for particular organisations’ strategic performance
management systems of emerging developments in the fields of information
technology (including e-commerce), business process re-engineering, benchmarking,
and corporate governance and propose changes to the strategic performance
management systems of a specific organisation described in a case study in the light
of such emerging developments.

Page 8

Syllabus:

1) Advanced Decision-Making
•

Pricing decisions (including strategic considerations, revenue management, profit
maximisation, services pricing, and product bundling).

•

Target costing & Lifecycle costing.

•

Product & segment profitability analysis.

•

Customer profitability analysis.

•

Theory of constraints, throughput and back-flush accounting.

•

Activity-based analysis for decision-making.

•

Measuring and managing uncertainty,risk (including risk appraisal, expected
values, maximax / maximin and sensitivity analysis).

•

Decision making with multiple limiting factors including the formulation of
problems and interpretation of results using linear programming and the simplex
algorithm).

•

Assessment of mutually exclusive projects, projects with unequal lives.

2) Planning and Control
•

The purpose of budgetary control systems.

•

Alternative approaches to budgeting (including incremental budgeting, zero-based
budgeting, activity-based budgeting, rolling budgets, and ‘beyond budgeting’
approaches).

•

Budgetary control of engineered, committed, and discretionary costs.

Page 9

•

Advanced variance analysis (including sales mix & yield; materials mix & yield;
planning & operational; market size & market share).

•

Critical appraisal of standard costing and variance analysis in modern
manufacturing environments.

•

Advantages and disadvantages of forecasting techniques including: time series,
trend analysis, smoothing techniques and seasonal variances.

3) Performance Evaluation
•

Divisional profitability: Return on investment and residual income.

•

The distinction between economic and managerial performance evaluation.

•

Economic value added ®.

•

Value-based management.

•

Transfer pricing (including Cost-plus, Market, Negotiated & Dual prices).

•

Interaction of transfer pricing and taxation.

4) Performance Measurement Systems
•

Mission statements, objectives, strategies and goals.

•

Performance measurement in modern manufacturing environments (including JIT,
TQM, world class manufacturing, and supply chain management issues).

•

Scenario planning (what if analysis).

•

Alternative competitive strategies.

•

Monitoring of the external environment (including competitor accounting).
Page 10

•

Financial and non-financial performance measurement.

•

Performance measurement models, including the balanced scorecard.

•

Incentive schemes.

5) Current Developments in Strategic Performance Management
•

Benchmarking.

•

Impact of developments in information technology and ecommerce.

•

Business process re-engineering.

•

Corporate governance.

Page 11

BLANK

Page 12

STUDY UNIT 1
Activity Based Costing
Contents

Page

Activity Based Costing ……………………………………………………….

15

Absorption Costing Versus ABC ……….…………………………………..

19

Merits And Criticisms Of ABC …………………………………………….

25

Implications Of Switching To ABC …………………………………………

27

Customer Profitability Analysis …………………………..…………………

33

Page 13

EXAM GUIDE
Activity Based Costing ABC was regularly examined in the professional 1 Strategic
Management Accounting paper and is an important topic on this paper also.

Page 14

ACTIVITY BASED COSTING
An alternative to absorption costing is activity based costing (ABC).
ABC involves the identification of the factors (drivers) which cause the costs of an
organisation's major activities. Support overheads are charged to products on the basis of
their usage of an activity.
− For costs that vary with production level in the short term, the cost driver will be
volume related (labour or machine hours).
− Overheads that vary with some other activity (and not volume of production) should be
traced to products using transaction-based cost drivers such as production runs or
number of orders received.

Reasons for the development of ABC
The traditional cost accumulation system of absorption costing was developed in a time
when most organisations produced only a narrow range of products (so that products
underwent similar operations and consumed similar proportions of overheads). And
overhead costs were only a very small fraction of total costs, direct labour and direct
material costs accounting for the largest proportion of the costs. The benefits of more
accurate systems for overhead allocation would probably have been relatively small. In
addition, information processing costs were high.
In recent years, however, there has been a dramatic fall in the costs of processing
information. And, with the advent of advanced manufacturing technology (AMT) and in a
production environment which uses machines such as computer controlled equipment
(lathes, drilling machines and hoists etc.) and computers in general, direct labour may
account for as little as 5% of a product's cost. It therefore now appears difficult to justify the
use of direct labour or direct material as the basis for absorbing overheads or to believe that
errors made in attributing overheads will not be significant.
Many resources are used in non-volume related support activities, (which have increased
due to AMT) such as setting-up, production scheduling, inspection and data processing, not
to mention finance, marketing and personnel departments etc. These support activities assist
the efficient manufacture of a wide range of products and are not, in general, affected by
changes in production volume. They tend to vary in the long term according to the range
and complexity of the products manufactured rather than the volume of output.
Page 15

The wider the range and the more complex the products, the more support services will be
required. Consider, for example, factory X which produces 10,000 units of one product, the
Alpha, and factory Y which produces 1,000 units each of ten slightly different versions of the
Alpha. Support activity costs in the factory Y are likely to be a lot higher than in factory X
but the factories produce an identical number of units. For example, factory X will only need
to set-up once whereas Factory Y will have to set-up the production run at least ten times for
the ten different products. Factory Y will therefore incur more set-up costs for the same
volume of production.
Traditional costing systems, which assume that all products consume all resources in
proportion to their production volumes, tend to allocate too great a proportion of
overheads to high volume products (which cause relatively little diversity and hence use
fewer support services) and too small a proportion of overheads to low volume products
(which cause greater diversity and therefore use more support services). Activity based
costing (ABC) attempts to overcome this problem.

Definition of ABC
Activity based costing (ABC) involves the identification of the factors which cause the costs
of an organisation's major activities. Support overheads are charged to products on the basis
of their usage of the factor causing the overheads.
The major ideas behind activity based costing are as follows.
a) Activities cause costs. Activities include ordering, materials handling, machining,
assembly, production scheduling and despatching.
b) Producing products creates demand for the activities.
c) Costs are assigned to a product on the basis of the product's consumption of the
activities.

Page 16

Outline of an ABC system
An ABC system operates as follows.
Step 1

Identify an organisation's major activities.

Step 2

Identify the factors which determine the size of the costs of an activity/cause the
costs of an activity. These are known as cost drivers.

A cost driver is a factor which causes a change in the cost of an activity.

Look at the following examples.
Costs

Possible cost driver

Ordering costs

Number of orders

Materials handling costs

Number of production runs

Production
costs

scheduling Number of production runs

Despatching costs

Number of despatches

Step 3

Collect the costs associated with each cost driver into what are known as cost
pools.

Step 4

Charge costs to products on the basis of their usage of the activity. A product's
usage of an activity is measured by the number of the activity's cost driver it
generates.

Page 17

BLANK

Page 18

ABSORPTION COSTING VERSUS ABC
The following example illustrates the point that traditional cost accounting techniques result
in a misleading and inequitable division of costs between low-volume and high-volume
products, and that ABC can provide a more meaningful allocation of costs.

Example: Activity based costing
Suppose that Coolplan manufactures four products, W, X, Y and Z. Output and cost data for
the period just ended are as follows:

Number

Output units

of production

Material cost Direct labour

Machine

runs in the period

per unit

hours per unit

hours per unit
2

W

10

2

20.00

2

1

X

10

2

80.00

6

3

Y

100

5

20.00

2

1

Z

100

5

80.00

6

3

220

14

200

16

8

Page 19

Direct labour cost per hour

RWF 5

Overhead costs

RWF

Short run variable costs

3,080

Set-up costs

10,920

Expediting and scheduling costs

9,100

Materials handling costs

7,700
30,800

Required
Prepare unit costs for each product using conventional costing and ABC.
Solution
Using a conventional absorption costing approach and an absorption rate for overheads
based on either direct labour hours or machine hours, the product costs would be as follows.
W

X

Y

Z

RWF ‘000

RWF ‘000

RWF ‘000

RWF ‘000

Direct material

200

800

2,000

8,000

11,000

Direct labour

20

60

200

600

880

Overheads *

700

2,100

7,000

21,000

30,800

920

2,960

9,200

29,600

42,680

Units produced

10

10

100

100

Cost per unit

92

296

92

296

* RWF30,800,000 ÷ 440 hours = RWF70,000 per machine hour.
Page 20

Total

Using activity based costing and assuming that the number of production runs is the cost
driver for set-up costs, expediting and scheduling costs and materials handling costs and that
machine hours are the cost driver for short-run variable costs, unit costs would be as follows.

Number of
production
runs in the

Material cost

Direct labour

Machine

Output units

period

RWF
‘000 hours per unit
per unit

hours per unit

W

10

2

20

2

1

X

10

2

80

6

3

Y

100

5

20

2

1

Z

100

5

80

6

3

220

14

200

Workings
Workings

Overheads

RWF '000

1

3,080

440

M/c hours

7

per machine hour

2

10,920

14

Runs

780

per run

3

9,100

14

Runs

650

per run

4

7,700

14

Runs

550

per run

Page 21

Summary

W

Conventional costing
unit cost
RWF '000
92

ABC
unit cost
RWF '000
425

Difference per
unit
RWF '000
333

Difference in
total
RWF '000
3,330

X

296

503

207

2,070

Y

92

128

36

3,600

Z

296

206

-90

-9,000

Product

0

The figures suggest that the traditional volume-based absorption costing system is flawed.
a) It under-allocates overhead costs to low-volume products (here, W and X) and
over-allocates overheads to higher-volume products (here Z in particular).
b) It under-allocates overhead costs to smaller-sized products (here W and Y with just
one hour of work needed per unit) and over allocates overheads to larger products
(here X and particularly Z).

ABC versus traditional costing methods
Both traditional absorption costing and ABC systems adopt the two stage allocation process.

Allocation of overheads
ABC establishes separate cost pools for support activities such as despatching. As the costs of
these activities are assigned directly to products through cost driver rates, reapportionment of
service department costs is avoided.
Page 22

Absorption of overheads
The principal difference between the two systems is the way in which overheads are absorbed
into products.
a) Absorption costing most commonly uses two absorption bases (labour hours and/or
machine hours) to charge overheads to products.
b) ABC uses many cost drivers as absorption bases (eg number of orders or despatches).
Absorption rates under ABC should therefore be more closely linked to the causes of
overhead costs.

Cost drivers
The principal idea of ABC is to focus attention on what causes costs to increase, i.e. the cost
drivers.
a) The costs that vary with production volume, such as power costs, should be traced to
products using production volume-related cost drivers, such as direct labour hours
or direct machine hours.
Overheads which do not vary with output but with some other activity should be
traced to products using transaction-based cost drivers, such as number of
production runs and number of orders received.
b) Traditional costing systems allow overheads to be related to products in rather more
arbitrary ways producing, it is claimed, less accurate product costs.

Page 23

BLANK

Page 24

MERITS AND CRITICISMS OF ABC
ABC has both advantages and disadvantages, and tends to be more widely used by larger
organisations and the service sector.
As you will have discovered when you attempted the question above, there is nothing
difficult about ABC. Once the necessary information has been obtained it is similar to
traditional absorption costing. This simplicity is part of its appeal. Further merits of ABC are
as follows.
a) The complexity of manufacturing has increased, with wider product ranges, shorter
product life cycles and more complex production processes. ABC recognises this
complexity with its multiple cost drivers.
b) In a more competitive environment, companies must be able to assess product
profitability realistically. ABC facilitates a good understanding of what drives
overhead costs.
c) In modern manufacturing systems, overhead functions include a lot of non-factoryfloor activities such as product design, quality control, production planning and
customer services. ABC is concerned with all overhead costs and so it takes
management accounting beyond its 'traditional' factory floor boundaries.
d) ABC is particularly useful for service industries where an individual member of staff
may be involved in transactions which concern different “product” or service lines
each day e.g. a bank teller who may receive cash to deposit, deal with a credit card, or
sell an insurance policy.

Page 25

Criticisms of ABC
It has been suggested by critics that activity based costing has some serious flaws.
a) Some measure of (arbitrary) cost apportionment may still be required at the cost
pooling stage for items like rent, rates and building depreciation.
b) Can a single cost driver explain the cost behaviour of all items in its associated pool?
c) Unless costs are caused by an activity that is measurable in quantitative terms and
which can be related to production output or a service activity, cost drivers are not
easily usable. What drives the cost of the annual external audit, for example?
d) ABC is sometimes introduced because it is fashionable, not because it will be used by
management to provide meaningful product costs or extra information. If
management is not going to use ABC information, an absorption costing system may
be simpler and therefore more cost-effective to operate.
e) The cost of implementing and maintaining an ABC system can exceed the benefits of
improved accuracy.
f) Implementing ABC is often problematic.
g) The calculations can be iterative, such as apportioning HR dept. costs to IT when IT
costs are also apportioned to production and to HR Department

Page 26

IMPLICATIONS OF SWITCHING TO ABC
Switching to ABC has implications for pricing, sales strategy, performance management and
decision making.
Switching to ABC is often problematic. Recent journal articles have highlighted the
following issues.
a) The incorrect belief that ABC can solve all an organisation's problems
b) Lack of the correct type of data
c) Difficulty in determining appropriate cost drivers
'World-wide adoption rates for ABC have peaked at 20 per cent and a declining number of
firms is giving it further consideration.' (Tom Kennedy, Financial Management, May 2000).
Recent UK studies have found ABC usage rates of about 25%, with larger organisations and
service sector companies being most likely to use it.

Pricing
An ABC system gives management a good understanding of the cost structures of making
and selling a wide range of products. Switching to ABC can change cost per unit
calculations substantially. If an organisation determines prices based on cost i.e. using costplus pricing, greater costing information will be very useful and prices will change.
Many organisations however price their products according to what the market will bear, so if
costs are re-calculated, it is the profit margin for a product that will change rather than its
price.
Consider a business that produces a large volume standard product and a number of variants
which are more refined versions of the basic product and sell in low volumes at a higher
price. Such companies are common in practice in the modern business environment. In
practice, also, such companies absorb fixed overheads on a conventional basis such as direct
labour hours, and price their products by adding a mark up to full cost.
In the situation described, the majority of the overheads would be allocated to the standard
range, and only a small percentage to the up-market products. The result would be that the

Page 27

profit margin achieved on the standard range would be much lower than that on the upmarket range.
Thus the traditional costing and pricing system indicates that the firm might be wise to
concentrate on its high margin, up-market products and drop its standard range. This is
absurd, however. Much of the overhead cost incurred in such an organisation is the cost of
support activities like production scheduling: the more different varieties of product there
are, the higher the level of such activities will become. The cost of marketing and
distribution also increases disproportionately to the volume of products being made.
The bulk of the overheads in such an organisation are actually the 'costs of complexity'.
Their arbitrary allocation on the basis of labour hours gives an entirely distorted view of
production line profitability; many products that appear to be highly profitable actually make
a loss if costs are allocated on the basis of what activities cause them.
The problem arises with marginal cost-plus approaches as well as with absorption cost
based approaches, particularly in a modern manufacturing environment, where a relatively
small proportion of the total cost is variable. The implication in both cases is that
conventional costing should be abandoned in favour of ABC.

Case Study
In a survey reported in Accountancy Age ('The price is right… or is it?', June 2002), it was
found that 'For many companies, pricing decisions are a “seat-of-the-pants” affair. There is
generally low take-up of the analytical tools and techniques an accountant would expect to
use in making financial decisions. The most extensively used technique was 'face to face'
research.' ABC was in third place, after competitive analysis but before breakeven analysis.
The survey also found that the leaders [most successful companies in the survey at using
price management to achieve business objectives] …''focus on customer segments,
differentiate products to serve them, pay attention to quality and deliver on customer care.
This costs money but they see it as a way of reducing unit cost and delivering the economies
of scale which lead to competitive prices and market leadership'.
Significantly too, the leaders are more likely to use realistic cost allocation methodologies,
such as activity-based costing, when they take pricing decisions. Some 62% of leaders ranked
this either 'very important' or 'important' compared with just 23% of laggards.
Page 28

'Confident – and profitable – pricing depends on knowing direct and indirect costs
attributable to a particular product or service … It's not surprising leader companies take
better pricing decisions when they are more likely to have this information at their fingertips.'

Sales strategy
As we have seen, the introduction of ABC has implications for the cost per unit, price and
profit margin. For example, a product with few set-ups, material movements or inspections
will have lower costs under ABC than traditional absorption costing. The organisation could
decide to reduce the product’s selling price but if it is a high volume product, the number of
units sold may not increase sufficiently to compensate for the loss in total revenue and
contribution.
ABC may result in a change in profit margins, with previously high margin products now
being seen as less profitable. This can result in increased sales efforts on different products,
especially if the sales department is rewarded on the basis of profits.

Performance management
The information provided by analysing activities can support performance management
provided it is used carefully and with full appreciation of its implications.

Planning
Before an ABC system can be implemented, management must analyse the organisation's
activities, determine the extent of their occurrence and establish the relationships between
activities, products/services and their cost.
The information database produced from such an exercise can then be used as a basis for
forward planning and budgeting. For example, once an organisation has set its budgeted
production level, the database can be used to determine the number of times that particular
activities will need to be carried out, thereby establishing necessary departmental staffing and
machine levels. Financial budgets can then be drawn up by multiplying the budgeted activity
levels by cost per activity.
Page 29

This activity-based approach may not produce the final budget figures but it can provide the
basis for different possible planning scenarios.

Control
The information database also provides an insight into the way in which costs are structured
and incurred in service and support departments. Traditionally it has been difficult to
control the costs of such departments because of the lack of relationship between
departmental output levels and departmental cost. With ABC, however, it is possible to
control or manage the costs by managing the activities which underlie them by monitoring
a number of key performance measures.

Decision making
Many of ABC's supporters claim that it can assist with decision making in a number of ways.
− Provides accurate and reliable cost information
− Establishes a long-run product cost
− Provides data which can be used to evaluate different ways of delivering business.
It is therefore particularly suited to the following types of decision.
− Pricing
− Promoting or discontinuing products or parts of the business
− Redesigning products and developing new products or new ways to do business

Note, however, that an ABC cost is not a true cost, it is simply an average cost because
some costs such as depreciation are still arbitrarily allocated to products. An ABC cost is
therefore not a relevant cost for all decisions.
The traditional cost behaviour patterns of fixed cost and variable cost are felt by advocates of
ABC to be unsuitable for longer-term decisions, when resources are not fixed and changes in
the volume or mix of business can be expected to have an impact on the cost of all resources
used, not just short-term variable costs.

Page 30

ABC attempts to relate the incidence of costs to the level of activities undertaken. A
hierarchy of activities has been suggested.

Type of activities

Costs are dependent on ….

Examples

Unit level

Volume of production

Machine power

Batch level

Number of batches

Set-up costs

Product sustaining

Existence of a product group/line

Product management

Facility sustaining

Organisation simply being in business

Rent and rates

The difference between a unit product cost determined using traditional absorption costing
and one determined using ABC will depend on the proportion of overhead cost which falls
into each of the categories above.
If most overheads are related to unit level and facility level activities, the costs will be
similar.
If the overheads tend to be associated with batch or product level activities they will be
significantly different.
Consider the following example.

Example: batch-level activities
XYZ produces a number of products including product D and product E and produces 500
units of each of products D and E every period at a rate of ten of each every hour. The
overhead cost is Rwf 500 million and a total of 400,000 direct labour hours are worked on all
products. A traditional overhead absorption rate would be Rwf1,250 per direct labour hour
and the overhead cost per product would be Rwf125.
Production of D requires five production runs per period, while production of E requires 20.
An investigation has revealed that the overhead costs relate mainly to 'batch-level' activities
associated with setting-up machinery and handling materials for production runs.

Page 31

There are 1,000 production runs per period and so overheads could be attributed to XYZ's
products at a rate ofRwf 500,000 per run.
− Overhead cost per D
− Overhead cost per E

=
=

(RWF500,000 × 5 runs)/500
(RWF500,000 × 20 runs)/500

=
=

RWF5,000
RWF20,000

These overhead costs are activity based and recognise that overhead costs are incurred due to
batch level activities. The fact that E has to be made in frequent small batches, perhaps
because it is perishable, means that it uses more resources than D. This is recognised by the
ABC overhead costs, not the traditional absorption costing overhead costs.
In the modern manufacturing environment, production often takes place in short,
discontinuous production runs and a high proportion of product costs are incurred at the
design stage. An increasing proportion of overhead costs are therefore incurred at batch or
product level.
Such an analysis of costs gives management an indication of the decision level at which
costs can be influenced. For example, a decision to reduce production costs will not simply
depend on making a general reduction in output volumes: production may need to be
organised to reduce batch volumes; a process may need to be modified or eliminated;
product lines may need to be merged or cut out; facility capacity may need to be altered.

Page 32

CUSTOMER PROFITABILITY ANALYSIS (CPA)
Customer profitability analysis uses an activity based approach to relate revenues and costs
to groups of customers in order to assess their relative profitability.
Traditionally, management accounting reports have been analysed on a product by
product basis. In the modern business environment, however, in which it is vital that
organisations respond promptly to the demands of customers, analysis on the basis of
customers can provide vital management information.

Analysing customers
Profitability can vary widely between different customers because various overhead costs
are, to some extent, variable and customer driven.
•
•
•
•

Discounts
Sales force
Quality control
Merchandising

•
•
•
•

Distribution
Promotions
Financing costs
Enquiries

Suppose a hotel offers a number of services such as a swimming pool, a gym and a nightly
dinner dance.
a) Older guests may attend the dinner dance.
b) Families may use the swimming pool.
c) People without children may appreciate the gym.

By charging services to the guests using them, a cost per bed night can be calculated for each
guest group. Strategies for attracting the most profitable guest group can then be adopted.

Page 33

Whether individual customers or groups of customers are costed largely depends on the number
of customers.
a) A manufacturing company supplying six companies would cost each customer
separately.
b) A supermarket or bank would cost groups of similar customers. UK banks divide their
customers into categories such as single and 30ish, married with young children, older
couples who are pensioners and so on.
Marketing departments should be aiming to attract and retain profitable customers but in
order to do this they need to know which customers are profitable and how much can be
spent on retaining them. The costing system should provide the necessary answers. Also,
banks know that a young “expnsive” customer may well become middle-aged and more
profitable.

Customer profitability analysis (CPA) is 'the analysis of the revenue streams and service
costs associated with specific customers or customer groups'.

Customer profitability analysis (CPA) provides important information which allows an
organisation to determine both which classes of customers it should concentrate on and the
prices it should charge for customer services. Its use ensures that those customers
contributing sizeably to the profitability of the organisation receive a comparable amount
of attention from the organisation.

Page 34

Customer revenues
Customer revenues are cash flows from customers. They are influenced by different factors,
mainly allowances and discounts.
a) Some types of customer store and distribute goods (e.g. wholesalers) or promote the
goods in return for an allowance.
b) By giving a discount a company may encourage bulk orders, which may be cheaper
to provide and may result in higher sales volume. Studies on customer profitability
have found large price discounting to be a key explanation for a group of customers
being below expected profitability, however. Sales representatives may have given
customers large price discounts unrelated to their current or potential value to the
company, perhaps to meet bonuses dependent on sales volumes. Two customers may
be purchasing the same volumes but the price discount given to one may make it
unprofitable, while the other is profitable.

Case Study
The USA company General Electric, which manufactures and sells refrigerators and so on,
used to give substantial discounts to customers who placed large orders. This did not result in
customers buying more products. Instead GE's sales orders bunched in particular weeks of
the year. In turn this led to an uneven production and distribution flow, which increased costs.
The company found that, by removing the discounts while at the same time guaranteeing
swift delivery, order size decreased and profits increased.

Customer costs and ABC
The creation of cost pools for activities in ABC systems allows organisations to arrange costs
in a variety of different ways. Because different customers use different amounts of activities,
it is possible to build up costs for individual customers or groups of customers on an
activity basis so that their relative profitability can be assessed.

Page 35

Examples of the build up of customer costs using an activity based system
Activity

Cost driver

Order taking

Number of orders taken

Sales visits

Number of sales visits

Emergency orders

Number of rushed orders

Delivery

Miles travelled

Case Study
Drury cites the case of Kanthal, a Swedish company in the Sandvik Group that sells electric
heating elements. Customer-related selling costs represented 34% of total costs. In the past
Kanthal had allocated these costs on the basis of sales value when customer profitability
studies were carried out. The company then introduced an ABC system in order to determine
the resources consumed by different customers.
An investigation identified two cost drivers for the resources used to service different
customers.
a) Number of orders placed. Each order had a large fixed cost, which did not vary with
the number of items ordered. A customer ordering 10 items 100 times cost more to
service than a customer placing a single order for 1,000 items.
b) Non-standard production items. These cost more to manufacture than standard
items.
A cost per order and the cost of handling standard and non-standard items were calculated
and a CPA carried out on the basis of the previous year's sales. The analysis showed that only
40% of customers were profitable, and a further 10% lost 120% of the profits. In other words,
10% of customers incurred losses equal to 120% of Kanthal's total profits. Two of the most
unprofitable customers were actually in the top three in terms of total sales volume but made
many small orders of non-standard items.

Page 36

Unprofitable customers identified by CPA should be persuaded to alter their buying
behaviour so they become profitable customers. In the Kanthal example above, unprofitable
customers should be discouraged from placing lots of small orders and/or from buying nonstandard products.
The activity based approach also highlights where cost reduction efforts should be
focused. Kanthal should concentrate on reducing ordering cost and the cost of handling nonstandard items.
Activity-based CPA allows an organisation to adopt a more market-orientated approach to
management accounting.

Customer profitability statement
There is no set format, but it would normally be similar to the one below. Note that financing
costs have been included.
RWFm

RWF m

Revenue at list prices

100

Less: discounts given

8

Net revenue

92

Less: cost of goods sold

50

Gross margin

42

Less:

customer specific costs (such as those listed above)

28

financing costs:
credit period

3

customer specific inventory

2
33

Net margin from customer

9
Page 37

Example
Seth supplies shoes to N and to K. Each pair of shoes has a list price of Rwf50,000 and costs
Seth Rwf25,000. As K buys in bulk it receives a 10% trade discount for every order for 100
pairs of shoes or more. N receives a 15% discount irrespective of order size, because that
company collects the shoes, thereby saving Seth any distribution costs. The cost of
administering each order is Rwf50,000 and the distribution cost is Rwf1,000,000 per order. N
makes 10 orders in the year, totalling 420 pairs of shoes, and K places 5 orders for 100 pairs.
Required: Fill in the blank in the sentence below.
The most profitable customer for Seth is …………… .

Answer

The correct answer is N.
It can be shown that Seth earns more from supplying Narayan, despite the larger discount
percentage.
K
N

Revenue
Less: discount
Net revenue
Less: cost of shoes
customer transport cost
customer administration cost
Net gain

RWF ‘000

RWF ‘000

25,000

21,000

2,500

3,150

22,500

17,850

(12,500)
(5,000)
(250)
4,750

(10,500)
–
( (500)
6,850

The difference on a unit basis is considerable.
Number of pair of shoes sold

500

Net gain per pair of shoes sold
Net gain per RWF 1,000 of sales revenue

Page 38

420

RWF9,500

RWF16,310

RWF190

RWF 326

Costing customers
Not all customers cost the same to serve even if they require the same products. A
customer will cost more to serve if based a long way from the factory (delivery costs
increase), or places rush orders (production scheduling is interrupted, special transport is
required), or requires a high level of after-sales service and technical assistance.
In order to analyse different customers it may therefore be useful to review non-financial
data.
Customer
X
Number of purchase orders

Y

Z

10

20

30

Number of sales visits

5

5

5

Number of deliveries

15

20

55

Distance per delivery

50

20

70

1

0

4

Number of emergency orders

Customer Y may be the cheapest to serve because of the number of deliveries per order, the
lower distance travelled and the lack of emergency orders.

Page 39

Categorising customers
It is not possible to 'cost' future dealings with customers accurately because the number and
size of orders and rush orders is likely to be unpredictable. It is, however, possible to gain a
broad idea of the amount of profit that can be expected from a particular category of
customer. Customers can be categorised in the following grid.

The aim is to attract as many accepting customers as possible. Such customers will have a
low 'cost to supply' perhaps because they are located close by or do not place rush orders, and
are prepared to accept a high price. Many large retail organisations fall into the
demanding category because they expect the supplier to deal with rush orders, change
production methods to suit them and so on. It is undesirable for a small supplier to be tied to
a large demanding customer who has the power to threaten the withdrawal of its custom if
the supplier does not acquiesce.
Alternatively, customers can be analysed using decision grid analysis (DGA), as illustrated
in the following diagram.

Page 40

Positive contribution as a % of
sales
POTENTIAL

WINNERS

Low
volume

High
volume

LOSERS

PROBLEM

Negative contribution as a % of
sales

Customers and life cycle costing
Customers can also be costed over their expected 'life cycle' and the expected future cash
flows relating to the customer may be discounted. It is rarely possible to predict accurately
the life cycle of a particular customer unless contracts are awarded for a specific time period.
Nevertheless the information is valuable as the longer the customer remains with the
organisation the more profitable the customer becomes. This is valuable information and
may show the importance of creating and retaining loyal customers.

Page 41

CHAPTER ROUNDUP
• An alternative to absorption costing is activity based costing (ABC).
ABC involves the identification of the factors (cost drivers) which cause the costs of an
organisation's major activities. Support overheads are charged to products on the basis
of their usage of an activity.
− For costs that vary with production level in the short term, the cost driver will be
volume related (labour or machine hours).
− Overheads that vary with some other activity (and not volume of production)
should be traced to products using transaction-based cost drivers such as
production runs or number of orders received.
• ABC has both advantages and disadvantages, and tends to be more widely used by
larger organisations and the service sector.
• Switching to ABC has implications for pricing, sales strategy, performance
management and decision making.

Page 42

STUDY UNIT 2
Pricing Decisions
Contents

Page

Demand ……………………………………………………………………….

45

Other Issues That Influence Pricing Decisions ……….………….………..

57

Profit Maximisation In Imperfect Markets ………….…………………….

63

Deriving The Demand Curve ……………………………….………….……

65

The Profit-Maximising Price / Output Level ……………..…………………

69

Page 43

BLANK

Page 44

DEMAND
In the first sections of this chapter you will be learning about the many issues that need to be
considered in decisions about the price which can be charged for a product or service. The
first issues relate to demand.

Issue 1: the relationship between price and demand
Demand is normally elastic because demand will increase as prices are lowered.

There are two extremes in the relationship between price and demand. A supplier can either
sell a certain quantity, Q, at any price (as in graph (a)). Demand is totally unresponsive to
changes in price and is said to be completely inelastic. Alternatively, demand might be
limitless at a certain price P (as in graph (b)), but there would be no demand above price P
and there would be little point in dropping the price below P. In such circumstances demand
is said to be completely elastic.
(a)

(b)

Page 45

A more normal situation is shown below. The downward-sloping demand curve shows the
inverse relationship between unit selling price and sales volume. As one rises, the other falls.
Demand is elastic because demand will increase as prices are lowered.

Price elasticity of demand (η)
Price elasticity of demand is a measure of the extent of change in market demand for a good
in response to a change in its price.

Price elasticity of demand (η), which is a measure of the extent of change in market demand
for a good in response to a change in its price, is measured as:
The change in quantity demanded, as a % of demand
The change in price, as a % of the price

Since the demand goes up when the price falls, and goes down when the price rises, the
elasticity has a negative value, but it is usual to ignore the minus sign.

Page 46

Example: price elasticity of demand
The price of a good is RWF1,200 per unit and annual demand is 800,000 units. Market
research indicates that an increase in price of RWF100 per unit will result in a fall in annual
demand of 75,000 units. What is the price elasticity of demand?

Solution
Annual demand at RWF1,200 per unit is 800,000 units.
Annual demand at RWF1,300 per unit is 725,000 units.
% change in demand
=
(75,000/800,000) × 100% = 9.375%
% change in price
=
(100/1,200) × 100% = 8.333%
Price elasticity of demand
=
(–9.375/8.333) = –1.125
Ignoring the minus sign, price elasticity is 1.125.

The demand for this good, at a price of Rwf1,200 per unit, would be referred to as elastic
because the price elasticity of demand is greater than 1.

Elastic and inelastic demand
The value of demand elasticity may be anything from zero to infinity.
Demand is referred to as inelastic if the absolute value is less than 1 and elastic if the
absolute value is greater than 1.

Think about what this means.
a) Where demand is inelastic, the quantity demanded falls by a smaller percentage than
the percentage increase in price.
b) Where demand is elastic, demand falls by a larger percentage than the percentage rise
in price.
Page 47

If demand is elastic, a reduction in price would lead to a rise in total sales revenue. If
demand is inelastic, a reduction in price would lead to a fall in total sales revenue.

Price elasticity and the slope of the demand curve
Generally, demand curves slope downwards. Consumers are willing to buy more at lower
prices than at higher prices. In general, elasticity will vary in value along the length of a
demand curve.
a) If a downward sloping demand curve becomes steeper over a particular range of
quantity, then demand is becoming more inelastic.
b) A shallower demand curve over a particular range indicates more elastic demand.
The ranges of price elasticity at different points on a downward sloping straight line demand
curve are illustrated in the diagram below.

a) At higher prices on a straight line demand curve (the top of the demand curve), small
percentage price reductions can bring large percentage increases in quantity
demanded. This means that demand is elastic over these ranges, and price
reductions bring increases in total expenditure by consumers on the commodity in
question.
Page 48

b) At lower prices on a straight line demand curve (the bottom of the demand curve),
large percentage price reductions can bring small percentage increases in
quantity. This means that demand is inelastic over these price ranges, and price
increases result in increases in total expenditure.

Two special values of price elasticity
a) Demand is perfectly inelastic (η = 0). There is no change in quantity demanded,
regardless of the change in price. The demand curve is a vertical straight line (as
in graph (a) in Issue 1).
b) Perfectly elastic demand (η = ∞). Consumers will want to buy an infinite amount,
but only up to a particular price level. Any price increase above this level will
reduce demand to zero. The demand curve is a horizontal straight line (as in graph
(b) in Issue 1).

Elasticity and the pricing decision
In practice, organisations will have only a rough idea of the shape of their demand curve:
there will only be a limited amount of data about quantities sold at certain prices over a
period of time and, of course, factors other than price might affect demand. Because any
conclusions drawn from such data can only give an indication of likely future behaviour,
management skill and expertise are also needed. Despite this limitation, an awareness of the
concept of elasticity can assist management with pricing decisions.
(a)

(i)
(ii)

(b)

With inelastic demand, increase prices because revenues will increase and total
costs will reduce (because quantities sold will reduce).
With elastic demand, increases in prices will bring decreases in revenue and
decreases in price will bring increases in revenue. Management therefore have to
decide whether the increase/decrease in costs will be less than/greater than
the increases/decreases in revenue.

In situations of very elastic demand, overpricing can lead to massive drops in quantity
sold and hence profits, whereas under-pricing can lead to costly inventory outs and,
again, a significant drop in profits. Elasticity must therefore be reduced by creating
Page 49

a customer preference which is unrelated to price (through advertising and
promotion).
(c)

In situations of very inelastic demand, customers are not sensitive to price. Quality,
service, product mix and location are therefore more important to a firm's pricing
strategy.

(d)

In practice, the prices of many products, such as consumer durables, need to fall over
time if demand is to rise. Costs must therefore fall by the same percentage to
maintain margins.

Determining factors
Factors that determine
the degree of elasticity

Detail

The price of the good
The price of other goods For two types of good the market demand is interconnected.
a) Substitutes, so that an increase in demand for one
version of a good is likely to cause a decrease in
demand for others. Examples include rival brands of the
same commodity (like Coca-Cola and Pepsi-Cola).
b) Complements, so that an increase in demand for one is
likely to cause an increase in demand for the other (e.g.
cups and saucers).
Income

A rise in income gives households more to spend and they will
want to buy more goods. However this phenomenon does not
affect all goods in the same way.
a) Normal goods are those for which a rise in income
increases the demand.
b) Inferior goods are those for which demand falls as
income rises, such as cheaper juice or beer.
c) For some goods demand rises up to a certain point and
then remains unchanged, because there is a limit to
which consumers can or want to consume. Examples
are basic foodstuffs such as salt and cassava.

Tastes and fashions

A change in fashion will alter the demand for a good, or a
particular variety of a good. Changes in taste may stem from
psychological, social or economic causes. There is an argument
Page 50

Factors that determine
the degree of elasticity

Detail
that tastes and fashions are created by the producers of
products and services. There is undeniably some truth in this,
but the modern focus on responding to customers' needs and
wants suggests otherwise.

Expectations

Where consumers believe that prices will rise or that shortages
will occur they will attempt to inventory up on the product,
thereby creating excess demand in the short term.

Obsolescence

Many products and services have to be replaced periodically.
a) Physical goods are literally 'consumed'. Carpets become
threadbare, glasses get broken, foodstuffs get eaten,
children grow out of clothes.
b) Technological developments render some goods
obsolete. Manual office equipment has been largely
replaced by electronic equipment, because it does a
better job, more quickly, quietly, efficiently and
effectively.
c) Software programmes become “overtaken” by new
innovations. Operations systems viz. Windows 7 has
replaced previous systems

Size of the market

The larger the market, the more inelastic the demand for the
product in broad terms. For example, the demand for bread is
relatively inelastic, whereas that the more expensive cuts of
beef may be more elastic.

Necessities

Demand for basic items such as rice toilet rolls and
cassava/manihot is, on the whole, price inelastic.

Page 51

Issue 2: demand and the market
Economic theory suggests that the volume of demand for a good in the market as a whole
is influenced by a variety of variables.
•
•
•

The price of the good
The price of other goods
Expectations

•
•
•
•

Obsolescence
Tastes and fashion
The perceived quality of the product
The size and distribution of household
income

Issue 3: demand and the individual firm
The volume of demand for one organisation's goods rather than another's is influenced
by three principal factors: product life cycle, quality and marketing.

Product life cycle
Product life cycle is 'The period which begins with the initial product specification, and ends
with the withdrawal from the market of both the product and its support. It is characterised by
defined stages including research, development, introduction, maturity, decline and
abandonment.'
(CIMA Official Terminology)

Page 52

Most products pass through the following phases.
Phase

Description

Introduction

The product is introduced to the market. Heavy capital expenditure
will be incurred on product development and perhaps also on the
purchase of new non-current assets and building up inventory for
sale. On its introduction to the market, the product will begin to
earn some revenue, but initially demand is likely to be small.
Potential customers will be unaware of the product or service, and
the organisation may have to spend further on advertising to bring
the product or service to the attention of the market.

Growth

The product gains a bigger market as demand builds up. Sales
revenues increase and the product begins to make a profit. The
initial costs of the investment in the new product are gradually
recovered.

Maturity

Eventually, the growth in demand for the product will slow down
and it will enter a period of relative maturity. It will continue to be
profitable. The product may be modified or improved, as a means
of sustaining its demand.

Saturation and
decline

At some stage, the market will have bought enough of the product
and it will therefore reach 'saturation point'. Demand will start to
fall. For a while, the product will still be profitable in spite of
declining sales, but eventually it will become a loss-maker and this
is the time when the organisation should decide to stop selling the
product or service, and so the product's life cycle should reach its
end.

Case Study
During 2001, low cost PC maker Dell had to discount prices heavily to show continual
growth despite market saturation.

Page 53

The life expectancy of a product will influence the pricing decision. Short-life products
must be quite highly priced so as to give the manufacturer a chance to recover the
investment and make a worthwhile return. This is why fashion goods and new high
technology goods, for example, tend to be highly priced.
The current tendency is towards shorter product life cycles. Notwithstanding this observation,
the life cycles of different products may vary in terms of length of phases, overall length
and shape.
a) Fashion products have a very short life and so do high technology products because
they become rapidly out-dated by new fashions and new technological developments.
b) Different versions of the same product may have different life cycles, and
consumers are often aware of this. For example, the prospective buyer of a new car is
more likely to purchase a recently introduced Ford than a Nissan that has been on the
market for several years, even if there is nothing to choose in terms of quality and
price.

Quality
One firm's product may be perceived to be better quality than another's, and may in some
cases actually be so, if it uses sturdier materials, goes faster or does whatever it is meant to do
in a 'better' way. Other things being equal, the better quality good will be more in demand
than other versions.

Page 54

Marketing
You may be familiar with the 'four Ps' of the marketing mix, all of which influence demand
for a firm's goods.
P…

Details

Price
Product
Place

This refers to the place where a good can be, or is likely to be,
purchased.
•
•

Promotion

If a good is difficult to obtain, potential buyers will turn to
substitutes.
Some goods have no more than local appeal.

This refers to the various means by which firms draw attention to
their products and services.
•

A good brand name is a strong influence on demand.

•

Demand can be stimulated by a variety of promotional tools,
such as free gifts, money off, shop displays, direct mail and
media advertising.

In recent years, emphasis has been placed, especially in marketing, on the importance of
non-price factors in demand. Thus the roles of product quality, promotion, personal selling
and distribution and, in overall terms, brands, have grown. While it can be relatively easy for
a competitor to copy a price cut, at least in the short term, it is much more difficult to copy a
successful brand image.
Some larger organisations go to considerable effort to estimate the demand for their products
or services at differing price levels; in other words, they produce estimated demand curves. A
knowledge of demand curves can be very useful: for example, a large transport company
such as KBS might be considering an increase in bus fares. The effect on total revenues and
profit of the fares increase could be estimated from a knowledge of the demand for transport
services at different price levels. If an increase in the price per ticket caused a large fall in
demand (that is, if demand were price-elastic) total revenues and profits would fall; whereas a
fares increase when demand is price-inelastic would boost total revenue and since a transport
authority's costs are largely fixed, would probably boost total profits too.

Page 55

BLANK

Page 56

OTHER ISSUES THAT INFLUENCE PRICING
DECISIONS
As well as demand, a range of other issues influence pricing decisions including the market
in which an organisation operates, competition, quality and price sensitivity.

Issue 4: markets
The price that an organisation can charge for its products will be determined to a greater or
lesser degree by the market in which it operates. Here are some familiar terms that might
feature as background for a question or that you might want to use in a written answer.

a) Perfect competition: many buyers and many sellers all dealing in an identical product.
Neither producer nor user has any market power and both must accept the prevailing
market price.
b) Monopoly: one seller who dominates many buyers. The monopolist can use his market
power to set a profit-maximising price.
c) Monopolistic competition: a large number of suppliers offer similar, but not identical,
products. The similarities ensure elastic demand whereas the slight differences give
some monopolistic power to the supplier.
d) Oligopoly: where relatively few competitive companies dominate the market. Whilst
each large firm has the ability to influence market prices the unpredictable reaction
from the other giants makes the final industry price indeterminate. Cartels are often
formed.

Page 57

Issue 5: competition
In established industries dominated by a few major firms, it is generally accepted that a price
initiative by one firm will be countered by a price reaction by competitors. In these
circumstances, prices tend to be fairly stable, unless pushed upwards by inflation or strong
growth in demand.
If a rival cuts its prices in the expectation of increasing its market share, a firm has several
options.
a) It will maintain its existing prices if the expectation is that only a small market share
would be lost, so that it is more profitable to keep prices at their existing level.
Eventually, the rival firm may drop out of the market or be forced to raise its prices.
b) It may maintain its prices but respond with a non-price counter-attack. This is a
more positive response, because the firm will be securing or justifying its current
prices with a product change, advertising, or better back-up services.
c) It may reduce its prices. This should protect the firm's market share so that the main
beneficiary from the price reduction will be the consumer.
d) It may raise its prices and respond with a non-price counter-attack. The extra
revenue from the higher prices might be used to finance an advertising campaign or
product design changes. A price increase would be based on a campaign to emphasise
the quality difference between the firm's own product and the rival's product.

Fighting a price war
Peter Bartram (Financial Management, March 2001) suggested a number of ways to fight a
price war.
a) Sell on value, not price, where value is made up of service, response, variety,
knowledge, quality, guarantee and price.
b) Target service, not product market niches, to build in the six non-price factors in (a)
above.

Page 58

Case Study
The Marriott hotel chain has chosen to compete in the premium market on service. When
guests arrive, instead of queuing at a busy reception, they are met at the front door by a host
who gives them their room key.

c) Use 'package pricing' to attract customers

Case Study
Computer retailers in Europe and USA such as Time and PC World have beaten discounters
by offering peripherals, discounted software and extended warranties as part of their more
expensive packages.

a) Make price comparisons difficult. Terrestrial and mobile phone companies offer a
bewildering variety of rates and discount offers which disguise the core price and
make comparisons almost impossible.
b) Build up key accounts, as it is cheaper to get more business from an existing customer
than to find a new one. Customer profitability analysis, covered in Chapter 16, is
important here.
c) Explore new pricing models. E-business provides opportunities to use new pricing
models.
(i)

On-line auctions for a wide range of products are carried out on certain websites.

(ii)

Other websites use a 'community shopping' pricing model, where the price of an
item falls as more people buy it.

(iii) Marginal cost pricing is used on certain websites to get rid of inventory such as
unsold theatre tickets and holidays.
Page 59

Case Study
The makers of desk-top printers sell printers at a heavily discounted price and make up the
“difference” by heavily priced ink cartridges. Coca Cola is experimenting with a vending
machine that varies the cost of a can of coke in line with changes in temperature: the hotter
the weather, the higher the price.

Other issues
Issue

Explanation/example

Price sensitivity This will vary amongst purchasers. Those that can pass on the cost of
purchases will be the least sensitive and will therefore respond more to
other elements of perceived value. For example, the business traveller
will be more concerned about the level of service and quality of food in
looking for an hotel than price, provided that it fits the corporate budget.
In contrast, the family on holiday is likely to be very price sensitive when
choosing an overnight stay.
Price
perception

This is the way customers react to prices. For example, customers may
react to a price increase by buying more. This could be because they
expect further price increases to follow (they are 'stocking up'). Some
believe that the more expensive an item the better the quality. Some
people buy expensive to show off !!

Compatibility
with other
products

A typical example is operating systems on computers, for which a user
would like to have a wide range of compatible software available. For
these types of product there is usually a cumulative effect on demand.
The more people who buy one of the formats, the more choice there is
likely to be of software for that format. This in turn is likely to influence
future purchasers. The owner of the rights to the preferred format will
eventually find little competition and will be able to charge a premium
price for the product.

Competitors

An organisation, in setting prices, sends out signals. Competitors are
likely to react to these signals in some way. In some industries (such as
petrol retailing) pricing moves in unison; in others, price changes by one
supplier may initiate a price war, with each supplier undercutting the
Page 60

Issue

Explanation/example
others. Competition is discussed in more detail below.

Competition
from substitute
products

These are products which could be transformed for the same use or which
might become desirable to customers at particular price levels. For
example, Coffee beans, ground coffee and Instant Coffee: As Instant
coffee becomes more expensive, so ordinary ground coffee or even
roasted beans become more attractive. One way around this is to sell
Instant coffee granules rather than powder.

Suppliers

If an organisation's suppliers notice a price rise for the organisation's
products, they may seek a rise in the price for their supplies to the
organisation on the grounds that it is now able to pay a higher price.

Inflation

In periods of inflation the organisation may need to change prices to
reflect increases in the prices of supplies and so on. Such changes may be
needed to keep relative (real) prices unchanged.

Quality

In the absence of other information, customers tend to judge quality by
price. Thus a price change may send signals to customers concerning the
quality of the product. A price rise may indicate improvements in quality,
a price reduction may signal reduced quality, for example through the use
of inferior components.

Incomes

In times of rising incomes, price may become a less important marketing
variable compared with product quality and convenience of access
(distribution). When income levels are falling and/or unemployment
levels rising, price will become a much more important marketing
variable.

Ethics

Ethical considerations are a further factor, for example whether or not to
exploit short-term shortages through higher prices.

Page 61

BLANK

Page 62

PROFIT MAXIMISATION IN IMPERFECT
MARKETS
In imperfect markets there will be an optimum price/output level at which profits are
maximised.

Some businesses enjoy a monopoly position in their market or something akin to a monopoly
position, even in a competitive market. This is because they develop a unique marketing mix,
for example a unique combination of price and quality, or a monopoly in a localised area.

The significance of a monopoly situation is as follows.
a) The business has choice and flexibility in the prices it sets.
b) Because the business has this freedom of choice in pricing, it will find that at higher
prices demand for its products or services will be less. Conversely, at lower prices,
demand for its products or services will be higher.
c) There will be an optimum price/output level at which profits will be maximised.
(Note. Imperfect markets are markets in which price is affected by the amount supplied to the
market and/or there is limited demand.)

Case Study
A large public transport organisation might be considering an increase in bus fares fares. The
effect on total revenues and profit of the fares increase could be estimated from a knowledge
of the demand for transport services at different price levels. If an increase in the price per
ticket caused a large fall in demand (that is, if demand were price-elastic) total revenues and
profits would fall; whereas a fares increase when demand is price-inelastic would boost total
revenue and since a transport organisation's costs are largely fixed, would probably boost
total profits too.

Page 63

BLANK

Page 64

DERIVING THE DEMAND CURVE
The demand curve shows the relationship between the price charged for a product and the
subsequent demand for that product.

When demand is linear the equation for the demand curve is P = a – bQ/∆Q
where

P

= the price

Q

= the quantity demanded

a

= the price at which demand would be nil

b

= the amount by which the price falls for each stepped change in demand

∆Q

= the stepped change in demand

The constant a is calculated as follows.

Current quantity at current price


a = current price + 
×b
 Change in quantity when price is changed by b 

You need to learn these formulae.
This looks rather complicated in words, but it is very easy once the numbers are substituted.
Note that you are not given these formulae in the exam.

Example: deriving the demand curve
Note the currency values are in thousands of francs( Rwf in this example = Rwf’000)

Page 65

The current price of a product is Rwf12. At this price the company sells 60 items a month.
One month the company decides to raise the price to Rwf14, but only 45 items are sold at this
price. Determine the demand equation.
Solution
Step 1

Find the price at which demand would be nil
Assuming demand is linear, each increase of RWF2 in the price would result in a
fall in demand of 15 units. For demand to be nil, the price needs to rise from its
current level by as many times as there are 15 units in 60 units (60/15 = 4) i.e. to
RWF12 + (4 × RWF2) = RWF20.
Using the formula above, this can be shown as a = RWF12 + ((60/15) × RWF2)=
RWF20

Step 2

Extract figures from the question
The demand equation can now be determined as P = a – bQ/∆Q = 20 – 2Q/15

Step 3

Check your equation
We can check this by substituting RWF12 and RWF14 for P.
12 = 20 − (2 × 60/15) = 20 − 8 = 12
14 = 20 – (2 × 45/15) = 20 − 6 = 14

The equation can also be re-arranged as Q =

(a × ΔQ)− (ΔQ× P)
b

Example: profit maximisation and the demand curve
Maximum demand for JL's product is 10,000 units per annum. Demand will reduce by 100
units for every RWF1,000 increase in the selling price. JL has calculated that the profitmaximising level of sales for the coming year will be 8,000 units.
Required
Calculate the price at which these units will be sold.
Page 66

Solution
a

= (10,000/100 × RWF1,000) = RWF100,000

b

= RWF1,000

∆Q

= 100

∴P

= 100,000 – Q/100,000

Now Q = 8,000
∴P

= RWF100,000 –(RWF 1,000 X 8,000)/100 = RWF20,000

Alternative approach without using the demand curve formula
When P = 0,

demand (Q) = 10,000

When P = 1,000,

demand (Q) = 9,900

∴Demand (Q) = 10,000 – 100 P, where P is the selling price in RWF’000

(because demand will drop by 100 for every increase (from RWF0) of RWF1,000 in the
selling price)
∴ If Q = 8,000, P = (10,000 – 8,000)/100 = RWF20 thousand

Page 67

BLANK

Page 68

THE PROFIT-MAXIMISING PRICE/OUTPUT LEVEL
Microeconomic theory and profit maximisation
Microeconomic theory suggests that as output increases, the marginal cost per unit might
rise (due to the law of diminishing returns) and whenever the firm is faced with a downward
sloping demand curve, the marginal revenue per unit will decline.
Eventually, a level of output will be reached where the extra cost of making one extra unit of
output is greater than the extra revenue obtained from its sale. It would then be unprofitable
to make and sell that extra unit.
Profits will continue to be maximised only up to the output level where marginal cost has
risen to be exactly equal to the marginal revenue.

Profits are maximised using marginalism theory when marginal cost (MC) = marginal
revenue (MR).

Page 69

Profits are maximised at the point where MC = MR, ie at a volume of Qn units. If we add a
demand curve to the graph, we can see that at an output level of Qn, the sales price per unit
would be Pn.
It is important to make a clear distinction in your mind between the sales price and
marginal revenue. In this example, the optimum price is Pn, but the marginal revenue is
much less. This is because the 'additional' sales unit to reach output Qn has only been
achieved by reducing the unit sales price from an amount higher than Pn for all the units to
be sold, not just the marginal extra one. The increase in sales volume is therefore partly offset
by a reduction in unit price; hence MR is lower than Pn.

Determining the profit-maximising selling price: using equations
The optimaum selling price can be determined using equations (i.e. where MC = MR).

You could be provided with equations for marginal cost and marginal revenue and/or
have to devise them from information in the question. By equating the two equations you
can determine the optimum price. Remember, marginal cost is the extra cost of producing
one extra unit, marginal revenue is the extra revenue from that one extra unit. Marginal
revenue may not be the same as the price charged for all units up to that demand level, as
to increase volumes the price may have to be reduced. The following example provides an
illustration.

Page 70

Example: MC = MR
MOC makes and sells a copyrighted, executive game for two distinct markets, in which it has
a monopoly. The fixed costs of production per month are Rwf20,000,000 and variable costs
per unit produced, and sold, are Rwf40,000. (The monthly sales can be thought of as X,
where X = X1 + X2, with X1 and X2 denoting monthly sales in their respective markets.)
Detailed market research has revealed the demand functions in the markets to be as follows,
with prices shown as P1, P2.
Market 1:

P1 = 55,000 – 50X1

Market 2:

P2 = 200,000 – 200X2

(Note. These formulae are simple linear equations. They show how the price (P) can be
determined for a given level of demand (X). So in market 1, at a level of demand of 100, the
price (P) will be 55,000 – (50 × 100) = 50,000.)
From these, the management accountant has derived that the marginal revenue functions in
the two markets are as follows.
Market 1:

MR1 = 55,000 – 100X1

Market 2:

MR2 = 200,000 – 400X2

(Note. In market 1, the marginal revenue if 100 units are sold is 55,000 – (100 × 100) =
45,000.)
The management accountant believes there should be price discrimination; the price is
currently RWF50,000 per game in both markets.
Required
Analyse the information for the executive game and, given the management accountant's
belief, do the following.
a) Calculate the price to charge in each market, and the quantity to produce (and sell)
each month, to maximise profit.
b) Determine the revenue function for each market and the maximum monthly profit in
total.
c) Calculate and comment on the change in total profitability and prices.
Page 71

Solution
a) In both markets, marginal cost = variable cost per unit = RWF40,000
Profit is maximised when marginal revenue = marginal cost.

Market 1
55,000 – 100X1

= 40,000

100X1

= 15,000

X1

= 15,000/100 = 150

and price P1 = 55,000 – (50 × 150) = RWF47,500.
Hence the price in market 1 should be RWF47,500 per unit and 150 units should be
produced.

Market 2
200,000 – 400X2

= 40,000

400X2

= 160,000

X2

= 160,000/400 = 400

and price P2 = 200,000 – (200 × 400) = RWF120,000
Hence the price in market 2 should be RWF120,000 per unit and 400 units should be
produced.
Total number of items to be produced per month is 550.

b) Revenue = unit price × number of units sold
Market 1
Page 72

Revenue = P1X1 = 55,000X1 – 50X12
Market 2
Revenue = P2X2 = 200,000X2 – 200X22
From (a), profit is maximised when
X1 = 150 and X2 = 400
P1 = 47,5005 and P2 = 120,000
At maximum profit:
Total revenue = (47,500 × 150) + (120,000 × 400) = RWF55,125,000
Total costs = 20,000,000 + (40,000 × 550) =RWF42,000,000
Total maximum monthly profit = RWF13,125,000

c) Currently the price is RWF50,000 in both markets.
Market 1

50,000 = 55 – 50X1
50X1 = 55,000 – 50,000 = 5,000
X1 = 5,000/50 = 100

Market 2

50,000 = 200,00 – 200X2
200X2 = 200,000 – 50,000 = 150,000
X2 = 150,000/200 = 750

Therefore the total number of units = 100 + 750 = 850.
Total revenue = RWF50,000 × 850 = RWF42,500,000.
Total cost = 20,000,000 + (40,000 × 850) = RWF54,000,000.
So the game currently makes a loss of RWF11,500,000.
Page 73

Hence, if the prices are changed to RWF47,500 in market 1 and RWF120,000 in market
2, the company can expect to turn a monthly loss of RWF11,500,000 into a profit of
RWF13,125,000.
One of the Section C, 25-mark questions in a pilot paper required the approach shown in the
solution above.

Formulae for MC and MR are often derived using a mathematical technique known as
differential calculus. This is well outside the scope of the syllabus, and so you will be
provided with equations representing MC and MR if they are needed.
Note, however, that if a question states that the extra cost of producing one extra item is
RWF20, say, you will be expected to realise that the MC is RWF20. Likewise, if you are told
that 100 units are sold for RWF10 each, but 101 can only be sold for RWF9.99, the MR
of the 101st item is (101 × RWF9.99) – (100 × RWF10) = RWF8.99.

This is a popular topic for exam questions.
The trickiest Section A question in the May 2005 exam, worth four marks, required
candidates to calculate a profit-maximising selling price using equations for MC and MR.

Determining the profit-maximising selling price: visual inspection
of a tabulation of data
The optimum selling price can also be determined using tabulation, graphs and gradients.
To determine the profit-maximising selling price:
a) Work out the demand curve and hence the price and the total revenue (PQ) at
various levels of demand.
b) Calculate total cost and hence marginal cost at each level of demand.
c) Finally calculate profit at each level of demand, thereby determining the price and
level of demand at which profits are maximised.
Page 74

Example
Learning outcome: A(iii)
An organisation operates in a market where there is imperfect competition, so that to sell
more units of output, it must reduce the sales price of all the units it sells. The following data
is available for prices and costs.
Total output

Sales price per unit (AR)

Average cost of output (AC)

Units

RWF‘000
–
504
471
439
407
377
346
317
288
259
232

RWF ‘000 per unit
–
720
402
288
231
201
189
182
180
186
198

0
1
2
3
4
5
6
7
8
9
10

The total cost of zero output is RWF600,000.
Complete the table below to determine the output level and price at which the organisation
would maximise its profits, assuming that fractions of units cannot be made.

Page 75

Units

Price

Total
revenue

Marginal
revenue

RWF ‘000

RWF ‘000

RWF ‘000

Total cost

Marginal
cost

Profit

RWF ‘000

RWF ‘000

RWF ‘000

0
1
2
3
4
5
6
7
8
9
10

Solution
The correct answer is that profit is maximised at seven units of output and a price of
RWF317,000 when MR is most nearly equal to MC.

Page 76

Price

Total
revenue

Marginal
revenue

RWF ‘000

RWF ‘000

RWF ‘000

0

0

0

0

600

-

(600)

1

504

504

504

720

120

(216)

2

471

942

438

804

84

138

3

439

1,317

375

864

60

453

4

407

1,628

311

924

60

704

5

377

1,885

257

1,005

81

880

6

346

2,076

191

1,134

129

942

7

317

2,219

143

1,274

140

945

8

288

2,304

85

1,440

166

864

9

259

2,331

27

1,674

234

657

10

232

2,320

(11)

1,980

306

340

price:

graphical

Units

Determining
approach

the

profit-maximising

Total cost

Marginal
cost

Profit

RWF ‘000

RWF ‘000

RWF ‘000

selling

The diagrams below show that profits are maximised at the point where the vertical
distance between the total revenue curve and the total costs curve is at a maximum (which is
fairly obvious if you think about it since profits are maximised when the difference between
cost and revenue is maximised). This profit-maximising demand level also corresponds to
the point at which the MC and MR curves intersect, as we would expect. Notice how the
profit-maximising price can be read off from the demand curve.

Page 77

Determining the profit-maximising selling price: using gradients
Suppose we were to draw tangents to the total revenue and total cost curves at the points at
which profit is maximised. As you can see, the gradients of these tangents are the same.

Page 78

The gradient of the total cost curve is the rate at which total cost changes with changes
in volume, which is simply marginal cost. Likewise, the gradient of the total revenue
curve is the rate at which total revenue changes with changes in volume, which is the
marginal revenue. At the point of profit maximisation, the two gradients are equal and
hence, once again, MC = MR.

Optimum pricing in practice
There are problems with applying the approach described above in practice for the following
reasons:
a) It assumes that the demand curve and total costs can be identified with certainty. This
is unlikely to be so.
b) It ignores the market research costs of acquiring knowledge of demand.
c) It assumes the firm has no production constraint which could mean that the
equilibrium point between supply and demand cannot be reached.
d) It assumes the objective is to maximise profits. There may be other objectives.

Case Study
Microsoft dominates the market for many types of computer software, but this domination
was not achieved by setting short-term profit-maximising selling prices for the MS-DOS and
Windows operating systems. By offering cheap licences to PC manufacturers for use of these
operating systems, Microsoft word processing, spread-sheet, graphics and database packages
have become almost industry-standard.

Page 79

e) It assumes that price is the only influence on quantity demanded. We saw in Sections
1 and 2 that this is far from the case.
f) It is complicated by the issue of price discrimination (the practice of charging
different unit selling prices for the same product). We look at price discrimination in
the next chapter.
g) Although there are arguments for the applicability of the concept of the profitmaximising unit selling price in traditional markets where homogenous, massproduced products are in continuous supply (such as public transport), the modern
trend is towards short product life cycles and a high degree of product
differentiation.

Further reading
Read the article that appeared in Financial Management in May 2006.
This article takes a humorous approach to the application of optimum pricing but could well
form the basis of a question in the exam.

Page 80

CHAPTER ROUNDUP
• Demand is normally elastic because demand will increase as prices are lowered.
• Price elasticity of demand is a measure of the extent of change in market demand for a
good in response to a change in its price.
• If demand is elastic a reduction in price would lead to a rise in total sales revenue. If
demand is inelastic, a reduction in price would lead to a fall in total sales revenue.
• The volume of demand for one organisation's goods rather than another's is
influenced by three principal factors: product life cycle, quality and marketing.
• As well as demand, a range of other issues influence pricing decisions including the
market in which an organisation operates, competition, quality and price sensitivity.
• In imperfect markets there will be an optimum price/output level at which profits are
maximised.
• When demand is linear the equation for the demand curve is P = a – bQ/∆Q
Where:
P
Q
a
b
∆Q

= the price
= the quantity demanded
= the price at which demand would be nil
= the amount by which the price falls for each stepped change in demand
= the stepped change in demand

The constant a is calculated as follows.



a = current price + 

Current quantity at currentprice



×b 

 Changein quantity when price is changedby b 

You need to learn these formulae.

Page 81

• Profits are maximised using marginalism theory when marginal cost (MC) =
marginal revenue (MR).
• The optimum selling price can be determined using equations (i.e. when MC = MR).
• The optimum selling price can also be determined using tabulation, graphs and
gradients.

Page 82

STUDY UNIT 3
Pricing Approaches And Strategies
Contents

Page

Full Cost-plus Pricing ……………………………………………………….

85

Marginal Cost-plus Or Mark-up Pricing ……….…………………..……..

95

Economists’ Versus Accountants’ Views On Pricing Decisions…………….

97

Pricing Based on Mark-up Per Unit Of Limiting Factor …………..………

101

Pricing Strategies For Special Orders ………………………………………

103

Pricing Strategies For New Products ………………………………………

105

Other Pricing Strategies ………………….…………………………………

109

Page 83

BLANK

Page 84

FULL COST-PLUS PRICING
In full cost-plus pricing the sales price is determined by calculating the full cost of the
product and then adding a percentage mark-up for profit. The most important criticism of full
cost-plus pricing is that it fails to recognise that since sales demand may be determined by the
sales price, there will be a profit-maximising combination of price and demand.

Reasons for its popularity
In practice cost is one of the most important influences on price. Many firms base price
on simple cost-plus rules (costs are estimated and then a mark-up is added in order to set the
price). A study by Lanzilotti gave a number of reasons for the predominance of this
method.
a) Planning and use of scarce capital resources are easier.
b) Assessment of divisional performance is easier.
c) It emulates the practice of successful large companies.
d) Organisations fear government action against 'excessive' profits.
e) There is a tradition of production rather than of marketing in many organisations.
f) There is sometimes tacit collusion in industry to avoid competition.
g) Adequate profits for shareholders are already made, giving no incentive to maximise
profits by seeking an 'optimum' selling price.
h) Cost-based pricing strategies based on internal data are easier to administer.
i) Over time, cost-based pricing produces stability of pricing, production and
employment.

Full cost-plus pricing is a method of determining the sales price by calculating the full cost
of the product and adding a percentage mark-up for profit.

Page 85

Setting full-cost plus prices
The 'full cost' may be a fully absorbed production cost only, or it may include some absorbed
administration, selling and distribution overhead.
A business might have an idea of the percentage profit margin it would like to earn, and so
might decide on an average profit mark-up as a general guideline for pricing decisions.
This would be particularly useful for businesses that carry out a large amount of contract
work or jobbing work, for which individual job or contract prices must be quoted regularly
to prospective customers. However, the percentage profit mark-up does not have to be rigid
and fixed, but can be varied to suit the circumstances. In particular, the percentage mark-up
can be varied to suit demand conditions in the market.

Question
Learning outcome: A(ii)
A product's full cost is RWF4,750 and it is sold at full cost plus 70%. A competitor has just
launched a similar product selling for RWF7,990.
Required
Fill in the gap in the sentence below.
The cost-plus percentage will need to be reduced by...... %.

Answer

The correct answer is that the cost-plus percentage will need to be reduced by 2%.
Profits = (7,990 – 4,750) = 3,240
Mark-up = (3,240/4,750) × 100% = 68%
∴% needs to be reduced by (70 – 68)% = 2%

Page 86

Example: full cost-plus pricing
Markup Ltd has begun to produce a new product, Product X, for which the following cost
estimates have been made.
RWF‘000
Direct materials

27

Direct labour:8 hrs at RWF2,5000 per hour

20

Variable production overheads: machining, ½ hr at RWF6,000 per hour

3
50

Production fixed overheads are budgeted at RWF300,000,000 per month and, because of the
shortage of available machining capacity, the company will be restricted to 10,000 hours of
machine time per month. The absorption rate will be a direct labour rate, however, and
budgeted direct labour hours are 50,000 per month. It is estimated that the company could
obtain a minimum contribution of RWF10,000 per machine hour on producing items other
than product X.
The direct cost estimates are not certain as to material usage rates and direct labour
productivity, and it is recognised that the estimates of direct materials and direct labour costs
may be subject to an error of ± 15%. Machine time estimates are similarly subject to an error
of ± 10%.
The company wishes to make a profit of 20% on full production cost from product X.
Required
Ascertain the full cost-plus based price.

Page 87

Solution
Even for a relatively 'simple' cost-plus pricing estimate, some problems can arise, and certain
assumptions must be made and stated. In this example, we can identify two problems.
a) Should the opportunity cost of machine time be included in cost or not?
b) What allowance, if any, should be made for the possible errors in cost estimates?

Different assumptions could be made.
a) Exclude machine time opportunity costs: ignore possible costing errors
RWF‘000

Direct materials

27.00

Direct labour (8 hours)

20.00

Variable production overheads
Fixed production overheads (at

3.00
300 m
25 m

48.00

= RWF6,000 per direct labour

hour)
Full production cost

98.00

Profit mark-up (20%)

19.60

Selling price per unit of product X

b)

117.60

Include machine time opportunity costs: ignore possible costing errors
RWF ‘000

Full production cost as in (a)

98.00

Opportunity cost of machine time: contribution forgone (½ hr ×
RWF10,000)
Page 88

5.00

Adjusted full cost

103.00

Profit mark-up (20%)

20.60

Selling price per unit of product X

123.60

c) Exclude machine time opportunity costs but make full allowance for possible underestimates of cost
RWF ‘000

Direct materials

27.00

Direct labour

20.00

RWF ‘000

47.00
Possible error (15%)

7.05
54.05

Variable production overheads

3.00

Possible error (10%)

0.30
3.30

Fixed production overheads (4 hrs × €12)
Possible error (labour time) (15%)

48.00
7.20
55.20

Potential full production cost

112.55

Profit mark-up (20%)

22.51

Selling price per unit of product X

135.06

Page 89

d) Include machine time opportunity costs and make a full allowance for possible
under-estimates of cost
Rwf ‘000
Potential full production cost as in (c)

112.55

Opportunity cost of machine time:
potential contribution forgone (½ hr × RWF 10,000 ×
110%)
Adjusted potential full cost

5.50

118.05

Profit mark-up (20%)

23.61

Selling price per unit of product X

141.66

Using different assumptions, we could arrive at any of four different unit prices in the range
RWF k117.60 to RWF k141.66.

Problems with and advantages of full cost-plus pricing
There are several serious problems with relying on a full cost approach to pricing.
a) It fails to recognise that since demand may be determining price, there will be a
profit-maximising combination of price and demand.
b) There may be a need to adjust prices to market and demand conditions.
c) Budgeted output volume needs to be established. Output volume is a key factor in the
overhead absorption rate.
d) A suitable basis for overhead absorption must be selected, especially where a
business produces more than one product.
However, it is a quick, simple and cheap method of pricing which can be delegated to junior
managers (which is particularly important with jobbing work where many prices must be
decided and quoted each day) and, since the size of the profit margin can be varied, a
Page 90

decision based on a price in excess of full cost should ensure that a company working at
normal capacity will cover all of its fixed costs and make a profit.
Example: full cost-plus versus profit-maximising prices
Tiger has budgeted to make 50,000 units of its product, timm. The variable cost of a timm is
RWF5,000 and annual fixed costs are expected to be RWF150,000,000.
The financial director of Tiger has suggested that a mark-up of 25% on full cost should be
charged for every product sold. The marketing director has challenged the wisdom of this
suggestion, and has produced the following estimates of sales demand for timms.

Price per unit (RWF‘000)

9

10

11

12

13

Demand (units)

42,000

38,000

35,000

32,000

27,000

Required
a) Calculate the profit for the year if a full cost-plus price is charged.
b) Calculate the profit for the year if a profit-maximising price is charged.
c)
Assume in both (a) and (b) that 50,000 units of timm are produced regardless of sales
volume.

Page 91

Solution
The full cost per unit comprises RWF k5 of variable costs plus RWF k3 of fixed costs (RWF
k8 in total). A 25% mark-up on this cost gives a selling price of RWF k10 per unit so that
sales demand would be 38,000 units. (Production is given as 50,000 units.) Profit using
absorption costing would be as follows.

RWF ‘000
Sales

RWF ‘000
380,000

Costs of production (50,000 units)
Variable (50,000 × RWF k5)

250,000

Fixed (50,000 × RWF k3)

150,000
400,000

Less increase in inventory (12,000 units × 8)
Cost of sales

(96,000)
304,000

Profit

76,000

Profit using marginal costing instead of absorption costing, so that fixed overhead costs are
written off in the period they occur, would be as follows. (The 38,000 unit demand level is
chosen for comparison.)
RWF ‘000
Contribution (38,000 × RWF k(10 – 5))

190,000

Fixed costs

150,000

Profit

40,000

Page 92

Since the company cannot go on indefinitely producing an output volume in excess of sales
volume, this profit figure is more indicative of the profitability of timms in the longer term.
A profit-maximising price is one which gives the greatest net (relevant) cash flow, which in
this case is the contribution-maximising price.

Price

Unit contribution

Demand

Total contribution

RWF ‘000

RWF ‘000

Units

RWF ‘000

9

4

42,000

168,000

10

5

38,000

190,000

11

6

35,000

210,000

12

7

32,000

224,000

13

8

27,000

216,000

The profit maximising price is RWF12,000, with annual sales demand of 32,000 units.
This example shows that a cost-plus based price is unlikely to be the profit-maximising
price, and that a marginal costing approach, calculating the total contribution at a variety of
different selling prices, will be more helpful for establishing what the profit-maximising
price ought to be.

Page 93

BLANK

Page 94

MARGINAL COST-PLUS PRICING OR MARK-UP
PRICING
Marginal cost-plus pricing involves adding a profit margin to the marginal cost of
production/sales. A marginal costing approach is more likely to help with identifying a profitmaximising price.
Whereas a full cost-plus approach to pricing draws attention to net profit and the net profit
margin, a variable cost-plus approach to pricing draws attention to gross profit and the
gross profit margin or contribution.

Marginal cost-plus pricing/mark-up pricing is a method of determining the sales price by
adding a profit margin on to either marginal cost of production or marginal cost of sales.

The advantages and disadvantages of a marginal cost-plus
approach to pricing
Here are the advantages.
a) It is a simple and easy method to use.
b) The mark-up percentage can be varied, and so mark-up pricing can be adjusted to
reflect demand conditions.
c) It draws management attention to contribution, and the effects of higher or lower
sales volumes on profit. In this way, it helps to create a better awareness of the
concepts and implications of marginal costing and cost-volume-profit analysis. For
example, if a product costs RWF k10 per unit and a mark-up of 150% is added to
reach a price of RWF k25 per unit, management should be clearly aware that every
additional RWF k1 of sales revenue would add 600 francs to contribution and profit.
d) In practice, mark-up pricing is used in businesses where there is a readilyidentifiable basic variable cost. Retail industries are the most obvious example, and
it is quite common for the prices of goods in shops to be fixed by adding a mark-up
(20% or 33.3%, say) to the purchase cost.

Page 95

There are, of course, drawbacks to marginal cost-plus pricing.
a) Although the size of the mark-up can be varied in accordance with demand conditions,
it does not ensure that sufficient attention is paid to demand conditions,
competitors' prices and profit maximisation.
b) It ignores fixed overheads in the pricing decision, but the sales price must be
sufficiently high to ensure that a profit is made after covering fixed costs.

In our study of decision making to date we have adopted a marginal cost approach in that we
have considered the effects on contribution and have classed (most) fixed overheads as
irrelevant. In pricing decisions, however, there is a conflict with such an approach because of
the need for full recovery of all costs incurred.

Page 96

ECONOMISTS’ VERSUS ACCOUNTANTS’ VIEWS
ON PRICING DECISIONS
Economic theory claims that profit is maximised by setting a price so that marginal cost
equals marginal revenue. But because most cost accounting systems are set up to provide
information for financial reporting purposes, it can be difficult to identify short-run or
long-run marginal cost, even if ABC is used.
Mike Lucas (Management Accounting, June 1999) looked at this topic. What follows is a
summary of his article.
Research by accountants has suggested that full costs play an important role in many pricing
and output decisions. The use of full cost is at odds with the economists’ view that prices
should be set at a level which equates marginal cost and marginal revenue, however.

Economic research findings
Economic research by Hall and Hitch in 1939 found that most organisations tended to set
prices by adding a fairly constant mark-up to full cost for three principal reasons.
a) Organisations have no knowledge of their demand curves because of a lack of
information about customers’ preferences and/or competitor reaction to price changes.
b) Price stickiness
As illustrated in the diagram below (a kinked demand curve), an organisation may
feel that if prices are increased above the current price P1, competitors will not match
the increase, demand being very elastic. The increase in profit per unit will not
compensate for the profit lost from the reduction in quantity sold, and so total profit
will fall.
If price is reduced below the current price P1, competitors will match the price
decrease, demand being inelastic. The small increase in the quantity sold will not
compensate for the drop in profit per unit, and so total profit will fall.
The organisation would therefore be reluctant to increase or decrease the price from
the current level if there are minor changes in costs or market conditions, giving rise
to apparent price stickiness.
Page 97

c) The frequent price changes which are likely to occur if profit-maximising prices are
set (as prices are changed whenever there is a change in demand or costs) can be
administratively expensive to bring in and can inconvenience sales staff,
distributors as well as customers.

Economists countered the suggested predominance of cost-based approaches, however, with
arguments of ‘implicit marginalism’, whereby organisations act as though they are
setting prices on the basis of equating MC and MR, even if this approach is not
consciously adopted. Evidence for this includes:
a) Discounting prices when market circumstances change and/or accepting a lower profit
margin when competition increases. This is similar to using a marginal revenue
function.
b) Reducing the overhead charged to products to reflect the short-term nature of some
fixed costs. This is similar to using a marginal cost function.

Page 98

Reconciling full cost pricing and marginalist, profit-maximising
principles
Some economists have tried to show that full cost pricing is compatible with marginalist
principles. One argument (by Koutsoylannis) is that:
Price (P) = average variable cost (AVC) + costing margin
where the costing margin = average fixed cost (AFC) + normal profit mark-up.
Taking AVC to be the best available approximation of long-run marginal cost, any
adjustments made to the costing margin because of competitive forces can be viewed as the
organisation attempting to establish its demand curve, and so – by implication – its marginal
revenue function.

Accounting research
Problems with the methods used for accounting research may not have picked up on the fact
that organisations are constantly making adjustments to prices in order to meet market
situations, and, while many organisations might believe they set prices on a cost plus basis,
these prices are the actual prices charged in just a few situations.

ABC and long-run marginal cost
As you will know from your Certificate level studies, ABC costs should be long-run
avoidable (marginal) costs and so in theory organisations using ABC costs are following
economists’ views of pricing. The treatment of ‘indivisibilities’ means that this is not
necessarily the case, however.
‘Indivisibilities’ occur when a reduction in the level of activity does not lead to a
proportionate reduction in resource inputs. For example, a process may be duplicated so
that output can be doubled but it may not necessarily be possible to halve the process if
demand drops by 50%.

Page 99

The cost of indivisible resources should therefore not be attributed to individual products
as, to the extent that they are indivisible, they will be incurred regardless of the activity level
and so are unavoidable in relation to a particular product.

Conclusion
Economic theory claims that profit is maximised by setting a price so that marginal cost
equals marginal revenue. But because most cost accounting systems are set up to provide
information for financial reporting purposes, it can be difficult to identify short-run or
long-run marginal cost, even if ABC is used.
It is difficult to know whether organisations are carrying out the analysis necessary to
determine marginal cost or whether the full cost provided by the accounting system is used
for pricing decisions.
The debate over the theory of pricing therefore continues.

Page 100

PRICING BASED ON MARK-UP PER UNIT OF
LIMITING FACTOR
Another approach to pricing might be taken when a business is working at full capacity,
and is restricted by a shortage of resources from expanding its output further. By deciding
what target profit it would like to earn, it could establish a mark-up per unit of limiting
factor.

Example: mark-up per unit of limiting factor
Suppose that a company provides a window cleaning service to offices and factories.
Business is brisk, but the company is restricted from expanding its activities further by a
shortage of window cleaners. The workforce consists of 12 window cleaners, each of whom
works a 40 hour week. They are paid RWF400 per hour. Variable expenses are RWF600 per
hour. Fixed costs are RWF1,500,000 per week. The company wishes to make a contribution
of at least RWF5,000 per hour.
The minimum charge per hour for window cleaning would then be as follows.
RWF per hour
Direct wages

400

Variable expenses

600

Contribution

5,000

Charge per hour

6,000

The company has a total workforce capacity of (12 × 40) 480 hours per week, and so total
revenue would be RWF2,880,000 per week, contribution would be (480 × RWF5,000)
RWF2,400,000, leaving a profit after fixed costs of RWF900,000per week.

Page 101

BLANK

Page 102

PRICING STRATEGIES FOR SPECIAL ORDERS
The basic approach to pricing special orders is minimum pricing.

What is a special order?
A special order is a one-off revenue-earning opportunity. These may arise in the following
situations.
a) When a business has a regular source of income but also has some spare capacity
allowing it to take on extra work if demanded. For example a brewery might have a
capacity of 500,000 barrels per month but only be producing and selling 300,000
barrels per month. It could therefore consider special orders to use up some of its
spare capacity.
b) When a business has no regular source of income and relies exclusively on its ability
to respond to demand. A building firm is a typical example as are many types of subcontractors. In the service sector consultants often work on this basis.
The reason for making the distinction is that in the case of (a), a firm would normally attempt
to cover its longer-term running costs in its prices for its regular product. Pricing for special
orders need therefore take no account of unavoidable fixed costs. This is clearly not the
case for a firm in (b)'s position, where special orders are the only source of income for the
foreseeable future.

Page 103

Minimum pricing
The minimum price is the price at which the organisation would break even if it undertook
the work. It would have to cover the incremental costs of producing and selling the item and
the opportunity costs of the resources consumed.

Firms with high overheads are faced with difficulties. Ideally some means should be found of
identifying the causes of such costs. Activity based analysis might reveal ways of attributing
overheads to specific jobs or perhaps of avoiding them altogether.

In today's competitive markets it is very much the modern trend to tailor products or
services to customer demand rather than producing for stock. This suggests that 'special'
orders may become the norm for most businesses.
In the motor trade in the UK, an order for a new car is often fulfilled not from stock, but by
programming that order into the manufacturing schedule. Each customer wants a different
colour, trim and other accessories and it is cheaper to make to order than to make for stock in
case a passing motorist wants just that car.
The same can be for furniture suites where many different combinations of colour/pattern and
materials are available.

Page 104

PRICING STRATEGIES FOR NEW PRODUCTS
Two pricing strategies for new products are market penetration pricing and market
skimming pricing.

Tabulation
Suppose that Novo is about to launch a new product with a variable cost of RWF10,000 per
unit. The company has carried out market research (at a cost of RWF15,000,000) to
determine the potential demand for the product at various selling prices.

Selling price

Demand

RWF‘000

Units

30

20,000

25

30,000

20

40,000

Its current capacity is for 20,000 units but additional capacity can be made available by using
the resources of another product line. If this is done the lost contribution from the other
product will be RWF35,000,000 for each additional 10,000 units of capacity.
How could we analyse this information for senior management in a way that helps them to
decide on the product's launch price?
Tabulation is the approach to use with a problem of this type.

Page 105

Selling

Variable

Opportunity

price

Demand

costs

costs

RWF ‘000

Units
('000)

RWF m

30

20

25
20

Sales
revenue

Contribution

RWFm

Total
costs
RWF m

RWF m

RWF m

200

–

200

600

400

30

300

35

335

750

415

40

400

70

470

800

330

The optimum price to maximise short-term profits is RWF25,000. However, it is quite
possible that the aim will not be to maximise short-term profits and a number of other
strategies may be adopted, as discussed below.
The main objections to the approach described above are that it only considers a limited
range of prices (what about charging RWF27,500?) and it takes no account of the
uncertainty of forecast demand. However, allowance could be made for both situations by
collecting more information.

First on the market?
A new product pricing strategy will depend largely on whether a company's product or
service is the first of its kind on the market.
a) If the product is the first of its kind, there will be no competition - yet and the
company, for a time at least, will be a monopolist. Monopolists have more influence
over price and are able to set a price at which they think they can maximise their
profits. A monopolist's price is likely to be higher, and its profits bigger, than those of
a company operating in a competitive market.
b) If the new product being launched by a company is following a competitor's product
onto the market, the pricing strategy will be constrained by what the competitor is
Page 106

already doing. The new product could be given a higher price if its quality is better, or
it could be given a price which matches the competition. Undercutting the
competitor's price might result in a price war and a fall of the general price level in the
market.

Market penetration pricing
Market penetration pricing is a policy of low prices when the product is first launched in
order to obtain sufficient penetration into the market.
Circumstances in which a penetration policy may be appropriate
a) If the firm wishes to discourage new entrants into the market
b) If the firm wishes to shorten the initial period of the product's life cycle in order to
enter the growth and maturity stages as quickly as possible
c) If there are significant economies of scale to be achieved from a high volume of
output, so that quick penetration into the market is desirable in order to gain unit cost
reductions
d) If demand is highly elastic and so would respond well to low prices.

Penetration prices are prices which aim to secure a substantial share in a substantial total
market. A firm might therefore deliberately build excess production capacity and set its
prices very low. As demand builds up the spare capacity will be used up gradually and unit
costs will fall; the firm might even reduce prices further as unit costs fall. In this way, early
losses will enable the firm to dominate the market and have the lowest costs.

Market skimming pricing
Market skimming pricing involves charging high prices when a product is first launched
and spending heavily on advertising and sales promotion to obtain sales.
As the product moves into the later stages of its life cycle, progressively lower prices will
be charged and so the profitable 'cream' is skimmed off in stages until sales can only be
sustained at lower prices.

Page 107

The aim of market skimming is to gain high unit profits early in the product's life. High
unit prices make it more likely that competitors will enter the market than if lower prices
were to be charged.
Circumstances in which such a policy may be appropriate
a) Where the product is new and different, so that customers are prepared to pay high
prices so as to be one up on other people who do not own it – e.g. the iPad.
b) Where the strength of demand and the sensitivity of demand to price are unknown.
It is better from the point of view of marketing to start by charging high prices and
then reduce them if the demand for the product turns out to be price elastic than to
start by charging low prices and then attempt to raise them substantially if demand
appears to be insensitive to higher prices.
c) Where high prices in the early stages of a product's life might generate high initial
cash flows. A firm with liquidity problems may prefer market-skimming for this
reason.
d) Where the firm can identify different market segments for the product, each
prepared to pay progressively lower prices. If product differentiation can be
introduced, it may be possible to continue to sell at higher prices to some market
segments when lower prices are charged in others. This is discussed further below.
e) Where products may have a short life cycle, and so need to recover their development
costs and make a profit relatively quickly.

Page 108

OTHER PRICING STRATEGIES
Product differentiation may be used to make products appear to be different. Price
discrimination is then possible.

Product differentiation and price discrimination
Price discrimination is the practice of charging different prices for the same product to
different groups of buyers when these prices are not reflective of cost differences.
In certain circumstances the same product can be sold at different prices to different
customers. There are a number of bases on which such discriminating prices can be set.

Basis

Detail

By market
segment

A cross-border bus company could market its services at different
prices in Rwanda and Uganda for say a return journey between
Kampala and Kigali.

By product
version

Many car models have optional extras which enable one brand to
appeal to a wider cross-section of customers. The final price need
not reflect the cost price of the optional extras directly: usually the
top of the range model would carry a price much in excess of the
cost of providing the extras - as a prestige appeal.

By place

Theatre seats are usually sold according to their location in the
theatre so that patrons pay different prices for the same performance
according to the seat type they occupy.

By time

This is perhaps the most popular type of price discrimination. Offpeak travel bargains, hotel prices and telephone charges are all
attempts to increase sales revenue by covering variable but not
necessarily average cost of provision. In Europe railway companies
are successful price discriminators, charging more to rush hour rail
commuters whose demand is inelastic at certain times of the day.

Page 109

Price discrimination can only be effective if a number of conditions hold.
a) The market must be segmentable in price terms, and different sectors must show
different intensities of demand. Each of the sectors must be identifiable, distinct and
separate from the others, and be accessible to the firm's marketing communications.
b) There must be little or no chance of a black market developing (this would allow
those in the lower priced segment to resell to those in the higher priced segment).
c) There must be little or no chance that competitors can and will undercut the firm's
prices in the higher priced (and/or most profitable) market segments.
d) The cost of segmenting and administering the arrangements should not exceed the
extra revenue derived from the price discrimination strategy.

'Own label' pricing: a form of price discrimination
Many supermarkets and multiple retail stores sell their 'own label' products, often at a lower
price than established branded products. The supermarkets or multiple retailers do this by
entering into arrangements with manufacturers, to supply their goods under the 'own brand'
label.

Premium pricing
This involves making a product appear 'different' through product differentiation so as to
justify a premium price. The product may be different in terms of, for example, quality,
reliability, durability, after sales service or extended warranties. Heavy advertising can
establish brand loyalty which can help to sustain a premium and premium prices will always
be paid by those customers who blindly equate high price with high quality.

Page 110

Product bundling
Product bundling is a variation on price discrimination which involves selling a number of
products or services as a package at a price lower than the aggregate of their individual
prices. For example a hotel might offer a package that includes the room, meals, use of
leisure facilities and entertainment at a combined price that is lower than the total price of the
individual components. This might encourage customers to buy services that they might
otherwise not have purchased and so increase cash-flow and contribution to fixed overheads.
The success of a bundling strategy depends on the expected increase in sales volume and
changes in margin. Other cost changes, such as in product handling, packaging and
invoicing costs, are possible. Longer-term issues such as competitors' reactions must also be
considered.
Another bundle could be a PC with software, printer and extended warranty insurance.

Pricing with optional extras
The decision here is very similar to that for product bundling. It rests on whether the increase
in sales revenue from the increased price that can be charged is greater than the increase
in costs required to incorporate extra features. Not all customers will be willing to pay a
higher price for additional features if they do not want or need those features.

Psychological pricing
Psychological pricing strategies include pricing a product at RWF19,990 instead of
RWF20,000 and withdrawing an unsuccessful product from the market and then relaunching
it at a higher price, the customer having equated the lower price with lower quality (which
was not the seller's intention).

Page 111

Multiple products and loss leaders
Most organisations sell a range of products. The management of the pricing function is likely
to focus on the profit from the whole range rather than the profit on each single product.
Take, for example, the use of loss leaders: a very low price for one product is intended to
make consumers buy additional products in the range which carry higher profit margins.

Case Study
Printers for PCs – The printers are priced very attractive and low prices, but the ink
cartridges are comparatively highly priced. Once you have bought the printer, the market for
the ink cartridges is almost secure.

Using discounts
Reasons for using discounts to adjust prices
−
−
−
−
−
−

To get rid of perishable goods that have reached the end of their shelf life
To sell off seconds – cheaper than disposal and does help towards contribution
Normal practice (e.g.2nd hand and antiques trade)
To increase sales volumes during a poor sales period without dropping prices permanently
To differentiate between types of customer (wholesale, retail and so on)
To get cash in quickly

Page 112

Controlled prices
State owned or nationalised industries operate in a monopolistic environment. Their prices
are regulated by the government or by a designated ministry. Over the next few years it is
expected that shares in some will be offered to the public and they then will operate within
the private sector. However they will probably be overseen by an industry regulator.
Regulators tend to concentrate on price so that these near monopolies cannot exploit their
position (although the regulators are also concerned with quality of service/product and
capital investment).
If a price is regulated, the elasticity of demand is zero: 'small' customers pay less than they
otherwise would, whereas 'large' customers pay more than in a competitive environment.
In general though, prices have become more flexible in recent year through:
a) Introduction of discounted prices for very large customers
b)Entry of other companies into the market

If asked to compare two pricing strategies and to determine which is the better, you
basically need to consider which produces the higher cash inflows.
When asked to assess the financial viability of the better strategy, however, you need to
perform a DCF appraisal on the forecast cash flows.

Page 113

CHAPTER ROUNDUP
• In full cost-plus pricing the sales price is determined by calculating the full cost of the
product and then adding a percentage mark-up for profit. The most important
criticism of full cost-plus pricing is that it fails to recognise, that since sales demand
may be determined by the sales price, there will be a profit-maximising combination
of price and demand.
• Marginal cost-plus pricing involves adding a profit margin to the marginal cost of
production/sales. A marginal costing approach is more likely to help with identifying
a profit-maximising price.
• Economic theory claims that profit is maximised by setting a price so that marginal
cost equals marginal revenue. But, because most cost accounting systems are set up
to provide information for financial reporting purposes, it can be difficult to identify
short-run or long-run marginal cost, even if ABC is used.
• Another approach to pricing might be taken when a business is working at full
capacity, and is restricted by a shortage of resources from expanding its output
further. By deciding what target profit it would like to earn, it could establish a
mark-up per unit of limiting factor.
• The basic approach to pricing special orders is minimum pricing.
• Two pricing strategies for new products are market penetration pricing and market
skimming pricing.
• Product differentiation may be used to make products appear to be different. Price
discrimination is then possible.

Page 114

STUDY UNIT 4
Target Costing
Contents

Page

What Is Target Costing? …………………………………………………….

117

Implementing Target Costing ……….………………………………..……..

119

Deriving A Target Cost …………………………………………………..….

121

Implications Of Using Target Costing …………..…………………………

123

Closing A Target Cost Gap …………………………………………………

125

Target Costing In Service Industries ………………………………………

127

Page 115

EXAM GUIDE
Target costing may form part of a question comparing its use to other costing techniques or it
may form an entire question including calculation of a target cost.

Page 116

WHAT IS TARGET COSTING?
Target costing involves setting a target cost by subtracting a desired profit margin from a
competitive market price.

To compete effectively, organisations must continually redesign their products (or services)
in order to shorten product life cycles (see Chapter 5). The planning, development and
design stage of a product is therefore critical to an organisation's cost management process.
Considering possible cost reductions at this stage of a product's life cycle (rather than
during the production process) is now one of the most important issues facing management
accountants in industry.
Here are some examples of decisions made at the design stage which impact on the cost of
a product.
− The number of different components
− Whether the components are standard or not
− The ease of changing over tools

Japanese companies have developed target costing as a response to the problem of
controlling and reducing costs over the product life cycle.

Target costing involves setting a target cost by subtracting a desired profit margin from a
competitive market price.
Target cost is an estimate of a product cost which is determined by subtracting a desired
profit margin from a competitive market price. This target cost may be less than the planned
initial product cost but it is expected to be achieved by the time the product reaches the
maturity stage of the product life cycle.

Page 117

BLANK

Page 118

IMPLEMENTING TARGET COSTING
In 'Product costing/pricing strategy' (ACCA Students Newsletter, August 1999),one of the
examiners provided a useful summary of the steps in the implementation of the target costing
process.

Step 1

Determine a product specification of which an adequate sales volume is
estimated.

Step 2

Set a selling price at which the organisation will be able to achieve a desired
market share.

Step 3

Estimate the required profit based on return on sales or return on investment.

Step 4

Calculate the target cost = target selling price – target profit.

Step 5

Compile an estimated cost for the product based on the anticipated design
specification and current cost levels.

Step 6

Calculate target cost gap = estimated cost – target cost.

Step 7

Make efforts to close the gap – assuming estimated cost is greater than target cost
and it usually is!. This is more likely to be successful if efforts are
made
to
'design out' costs prior to production, rather than to 'control out' costs during the
production phase.

Step 8

Negotiate with the customer before making the decision about whether to go
ahead with the project.

Case Study
The following comments appeared in an article in the Financial Times. (Emphasis is ours.)
'Mercedes-Benz, one of the world's most prestigious and tradition-laden carmakers, has taken
its time to wake up to the daunting dimensions of the challenges it faces in the rapidlychanging world car market of the 1990s.
Page 119

The company has accepted that radical changes in the world car market mean that MercedesBenz will no longer be able to demand premium prices for its products based on an image of
effortless superiority and a content of the ultimate in automotive engineering.
Instead of developing the ultimate car and then charging a correspondingly sky-high price as
in the past, Mercedes-Benz is taking the dramatic and radical step of moving to 'target
pricing'. It will decide what the customer is willing to pay in a particular product category
– priced against its competitors – it will add its profit margin and then the real work will
begin to cost every part and component to bring in the vehicle at the target price.
The following extracts are from an article which appeared three months later.
'The marketing motto for the Mercedes-Benz compact C-class is that it offers customers more
car for their money.
It is the first practical example of the group's new pricing policy. The range embodies a
principle new to Mercedes which states that before any work starts a new product will be
priced according to what the market will bear and what the company considers an
acceptable profit. Then each component and manufacturing process will be costed to
ensure the final product is delivered at the target price.
Under the old system of building the car, adding up the costs and then fixing a price, the Cclass would have been between 15 per cent and 20 per cent dearer than the 10-year-old
outgoing 190 series, Mr Vöhringer said.
Explaining the practical workings of the new system, he explained that project groups for
each component and construction process were instructed without exception to increase
productivity by between 15 and 25 per cent. And they had to reach their targets in record
time.
One result was that development time on the new models was cut to 40 months, about a third
less than usual. But the most important effect, according to Mr Vöhringer, has been to reduce
the company's cost disadvantages vis-à-vis Japanese competitors in this class from 35
per cent to only 15 per cent.'

Page 120

DERIVING A TARGET COST
The target cost is calculated by starting with a market-based price and subtracting a desired
profit margin. The target cost is simply the price minus the profit.

Example: target costing
A car manufacturer wants to calculate a target cost for a new car, the price of which will be
set at RWF17,950,000. The company requires an 8% profit margin.

Required
What is the target cost?

Solution
Profit required
Target cost

= 8% × RWF17,950,000
= RWFk(17,950 – 1,436)

= RWF1,436,000
= RWF16,514,000

The car manufacturer will then need carefully to compile an estimated cost for the new car.
ABC will help to ensure that costs allocated to the new model are more accurate.

Page 121

BLANK

Page 122

IMPLICATIONS OF USING TARGET COSTING
Target costing requires managers to change the way they think about the relationship
between cost, price and profit.
a) Traditionally the approach is to develop a product, determine the production cost of
that product, set a selling price, with a resulting profit or loss.
b) The target costing approach is to develop a product, determine the market selling
price and desired profit margin, with a resulting cost which must be achieved.

With target costing there is a focus on:
a) Price-led costing.
b) Customers. Customer requirements for quality, cost and time are incorporated into
product and process decisions. The value of product features to the customers must be
greater than the cost of providing them.
c) Design. Cost control is emphasised at the design stage so any engineering changes
must happen before production starts.
d) Faster time to market. The early external focus enables the business to get the
process right first time and avoids the need to go back and change aspects of the
design and/or production process. This then reduces the time taken to get a product to
the market.
e) With Target Costing, it is too easy to design down to ensure the target cost is met and
the manufacturer could be left with an inferior product

Page 123

BLANK

Page 124

CLOSING A TARGET COST GAP
The target cost gap is the estimated cost less the target cost. When a product is first
manufactured, its target cost may well be much lower than its currently-attainable cost, which
is determined by current technology and processes. Management can then set benchmarks
for improvement towards the target costs, by improving technologies and processes. Various
techniques can be employed.

−
−
−
−
−
−
−

Reducing the number of components
Using cheaper staff
Using standard components wherever possible
Acquiring new, more efficient technology
Training staff in more efficient techniques
Cutting out non-value-added activities
Using different materials (identified using activity analysis etc)

Even if the product can be produced within the target cost the story does not end there. Target
costing can be applied throughout the entire life cycle. Once the product goes into production
target costs will therefore gradually be reduced. These reductions will be incorporated into
the budgeting process. This means that cost savings must be actively sought and made
continuously over the life of the product.

When answering a question on closing a target cost gap, make sure you refer to the specific
circumstances of the business in the question.

Page 125

BLANK

Page 126

TARGET COSTING IN SERVICE INDUSTRIES
Target costing is difficult to use in service industries due to the characteristics and
information requirements of service businesses.

Characteristics of services
Unlike manufacturing companies, services are characterised by intangibility, inseparability,
variability, perishability and no transfer of ownership.

Examples of service businesses include:
a) Mass service eg the banking sector, transportation (rail, air), mass entertainment
b) Either / or eg fast food, teaching, hotels and holidays, psychotherapy
c) Personal service eg pensions and financial advice, car maintenance
d)
Services are any activity of benefit that one party can offer to another which is essentially
intangible and does not result in the ownership of anything. Its production may or may not be
tied to a physical product.'
(P Kotler, Social Marketing)

There are five major characteristics of services that distinguish them from manufacturing.
a) Intangibility refers to the lack of substance which is involved with service delivery.
Unlike goods (physical products such as confectionery, books or even a CD with a
software programme), there is no substantial material or physical aspects to a service:
no taste, feel, visible presence and so on. For example, if you go to the theatre, you
cannot take the 'play' with you when you leave.
b) Inseparability/simultaneity. Many services are created at the same time as they are
consumed. (Think of dental treatment.) No service exists until it is actually being
experienced/consumed by the person who is buying it.
Page 127

c) Variability/heterogeneity. Many services face the problem of maintaining
consistency in the standard of output. It may be hard to attain precise standardisation
of the service offered, but customers expect it (such as with fast food).
d) Perishability. Services are innately perishable. The services of a beautician are
purchased for a period of time. An audit may last 2 weeks and after that it is no more.
e) No transfer of ownership. Services do not result in the transfer of property. The
purchase of a service only confers on the customer access to or a right to use a
facility.

Information requirements of services
Service businesses need the same aggregate information as manufacturing firms, but also
need performance data as to their cost and volume drivers. Operational information is likely
to be more qualitative.
A service business needs a mix of quantitative and non-quantitative information to price its
services properly, to optimise capacity utilisation and to monitor performance.
a) They need to control the total cost of providing the service operation.
b)They need positive cash flow to finance activities.
c) They need operating information to identify how costs are incurred and on what
services.

Arguably, small service businesses, whose expenses are mainly overheads, provide a model,
in miniature, of the requirements of activity based costing.
Are 'mass services' any different?
a) Because mass services, such as cheque clearing, are largely automated, there may be a
large fixed cost base.
b) Even if a service is heavily automated, each time the service is performed is a 'moment
of truth' for the customer. Ensuring consistency and quality is important but this is
true for small service businesses too.

Page 128

Service industries, perhaps more than manufacturing firms, rely on their staff. Front-line
staff are those who convey the 'service' – and the experience of the brand – to the consumer.
For service businesses, management accounting information should incorporate the key
drivers of service costs.
−
−
−
−
−

Repeat business
Opportunity costs of not providing a service
Churn rate (for subscriptions)*
Avoidable / unavoidable costs
Customer satisfaction surveys, complaints

* For any given period of time, the number of participants who discontinue their use of a
service divided by the average number of total participants is the churn rate. Churn rate
provides insight into the growth or decline of the subscriber base as well as the average
length of participation in the service.

Page 129

CHAPTER ROUNDUP
• Target costing involves setting a target cost by subtracting a desired profit margin
from a competitive market price.
• Unlike manufacturing companies, services are characterised by intangibility,
inseparability, variability, perishability and no transfer of ownership.
• Service businesses need the same aggregate information as manufacturing firms, but
also need performance data as to their cost and volume drivers. Operational
information is likely to be more qualitative.

Page 130

STUDY UNIT 5
Lifecycle Costing
Contents

Page

What Are Life Cycle Costs? …………………………………….………….

133

The Product Life Cycle ……….……………………………………..……..

135

The Implications Of Life Cycle Costing ………………………………….

137

Page 131

EXAM GUIDE
Life cycle costing will probably form part of a question on costing techniques but it has equal
weighting in the syllabus as the other management accounting techniques, so could form an
entire question.

Page 132

WHAT ARE LIFE CYCLE COSTS?
Life cycle costing tracks and accumulates costs and revenues attributable to each product
over the entire product life cycle.

A product's life cycle costs are incurred from its design stage through development to
market launch, production and sales, and finally to its eventual withdrawal from the
market. The component elements of a product's cost over its life cycle could therefore
include the following.
•

Research & development costs
–
–
–

Design
Testing
Production process and equipment

•

The cost of purchasing any technical data required

•

Training costs (including initial operator training and skills updating)

•

Production costs

•

Distribution costs. Transportation and handling costs

•

Marketing costs
–
–
–

Customer service
Field maintenance
Brand promotion

•

Inventory costs (holding spare parts, warehousing and so on)

•

Retirement and disposal costs. Costs occurring at the end of a product's life

Life cycle costs can apply to services, customers and projects as well as to physical products.
Traditional cost accumulation systems are based on the financial accounting year and tend
to dissect a product's life cycle into a series of 12-month periods. This means that traditional
management accounting systems do not accumulate costs over a product's entire life cycle
and do not therefore assess a product's profitability over its entire life. Instead they do it
on a periodic basis.

Page 133

Life cycle costing, on the other hand, tracks and accumulates actual costs and revenues
attributable to each product over the entire product life cycle. Hence the total profitability
of any given product can be determined.

Life cycle costing is the accumulation of costs over a product's entire life.

Page 134

THE PRODUCT LIFE CYCLE
A product life cycle can be divided into five phases.
•

Development

•

Maturity

•

Introduction

•

Decline

•

Growth

Every product goes through a life cycle.
a) Development. The product has a research and development stage where costs are
incurred but no revenue is generated.
b) Introduction. The product is introduced to the market. Without advertising or some
similar means, the potential customers will be unaware of the product or service –
more cost without revenue.
c) Growth. The product gains a bigger market as demand builds up. Sales revenues
increase and the product begins to make a profit.
d) Maturity. Eventually, the growth in demand for the product will slow down and it will
enter a period of relative maturity. It will continue to be profitable. The product may
be modified or improved, as a means of sustaining its demand.
e) Decline. At some stage, the market will have bought enough of the product and it will
therefore reach 'saturation point'. Demand will start to fall. Eventually it will become
a loss-maker and this is the time when the organisation should decide to stop selling
the product or service.

The level of sales and profits earned over a life cycle can be illustrated diagrammatically as
follows.

Page 135

The horizontal axis measures the duration of the life cycle, which can last from, say, 18
months to several hundred years. Children's crazes or fad products have very short lives
while some products, such as binoculars (invented in the eighteenth century) can last a very
long time.

Problems with traditional cost accumulation systems
Traditional cost accumulation systems do not tend to relate research and development costs
to the products that caused them. Instead they write off these costs on an annual basis
against the revenue generated by existing products. This makes the existing products seem
less profitable than they really are. If research and development costs are not related to the
causal product the true profitability of that product cannot be assessed.
Traditional cost accumulation systems usually total all non-production costs and record
them as a period expense.

Page 136

THE IMPLICATIONS OF LIFE CYCLE COSTING
Life cycle costing has implications on pricing, performance management and decisionmaking.
With life cycle costing, non-production costs are traced to individual products over complete
life cycles.
a) The total of these costs for each individual product can therefore be reported and
compared with revenues generated in the future.
b) The visibility of such costs is increased.
c) Individual product profitability can be better understood by attributing all costs to
products.
d) As a consequence, more accurate feedback information is available on the
organisation's success or failure in developing new products. In today's competitive
environment, where the ability to produce new or updated versions of products is
paramount to the survival of an organisation, this information is vital.

The importance of the early stages of the life cycle
It is reported that some organisations operating within an advanced manufacturing
technology environment find that approximately 90% of a product's life cycle cost is
determined by decisions made early within the cycle - at the design stage. Life cycle costing
is therefore particularly suited to such organisations and products, monitoring spending and
commitments to spend during the early stages of a product's life cycle.
In order to compete effectively in today's market, organisations need to redesign continually
their products with the result that product life cycles have become much shorter. The
planning, design and development stages of a product's cycle are therefore critical to an
organisation's cost management process. Cost reduction at this stage of a product's life cycle,
rather than during the production process, is one of the most important ways of reducing
product cost.

Page 137

Maximising the return over the product life cycle
Design costs out of products
Between 70% to 90% of a product's life cycle costs are determined by decisions made early
in the life cycle, at the design or development stage. Careful design of the product and
manufacturing and other processes will keep cost to a minimum over the life cycle.

Minimise the time to market
This is the time from the conception of the product to its launch. More products come onto
the market nowadays and development times have been reduced over the years. Competitors
watch each other very carefully to determine what types of product their rivals are
developing. If an organisation is launching a new product it is vital to get it to the market
place as soon as possible. This will give the product as long a period as possible without a
rival in the market place and should mean increased market share in the long run.
Furthermore, the life span may not proportionally lengthen if the product's launch is delayed
and so sales may be permanently lost. It is not unusual for the product's overall profitability
to fall by 25% if the launch is delayed by six months. This means that it is usually worthwhile
incurring extra costs to keep the launch on schedule or even to speed up the launch.

Minimise breakeven time (BET)
A short BET is very important in keeping an organisation liquid. The sooner the product is
launched the quicker the research and development costs will be repaid, providing the
organisation with funds to develop further products.

Maximise the length of the life span
Product life cycles are not predetermined; they are set by the actions of management and
competitors. Once developed, some products lend themselves to a number of different uses;
this is especially true of materials, such as plastic, PVC, nylon and other synthetic materials.
The life cycle of the material is then a series of individual product curves nesting on top of
each other as shown below.

Page 138

By entering different national or regional markets one after another an organisation may be
able to maximise revenue. This allows resources to be better applied, and sales in each
market to be maximised. On the other hand, in today's fast moving world, an organisation
could lose out to a competitor if it failed to establish an early presence in a particular market.

Minimise product proliferation
If products are updated or superseded too quickly, the life cycle is cut short and the product
may just cover its R and D costs before its successor is launched.

Manage the product's cashflows
Hewlett-Packard developed a return map to manage the lifecycle of their products. Here is
an example.

Page 139

Key time periods are measured by the map:
a) Time to market
b) Breakeven time
c) Breakeven time after product launch
d) Return factor (the excess of profit over the investment)

Changes to planned time periods can be incorporated into the map (for example, if the
development plan takes longer than expected) and the resulting changes to the return factor at
set points after release highlighted.

Page 140

Service and project life cycles
A service organisation will have services that have life cycles. The only difference is that the
R & D stages will not exist in the same way and will not have the same impact on
subsequent costs. The different processes that go to form the complete service are important,
however, and consideration should be given in advance as to how to carry them out and
arrange them so as to minimise cost.
Products that take years to produce or come to fruition are usually called projects, and
discounted cash flow calculations are invariably used to cost them over their life cycle in
advance. The projects need to be monitored very carefully over their life to make sure that
they remain on schedule and that cost overruns are not being incurred.

Customer life cycles
Customers also have life cycles, and an organisation will wish to maximise the return from
a customer over their life cycle. The aim is to extend the life cycle of a particular
customer or decrease the 'churn' rate, as the Americans say. This means encouraging
customer loyalty. For example, Europe some large chain retail outlets issue loyalty cards
that offer discounts to loyal customers who return to the shop and spend a certain amount
with the organisation. As existing customers tend to be more profitable than new ones they
should be retained wherever possible.
Customers become more profitable over their life cycle. The profit can go on increasing
for a period of between approximately four and 20 years. For example, if you open a bank
account, take out insurance or invest in a pension, the company involved has to set up the
account, run checks and so on. The initial cost is high and the company will be keen to retain
your business so that it can recoup this cost. Once customers get used to their supplier they
tend to use them more frequently, and so there is a double benefit in holding on to customers.
For example, you may use the bank to purchase shares on your behalf, or you may take out a
second insurance policy with the same company.
The projected cash flows over the full lives of customers or customer segments can be
analysed to highlight the worth of customers and the importance of customer retention. It
may take a year or more to recoup the initial costs of winning a customer, and this could be
referred to as the payback period of the investment in the customer.
Page 141

Example
Solaris specialises in the manufacture of solar panels. It is planning to introduce a new
slimline solar panel specially designed for small houses. Development of the new panel is to
begin shortly and Solaris is in the process of determining the price of the panel. It expects the
new product to have the following costs.

Year 1
Units manufactured and sold

R&D costs
Marketing costs
Production cost per unit
Customer service costs per unit

Year 2

Year 3

Year 4

2,000

15,000

20,000

5,000

RWF ‘000

RWF ‘000

RWF ‘000

RWF‘000

1,900,000

100,000

-

-

100,000

75,000

50,000

10,000

500

450

400

450

50

40

40

40

Disposal of specialist equipment

300,000

The Marketing Director believes that customers will be prepared to pay RWF500,000 for a
solar panel but the Financial Director believes this will not cover all of the costs throughout
the lifecycle.

Required
Calculate the cost per unit looking at the whole life cycle and comment on the suggested
price.

Page 142

Answer

Lifecycle costs
RWFmillions
R&D (1,900 + 100)

2,000

Marketing (100 + 75 + 50 + 10)

235

Production (1,000 + 6,750 + 8,000 + 2,250)

18,000

Customer service (100 + 600 + 800 + 200)

1,700

Disposal

300

Total lifecycle costs RWF m

22,235

Total production ('000 units)

42

Cost per unit RWF‘000

529.40

The total lifecycle costs are RWF 529,400 per solar panel which is higher than the price
proposed by the marketing director. Solaris will either have to charge a higher price or look at
ways to reduce costs.

Page 143

CHAPTER ROUNDUP
• Life cycle costing tracks and accumulates costs and revenues attributable to each
product over the entire product life cycle.
• A product life cycle can be divided into five phases.
−
−
−
−
−

Development
Introduction
Growth
Maturity
Decline

• Life cycle costing has implications on pricing, performance management and decisionmaking.

Page 144

STUDY UNIT 6
Backflush Accounting
Contents

Page

Costing Systems And Manufacturing Philosophy ……………………….

147

Backflush Costing ……….…………………………………………..……..

151

Page 145

EXAM GUIDE
You will need to understand how modern manufacturing techniques have led to new
management accounting techniques. This topic could easily form an entire 25 mark question
but you will not be required to produce the double entry accounts.

Page 146

COSTING SYSTEMS AND MANUFACTURING
PHILOSOPHY
Costing systems have evolved to reflect a manufacturing philosophy that is based on the
need to achieve competitive advantage.
•
•
•

Flexibility and the ability to respond quickly to customer demands are vital.
Product life cycles are shorter and products must be brought to the market quickly.
New technology has been introduced.

Costing systems
a) Designed to complement the organisation's operations flow
b) Should reflect management philosophy and style
c) Provide information which management can use to plan and control operations on a
daily, monthly and longer-term basis
d) Changes in manufacturing philosophy and new technology (CAM (computer aided
manufacturing) and FMS (flexible manufacturing system)) require changes in
information and cost reporting systems
− Collecting information in a different way
− Rethinking what data need to be collected
− Rethinking what information should be reported

e) New Systems
− Unit quantities (rather than monthly monetary values) reported to production
employees
− Performance measures based on output (rather than hours worked) reported to
management

Page 147

f) Activities important to the organisation's success should determine the information
required. These might include:
−
−
−
−

Accurate product costing
Information to control costs
Knowledge of customer costs
Cost reduction

Traditional manufacturing philosophy
a) Labour and manufacturing equipment are so valuable they should not be left idle.
b) Resulting raw materials and stock not needed should be stored (thus hiding inefficient
and uneven production methods).
c) To increase efficiency and reduce production cost per unit, batch sizes and production
runs should be as large as possible.
d) Concerned with balancing production run costs and inventory holding costs.

Modern manufacturing philosophy
a) Smooth, steady production flow (throughput)
b) Flexibility, providing the customer with exactly what is wanted, exactly when it is
wanted (making the organisation a more complex affair to manage), so as to achieve
competitive advantage.
c) Volume versus variety
d) JIT

Page 148

Just-in-time (JIT)
Just-in-time is an approach to operations planning and control based on the idea that goods
and services should be produced only when they are needed, and neither too early (so that
inventories build up) nor too late (so that the customer has to wait).

Just-in-time is a system whose objective is to produce or procure products or components as
they are required rather than for inventory.

In traditional manufacturing, where there is a production process with several stages,
management seek to insulate each stage in the process from disruption by another stage, by
means of producing for, and holding, inventory.
For example, suppose a manufacturing process consists of four consecutive stages. In a
traditional manufacturing system, there would be inventories of raw materials and finished
goods, and also inventories of part-finished items between stage 1 and stage 2, between stage
2 and stage 3 and between stage 3 and stage 4. If there is disruption to production at, say,
stage 2, the other stages would not be immediately affected. Stages 3 and 4 would continue to
operate, using the inventories of part-finished items from stages 2 and 3. Stage 1 would also
continue to operate, producing inventory for stage 2. The responsibility for resolving the
disruption would fall mainly on the managers of the stage affected, which in this example
would be the management of stage 2.
In contrast, in its extreme form, a JIT system seeks to hold zero inventories. In the same fourstage process described above, a disruption at any stage would immediately have an impact
on all the other stages. For example, if a disruption occurs at stage 2, stages 3 and 4 will have
to stop working because they have no output from stage 2. Stage 1 will also have to stop
working, because it will only produce when stage 2 is ready to receive and use its output.
With JIT, a disruption at any point in the system becomes a problem for the whole
operation to resolve. Supporters of JIT management argue that this will improve the
likelihood of the problem being resolved, because it is in the interests of everyone to resolve
it. They also argue that inventories help to hide problems within the system, so that problems
go unnoticed for too long.
JIT can be regarded as an approach to management that encompasses a commitment to
continuous improvement and the search for excellence in the design and operation of the
Page 149

production management system. Its aim is to streamline the flow of products through the
production process and into the hands of customers.

Page 150

BACKFLUSH COSTING
Backflush costing is suitable for use in a JIT environment. Costs are attached to output only,
thereby simplifying the costing system.
Backflush accounting is the name given to the method of keeping cost accounts employed if
backflush costing is used. The two terms are almost interchangeable.

Traditional costing systems v backflush costing
Traditional costing systems use sequential tracking (also known as synchronous
tracking) to track costs sequentially as products pass from raw materials to work in progress,
to finished goods and finally to sales. In other words, material costs are charged to WIP when
materials are issued to production, direct labour and overhead costs are charged in a similar
way as the cost is incurred or very soon after.
If a production system such as JIT is used, sequentially tracking means that all entries are
made at almost the same moment and so a different accounting system can be used. In
backflush accounting, costs are calculated and charged when the product is sold, or
when it is transferred to the finished goods store.

Backflush costing is 'a more simplified costing system for allocating costs between stocks
and cost of goods sold…... The purpose of this is to eliminate detailed accounting
transactions. Rather than tracking the movement of materials through the production process,
a backflush costing system focuses first on the output of the organisation and then works
backwards when allocating cost between costs of goods sold and stocks, with no separate
accounting for WIP.'
(Drury)

Page 151

Backflush costing and standard costs
Budgeted or standard costs are used to work backwards to 'flush' out manufacturing costs
for the units produced. (Hence the rather unattractive name for the system!) The application
of standard costs to finished goods units, or to units sold, is used in order to calculate cost of
goods sold, thereby simplifying the costing system and creating savings in administrative
effort. In a true backflush accounting system, records of materials used and work in progress
are not required as material cost can be calculated from either finished goods or goods sold.

When backflush costing is appropriate
Backflush costing is appropriate for organisations trying to keep inventories to the very
minimum. In such circumstances, the recording of every little increase in inventory value, as
each nut and bolt is added, is simply an expensive and non-value-added activity that should
be eliminated.

Example: Backflush accounting
A company operates a backflush costing system.
The standard cost of product X is:
Materials
Conversion

RWF ‘000
6
8
14

Details of transactions in the month were:
Rwf ‘000
2,000
10,000
15,000
20,500

Raw materials b/f
Purchases
Conversion
Cost of goods sold

Page 152

Required
What is the closing balance on raw materials account?

Solution
RAW MATERIAL
Rwf ‘000

RWF ‘000
B/f
Cash

2,000

(W1)

8,200

10,000

C/f

3,800

12,000

12,000

CONVERSION
RWF ‘000
15,000

RWF ‘000
(W1)

12,300

COST OF GOODS SOLD
RWF ‘000

RWF ‘000

20,500

Working 1
Raw materials

10/25 × 20,500

=

8,200

Conversion

15/25 × 20,500

=

12,300

Page 153

You will not be required to produce double entry accounts in your exam. The emphasis will
be on explanation and discussion.

Possible problems with backflush costing
The successful operation of backflush costing rests upon predictable levels of efficiency and
stable material prices and usage. In other words there should be insignificant cost
variances.
a) It is only appropriate for JIT operations where production and sales volumes are
approximately equal.
b) Some people claim that it should not be used for external reporting purposes. If,
however, inventories are low or are practically unchanged from one accounting
period to the next, operating income and inventory valuations derived from backflush
accounting will not be materially different from the results using conventional
systems. Hence, in such circumstances, backflush accounting is acceptable for
external financial reporting.
c) It is vital that adequate production controls exist so that cost control during the
production process is maintained.

Advantages of backflush costing
a) It is much simpler, as there is no separate accounting for WIP.
b) The number of accounting entries should be greatly reduced, as are the supporting
vouchers, documents and so on.
c) The system should discourage managers from producing simply for inventory since
working on material does not add value until the final product is completed or sold.

Backflush costing and modern database/warehouse accounting
a) With modern barcode readers and ordering systems, stock control and accounting can
be built into modern ERP (enterprise resource planning) and accounting database
systems such as SAP or Oracle. From the authorisation of the initial request/order, no
further special voucher entries are required.

Page 154

b) Stock/WIP/FG are automatically taken care of in this type of accounting and so there is
no saving here.
c) The number of accounting entries are not affected.
d) BUT the system can be instructed to produce “backflush costing” reports to discourage
managers from producing simply for inventory since working on material does not
add value until the final product is completed or sold.

Page 155

CHAPTER ROUNDUP
• Costing systems have evolved to reflect a manufacturing philosophy that is based on
the need to achieve competitive advantage.
− Flexibility and the ability to respond quickly to customer demands are vital.
− Product life cycles are shorter and products must be brought to the market quickly.
− New technology has been introduced.
• Just-in-time is an approach to operations planning and control based on the idea that
goods and services should be produced only when they are needed, and neither too
early (so that inventories build up) nor too late (so that the customer has to wait).
• Backflush costing is suitable for use in a JIT environment. Costs are attached to output
only, thereby simplifying the costing system.

Page 156

STUDY UNIT 7
Throughput Accounting
Contents

Page

Theory Of Constraints ……………………………………………………….

159

Throughput Accounting ……….……………………………………...……..

161

Performance Measures In Throughput Accounting …………...………….

165

Throughput And Decision Making …………………………………………

167

Page 157

EXAM GUIDE
Questions on this topic are likely to be a mixture of calculation and discussion. You may be
required to use your knowledge of limiting factors from previous studies.

Page 158

THEORY OF CONSTRAINTS
Throughput accounting is a product management system which aims to maximise
throughput, and therefore cash generation from sales, rather than profit. A just in time (JIT)
environment is operated, with buffer inventory kept only when there is a bottleneck.
The Theory of Constraints (TOC) is an approach to production management. Its key
financial concept is to turn materials into sales as quickly as possible, thereby maximising the
net cash generated from sales. This is achieved by striving for balance in production
processes, and so evenness of production flow is also an important aim.

Theory of constraints (TOC) is an approach to production management which aims to
maximise sales revenue less material and variable overhead cost. It focuses on factors such as
bottlenecks which act as constraints to this maximisation.
Bottleneck or binding constraint – an activity which has a lower capacity than preceding or
subsequent activities, thereby limiting throughput.

One process will inevitably act as a bottleneck (or limiting factor) and constrain throughput –
this is known as the binding constraint in TOC terminology. Steps should be taken to
remove this by buying more equipment, improving production flow and so on. But ultimately
there will always be a binding constraint, unless capacity is far greater than sales demand or
all processes are totally in balance, which is unlikely.
Output through the binding constraint should never be delayed or held up otherwise sales will
be lost. To avoid this happening a buffer inventory should be built up immediately prior to
the bottleneck or binding constraint. This is the only inventory that the business should hold,
with the exception of possibly a very small amount of finished goods inventory and raw
materials that are consistent with the JIT approach.
Operations prior to the binding constraint should operate at the same speed as the binding
constraint, otherwise work in progress (other than the buffer inventory) will be built up.
According to TOC, inventory costs money in terms of storage space and interest costs, and so
inventory is not desirable.

Page 159

The overall aim of TOC is to maximise throughput contribution (sales revenue – material
cost) while keeping conversion cost (all operating costs except material costs) and
investment costs (inventory, equipment and so on) to the minimum. A strategy for
increasing throughput contribution will only be accepted if conversion and investment costs
increase by a lower amount than the increase in contribution.

Page 160

THROUGHPUT ACCOUNTING
The concept of throughput accounting has been developed from TOC as an alternative system
of cost and management accounting in a JIT environment.

Throughput Accounting (TA) is an approach to accounting which is largely in sympathy
with the JIT philosophy. In essence, TA assumes that a manager has a given set of resources
available. These comprise existing buildings, capital equipment and labour force. Using these
resources, purchased materials and parts must be processed to generate sales revenue. Given
this scenario the most appropriate financial objective to set for doing this is the maximisation
of throughput (Goldratt and Cox, 1984) which is defined as: sales revenue less direct material
cost.
(Tanaka, Yoshikawa, Innes and Mitchell, Contemporary Cost Management)

TA for JIT is said to be based on three concepts.
a) Concept 1
In the short run, most costs in the factory (with the exception of materials costs) are
fixed (the opposite of ABC, which assumes that all costs are variable). These fixed
costs include direct labour. It is useful to group all these costs together and call them
Total Factory Costs (TFC).
b) Concept 2
In a JIT environment, all stock is a 'bad thing' and the ideal stock level is zero.
Products should not be made unless a customer has ordered them. When goods are
made, the factory effectively operates at the rate of the slowest process, and there will
be unavoidable idle capacity in other operations.
Work in progress should be valued at material cost only until the output is
eventually sold, so that no value will be added and no profit earned until the sale takes
place. Working on output just to add to work in progress or finished goods inventory
creates no profit, and so should not be encouraged.

Page 161

c) Concept 3
Profitability is determined by the rate at which 'money comes in at the door' (that is,
sales are made) and, in a JIT environment, this depends on how quickly goods can be
produced to satisfy customer orders. Since the goal of a profit-oriented organisation is
to make money, inventory must be sold for that goal to be achieved. The bottleneck
slows the process of making money.

Bottleneck.
The aim of modern manufacturing approaches is to match production resources with the
demand for them. This implies that there are no constraints, termed binding constraint in
TA, within an organisation. The throughput philosophy entails the identification and
elimination of these bottlenecks by overtime, product changes and process alterations to
reduce set-up and waiting times.
Where throughput cannot be eliminated by say prioritising work, and to avoid the build-up of
work in progress, production must be limited to the capacity of the bottleneck but this
capacity must be fully utilised. If a rearrangement of existing resources or buying-in
resources does not alleviate the bottleneck, investment in new equipment may be necessary.
The elimination of one bottleneck is likely to lead to the creation of another at a previously
satisfactory location, however. The management of bottlenecks therefore becomes a primary
concern of the manager seeking to increase throughput.
There are other factors which might limit throughput other than a lack of production
resources (bottlenecks) and these need to be addressed as well.
a) The existence of an uncompetitive selling price
b)The need to deliver on time to particular customers
c) The lack of product quality and reliability
d)The lack of reliable material suppliers
e) The shortage of production resources

Page 162

Is it good or bad?
TA is seen by some as too short term, as all costs other than direct material are regarded as
fixed. Moreover, it concentrates on direct material costs and does nothing for the control of
other costs such as overheads. These characteristics make throughput accounting a good
complement for ABC, however, since aspects of ABC focus on labour and overhead costs.
TA attempts to maximise throughput whereas traditional systems attempt to maximise profit.
By attempting to maximise throughput, an organisation could be producing in excess of the
profit-maximising output. Production scheduling problems inevitably mean that the
maximising of throughput is never attained, however, and so a throughput maximising
approach could well lead to the profit-maximising output being achieved.
TA helps to direct attention to bottlenecks and focus management on the key elements in
making profits, inventory reduction and reducing the response time to customer demand.

Page 163

BLANK

Page 164

PERFORMANCE MEASURES IN THROUGHPUT
ACCOUNTING
Performance measures in throughput accounting are based around the concept that only
direct materials are regarded as variable costs.

a) Return per factory hour
Sales−direct materialcosts
Usageof bottleneckin hours(factoryhours)

This enables businesses to take short-term decisions when a resource is in scarce supply.

b) Throughput accounting ratio
Return per factory hour
Total conversioncost per factory hour

Again factory hours are measured in terms of use of the bottleneck. Businesses should
try to maximise the throughput accounting ratio by making process improvements or
product specification changes.
This measure has the advantage of including the costs involved in running the factory.
The higher the ratio, the more profitable the company. (If a product has a ratio of less
than one, the organisation loses money every time the product is made.)

Page 165

Here's an example.
Product A

Product B

RWF ‘000 per factory hour

RWF ‘000 per factory hour

Sales

100

150

Material cost

(40)

(50)

Conversion cost

(50)

(50)

10

50

Profit
TA ratio

60
50

= 1.2

100
50

= 2.0

Profit will be maximised by manufacturing as much of product B as possible.

Page 166

THROUGHPUT AND DECISION MAKING
In a throughput environment, production priority must be given to the products best able to
generate throughput, that is those products that maximise throughput per unit of
bottleneck.
The TA ratio can be used to assess the relative earning capabilities of different products and
hence can help with decision making.

Example: throughput accounting
Corrie produces three products, X, Y and Z. The capacity of Corrie's plant is restricted by
process alpha. Process alpha is expected to be operational for eight hours per day and can
produce 1,200 units of X per hour, 1,500 units of Y per hour, and 600 units of Z per hour.
Selling prices and material costs for each product are as follows.

Product

Selling price

Material cost

Throughput contribution

per unit

per unit

per unit

X

150

80

70

Y

120

40

80

Z

300

100

200

Conversion costs are RWF720,000 per day.

Page 167

Required
a) Calculate the profit per day if daily output achieved is 6,000 units of X, 4,500 units of
Y and 1,200 units of Z.
b) Calculate the TA ratio for each product.
c) In the absence of demand restrictions for the three products, advise Corrie's
management on the optimal production plan.
Solution
a) Profit per day

= throughput contribution – conversion cost

= [(RWF70 × 6,000) + (RWF80 × 4,500) + (RWF200 × 1,200)] – RWF720,000
= RWF300,000

b) TA ratio = throughput contribution per factory hour/conversion cost per factory hour
Conversion cost per factory hour = Rwf720,000/8 = Rwf90,000
Product

Throughput contribution per factory hour

Cost per factory
hour

TA ratio

X

RWF70 × 1,200 = Rwf84,000

RWF90,000

0.93

Y

RWF80 × 1,500 = Rwf120,000

RWF90,000

1.33

Z

RWF200 × 600 = Rwf120,000

RWF90,000

1.33

c) An attempt should be made to remove the restriction on output caused by process
alpha's capacity. This will probably result in another bottleneck emerging elsewhere.
The extra capacity required to remove the restriction could be obtained by working
overtime, making process improvements or product specification changes. Until the
volume of throughput can be increased, output should be concentrated upon products
Y and Z (greatest TA ratios), unless there are good marketing reasons for continuing
the current production mix.
Page 168

Product X is losing money every time it is produced so, unless there are good reasons
why it is being produced, for example it has only just been introduced and is expected
to become more profitable, Corrie should consider ceasing production of X.

How can a business improve a throughput accounting ratio?

Measures

Consequences

•

Increase sales price per unit

•

•

Reduce material costs per unit, eg •
change materials and/or suppliers

•

Reduce operating expenses

•

Demand for the product may fall
Quality may fall and bulk discounts
may be lost
Quality may
increase

fall

and/or

errors

Limitations of the throughput accounting ratio
As we have seen, the TA ratio can be used to decide which products should be produced.
However, the huge majority of organisations cannot produce and market products based on
short-term profit considerations alone. Strategic-level issues such as market developments,
product developments and the stage reached in the product life cycle must also be taken into
account.

Throughput and limiting factor analysis
The throughput approach is very similar to the approach of maximising contribution per
unit of scarce resource, which you will have covered in your earlier studies.

Page 169

Knowledge brought forward from previous studies
Limiting factor analysis
•

An organisation might be faced with just one limiting factor (other than maximum sales
demand) but there might also be several scarce resources, with two or more of them
putting an effective limit on the level of activity that can be achieved.

•

Examples of limiting factors include sales demand and production constraints.
–
–
–

•

Labour. The limit may be either in terms of total quantity or of particular skills.
Materials. There may be insufficient available materials to produce enough units
to satisfy sales demand.
Manufacturing capacity. There may not be sufficient machine capacity for the
production required to meet sales demand.

It is assumed in limiting factor analysis that management would make a product mix
decision or service mix decision based on the option that would maximise profit and
that profit is maximised when contribution is maximised (given no change in fixed cost
expenditure incurred). In other words, marginal costing ideas are applied.
–

–
–

Contribution will be maximised by earning the biggest possible contribution per
unit of limiting factor. For example if grade A labour is the limiting factor,
contribution will be maximised by earning the biggest contribution per hour of
grade A labour worked.
The limiting factor decision therefore involves the determination of the
contribution earned per unit of limiting factor by each different product.
If the sales demand is limited, the profit-maximising decision will be to produce
the top-ranked product(s) up to the sales demand limit.

•

In limiting factor decisions, we generally assume that fixed costs are the same whatever
product or service mix is selected, so that the only relevant costs are variable costs.

•

When there is just one limiting factor, the technique for establishing the contributionmaximising product mix or service mix is to rank the products or services in order of
contribution-earning ability per unit of limiting factor.

Page 170

Throughput is defined as sales less material costs whereas contribution is defined as sales less
all variable costs. Throughput assumes that all costs except materials are fixed in the short
run.

Example: throughput v limiting factor analysis
A company produces two products, Beano and Nutto, which have the following production
costs.
Beano

Nutto

RWF

RWF

12

12

Direct labour cost

6

10

Variable overhead

6

10

Fixed overhead

6

10

30

42

Direct material cost

Total product cost

Fixed overheads are absorbed on the basis of direct labour cost. Beano and Nutto pass
through two processes, grinding and pasting which incur direct labour time as follows.
Time taken
Process

Beano

Nutto

Grinding

15 mins

25 mins

Pasting

25 mins

20 mins

The current market price for Beano is RWF75 and for Nutto RWF60 and, at these prices,
customers will buy as many units as are available.

Page 171

The capacity of the two processes limits the amount of units of products that can be
produced. Grinding can be carried out for 8 hours per day but pasting can only operate for 6
hours per day.
Required
What production plan should the company follow in order to maximise profits?
a) Using contribution per minute
b) Using throughput per minute

Solution
The constraint in this situation is the ability to process the product. The total daily processing
time for the two processes is as follows.
Maximum grinding time
Maximum pasting time

=
=

8 × 60 mins
6 × 60 mins

=
=

480 mins
360 mins

The maximum number of each product that can be produced is therefore:
Beano

Nutto

Units

Units

Grinding

480
= 32
15

480
= 19
25

Pasting

360
= 14
25

360
= 18
20

The total number of units that can be processed is greater for grinding so pasting capacity is
the binding constraint or limiting factor.
(a)

Maximising contribution per minute RWF in thousands
Contribution of Beano =

RWF(75 – 12 – 6 – 6)

=

RWF51

Contribution of Nutto =

RWF(60 – 12 – 10 – 10)

=

RWF28

Page 172

Contribution of Beano per minute in pasting process =

51
=
25

RWF2.04

Contribution of Nutto per minute in pasting process =

28
20

=

RWF1.04

The profit maximising solution is therefore to produce the maximum number of units of
Beano, giving a contribution of 14 × RWF51 = RWF714

b)

Maximising throughput per minute
Contribution of Beano =

RWF(75 – 12)

=

RWF63

Contribution of Nutto =

RWF(60 – 12)

=

RWF48

Throughput per minute of Beano in pasting process

=

63
25

= RWF2.52

Throughput per minute of Nutto in pasting process

=

48
20

= RWF2.40

The profit maximising approach is therefore again to produce the maximum number of
units of Beano, but the result is not as clear cut.

Page 173

CHAPTER ROUNDUP
• Throughput accounting is a product management system which aims to maximise
throughput, and therefore cash generation from sales, rather than profit. A just in
time (JIT) environment is operated, with buffer inventory kept only when there is a
defined bottleneck .
• Performance measures in throughput accounting are based around the concept that only
direct materials are regarded as variable costs.
• In a throughput environment, production priority must be given to the products best
able to generate throughput, that is those products that maximise throughput per
unit of bottleneck.

Page 174

STUDY UNIT 8
Limiting Factor Analysis
Contents

Page

Limiting Factors …………………………………………………………….

177

Limiting Factor Analysis And Restricted Freedom Of Action .…..……..

185

Make Or Buy Decisions And Scarce Resources ………………………….

191

Limiting Factors And Shadow Prices …………………………….………

195

Using Limiting Factor Analysis …………………………..………………

197

Page 175

BLANK

Page 176

LIMITING FACTORS
A scarce resource is a resource of which there is a limited supply. Once a scarce resource
affects the ability of an organisation to earn profits, a scarce resource becomes known as a
limiting factor.

A limiting factor or key factor is 'Anything which limits the activity of an entity. An entity
seeks to optimise the benefit it obtains from the limiting factor'.

Knowledge brought forward from earlier studies
Limiting factor analysis
•

An organisation might be faced with just one limiting factor (other than maximum sales
demand) but there might also be several scarce resources, with two or more of them
putting an effective limit on the level of activity that can be achieved.

•

Examples of limiting factors include sales demand and production constraints.
–
–
–

•

Labour. The limit may be either in terms of total quantity or of particular skills.
Materials. There may be insufficient available materials to produce enough units
to satisfy sales demand.
Manufacturing capacity. There may not be sufficient machine capacity for the
production required to meet sales demand.

It is assumed in limiting factor analysis that management would make a product mix
decision or service mix decision based on the option that would maximise profit and
that profit is maximised when contribution is maximised (given no change in fixed cost
expenditure incurred). In other words, marginal costing ideas are applied.
–

–
–

Contribution will be maximised by earning the biggest possible contribution per
unit of limiting factor. For example if grade A labour is the limiting factor,
contribution will be maximised by earning the biggest contribution per hour of
grade A labour worked.
The limiting factor decision therefore involves the determination of the
contribution earned per unit of limiting factor by each different product.
If the sales demand is limited, the profit-maximising decision will be to produce
the top-ranked product(s) up to the sales demand limit.
Page 177

•

In limiting factor decisions, we generally assume that fixed costs are the same whatever
product or service mix is selected, so that the only relevant costs are variable costs.

•

When there is just one limiting factor, the technique for establishing the contributionmaximising product mix or service mix is to rank the products or services in order of
contribution-earning ability per unit of limiting factor.

If resources are limiting factors, contribution will be maximised by earning the biggest
possible contribution per unit of limiting factor.
Where there is just one limiting factor, the technique for establishing the contributionmaximising product or service mix is to rank the products or services in order of
contribution-earning ability per unit of limiting factor.

Example: limiting factor decision
Sausages Ltd makes two products, the Mash and the Sauce. Unit variable costs are as follows.
Mash
RWF ‘000

Sauce
RWF’000

Direct materials

1

3

Direct labour (RWF3,000 per hour)

6

3

Variable overhead

1

1

8

7

The sales price per unit is RWF14,000 for Mash and RWF11,000 for Sauce. During July the
available direct labour is limited to 8,000 hours. Sales demand in July is expected to be as
follows.
Mash

3,000 units

Sauce

5,000 units
Page 178

Required
Determine the production budget that will maximise profit, assuming that fixed costs per
month are RWF20m and that there is no opening inventory of finished goods or work in
progress.

Solution
Step 1

Confirm that the limiting factor is something other than sales demand.
Mash
2 hrs

Labour hours per unit
Sales demand
Labour hours needed

Sauces

3,000 units
6,000 hrs

Total

1 hr
5,000 units
5,000 hrs

11,000 hrs

Labour hours available

8,000 hrs

Shortfall

3,000 hrs

Labour is the limiting factor on production.

Step 2

Identify the contribution earned by each product per unit of scarce resource, that
is, per labour hour worked.
Mash

Sauce

RWF ‘000
Sales price

RWF ‘000

14

11

Variable cost

8

7

Unit contribution

6

4

Labour hours per unit

2 hrs

1 hr

3

4

Contribution per labour hour (= per unit of
limiting factor)
Page 179

Although Mash has a higher unit contribution than Sauce, two Sauces can be
made in the time it takes to make one Mash. Because labour is in short supply it is
more profitable to make Sauces than Mashes.

Step 3

Determine the budgeted production and sales. Sufficient Sauces will be made to
meet the full sales demand, and the remaining labour hours available will then be
used to make Mashes.

Product

Demand

Hours
required

Sauce

5,000

5,000

Mashes

3,000

(a)

6,000

11,000

(b)
Product

Units

Hours
needed

Hours
available

Priority for
manufacture

5,000

1st

3,000 (bal)

2nd

8,000

Contribution
per unit

Total

RWF ‘000

RWF ‘000

Sauce

5,000

5,000

4

20,000

Mashes (balance)

1,500

3,000

6

9,000

8,000
Less fixed costs

29,000
20,000

Profit

9,000

Page 180

Conclusion
a) Unit contribution is not the correct way to decide priorities.
b) Labour hours are the scarce resource, therefore contribution per labour hour is the
correct way to decide priorities.
c) The Sauce earns RWF4,000 contribution per labour hour, and the Mash earns
RWF3,000 contribution per labour hour. Sauce therefore makes more profitable use
of the scarce resource, and should be manufactured first.

Two potentially limiting factors
You may be asked to deal with situations where two limiting factors are potentially limiting
(and there are also product/service demand limitations). The approach in these situations is to
find out which factor (if any) prevents the business from fulfilling maximum demand.

Where there is a maximum potential sales demand for an organisation's products or
services, they should still be ranked in order of contribution-earning ability per unit of the
limiting factor. The contribution-maximising decision, however, will be to produce the topranked products (or to provide the top-ranked services) up to the sales demand limit.

Example: two potentially limiting factors
Lucky manufactures and sells three products - X, Y and Z, for which budgeted sales demand,
unit selling prices and unit variable costs are as follows.

Budgeted sales demand

X

Y

Z

550 units

500 units

400 units

RWF
‘000
Unit sales price
Variable costs: materials
labour

RWF
‘000

RWF
‘000

16
8
4

RWF
‘000
18

6
6
12
Page 181

RWF
‘000

RWF
‘000
14

2
9
12

11

Unit contribution

4

6

3

The organisation has existing inventory of 250 units of X and 200 units of Z, which it is quite
willing to use up to meet sales demand. All three products use the same direct materials and
the same type of direct labour. In the next year, the available supply of materials will be
restricted to RWF4,800,000 (at cost) and the available supply of labour to RWF6,600,000 (at
cost).

Required
Determine what product mix and sales mix would maximise the organisation's profits in the
next year.

Solution
There appear to be two scarce resources, direct materials and direct labour. This is not
certain, however, and because there is a limited sales demand as well, either of the following
might apply.
•
•

Step 1

There is no limiting factor at all, except sales demand.
There is only one scarce resource that prevents the full potential sales demand being
achieved.

Establish which of the resources, if any, is scarce.
X

Y

Z

Units

Units

Units

Budgeted sales

550

500

400

Inventory in hand

250

0

200

Minimum production to meet demand

300

500

200

Page 182

Minimum production to
meet sales demand
Units

Required
materials at cost
Rw RWF‘000

f

Required labour at
cost
RWF‘000

X

300

2,400

1,200

Y

500

3,000

3,000

Z

200

400

1,800

Total required

5,800

6,000

Total available

4,800

6,600

(Shortfall)/Surplus

(1,000)

600

Materials are a limiting factor, but labour is not.

Step 2

Rank X, Y and Z in order of contribution earned per Rwf1 of direct materials
consumed.
X
Y
Z
RWF ‘000 RWF ‘000

RWF ‘000

Unit contribution

4

6

3

Cost of materials

8

6

2

Contribution per RWF1,000 materials

0.50

1.00

1.50

Ranking

3rd

2nd

1st

Page 183

Step 3

Determine a production plan. Z should be manufactured up to the limit where
units produced plus units held in inventory will meet sales demand, then Y
second and X third, until all the available materials are used up.

Ranking

Product

Sales demand

Production

Materials

less units held

quantity

cost

Units

Units

RWF
‘000

1st

Z

200

200

(× 2)

400

2nd

Y

500

500

(× 6)

3,000

3rd

X

300

175

(× 8)

*1,400

Total available

4,800

* Balancing amount using up total available.

Step 4

Draw up a budget. The profit-maximising budget is as follows.
X

Y

Z

Units

Units

Units

Opening inventory

250

0

200

Add production

175

500

200

Sales

425

500

400

Y
RWF
‘000
9,000

Z
RWF
‘000

Revenue

X
RWF
‘000
6,800

5,600

Total
RWF
‘000
21,400

Variable costs

5,100

6,000

4,400

15,500

Contribution

1,700

3,000

1,200

5,900

Page 184

LIMITING FACTOR ANALYSIS AND RESTRICTED
FREEDOM OF ACTION
In certain circumstances an organisation faced with a limiting factor on production and sales
might not be able to produce the profit-maximising product mix because the mix and/or
volume of products that can be produced and sold is also restricted by a factor other than a
scarce resource.
a) A contract to supply a certain number of products to a customer
b) Production/sales of a minimum quantity of one or more products to provide a
complete product range and/or to maintain customer goodwill
c) Maintenance of a certain market share of one or more products
In each of these cases, the organisation might have to produce more of a particular product
or products than the level established by ranking according to contribution per unit of
limiting factor.

If an organisation has to produce more of a particular product or products than the level
established by ranking according to contribution per unit of limiting factor, the products
should be ranked in the normal way but the optimum production plan must first take into
account the minimum production requirements. The remaining resource must then be
allocated according to the ranking.

Page 185

Example: restricted freedom of action
Harvey is currently preparing its budget for the year ending 30 September 20X2. The
company manufactures and sells three products, Beta, Delta and Gamma.
The unit selling price and cost structure of each product is budgeted as follows.
Beta

Delta

Gamma

RWF
‘000

RWF
‘000

RWF
‘000

100

124

32

Labour

24

48

6

Materials

26

7

8

Overhead

10

5

6

60

60

20

Selling price
Variable costs:

Direct labour rate is budgeted at RWF6,000 per hour, and fixed costs at RWF1,300m per
annum. The company has a maximum production capacity of 228,000 direct labour hours.
A meeting of the board of directors has been convened to discuss the budget and to resolve the
problem as to the quantity of each product which should be made and sold. The sales director
presented the results of a recent market survey which reveals that market demand for the
company's products will be as follows.

Product

Units

Beta

24,000

Delta

12,000

Gamma

60,000

Page 186

The production director proposes that since Gamma only contributes RWF12,000 per unit,
the product should no longer be produced, and the surplus capacity transferred to produce
additional quantities of Beta and Delta. The sales director does not agree with the proposal.
Gamma is considered necessary to complement the product range and to maintain customer
goodwill. If Gamma is not offered, the sales director believes that sales of Beta and Delta will
be seriously affected. After further discussion the board decided that a minimum of 10,000
units of each product should be produced. The remaining production capacity would then be
allocated so as to achieve the maximum profit possible.

Required
Prepare a budget statement which clearly shows the maximum profit which could be
achieved in the year ending 30 September 20X2.

Solution
Step 1

Ascertain whether labour hours are a scarce resource.
Units demanded

Labour hours per unit

Total labour hours

Beta

24,000

4 (RWF24/RWF6)

96,000

Delta

12,000

8 (RWF48/RWF6)

96,000

Gamma

60,000

1 (RWF6/RWF6)

60,000
252,000

Page 187

Step 2

Rank the products.
Since only 228,000 hours are available we need to establish which product earns
the greatest contribution per labour hour.
Beta
Contribution (RWF ‘000)

Ranking

Step 3

Gamma

40

64

12

4

8

1

10

8

12

Labour hours

Contribution per labour hour – RWF‘000

Delta

2nd

3rd

1st

Determine a production plan.
The optimum production plan must take into account the requirement that 10,000
units of each product are produced, and then allocate the remaining hours according
to the above ranking.
Hours
Beta

10,000 units × 4 hours

40,000

Delta

10,000 units × 8 hours

80,000

Gamma

10,000 units × 1 hour

10,000
130,000

Gamma

50,000 units × 1 hour (full demand)

Beta

12,000 units × 4 hours (balance)

50,000
48,000
228,000

Page 188

Step 4

Draw up a budget.
BUDGET STATEMENT
Contribution

RWF ‘000

Beta (22,000 units × RWF40,000)

880,000

Delta (10,000 units × RWF64,000)

640,000

Gamma (60,000 units × RWF12,000)

720,000
2,240,000

Fixed costs

1,300,000

Profit

940,000

Page 189

BLANK

Page 190

MAKE OR BUY DECISIONS AND SCARE
RESOURCES
An organisation might want to do more things than it has the resources for, and so its
alternatives would be as follows.
a) Make the best use of the available resources and ignore the opportunities to buy help
from outside
b) Combine internal resources with buying externally so as to do more and increase
profitability
Buying help from outside is justifiable if it adds to profits. A further decision is then required
on how to split the work between internal and external effort. What parts of the work should
be given to suppliers or sub-contractors so as to maximise profitability?

In a situation where a company must sub-contract work to make up a shortfall in its own
in-house capabilities, its total costs will be minimised if those units bought have the lowest
extra variable cost of buying per unit of scarce resource saved by buying.

Example: make or buy decision with scarce resources
MM manufactures three components, S, A and T using the same machines for each. The
budget for the next year calls for the production and assembly of 4,000 of each component.
The variable production cost per unit of the final product is as follows.

Machine hours

Variable cost
RWF ‘000

1 unit of S

3

20

1 unit of A

2

36

1 unit of T

4

24

Assembly

20
100
Page 191

Only 24,000 hours of machine time will be available during the year, and a sub-contractor
has quoted the following unit prices for supplying components: S RWF29,000; A
RWF40,000; T RWF34,000.

Required
Advise MM.

Solution
The organisation's budget calls for 36,000 hours of machine time, if all the components are to
be produced in-house. Only 24,000 hours are available, and so there is a shortfall of 12,000
hours of machine time, which is therefore a limiting factor. The shortage can be overcome by
subcontracting the equivalent of 12,000 machine hours' output to the subcontractor.
The assembly costs are not relevant costs because they are unaffected by the decision.
The decision rule is to minimise the extra variable costs of sub-contracting per unit of
scarce resource saved (that is, per machine hour saved).
S

A

T

RWF
‘000

RWF ‘000

RWF
‘000

Variable cost of making

20

36

24

Variable cost of buying

29

40

34

Extra variable cost of buying

9

4

10

Machine hours saved by buying

3 hrs

2 hrs

4 hrs

2

2.50

Extra variable cost of buying per hour
saved

3

This analysis shows that it is cheaper to buy A than to buy T and it is most expensive to
buy S. The priority for making the components in-house will be in the reverse order: S,
then T, then A. There are enough machine hours to make all 4,000 units of S (12,000 hours)
Page 192

and to produce 3,000 units of T (another 12,000 hours). 12,000 hours' production of T and A
must be sub-contracted.
The cost-minimising and so profit-maximising make and buy schedule is as follows.

Machine
hours
used/saved

Component

Make:
T

S

Number of units

Unit variable

Total variable

cost

cost

RWF ‘000

RWF ‘000

12,000

4,000

20

80,000

12,000

3,000

24

72,000

24,000

152,000

Buy: T

4,000

1,000

34

34,000

A

8,000

4,000

40

160,000

12,000

346,000

Total variable cost of components, excluding assembly costs

Page 193

BLANK

Page 194

LIMITING FACTORS AND SHADOW PRICES
Whenever there are limiting factors, there will be opportunity costs. As you know, these are
the benefits forgone by using a limiting factor in one way instead of in the next most
profitable way.
For example, suppose that an organisation provides two services X and Y, which earn a
contribution of RWF24,000 and RWF18,000 per unit respectively. Service X requires 4
labour hours, and service Y 2 hours. Only 5,000 labour hours are available, and potential
demand is for 1,000 of each of X and Y.
Labour hours would be a limiting factor, and with X earning RWF6,000 per hour and Y
earning RWF9,000 per hour, the profit-maximising decision would be as follows.

Contribution

Y
X (balance)

Services

Hours

RWF‘000

1,000

2,000

18,000

750

3,000

18,000

5,000

36,000

Priority is given to Y because the opportunity cost of providing Y instead of more of X is
RWF6,000 per hour (X's contribution per labour hour), and since Y earns RWF9,000 per
hour, the incremental benefit of providing Y instead of X would be RWF3,000 per hour.
If extra labour hours could be made available, more X (up to 1,000) would be provided, and
an extra contribution of RWF6,000 per hour could be earned. Similarly, if fewer labour hours
were available, the decision would be to provide fewer X and to keep provision of Y at 1,000,
and so the loss of labour hours would cost the organisation RWF6,000 per hour in lost
contribution. This RWF6,000 per hour, the marginal contribution-earning potential of the
limiting factor at the profit-maximising output level, is referred to as the shadow price (or
dual price) of the limiting factor.

Page 195

The shadow price or dual price of a limiting factor is the increase in value which would be
created by having one additional unit of the limiting factor at the original cost.

A shadow price is 'An increase in value which would be created by having available one
additional unit of a limiting resource at the original cost'.

Note that the shadow price only applies while the extra unit of resource can be obtained at its
normal variable cost. The shadow price also indicates the amount by which contribution
could fall if an organisation is deprived of one unit of the resource.
The shadow price of a resource is its internal opportunity cost. This is the marginal
contribution towards fixed costs and profit that can be earned for each unit of the limiting
factor that is available. A knowledge of the shadow price of a resource will help managers to
decide how much it is worth paying to acquire another unit of the resource.

Page 196

USING LIMITING FACTOR ANALYSIS
Don't ignore this wordy session – if you were to get a full limiting factor analysis question in
the exam there would undoubtedly be marks for discussion of pertinent non-quantifiable
issues.
Limiting factor analysis provides us with a profit-maximising product mix, within the
assumptions made. It is important to remember, however, that other considerations, so far not
fully considered in our examples, might entirely alter the decision reached.

Non-quantifiable factors
Non-quantifiable factors, such as effect on customer goodwill, ability to restart production
and reasons for a resource being a limiting factor, should also be borne in mind in product
mix decisions.
Factor

Examples

Demand

Will the decision reached (perhaps to make and sell just one product rather than
two) have a harmful effect on customer loyalty and sales demand? For example, a
manufacturer of knives and forks could not expect to cease production of knives
without affecting sales demand for the forks.

Longterm
effects

Is the decision going to affect the long-term as well as the short-term plans of the
organisation? If a particular product is not produced, or produced at a level below
sales demand, is it likely that competitors will take over vacated markets? Labour
skilled in the manufacture of the product may be lost and a decision to reopen or
expand production of the product in the future may not be possible.

Labour

If labour is a limiting factor, is it because the skills required are difficult to obtain,
perhaps because the organisation is using very old-fashioned production methods,
or is the organisation a high-tech newcomer in a low-tech area? Or perhaps the
conditions of work are so unappealing that people simply do not want to work for
the organisation.

Other
limiting
factors

The same sort of questions should be asked whatever the limiting factor. If
machine hours are in short supply is this because more machines are needed, or
newer, more reliable and efficient machines? If materials are in short supply,
what are competitors doing? Have they found an equivalent or better substitute?
Is it time to redesign the product?

Page 197

Assumptions in limiting factor analysis
Various assumptions are made in limiting factor analysis.
–
–
–
–

Fixed costs remain the same regardless of the decision taken.
Unit variable cost is constant regardless of the decision taken.
Estimates of sales demand and resources required are known with certainty.
Units of output are divisible.

In the examples covered in the chapter, certain assumptions were made. If any of the
assumptions are not valid, then the profit-maximising decision might be different. These
assumptions are as follows.

a) Fixed costs will be the same regardless of the decision that is taken, and so the
profit-maximising and contribution-maximising output level will be the same.
This will not necessarily be true, since some fixed costs might be directly attributable to
a product or service. A decision to reduce or cease altogether activity on a product or
service might therefore result in some fixed cost savings, which would have to be taken
into account.

b) The unit variable cost is constant, regardless of the output quantity of a product or
service. This implies the following.
(i)

The price of resources will be unchanged regardless of quantity; for example,
there will be no bulk purchase discount of raw materials.

(ii)

Efficiency and productivity levels will be unchanged; regardless of output
quantity the direct labour productivity, the machine time per unit, and the
materials consumption per unit will remain the same.

c) The estimates of sales demand for each product, and the resources required to
make each product, are known with certainty.
In the example in Section 1.2.1, there were estimates of the budgeted sales demand for
each of three products, and these estimates were used to establish the profit-maximising
product mix. Suppose the estimates were wrong? The product mix finally chosen would
then either mean that some sales demand of the most profitable item would be
Page 198

unsatisfied, or that production would exceed sales demand, leaving some inventory
unsold. Clearly, once a profit-maximising output decision is reached, management will
have to keep their decision under continual review, and adjust their decision as
appropriate in the light of actual results.

d) Units of output are divisible, and a profit-maximising solution might include
fractions of units as the optimum output level.
Where fractional answers are not realistic, some rounding of the figures will be
necessary.

Exam Focus Point
An examination problem might present you with a situation in which there is a limiting
factor, without specifically stating that this is so, and you will have the task of recognising
what the situation is. You may be given a hint with the wording of the question.
a) 'It is possible that the main raw material used in manufacturing the products will be
difficult to obtain in the next year.'
b) 'The company employs a fixed number of employees who work a maximum overtime
of eight hours on top of the basic 36 hour week. The company has also agreed that no
more staff will be recruited next year.'
c)
In (a) there is a hint that raw materials might be a limiting factor. In (b), perhaps less
obviously, a maximum limit is placed on the available labour hours, and so the possibility
should occur to you that perhaps labour is a limiting factor.

If you suspect the existence of a limiting factor, some quick computations should confirm
your suspicions.
a) Calculate the amount of the scarce resource (material quantities, labour hours,
machine hours and so on) needed to meet the potential sales demand.
b) Calculate the amount of the scarce resource available (for example number of
employees multiplied by maximum working hours per employee).
c) Compare the two figures. Obviously, if the resources needed exceed the resources
available, there is a limiting factor on output and sales.
Page 199

CHAPTER ROUNDUP
•

A scarce resource is a resource of which there is a limited supply. Once a scarce
resource affects the ability of an organisation to earn profits, a scarce resource becomes
known as a limiting factor.

•

If resources are limiting factors, contribution will be maximised by earning the
biggest possible contribution per unit of limiting factor.

•

Where there is just one limiting factor, the technique for establishing the
contribution-maximising product or service mix is to rank the products or services in
order of contribution-earning ability per unit of limiting factor.

•

Where there is a maximum potential sales demand for an organisation's products or
services, they should still be ranked in order of contribution-earning ability per unit of
the limiting factor. The contribution-maximising decision, however, will be to produce
the top-ranked products (or to provide the top-ranked services) up to the sales demand
limit.

•

If an organisation has to produce more of a particular product or products than the
level established by ranking according to contribution per unit of limiting factor, the
products should be ranked in the normal way but the optimum production plan must
first take into account the minimum production requirements. The remaining resource
must then be allocated according to the ranking.

•

In a situation where an organisation must subcontract work to make up a shortfall in
its own in-house capabilities, its total costs will be minimised if the units bought have
the lowest extra variable cost of buying per unit of scarce resource saved by buying.

•

The shadow price or dual price of a limiting factor is the increase in value which
would be created by having one additional unit of the limiting factor at the original
cost.

•

Non-quantifiable factors, such as the effect on customer goodwill, ability to restart
production and reasons for a resource being a limiting factor, should also be borne in
mind in product mix decisions.

•

Various assumptions are made in limiting factor analysis.
− Fixed costs remain the same regardless of the decision taken.
− Unit variable cost is constant regardless of the decision taken.
− Estimates of sales demand and resources required are known with certainty.
Page 200

− Units of output are divisible

Page 201

STUDY UNIT 9
Linear Programming: The Graphical Method
Contents

Page

The Graphical Method ………………………….………………………….

203

The Graphical Method Using Simultaneous Equations …………..……..

223

Sensitivity Analysis …………………………………………………..…….

229

Page 201

BLANK

Page 202

THE GRAPHICAL METHOD
The graphical method of linear programming is used for problems involving two products.

Formulating the problem
Let us suppose that WX manufactures two products, A and B. Both products pass through
two production departments, mixing and shaping. The organisation's objective is to maximise
contribution to fixed costs.
Product A is sold for RWF1,500 whereas product B is priced at RWF2,000. There is
unlimited demand for product A but demand for B is limited to 13,000 units per annum. The
machine hours available in each department are restricted to 2,400 per annum. Other relevant
data are as follows.

Machine hours required

Mixing

Shaping

Hrs

Hrs

Product A

0.06

0.04

Product B

0.08

0.12

Variable cost per unit

RWF ‘000

Product A

1.30

Product B

1.70

Before we work through the steps involved in solving this constraints problem using the
graphical approach to linear programming, it is worth reading the CIMA Official
Terminology definition of linear programming to get a glimpse of what we will be doing.
Linear programming is 'The use of a series of linear equations to construct a mathematical
model. The objective is to obtain an optimal solution to a complex operational problem,
Page 203

which may involve the production of a number of products in an environment in which there
are many constraints'.
Example
What are the constraints in the situation facing WX?
(i)
(ii)
(iii)
(iv)

Machine hours in each department
Labour hours in each department
Sales demand for product B
Selling price of product A

A
B
C
D

(i) and (iii)
(i) only
(ii) and (iv)
(i), (ii) and (iii)

Soultion
The correct answer is A. There is no restriction on the availability of labour hours. Selling
price cannot be a constraint.

The steps in the graphical method are as follows.
•
•
•
•
•
•

Define variables.
Establish objective function.
Establish constraints.
Draw a graph of the constraints.
Establish the feasible region.
Determine the optimal product mix.

Page 204

Let's start solving WX's problem.
Step 1

Define variables
What are the quantities that WX can vary? Obviously not the number of
machine hours or the demand for product B. The only things which it can vary are
the number of units of each type of product produced. It is those numbers
which the company has to determine in such a way as to obtain the maximum
possible profit. Our variables (which are usually products being produced) will
therefore be as follows.
Let x = number of units of product A produced.
Let y = number of units of product B produced.

Step 2

Establish objective function

The objective function is a quantified statement of the aim of a resource allocation decision.
We now need to introduce the question of contribution or profit. We know that
the contribution on each type of product is as follows.

RWFper unit
Product A

RWF(1.50 – 1.30) =

0.20

Product B

RWF(2.00 – 1.70) =

0.30

The objective of the company is to maximise contribution and so the objective
function to be maximised is as follows.
Contribution (C) = 0.2x + 0.3y

Step 3

Establish constraints

A constraint is 'An activity, resource or policy that limits the ability to achieve objectives'.
The value of the objective function (the maximum contribution achievable from
producing products A and B) is limited by the constraints facing WX, however.
To incorporate this into the problem we need to translate the constraints into
inequalities involving the variables defined in Step 1. An inequality is an
equation taking the form 'greater than or equal to' or 'less than or equal to'.

Page 205

a)

Consider the mixing department machine hours constraint.
(i) Each unit of product A requires 0.06 hours of machine time. Producing
five units therefore requires 5 × 0.06 hours of machine time and, more
generally, producing x units will require 0.06x hours.
(ii) Likewise producing y units of product B will require 0.08y hours.
(iii) The total machine hours needed in the mixing department to make x units
of product A and y units of product B is 0.06x + 0.08y.
(iv) We know that this cannot be greater than 2,400 hours and so we arrive
at the following inequality.
0.06x + 0.08y ≤ 2,400

Example
How can the constraint facing the shaping department be written as an inequality?
A
B
C
D

0.4x + 0.012y ≥ 2,400
0.04x + 0.12y ≤ 2,400
0.4x + 0.012y ≤ 2,400
0.04x + 0.12y ≥ 2,400

Solution

The correct answer is B. The constraint has to be a 'less than equal to' inequality, because
the amount of resource used (0.04x + 0.12y) has to be 'less than equal to' the amount
available of 2,400 hours.

b)

The final inequality is easier to obtain. The number of units of product B
produced and sold is y but this has to be less than or equal to 13,000. Our
inequality is therefore as follows.
y ≤ 13,000

Page 206

c)

We also need to add non-negativity constraints (x ≥ 0, y ≥ 0) since negative
numbers of products cannot be produced. (Linear programming is simply a
mathematical tool and so there is nothing in this method which guarantees that
the answer will 'make sense'. An unprofitable product may produce an answer
which is negative. This is mathematically correct but nonsense in operational
terms. Always remember to include the non-negativity constraints. The examiner
will not appreciate 'impossible' solutions.)

The problem has now been reduced to the following four inequalities and one equation.
Maximise contribution (C) = 0.2x + 0.3y, subject to the following constraints:
0.06x + 0.08y

≤

2,400

0.04x + 0.12y

≤

2,400

≤

13,000

≤

x

0≤y
0

Example
An organisation makes two products, X and Y. Product X has a contribution of RWF124 per
unit and product Y RWF80 per unit. Both products pass through two departments for
processing and the times in minutes per unit are as follows.
Product X

Product Y

Department 1

150

90

Department 2

100

120

Currently there is a maximum of 225 hours per week available in department 1 and 200 hours
in department 2. The organisation can sell all it can produce of X but EAC quotas restrict the
sale of Y to a maximum of 75 units per week. The organisation, which wishes to maximise
contribution, currently makes and sells 30 units of X and 75 units of Y per week.

Page 207

Required
Assume x and y are the number of units of X and Y produced per week. Formulate a
linear programming model of this problem, filling in the blanks in (a) and (b) below.
a)

The objective function is to maximise weekly contribution, given by C =
………………….. .

b)

The constraints are:
Department 1 ……………………. EU quota …………………….
Department 2 ……………………. Non-negativity ………………

Solution

a) The objective function is to maximise weekly contribution, given by C = 124x + 80y.
b) The constraints are:
Department 1:

150x + 90y

≤

225 × 60 minutes

Department 2:

100x + 120y

≤

200 × 60 minutes

EAC quota y

≤

75

≥

0

Non-negativity

x, y

These constraints can be simplified to:
Department 1:

15x + 9y

≤

1,350

Department 2:

10x + 12y

≤

1,200

EAC quota

y

≤

75

Non-negativity

x, y

≥

0

Page 208

Graphing the problem
A graphical solution is only possible when there are two variables in the problem. One
variable is represented by the x axis of the graph and one by the y axis. Since non-negative
values are not usually allowed, the graph shows only zero and positive values of x and y.

Graphing equations and constraints
A linear equation with one or two variables is shown as a straight line on a graph. Thus y
= 6 would be shown as follows.

If the problem included a constraint that y could not exceed 6, the inequality y ≤ 6 would
be represented by the shaded area of the graph below.

Page 209

The equation 4x + 3y = 24 is also a straight line on a graph. To draw any straight line, we
need only to plot two points and join them up. The easiest points to plot are the following.
•
•

x = 0 (in this example, if x = 0, 3y = 24, y = 8)
y = 0 (in this example, if y = 0, 4x = 24, x = 6)

By plotting the points, (0, 8) and (6, 0) on a graph, and joining them up, we have the line for
4x + 3y = 24.

Any combination of values for x and y on the line satisfies the equation. Thus at a point
where x = 3 and y = 4, 4x + 3y = 24. Similarly, at a point where x = 4.5 and y = 2, 4x + 3y =
24.
If we had a constraint 4x + 3y ≤ 24, any combined value of x and y within the shaded
area below (on or below the line) would satisfy the constraint.

Page 210

Consider point P which has coordinates of (2, 2). Here 4x + 3y = 14, which is less than 24;
and at point Q where x = 5½, y = 2/3, 4x + 3y = 24. Both P and Q lie within the feasible
area or feasible region. A feasible area enclosed on all sides may also be called a feasible
polygon.

A feasible region is 'The area contained within all of the constraint lines shown on a
graphical depiction of a linear programming problem. All feasible combinations of output are
contained within or located on the boundaries of the feasible region'.

When there are several constraints, the feasible area of combinations of values of x and y
must be an area where all the inequalities are satisfied. Thus, if y ≤ 6 and 4x + 3y ≤ 24 the
feasible area would be the shaded area in the following graph.

a) Point R (x = 0.75, y = 7) is not in the feasible area because although it satisfies the
inequality 4x + 3y ≤ 24, it does not satisfy y ≤ 6.
b) Point T (x = 5, y = 6) is not in the feasible area, because although it satisfies the
inequality y ≤ 6, it does not satisfy 4x + 3y ≤ 24.
c) Point S (x = 1.5, y = 6) satisfies both inequalities and lies just on the boundary of the
feasible area since y = 6 exactly, and 4x + 3y = 24. Point S is thus at the intersection
of the two lines.

Page 211

Similarly, if y ≥ 6 and 4x + 3y ≥ 24 but x is ≤ 6, the feasible area would be the shaded area in
the graph below.

Example
Draw the feasible region which arises from the constraints facing WX on the graph below.

Solution
If 0.06x + 0.08y = 2,400, then if x = 0, y = 30,000 and if y = 0, x = 40,000.
If 0.04x + 0.12y = 2,400, then if x = 0, y = 20,000 and if y = 0, x = 60,000.

Page 212

Finding the optimum allocation of resources
The optimal solution can be found by 'sliding the iso-contribution (or profit) line out'.

Having found the feasible region (which includes all the possible solutions to the problem)
we need to find which of these possible solutions is 'best' or optimal in the sense that it
yields the maximum possible contribution.
Look at the feasible region of the problem faced by WX (see the solution to the question
above). Even in such a simple problem as this, there are a great many possible solution
points within the feasible area. Even to write them all down would be a time-consuming
process and also an unnecessary one, as we shall see.

Page 213

Here is the graph of WX's problem.

a) Consider point A at which 10,000 units of product A and 5,000 units of product B are
being manufactured. This will yield a contribution of (1,000 × RWF0.20) + (5,000 ×
RWF0.30) = RWF3,500.
b) We would clearly get more contribution at point B, where the same number of units of
product B are being produced but where the number of units of product A has
increased by 5,000.
c) We would also get more contribution at point C where the number of units of product
A is the same but 2,500 more units of product B are being produced.

This argument suggests that the 'best' solution is going to be at a point on the edge of the
feasible area rather than in the middle of it.
This still leaves us with quite a few points to look at but there is a way in which we can
narrow down still further the likely points at which the best solution will be found.
Suppose that WX wishes to earn contribution of RWF3,000. The company could sell the
following combinations of the two products.
Page 214

a) 15,000 units of A, no B.
b) No A, 10,000 units of B.
c) A suitable mix of the two, such as 7,500 A and 5,000 B.

The possible combinations required to earn contribution of RWF3,000 could be shown
by the straight line 0.2x + 0.3y = 3,000.

Likewise for profits of RWF6,000 and RWF1,500, lines of 0.2x + 0.3y = 6,000 and 0.2x +
0.3y = 1,500 could be drawn showing the combination of the two products which would
achieve contribution of RWF6,000 or RWF1,500.

Page 215

The contribution lines are all parallel. (They are called iso-contribution lines, 'iso'
meaning equal.) A similar line drawn for any other total contribution would also be parallel to
the three lines shown here. Bigger contribution is shown by lines further from the origin
(0.2x + 0.3y = 6,000), smaller contribution by lines closer to the origin (0.2x + 0.3y = 1,500).
As WX tries to increase possible contribution, we need to 'slide' any contribution line
outwards from the origin, while always keeping it parallel to the other contribution lines.
As we do this there will come a point at which, if we were to move the contribution line out
any further, it would cease to lie in the feasible region. Greater contribution could not be
achieved, because of the constraints. In our example concerning WX this will happen, as you
should test for yourself, where the contribution line just passes through the intersection of
0.06x + 0.08y = 2,400 and 0.04x + 0.12y = 2,400 (at coordinates (24,000, 12,000)). The point
(24,000, 12,000) will therefore give us the optimal allocation of resources (to produce 24,000
units of A and 12,000 units of B).

We can usefully summarise the graphical approach to linear programming as follows.
Step 1

Define variables.

Step 4

Graph the problem.

Step 2

Establish objective function.

Step 5

Define feasible area.

Step 3

Establish constraints.

Step 6

Determine optimal solution.

Example: the graphical solution with a twist
This example shows that it is not always necessarily easy to identify the decision variables in
a problem.
DCC operates a small plant for the manufacture of two joint chemical products X and Y. The
production of these chemicals requires two raw materials, A and B, which cost RWF5,000
and RWF8,000 per litre respectively. The maximum available supply per week is 2,700 litres
of A and 2,000 litres of B.
The plant can operate using either of two processes, which have differing operating costs and
raw materials requirements for the production of X and Y, as follows.
Page 216

Process 1
Process

Process 2

Raw materials consumed

Output

Cost

Litres per processing hour

Litres per hour

RWF ‘000
per hour

A

B

X

Y

1

20

10

15

20

500

2

30

20

20

10

230

The plant can run for 120 hours per week in total, but for safety reasons, process 2 cannot be
operated for more than 80 hours per week.
X sells for RWF18,000 per litre, Y for RWF24,000 per litre.

Required
Formulate a linear programming model, and then solve it, to determine how the plant should
be operated each week.

Solution
Step 1

Define variables
You might decide that there are two decision variables in the problem, the
quantity of X and the quantity of Y to make each week. If so, begin by letting
these be x and y respectively.
You might also readily recognise that the aim should be to maximise the total
weekly contribution, and so the objective function should be expressed in terms
of maximising the total contribution from X and Y.
The contribution per litre from X and Y cannot be calculated because the
operating costs are expressed in terms of processing hours.

Page 217

RWF ‘000 per RWF ’000
hour
per hour

RWF ‘000 per
hour

RWF ‘000
per hour

Costs:
Material A

100

150

Material B

80

160

500

230

680

540

Operating cost

Revenue:
X (15 ×RWF k18)

270

(20 × 18,000)

360

Y (20 × RWF k24)

480

(10 × 24,000)

240

Contribution

750

600

70

60

The decision variables should be processing hours in each process, rather than litres of X
and Y. If we let the processing hours per week for process 1 be P1 and the processing hours
per week for process 2 be P2 we can now formulate an objective function, and constraints, as
follows.

Step 2

Establish objective function
(total contribution) subject to the constraints below
Maximise 70P1 + 60P2

Page 218

Step 3

Establish constraints
20P1 + 30P2 ≤ 2,700
10P1 + 20P2

(material A supply)

≤ 2,000 (material B supply)

P2

≤

80

(maximum time for P2)

P1 + P2

≤

120

(total maximum time)

P1, P2

≥

0

Step 4

Graph the problem
The graphical solution looks like this.

Step 5

Define feasible area
The material B constraint is not critical, and the feasible area for a solution is
shown as ABCDO on the graph.

Step 6

Determine optimal solution
The optimal solution, determined using the iso-contribution line 70P1 + 60P2 =
4,200, is at point A, where P1 = 120 and P2 = 0.
Production would be (120 × 15) 1,800 litres of X and (120 × 20) 2,400 litres of Y.
Total contribution would be (120 × RWF k70) = RWF k8,400 per week.

Page 219

Example
On 20 days of every month GS makes two products, the Crete and the Corfu. Production is
carried out in three departments – tanning, plunging and watering. Relevant information is as
follows.
Note: All RWFvalues are in thousands
Crete

Corfu

RWF75

RWF50

Minutes in tanning department per unit

10

12

Minutes in plunging department per unit

15

10

Minutes in watering department per unit

6

15

3,500

4,000

Plunging

Watering

Contribution per unit

Maximum monthly sales (due to government quota
restrictions)
Tanning
Number of employees

7

10

5

Hours at work per day per employee

7

6

10

Number of idle hours per day per employee

0.5

1

0.25

Due to various restrictions, employees cannot be at work for longer than the hours detailed
above.

Required
Use the graphical method of linear programming to determine the optimum monthly
production of Cretes and Corfus and the monthly contribution if GS's objective is to
maximise contribution.

Page 220

Solution
Calculate the number of productive hours worked in each department each month
Number of employees x number of productive hours worked each day x number of days each
month.
Tanning

=

7 x (7 – 0.5) × 20 = 910 hours

Plunging

=

10 x (6 – 1) × 20 = 1,000 hours

Watering

=

5 x (10 – 0.25) × 20 = 975 hours

Step 1

Define variables
Let the number of Cretes produced each month = x and the number of Corfus
produced each month = y.

Step 2

Establish objective function
The contribution is Rwf75 per Crete and Rwf50 per Corfu. The objective function
is therefore maximise C = 75x + 50y subject to the constraints below.

Step 3

Establish constraints
Tanning
x/6 + y/5

≤

910

Plunging

x/4 + y/6

≤

1,000

Watering

x/10 + y/4 ≤

Monthly sales units

x ≤ 3,500, y ≤ 4,000

Non negativity

x ≥ 0,

Step 4

975

y≥0

Graph the problem
The problem can be solved using the following graph which includes a sample
contribution line 75x + 50y = 150,000.

Page 221

Step 5

Define the feasible area
The feasible region for a solution is OABCDE.

Step 6

Determine the optimal solution
Moving the sample contribution line across the feasible region it can be seen that
the optimum solution is at any point along the line x/4 + y/6 = 1,000 between C
and D (as the sample contribution line has the same gradient as the plunging
constraint). The coordinates of point C are (2,175, 2,737.5) while those of point D
are (3,500, 750).
The contribution from any of these solutions is RWF((75 × 3,500) + (50 × 750))
= RWF300,000 (using the coordinates of D).

Page 222

THE GRAPHICAL METHOD USING
SIMULTANEOUS EQUATIONS
Instead of a 'sliding the contribution line out' approach, simultaneous equations can be used
to determine the optimal allocation of resources, as shown in the following example.

The optimal solution can also be found using simultaneous equations.

Example: using simultaneous equations
An organisation manufactures plastic-covered steel fencing in two qualities: standard and
heavy gauge. Both products pass through the same processes involving steel forming and
plastic bonding.
The standard gauge sells at RWF15,000 a roll and the heavy gauge at RWF20,000 a roll.
There is an unlimited market for the standard gauge but outlets for the heavy gauge are
limited to 13,000 rolls a year. The factory operations of each process are limited to 2,400
hours a year. Other relevant data is given below.

Variable costs per roll
Direct

Direct

Direct

material

wages

expense

RWF ‘000

RWF‘000

RWF ‘000

Standard

5

7

1

Heavy

7

8

2

Processing hours per 100 rolls
Steel

Plastic

forming

bonding

Hours

Hours

Standard

6

4

Heavy

8

12

Page 223

Required
Calculate the allocation of resources and hence the production mix which will maximise total
contribution.

Solution
Step 1

Define variables
Let the number of rolls of standard gauge to be produced be x and the number of
rolls of heavy gauge be y.

Step 2

Establish objective function
Standard gauge produces a contribution of RWF2,000 per roll RWFk15 – RWF
k(5 + 7 + 1)) and heavy gauge a contribution of RWFk3 (RWF k20 – RWF k(7 +
8 + 2)).
Therefore the objective is to maximise contribution (C) = 2x + 3y subject to the
constraints below.

Step 3

Step 4

Establish constraints
The constraints are as follows.
0.06x + 0.08y

≤

2,400

(steel forming hours)

0.04x + 0.12y

≤

2,400

(plastic bonding hours)

y

≤

13,000

(demand for heavy gauge)

x, y

≥

0

(non-negativity)

Graph problem
The graph of the problem can now be drawn.

Page 224

Step 5

Define feasible area
The combinations of x and y that satisfy all three constraints are represented by
the area OABCD.

Step 6

Determine optimal solution
Which combination will maximise contribution? Obviously, the more units of x
and y, the bigger the contribution will be, and the optimal solution will be at point
B, C or D. It will not be at A, since at A, y = 13,000 and x = 0, whereas at B, y =
13,000 (the same) and x is greater than zero.
Using simultaneous equations to calculate the value of x and y at each of points
B, C and D, and then working out total contribution at each point from this, we
can establish the contribution-maximising product mix.

Point B
y

= 13,000

(1)

0.04x + 0.12y = 2,400 (2)
0.12y

= 1,560 (3) ((1) × 0.12)

0.04x

= 840 (4) ((2) – (3))

x

= 21,000

(5)

Total contribution = (21,000 × RWF2,000) + (13,000 × RWF3,000) =
RWF81,000,000.

Point C
0.06x + 0.08y = 2,400 (1)
0.04x + 0.12y = 2,400 (2)
0.12x + 0.16y = 4,800 (3) ((1) × 2)
0.12x + 0.36y = 7,200 (4) ((2) × 3)
0.2y
y
0.06x + 960
x

= 2,400 (5) ((4) – (3))
= 12,000

(6)

= 2,400 (7) (substitute in (1))
= 24,000

(8)

Total contribution = (24,000 × RWF2,000) + (12,000 × RWF3,000) =
RWF84,000,000.
Page 225

Point D
Total contribution = 40,000 × RWF2,000 = RWF80,000,000.
Comparing B, C and D, we can see that contribution is maximised at C, by
making 24,000 rolls of standard gauge and 12,000 rolls of heavy gauge, to earn a
contribution of RWF k84,000.

Slack and surplus
Slack occurs when maximum availability of a resource is not used. Surplus occurs when
more than a minimum requirement is used.
If, at the optimal solution, the resource used equals the resource available there is no spare
capacity of a resource and so there is no slack.
If a resource which has a maximum availability is not binding at the optimal solution, there
will be slack.

In the example above, the optimal solution is x = 24,000, y = 12,000.
If we substitute these values into the inequalities representing the constraints, we can
determine whether the constraints are binding or whether there is slack.
Steel forming hours:

(0.06 × 24,000) + (0.08 × 12,000) = 2,400 = availability
Constraint is binding.

Plastic bonding hours: (0.04 × 24,000) + (0.12 × 12,000) = 2,400 = availability
Constraint is binding.
Demand:

Demand of 12,000 ≤ maximum demand of 13,000
There is slack.

Note that because we had already determined the optimal solution to be at the intersection of
the steel forming hours and plastic bonding hours constraints, we knew that they were
binding!

Page 226

If a minimum quantity of a resource must be used and, at the optimal solution, more than
that quantity is used, there is a surplus on the minimum requirement.
For example, suppose in a particular scenario a minimum of 8,000 grade A labour hours had
to be worked in the production of products x and y, such that (say) 3x + 2y ≥ 8,000. If 10,000
hours are used to produce the optimal solution, there is a surplus of 2,000 hours.
We will be looking at this form of constraint in the next section.

Page 227

BLANK

Page 228

SENSITIVITY ANALYSIS
Once a graphical linear programming solution has been found, it should be possible to
provide further information by interpreting the graph more fully to see what would happen if
certain values in the scenario were to change.
a) What if the contribution from one product was Rwf1 lower than expected?
b) What if the sales price of another product was raised byRwf2?
c) What would happen if less or more of a limiting factor were available, such as
material?
Sensitivity analysis with linear programming can be carried out in one of two ways.
a) By considering the value of each limiting factor or binding resource constraint
b) By considering sale prices (or the contribution per unit)

Limiting factor sensitivity analysis
We use the shadow price to carry out sensitivity analysis on the availability of a limiting
factor.

Shadow prices

The shadow price of a resource which is a limiting factor on production is the amount by
which total contribution would fall if the organisation were deprived of one unit of the
resource. The shadow price also indicates the amount by which total contribution would rise
if the organisation were able to obtain one extra unit of the resource, provided that the
resource remains an effective constraint on production and provided also that the extra unit of
resource can be obtained at its normal variable cost.

Page 229

So in terms of linear programming, the shadow price is the extra contribution or profit that
may be earned by relaxing by one unit a binding resource constraint.
Suppose the availability of materials is a binding constraint. If one extra kilogram becomes
available so that an alternative production mix becomes optimal, with a resulting increase
over the original production mix contribution of RWF2, the shadow price of a kilogram of
material is RWF.
Note, however, that this increase in contribution of RWF2 per extra kilogram of material
made available is calculated on the assumption that the extra kilogram would cost the
normal variable amount.

Note the following points.
a) The shadow price therefore represents the maximum premium above the basic rate
that an organisation should be willing to pay for one extra unit of a resource.
b) Since shadow prices indicate the effect of a one unit change in a constraint, they
provide a measure of the sensitivity of the result.
c) The shadow price of a constraint that is not binding at the optimal solution is zero.
d) Shadow prices are only valid for a small range before the constraint becomes nonbinding or different resources become critical.
Depending on the resource in question, shadow prices enable management to make better
informed decisions about the payment of overtime premiums, bonuses, premiums on small
orders of raw materials and so on.

Calculating shadow prices

In the earlier example of WX, the availability of time in both departments are limiting factors
because both are used up fully in the optimal product mix. Let us therefore calculate the
effect if one extra hour of shaping department machine time was made available so that
2,401 hours were available.
The new optimal product mix would be at the intersection of the two constraint lines
0.06x + 0.08y = 2,400 and 0.04x + 0.12y = 2,401.
Solution by simultaneous equations gives x = 23,980 and y = 12,015.
Page 230

(You should solve the problem yourself if you are doubtful about the derivation of the
solution.)
Contribution
per unit

Product

Total
contribution

Units

RWF

RWF

A

23,980

0.20

4,796.0

B

12,015

0.30

3,604.5
8,400.5

Contribution in original problem
((24,000 ×
RWF0.30))

RWF0.20)

+

(12,000

×

8,400.0

0.5

Increase in contribution from one extra hour of shaping time
The shadow price of an hour of machining time in the shaping department is therefore
RWF0.50.
The shadow price of a limiting factor also shows by how much contribution would fall if
the availability of a limiting resource fell by one unit. The shadow price (also called dual
price) of an hour of machine time in the shaping department would again be calculated as
RWF0.50. This is the opportunity cost of deciding to put an hour of shaping department
time to an alternative use.

We can now make the following points.
a) The management of WX should be prepared to pay up to RWF0.50 extra per hour
(i.e. RWF0.50 over and above the normal price) of shaping department machine time
to obtain more machine hours.
b) This value of machine time only applies as long as shaping machine time is a
limiting factor. If more and more machine hours become available, there will
eventually be so much machine time that it is no longer a limiting factor.
Page 231

Example
What is the shadow price of one hour of machine time in the mixing department?
A
B
C
D

RWF3
RWF7
RWF10.50
RWF1,193

Solution
The correct answer is A.
If we assume one less hour of machine time in the mixing department is available, the new
optimal solution is at the intersection of 0.06x + 0.08y = 2,399 and 0.04x + 0.12y = 2,400
Solution by simultaneous equations gives x = 23,970, y = 12,010

Product

Contribution

Total

per unit

contribution

Units

RWF

RWF

A

23,970

0.20

4,794

B

12,010

0.30

3,603
8,397

Contribution in original
problem

8,400

Reduction in contribution

3

∴ Shadow price of one hour of machine time in the mixing department is RWF3.

Page 232

Ranges for limiting factors
We can calculate how many hours will be available before machine time in the shaping
department ceases to be a limiting factor.
Look back at the third graph in Section 1.3. As more hours become available the constraint
line moves out away from the origin. It ceases to be a limiting factor when it passes through
the intersection of the sales constraint and the mixing department machine time constraint
which is at the point (22,667, 13,000).
So, if x = 22,667 and y = 13,000, our new constraint would be 0.04x + 0.12y = H (hours)
where H = (0.04 × 22,667) + (0.12 × 13,000) = 2,466.68 hours.
The shadow price of shaping department machine time is therefore RWF0.50 but only up to a
maximum supply of 2,466.68 hours (that is 66.68 hours more than the original 2,400 hours).
Extra availability of machine time above 2,466.68 hours would not have any use, and the two
limiting factors would become sales demand for product B and machine time in the mixing
department.

Sales price sensitivity analysis
Sales price sensitivity analysis is carried out by changing the slope of the 'iso-contribution'
line.
The optimal solution in our WX example was to make 24,000 units of product A and 12,000
units of product B. Would this solution change if the unit sales price of A increased by
10p?
The contribution would increase to 0.3x + 0.3y (in place of 0.2x + 0.3y). The isocontribution lines would now have a steeper slope than previously, parallel (for example)
to 0.3x + 0.3y = 3,000.

Page 233

If you were to place a ruler along the iso-contribution line and move it away from the origin
as usual, you would find its last point within the feasible region was the point (40,000, 0).
Therefore if the sales price of A is raised by RWF 0.10, WX's contribution-maximising
product mix would be to produce 40,000 units of A and none of B.

Example: sensitivity analysis
SW makes two products, X and Y, which each earn a contribution of RWF8 per unit. Each
unit of X requires four labour hours and three machine hours. Each unit of Y requires three
labour hours and five machine hours.
Total weekly capacity is 1,200 labour hours and 1,725 machine hours. There is a standing
weekly order for 100 units of X which must be met. In addition, for technical reasons, it is
necessary to produce at least twice as many units of Y as units of X.

Required
a) Determine the contribution-maximising production plan each week.
b) Calculate the shadow price of the following.
(i) Machine hours
(ii) Labour hours
(iii) The minimum weekly demand for X of 100 units
Page 234

Solution (a): production plan
The linear programming problem may be formulated as follows.

Step 1

Define variables
Let x = number of units of X produced and y = number of units of Y produced.

Step 2

Establish objective function
Maximise contribution (c) = 8x + 8y subject to the constraints below.

Step 3

Establish constraints

Step 4

4x + 3y

≤

1,200

(labour hours)

3x + 5y

≤

1,725

(machine hours)

x ≥

100

(minimum demand)

y ≥

2x

(technical constraint)

y ≥

0

(non-negativity)

Graph the problem
The graph of this problem would be drawn as follows, using 8x + 8y = 2,400 as
an iso-contribution line.

Page 235

Step 5

Establish feasible polygon
The feasible polygon is ABC. Using the slope of the iso-contribution line, we can
measure that the contribution-maximising point is point A.

Step 6

Determine optimal solution
At point A, the effective constraints are x = 100 and 4x + 3y = 1,200.
∴If x = 100, (4 × 100) + 3y = 1,200
∴3y = 1,200 – 400 and so y = 2662/3
It is important to be aware that in linear programming, the optimal solution is
likely to give values to the decision variables which are in fractions of a unit. In
this example, contribution will be maximised by making 2662/3 units of Y.

Contribution
RWF
Make 100 units of X

800.00

2662/3 units of Y

2,133.33

Total weekly contribution

2,933.33

Solution (b): sensitivity analysis
(i)

Machine hours are not fully utilised in the optimal solution. 100 units of X and 2662/3
units of Y need (300 + 1,333.33) = 1,633.33 machine hours, leaving 91.67 machine
hours unused. Machine hours, not being an effective constraint in the optimal solution,
have a shadow price of RWF0. Obtaining one extra machine hour would add nothing
to the contribution.

(ii)

The shadow price of labour hours would be obtained by calculating the total weekly
contribution if the labour hours constraint were 1,201 hours. It should be possible to see
fairly easily that the new optimal solution would be where x = 100 and 4x + 3y =
1,201. Therefore x = 100, y = 267 and total weekly contribution would be (100 + 267)
× RWF8 =RWF2,936.

Page 236

Since contribution with 1,200 labour hours as the constraint was RWF2,933.33, the
shadow price of labour hours is RWF(2,936 – 2,933.33) = RWF2.67 per hour. This is
the amount by which total contribution would rise if one extra labour hour per week
were made available.
Note that there is a limitation to the number of extra labour hours that could be used to
earn extra contribution. As more and more labour hours are added, the constraint line
will move further and further away from the origin. For example if we added 800
labour hours capacity each week, the constraint 4x + 3y ≤ (1,200 + 800) (i.e. 4x + 3y ≤
2,000) would be so much further away from the origin that it would no longer be an
effective constraint. Machine hours would now help to impose limitations on
production, and the profit-maximising output would be at point P on the graph.
Labour hours could only be added to earn more contribution up to point P, after which
they would cease to be an effective constraint. At point P, x = 100 and 3x + 5y = 1,725.
Therefore y = 285.
The labour hours required to make 100 units of X and 285 units of Y are (4 × 100) + (3
× 285) = 1,255 hours, which is 55 hours more than the initial constraint limit.
Total contribution at point P = (100 + 285) × RWF8 = RWF3,080. Since total
contribution at point A, where labour hours were limited to 1,200 hours, was
RWF2,933.33, the extra contribution from the 55 extra labour hours would be
RWF(3,080 – 2,933.33)/55 = RWF2.67 per hour (as calculated previously).
Thus, the shadow price of labour hours is RWF2.67 per hour, for a maximum of 55
extra hours per week, after which additional labour hours would add nothing to the
weekly contribution.
(iii) The shadow price of the minimum weekly demand for X may be obtained by
calculating the weekly contribution if the minimum demand is reduced by one unit to
99, so that x ≥ 99, given no change in the other original constraints in the problem.
The new optimal solution would occur where x = 99 and 4x + 3y = 1,200. Therefore y
= 268.
Total contribution per week when x = 99 and y = 268 is (99 + 268) × RWF8 =
RWF2,936. Since the contribution when x ≥ 100 was RWF2,933.33, the shadow price
of the minimum demand for X is RWF(2,936 – 2,933.33) = RWF2.67 per unit. In
other words, by reducing the minimum demand for X, the weekly contribution can be
raised by RWF2.67 for each unit by which the minimum demand is reduced below 100
per week.
Page 237

As with the constraint on labour hours, this shadow price is only applicable up to a
certain amount. If you refer back to the graph of the problem, you should be able to
see that if the minimum constraint on X is reduced beyond point Z, it will cease to be
an effective constraint in the optimal solution, because at point Z the machine hours
limitation will begin to apply.

Example
By how many units per week can the minimum demand be reduced before the shadow price
of RWF2.67 per unit referred to above ceases to apply?
A
B
C
D

300 units
100 units
75 units
25 units

Solution
The correct answer is D.
At point Z:

4x + 3y = 1,200

……. (1)

3x + 5y = 1,725

……. (2)

Multiply (1) by 3

12x + 9y = 3,600

……. (3)

Multiply (2) by 4

12x + 20y = 6,900

……. (4)

Subtract (3) from (4)

11y = 3,300
y = 300

Substituting in (1)

4x + 900 = 1,200
4x = 300
x = 75

The shadow price of the minimum demand for X is RWF2.67 per unit demanded, but only up
to a total reduction in the minimum demand of (100 – 75) = 25 units per week.

Page 238

CHAPTER ROUNDUP
•

The graphical method of linear programming is used for problems involving two
products.

•

The steps in the graphical method are as follows.
− Define variables.
−
−
−
−
−

Establish objective function.
Establish constraints.
Draw a graph of the constraints.
Establish the feasible region.
Determine the optimal product mix.

•

The optimal solution can be found by 'sliding the iso-contribution (or profit) line out'.

•

The optimal solution can also be found using simultaneous equations.

•

Slack occurs when maximum availability of a resource is not used. Surplus occurs
when more than a minimum requirement is used.

•

The shadow price of a resource which is a limiting factor on production is the
amount by which total contribution would fall if the organisation were deprived of
one unit of the resource. The shadow price also indicates the amount by which total
contribution would rise if the organisation were able to obtain one extra unit of the
resource, provided that the resource remains an effective constraint on production and
provided also that the extra unit of resource can be obtained at its normal variable
cost.

•

Sales price sensitivity analysis is carried out by changing the slope of the 'isocontribution' line.

Page 239

BLANK

Page 240

STUDY UNIT 10
Linear Programming: The Simplex Method
Contents

Page

The Principles Of The Simplex Method …………………………..……….

243

Sensitivity Analysis ……….…………………………………………..……..

257

Using Computer Packages ………………………………………………….

265

Using Linear Programming …………..………………………………….…

267

Page 241

BLANK

Page 242

THE PRINCIPLES OF THE SIMPLEX METHOD
The simplex method is a method of solving linear programming problems with two or more
decision variables.

The formulation of the problem using the simplex method is similar to that required when
the graphical method is used but slack variables must be incorporated into the constraints
and the objective function.

General points about the simplex method
A slack variable represents the amount of a constraint that is unused.
In any feasible solution, if a problem involves n constraints and m variables (decision plus
slack), n variables will have a positive value and (m–n) variables will have a value of zero.
Feasible solutions to a problem are shown in a table.

Before introducing an example to explain the technique, we will make a few introductory
points. Don't worry if you get confused, working through the example will make things
clearer.
a) The simplex method involves testing one feasible solution after another, in a
succession of tables, until the optimal solution is found. It can be used for problems
with any number of decision variables, from two upwards.
b) In addition to the decision variables, the method introduces additional variables,
known as slack variables or surplus variables. There will be one slack (or surplus)
variable for each constraint in the problem (excluding non-negativity
constraints).
For example, if a linear programming problem has three decision variables and four
constraints, there will be four slack variables. With the three decision variables, there
will therefore be a total of seven variables and four constraints in the problem.
c) The technique is iterative (a repetitive, step-by-step process), with each iteration
(step) having the following purposes.
Page 243

(i)

To establish a feasible solution (in other words, a feasible combination of
decision variable values and slack variable values) and the value of the objective
function for that solution.

(ii)

To establish whether that particular solution is one that optimises the value of
the objective function.

d) Each feasible solution is tested by drawing up a matrixwith the following rows and
columns.
(i)

One row per constraint, plus a solution row

(ii)

One column per decision variable and per slack variable, plus a solution
column

e) Every variable, whether a decision variable, slack variable or surplus variable, must
be ≥ 0 in any feasible solution.
f) A feature of the simplex method is that if there are n constraints, there will be n
variables with a value greater than 0 in any feasible solution. Thus, if there are
seven variables in a problem, and four constraints, there will be four variables with a
positive value in the solution, and three variables with a value equal to 0.
Keep these points in mind as we work through an example.

Example: the simplex method
An organisation produces and sells two products, X and Y. Relevant information is as
follows.
Contribution
Materials

Labour

Machine
time

per unit

units

hours

hours

RWF

X, per unit

5

1

3

20

Y, per unit

2

3

2

16

3,000

1,750

2,100

Total available, each
week

Page 244

Required
Use the simplex method to determine the profit-maximising product mix.

Formulating the problem
We have just two decision variables in this problem, but we can still use the simplex method
to solve it.
Step 1

Define variables
Let x be the number of units of X that should be produced and sold.
Let y be the number of units of Y that should be produced and sold.

Step 2

Establish objective function
Maximum contribution (C) = 20x + 16y subject to the constraints below.

Step 3

Establish constraints
The constraints are as follows.
Materials
Labour

Step 4

5x + 2y ≤ 3,000
x + 3y ≤ 1,750

Machine time 3x + 2y ≤ 2,100
Non-negativity x ≥ 0, y ≥ 0

Introduce slack variables
Begin by turning each constraint (ignoring the non-negativity constraints now)
into an equation. This is done by introducing slack variables.
Let a be the quantity of unused materials, b be the number of unused labour hours
and c be the number of unused machine hours.

Slack variable. ‘Amount of each resource which will be unused if a specific linear
programming solution is implemented.’

Page 245

Example
A problem to be solved using linear programming has three decision variables, six constraints
(including two non-negativity constraints) and one objective function.
How many slack variables will be required if the simplex method is used?
A
B
C
D

3
4
5
6

Answer

The correct answer is B.
A slack variable is required for each constraint (ignoring non-negativity constraints). There
are 6 – 2 = 4 such constraints.

We can now express the original constraints as equations.
5x + 2y + a = 3,000
x + 3y + b = 1,750
3x + 2y + c = 2,100
The slack variables a, b and c will be equal to 0 in the final solution only if the
combined production of X and Y uses up all the available materials, labour hours
and machine hours.

Step 5

Values of variables – non-negative or zero?
In this example, there are five variables (x, y, a, b and c) and three equations,
and so in any feasible solution that is tested, three variables will have a nonnegative value (since there are three equations) which means that two variables
will have a value of zero.

Page 246

Example
A problem to be solved using linear programming has seven variables and four equations
based on the original constraints.
How many variables will have a value of zero in any feasible solution determined using the
simplex method?
A
B
C
D

7
5
4
3

Answer
The correct answer is D.
Four variables will have a non-negative value (since there are four equations), which means
that 7 – 4 = 3 variables will have a value of zero.

Step 6

Express objective function as an equation
It is usual to express the objective function as an equation with the right hand side
equal to zero. In order to keep the problem consistent, the slack (or surplus)
variables are inserted into the objective function equation, but as the quantities
they represent should have no effect on the objective function they are given zero
coefficients. In our example, the objective function will be expressed as follows.
Maximise contribution (C) given by C – 20x – 16y + 0a + 0b + 0c = 0.

Page 247

Drawing up the initial table and testing the initial feasible solution
You will not be required to do this in the exam but seeing how the initial table is drawn up
will give you additional insight into the technique.

We begin by testing a solution that all the decision variables have a zero value, and all the
slack variables have a non-negative value.
Obviously, this is not going to be the optimum solution, but it gives us a starting point from
which we can develop other feasible solutions.
Simplex tables can be drawn in several different ways, and if you are asked to interpret a
given table in an examination question, you may need to adapt your understanding of the
table format in this Study Text to the format in the question. The following points apply to all
tables, however.
a) There should be a column for each variable and also a solution column.
b) It helps to add a further column on the left, to indicate the variable which is in the
solution to which the corresponding value in the solution column relates.
c) There is a row for each equation in the problem, and a solution row.
Here is the initial matrix for our problem. Information on how it has been derived is given
below.
Variables in solution

x

y

a

b

c

Solution

A materials

5

2

1

0

0

3,000

B Labour hours

1

3

0

1

0

1,750

C Machine hours

3

2

0

0

1

2,100

–20

–16

0

0

0

0

Solution

a) The figures in each row correspond with the coefficients of the variables in each of
the initial constraints. The bottom row or solution row holds the coefficients of the
objective function. For example the materials constraint 5x + 2y + a = 3,000 gives us
the first row, 5 (number of x's), 2 (number of y's), 1 (number of a's), then zeros in the
b and c columns (since these do not feature in the constraint equation) and finally
3,000 in the solution column.
Page 248

b) The variables in the solution are a, b and c (the unused resources).
(i)

The value of each variable is shown in the solution column. We are testing a
solution that all decision variables have a zero value, so there is no production
and hence no resources are used. The total resource available is therefore unused.

(ii)

The column values for each variable in the solution are as follows.
− 1 in the variable's own solution row
− 0 in every other row, including the solution row.

c) The contribution per unit obtainable from x and y is given in the solution row.
These are the dual prices or shadow prices of the products X and Y. The minus signs
are of no particular significance, except that in the solution given here they have the
following meanings.
(i)

A minus shadow price indicates that the value of the objective function can be
increased by the amount of the shadow price per unit of the variable that is
introduced into the solution, given no change in the current objective function or
existing constraints.

(ii)

A positive shadow price indicates the amount by which the value of the
objective function would be decreased per unit of the variable introduced into
the solution, given no change in the current objective function or the existing
constraints.

Interpreting the table and testing for improvement
We can see that the solution is testing a = 3,000, b = 1,750 and c = 2,100, contribution = 0.
The co-efficients for the variables not in this solution, x and y, are the dual prices or
shadow prices of these variables, given the solution being tested. A negative value to a dual
price means that the objective function can be increased; therefore the solution in the table
is not the optimal solution.
The shadow prices in the initial solution (table) indicate the following.

a) The profit would be increased by RWF20 for every extra unit of x produced (because
the shadow price of x is RWF20 per unit).
Page 249

b) Similarly, the profit would be increased by RWF16 for every extra unit of y produced
(because its shadow price is RWF16 per unit).
Since the solution is not optimal, the contribution may be improved by introducing
either x or y into the solution.

The next step
The next step is to test another feasible solution. We do this by introducing one variable
into the solution, in the place of one variable that is now removed. In our example, we
introduce x or y in place of a, b or c.
The simplex technique continues in this way, producing a feasible solution in each successive
table, until the optimal solution is reached.

Interpreting the final table
If the shadow prices on the bottom (solution) row of a table are all positive, the table shows
the optimal solution.
•

The solution column shows the optimal production levels and the units of unused
resource.

•

The figure at the bottom of the solution column/right-hand side of the solution row
shows the value of the objective function.

•

The figures in the solution row indicate the shadow prices of resources.

Page 250

After a number of iterations, the following table is produced.
Variables in solution

x

y

a

b

c

Solution
column

X

1

0

0

–
0.2857

0.4286

400

A

0

0

1

0.5714

–
1.8571

100

y

0

1

0

0.4286

–
0.1429

450

Solution row

0

0

0

1.1428

6.2858

15,200

This can be interpreted as follows.
a) The solution in this table is the optimum one, because the shadow prices on the
bottom row are all positive.
b) The optimal solution is to make and sell 400 units of X and 450 units of Y, to earn
a contribution of RWF15,200.
c) The solution will leave 100 units of material unused, but will use up all available
labour and machine time.
d) The shadow price of labour time (b) is RWF1.1428 per hour, which indicates the
amount by which contribution could be increased if more labour time could be
made available at its normal variable cost.
e) The shadow price of machine time (c) is RWF6.2858 per hour, which indicates
the amount by which contribution could be increased if more machine time could
be made available, at its normal variable cost.
f) The shadow price of materials is nil, because there are 100 units of unused
materials in the solution.

Page 251

Question
TDS manufactures two products, X and Y, which earn a contribution of RWF8 and RWF14
per unit respectively. At current selling prices, there is no limit to sales demand for Y, but
maximum demand for X would be 1,200 units. The company aims to maximise its annual
profits, and fixed costs are RWF15,000 per annum.
In the year to 30 June 2012, the company expects to have a limited availability of resources
and estimates of availability are as follows.
Skilled labour

maximum 9,000 hours

Machine time

maximum 4,000 hours

Material M

maximum 1,000 tonnes

The usage of these resources per unit of product are as follows.
X

Y

Skilled labour time

3 hours

4 hours

Machine time

1 hour

2 hours

Material M

½ tonne

¼ tonne

Required
a) Formulate the problem using the simplex method of linear programming.
b) Determine how many variables will have a positive value and how many a value of
zero in any feasible solution.

Page 252

Solution

a) The linear programming problem would be formulated as follows.
Define variables
Let x and y be the number of units made and sold of product X and product Y
respectively.
Establish objective function
Maximise contribution (C) = 8x + 14y subject to the constraints below.
Establish constraints
3x + 4y

≤

9,000 (skilled labour)*

x + 2y

≤

4,000 (machine time)

0.5x + 0.25y

≤

1,000 (material M)

x

≤

1,200 (demand for X)

x, y ≥

0

* This constraint is that skilled labour hours cannot exceed 9,000 hours, and since a unit
of X needs 3 hours and a unit of Y needs 4 hours, 3x + 4y cannot exceed 9,000. The
other constraints are formulated in a similar way.
Introduce slack variables
Introduce a slack variable into each constraint, to turn the inequality into an equation.
Let

a = the number of unused skilled labour hours
b = the number of unused machine hours
c = the number of unused tonnes of material M
d = the amount by which demand for X falls short of 1,200 units

Then
3x + 4y + a =

9,000

(labour hours)

x + 2y + b

4,000

(machine hours)

= 1,000

(tonnes of M)

=

0.5x + 0.25y + c

Page 253

x+d =

1,200

(demand for X)

and maximise contribution (C) given by C – 8x – 14y + 0a + 0b + 0c + 0d = 0

b) There are six variables (x, y, a, b, c, d) and four equations. In any feasible solution
four variables will have a non-negative value (as there are four equations), while two
variables will have a value of zero.

Example
The final table to the problem in Question: formulation of problem is shown below.
Required
Interpret the table.
Variables
solution

in

the x

y

a

b

c

d

Solution
column

x

1

0

0

0

0

1,000

y

0

1

–0.5

1.5 0

0

1,500

c

0

0

–0.375

0.625 1

0

125

d

0

0

–1

2

0

1

200

Solution row

0

0

1

5

0

0

29,000

–2

Answer
There is a column in the table for every variable, including the slack variables, but the
important parts of the table are the 'Variables in the solution' column, the Solution row, and
the Solution column. These tell us a number of things.

Page 254

Identifying the variables in the solution
The variables in the solution are x, y, c and d. It follows that a and b have zero values. To be
the variable in the solution on a particular row of the table, a value of 1 must appear in the
column for that variable, with zero values in every other row of that column. For example, x
is the variable in the solution for the row which has 1 in the x column. There are zeros in
every other row in the x column.

The value of the variables
The solution column gives the value of each variable.
x
y
c
d

1,000
1,500
125
200

(units made of X)
(units made of Y)
(unused material M)
(amount below the 1,200 maximum of demand for X)

This means that contribution will be maximised by making and selling 1,000 units of X and
1,500 units of Y. This will leave 125 unused tonnes of material M, and production and sales
of X will be 200 units below the limit of sales demand. Since a and b are both zero, there is
no unused labour and machine time; in other words, all the available labour and machine
hours will be fully utilised.

The total contribution
The value of the objective function – here, the total contribution – is in both the solution row
and the solution column. Here it is RWF29,000.

Shadow prices
The solution row gives the shadow prices of each variable. Here, the shadow price of a is
RWF1 per labour hour and that for b is RWF5 per machine hour.
This means that if more labour hours could be made available at their normal variable cost
per hour, total contribution could be increased by RWF1 per extra labour hour. Similarly, if
more machine time could be made available, at its normal variable cost, total contribution
could be increased by RWF5 per extra machine hour. Here is the final table of a problem
involving the production of products X and Y solved using the simplex method of linear
programming.
Page 255

Variables
solution

in x

y

a

b

c

d

e

x

1

0

–2.0

0

y

0

1

–0.8

b

0

0

d

0

e
Solution row

3.0

0

0

550

0

0.5

0

0

720

1.5

1

1.0

0

0

95

0

0.7

0

–1.1

1

0

50

0

0

2.0

0

1.8

0

1

104

0

0

7.0

0

4.0

0

0

14,110

Page 256

Solution
column

SENSITIVITY ANALYSIS
You might be asked to carry out some sensitivity analysis on a simplex matrix giving the
optimal solution to a linear programming problem. This could involve the following.
a) Testing how the optimal solution would change if there were either more or less of
a scarce resource.
b) Testing whether it would be worthwhile obtaining more of a scarce resource by
paying a premium for the additional resources, for example by paying an overtime
premium for extra labour hours, or by paying a supplier a higher price for extra raw
materials.

The effect of having more or less of a scarce resource
Sensitivity analysis can be applied to the final matrix to determine the effect of having more
or less of a scarce resource (indicated by figures in the column for the resource's slack
variable).
The optimal solution to a linear programming problem is based on the assumption that the
constraints are known with certainty, and fixed in quantity. Sensitivity analysis enables us to
test how the solution would alter if the quantity of a scarce resource (the size of a constraint)
were to change.

Example: the effect of having more or less of a scarce resource
Return to our previous example, and the optimal solution in section named ‘Interpreting the
final tableau’ , in which both labour hours and machine hours are fully used. How would the
solution change if more labour hours (variable b) were available?
Solution
The simplex matrix, and in particular the figures in the b column, provide the following
information for each extra labour hour that is available.
a) The contribution would increase by RWF1.1428
Page 257

b) The value of x would fall by 0.2857 units
c) The value of a (unused materials) would increase by 0.5714 units
d) The value of y would increase by 0.4286 units

In other words, we would be able to make 0.4286 units of Y extra, to earn contribution of (×
RWF16) RWF6.8576, but we would make 0.2857 units less of X and so lose contribution of
(× RWF20) RWF5.714, leaving a net increase in contribution of RWF(6.8576 – 5.714)
=RWF1.1436. Allowing for rounding errors of RWF0.0008, this is the figure already given
above for the increase in contribution.
Since x = 400 in the optimal table, and extra labour hours would lead to a reduction of 0.2857
units of x, there is a limit to the number of extra labour hours that would earn an extra
RWF1.1428. This limit is calculated as 400/0.2857 = 1,400 extra labour hours.
In other words, the shadow price of RWF1.1428 per hour for labour is only valid for about
1,400 extra labour hours on top of the given constraint in the initial problem, which was
1,750 hours, (that is up to a total limit of 3,150 hours).
If there were fewer labour hours available, the same sort of analysis would apply, but in
reverse.
a)
b)
c)
d)

The contribution would fall by RWF1.1428 per hour unavailable
The value of x would increase by 0.2857 units
The value of a would fall by 0.5714 units
The value of y would fall by 0.4286 units

Example: obtaining extra resources at a premium on cost
Sensitivity analysis can also be applied to test whether or not it would be worthwhile to
obtain more of a scarce resource by paying a premium for additional supplies (only if the
shadow price is greater than the additional cost).
Suppose we are given the following additional information about our example.
a) The normal variable cost of labour hours (variable b) isRwf4 per hour, but extra
labour hours could be worked in overtime, when the rate of pay would be time-and-ahalf.
Page 258

b) The normal variable cost of machine time is RWF1.50 per hour, but some extra
machine time could be made available by renting another machine for 40 hours per
week, at a rental cost of RWF160. Variable running costs of this machine would be
RWF1.50 per hour.
Would it be worth obtaining the extra resources?

Solution
We know that the shadow price of labour hours is RWF1.1428 and of machine hours is
RWF6.2858. We can therefore deduce the following.
a) Paying an overtime premium of RWF2 per hour for labour would not be
worthwhile, because the extra contribution of RWF1.1428 per hour would be more
than offset by the cost of the premium, leaving the company worse off by RWF0.8572
per hour worked in overtime.
b) Renting the extra machine would be worthwhile, but only by RWF91.43 (which is
perhaps too small an amount to bother with).
c)
V

RWF

Extra contribution from 40 hours of machine time (×RWF6.2858)

251.43

Rental cost

160.00

Net increase in profit

91.43

Note that the variable running costs do not enter into this calculation since they are identical
to the normal variable costs of machine time. We are concerned here only with the
additional costs.

Page 259

Example
An organisation manufactures three products, tanks, trays and tubs, each of which passes
through three processes, X, Y and Z.
Process hours per unit
Tanks
Trays

Tubs

X

5

2

4

12,000

Y

4

5

6

24,000

Z

3

5

4

18,000

Process

Total process
hours
available

The contribution to profit of each product is RWF2 for each tank, RWF3 per tray and RWF4
per tub.

Required
Fill in the blanks in (a) and (b) below, which relate to the formulation of the above data
into a simplex linear programming model. Use the following notation.
Let

a be the number of units of tanks produced
b be the number of units of trays produced
c be the number of units of tubs produced
x = quantity of unused process X hours
y = quantity of unused process Y hours
z = quantity of unused process Z hours

a) Maximise contribution (C) given by …………………………. subject to the
following constraints in (b).
b) …………………………. (process X hours)
…………………………. (process Y hours)
Page 260

…………………………. (process Z hours)

Answer
a) C is given by C – 2a – 3b – 4c + 0x + 0y + 0z
b) Constraint for process X hours: 5a + 2b + 4c + x = 12,000
Constraint for process Y hours: 4a + 5b + 6c + y = 24,000
Constraint for process Z hours: 3a + 5b + 4c + z = 18,000

Example
The final simplex table, based on the data in the question above, look like this.
Variables
solution

b

c

c

1.583 0

1

0.417 0

–0.167

2,000

y

–2.167 0

0

–0.833 1

–0.667

2,000

b

–0.667 1

0

–0.333 0

0.333

2,000

2.333 0

0

0.667 0

0.333

14,000

Solution row

in a

x

y

z

Solution
column

Required
a) Determine how many of each product should be produced and the maximum
contribution. Calculate how much slack time, if any, is available in the processes.
b) Explain how your solution would vary if an extra 3,000 hours of process X time could
be made available.
c) Describe what would happen to the production schedule and budgeted contribution if
an order were received for 300 units of tanks which the company felt that it had to
Page 261

accept, because of the importance of the customer. Ignore the increase of process X
time in part (b) above.

Solution
a) Contribution is maximised at RWF14,000 by making 2,000 units of tubs and 2,000
units of trays. No tanks would be made.
There will be 2,000 slack hours in process Y. Process X and process Z hours will be
fully utilised.
b) The shadow price of process X time is RWF0.667 per hour, and for every extra hour
of process X time that can be made available (at its normal variable cost), the
production quantities could be altered in such a way that the following would happen.
(i)

Contribution would go up by Rwf0.667 per extra process X hour used.

(ii)

c (the quantity of tubs) would go up by 0.417 units.

(iii) b (the quantity of trays) would go down by 0.333 units.
(iv) y (unused process Y time) would fall by 0.833 hours.

This is only true up to the point where so many extra process X hours have been made
available that either b or y reaches 0 in value. This will be at the following points.
(i)

For y, after

(ii)

For b, after

2, 000
0.833
2, 000
0.333

= 2,400 extra process X hours
= 6,000 extra process X hours

2,400 is the lowest of these two limits.
The shadow price is therefore valid only for up to 2,400 extra process X hours, so
that the full 3,000 available would not be required.
The new optimal solution would therefore be to make and sell the following.
c

2,000 + (2,400 × 0.417) = 3,000 units

b

2,000 – (2,400 × 0.333) = 1,200 units
Page 262

These would require a total of 14,400 hours in process X, 24,000 hours in process Y
and 18,000 hours in process Z.
Contribution would be as follows.
RWF
Tubs 3,000 × RWF4

12,000

Trays 1,200 × RWF3

3,600
15,600

Contribution in initial solution

14,000

Increase in contribution (2,400 × RWF0.667)

1,600

c) Going back to the original solution, if an order is received for 300 units of tanks, the
production schedule would be re-arranged so that for each unit of tank made the
following would happen.
(i)

Contribution would fall by RWF2.333.

(ii)

1.583 units less of tubs (variable c) would be made.

(iii) 0.667 units more of trays (variable b) would be made.
(iv) Unused process Y time would increase by 2.167 hours.

Page 263

The new production and contribution budget would be as follows.

Product

Process X

Process Y

Process Z

time

time

time

Contribution

Units

Hours

Hours

Hours

RWF

Tanks

(a)

300

1,500

1,200

900

600

Trays

(b)

2,200*

4,400

11,000

11,000

6,600

1,525**

6,100

9,150

6,100

6,100

12,000

21,350

18,000

13,300

Tubs (c)

*

2,000 + (300 × 0.667)

**

2,000 – (300 × 1.583)

The contribution is RWF700 lower than in the original optimal solution (which
represents 300 tanks × RWF2.333).
Unused process Y time is 2,650 hours, which is 650 more than in the original solution
(which represents 300 × 2.167)

Page 264

USING COMPUTER PACKAGES
Spreadsheet packages can be used to solve linear programming problems.
•

The slack/surplus columns provide information about the slack values of ≤
constraints and the surplus values of any ≥ constraints.

•

The worth column shows the positive shadow price of resources.

•

The relative loss shows by how much contribution (usually) would fall if extra units
of particular decision variables were produced.

Nowadays, modern spreadsheet packages can be used to solve linear programming problems.
Suppose an organisation produces three products, X and Y and Z, subject to four constraints
(1, 2, 3, 4).
a) Constraints 1 and 2 are 'less than or equal to' resource constraints.
b) Constraint 3 provides a limit on the number of X that can be produced.
c) Constraint 4 is a 'greater than or equal to' constraint and provides for a
minimum number of Z to be produced (400).

Page 265

The organisation wishes to maximise contribution.
Typical output from a spreadsheet package for such a problem is shown below.

Objective function (c)

137,500

Variable

Value

Relative loss

x

475.000

0.000

y

0.000

105.000

z

610.000

0.000

Constraint

Slack/surplus

Worth

1

17.000

0.000

2

0.000

290.000

3

0.000

1,150.000

4

210.000

0.000

Interpretation
a) Total optimal contribution (c) will be RWF137,500.
b) The variable and value columns mean that x = 475, y = 0 and z = 610.
To maximise contribution, 475 units of X and 610 units of Z should therefore be
produced. No units of Y should be produced.
c) The constraint and slack/surplus columns provide information about the slack
values of 'less than or equal to' constraints and the surplus values for any 'greater than
or equal to' constraints.
(i)

Constraint 1 is a 'less than or equal to' resource constraint. The slack is 17
and so 17 units of resource 1 will be unused in the optimal solution.

Page 266

(ii)

Constraint 2 is a 'less than or equal to' resource constraint. The slack is
zero, indicating that all available resource 2 will be used in the optimal
solution.
(iii) Constraint 3 provides a limit on x. The slack is zero, showing that the limit
has been met.
(iv) Constraint 4 provides for a minimum z. The surplus is 210, meaning 400 +
210 = 610 units of Z are made.
d) Worth. This column shows the positive shadow price of resources (the amount that
contribution (or, in general terms, c) alters if the availability of the resource is
changed by one unit).
(i)

Contribution would increase by RWF290 if one extra unit of resource 2
were made available.
(ii) Contribution would increase by RWF1,150 if the limit on the minimum
number of Z to be produced altered by 1.
(iii) Resource 1 has a worth of 0 because 17 units of the resource are unused in
the optimal solution.

In general, any constraint with a slack of zero has a positive worth figure, while any
constraint with a positive slack figure will have a worth of zero.

e) Relative loss. This indicates that if one unit of Y were produced, total contribution (or
generally c) would fall by RWF105. A relative loss of RWF105 would therefore be
made for every unit of Y made. Units of Y should only be made if unit contribution of
Y increases by RWF105.
X and Z have relative losses of zero, indicating that they should be made.

In general, only those decision variables with a relative loss of zero will have a positive
value in the optimal solution.

Page 267

BLANK

Page 268

USING LINEAR PROGRAMMING
There are a number of assumptions and practical difficulties in the use of linear
programming.
The considerations, non-quantifiable factors and assumptions in limiting factor analysis that
we looked at in Chapter 8 apply equally to linear programming.

Further assumptions
In addition, there are further assumptions if we are dealing with product mix decisions
involving several limiting factors.
a) The total amount available of each scarce resource is known with accuracy.
b) There is no interdependence between the demand for the different products or
services, so that there is a completely free choice in the product or service mix
without having to consider the consequences for demand or selling prices per unit.

In spite of these assumptions, linear programming is a useful technique in practice. Some
statistical studies have been carried out suggesting that linear cost functions do apply over
fairly wide ranges of output, and so the assumptions underlying linear programming may be
valid.

Uses of linear programming
a) Budgeting. If scarce resources are ignored when a budget is prepared, the budget is
unattainable and is of little use for planning and control. When there is more than one
scarce resource, linear programming can be used to identify the most profitable use of
resources.
b) Calculation of relevant costs. The calculation of relevant costs is essential for
decision making. The relevant cost of a scarce resource is calculated as acquisition
cost of the resource plus opportunity cost. When more than one scarce resource
exists, the opportunity cost (or shadow price) should be established using linear
programming techniques.
Page 269

c) Selling different products. Suppose that an organisation faced with resource
constraints manufactures products X and Y and linear programming has been used to
determine the shadow prices of the scarce resources. If the organisation now wishes to
manufacture and sell a modified version of product X (Z), requiring inputs of the
scarce resources, the relevant costs of these scarce resources can be determined (see
above) to ascertain whether the production of X and Y should be restricted in order to
produce Z.
d) Maximum payment for additional scarce resources. This use of shadow prices has
been covered in this chapter.
e) Control. Opportunity costs are also important for cost control: standard costing can
be improved by incorporating opportunity costs into variance calculations. For
example, adverse material usage variances can be an indication of material wastage.
Such variances should be valued at the standard cost of the material plus the
opportunity cost of the loss of one scarce unit of material. Such an approach
highlights the true cost of the inefficient use of scarce resources and encourages
managers of responsibility centres to pay special attention to the control of scarce
factors of production. For organisations using an optimised production technology
(OPT) strategy, this approach is particularly useful because variances arising from
bottleneck operations will be reported in terms of opportunity cost rather than
purchase cost.
f) Capital budgeting. Linear programming can be used to determine the combination of
investment proposals that should be selected if investment funds are restricted in more
than one period.

Practical difficulties with using linear programming
Difficulties with applying the linear programming technique in practice include the
following.
a) It may be difficult to identify which resources are likely to be in short supply and
what the amount of their availability will be.
With linear programming, the profit-maximising product mix and the shadow price of
each limiting factor depend on the total estimated availability of each scarce resource.
So it is not sufficient to know that labour hours and machine hours will be in short
supply, it is also necessary to guess how many labour hours and machine hours will be
Page 270

available. Estimates of future availability will inevitably be prone to inaccuracy and any
such inaccuracies will invalidate the profit-maximising product mix derived from the
use of linear programming.
b) Management may not make product mix decisions which are profit-maximising.
They may be more concerned to develop a production/sales plan which has the
following features.
(i)
(ii)
(iii)
(iv)

Realistic
Acceptable to the individual managers throughout the organisation
Acceptable to the rest of the workforce
Promises a 'satisfactory' profit and accounting return

In other words, management might look for a satisfactory product mix which achieves
a satisfactory return, sales revenue and market share whilst at the same time plans
operations and targets of achievement which employees can accept as realistic, not too
demanding or unreasonable, and not too threatening to their job security.
If a 'satisfactory' output decision is adopted, the product mix or service mix
recommended by the linear programming (profit-maximising) technique will
inevitably be 'watered down', amended or ignored.
c) The assumption of linearity may be totally invalid except over smaller ranges.
For example, in a profit maximisation problem, it may well be found that there are
substantial changes in unit variable costs arising from increasing or decreasing returns
to scale.
d) The linear programming model is essentially static and is therefore not really suitable
for analysing in detail the effects of changes in the various parameters, for example
over time.
e) In some circumstances, a practical solution derived from a linear programming model
may be of limited use as, for example, where the variables may only take on integer
values. A solution must then be found by a combination of rounding up and trial and
error.
f) The shadow price of a scarce resource only applies up to a certain limit.

Page 271

CHAPTER ROUNDUP
•

The formulation of the problem using the simplex method is similar to that required
when the graphical method is used but slack variables must be incorporated into the
constraints and the objective function.

•

A slack variable represents the amount of a constraint that is unused.

•

In any feasible solution, if a problem involves n constraints and m variables (decision
plus slack), n variables will have a positive value and (m–n) variables will have a
value of zero.

•

Feasible solutions to a problem are shown in a matrix table.

•

If the shadow prices on the bottom (solution) row of a table are all positive, the table
shows the optimal solution.
− The solution column shows the optimal production levels and the units of
unused resource.
− The figure at the bottom of the solution column/right-hand side of the solution
row shows the value of the objective function.
− The figures in the solution row indicate the shadow prices of resources.

•

Sensitivity analysis can be applied to the final matrix to determine the effect of
having more or less of a scarce resource (indicated by figures in the column for the
resource's slack variable).

•

Sensitivity analysis can also be applied to test whether or not it would be worthwhile
to obtain more of a scarce resource by paying a premium for additional supplies
(only if the shadow price is greater than the additional cost).

•

Spreadsheet packages can be used to solve linear programming problems.
− The slack/surplus columns provide information about the slack values of ≤
constraints and the surplus values of any ≥ constraints.
− The worth column shows the positive shadow price of resources.
− The relative loss shows by how much contribution (usually) would fall if
extra units of particular decision variables were produced.

•

There are a number of assumptions and practical difficulties in the use of linear
programming.
Page 272

STUDY UNIT 11
Risk And Uncertainty
Contents

Page

Risk And Uncertainty ……………………………………………………….

275

Allowing For Uncertainty ………………………..…………………..……..

277

Probabilities And Expected Values …………………………………….….

281

Decision Rules ……………………………………………………….………

285

Decision Trees ……………………………………………………….………

293

The Value Of Information ………………………………………………….

303

Sensitivity Analysis ………………………………………………….………

315

Simulation Methods ………………….……………………………..………

317

Page 273

EXAM GUIDE
Management accounting exams have increasingly expected candidates to have a good
understanding of risk. Questions are likely to be a mixture of calculations and explanation.

Page 274

RISK AND UNCERTAINTY
An example of a risky situation is one in which we can say that there is a 70% probability
that returns from a project will be in excess of RWF100 million but a 30% probability that
returns will be less than RWF100 million. If we cannot predict an outcome or assign
probabilities, we are faced with an uncertain situation.
Risk involves situations or events which may or may not occur, but whose probability of
occurrence can be calculated statistically and the frequency of their occurrence predicted
from past records. Thus insurance deals with risk.
Uncertain events are those whose outcome cannot be predicted with statistical confidence.

In everyday usage the terms risk and uncertainty are not clearly distinguished. If you are
asked for a definition, do not make the mistake of believing that the latter is a more extreme
version of the former. It is not a question of degree, it is a question of whether or not
sufficient information is available to allow the lack of certainty to be quantified. As a rule,
however, the terms are used interchangeably.

Risk preference
People may be risk seekers, risk neutral or risk averse.

A risk seeker is a decision maker who is interested in the best outcomes no matter how small
the chance that they may occur.
A decision maker is risk neutral if s/he is concerned with what will be the most likely
outcome.
A risk averse decision maker acts on the assumption that the worst outcome might occur.

Page 275

This has clear implications for managers and organisations. A risk seeking manager
working for an organisation that is characteristically risk averse is likely to make decisions
that are not congruent with the goals of the organisation. There may be a role for the
management accountant here, who could be instructed to present decision-making
information in such a way as to ensure that the manager considers all the possibilities,
including the worst.

Page 276

ALLOWING FOR UNCERTAINTY
Management accounting directs its attention towards the future and the future is uncertain.
For this reason a number of methods of taking uncertainty into consideration have evolved.

Research techniques to reduce uncertainty
Market research can be used to reduce uncertainty.

Market research is the systematic process of gathering, analysing and reporting data about
markets to investigate, describe, measure, understand or explain a situation or problem facing
a company or organisation.

Market research involves tackling problems. The assumption is that these problems can be
solved, no matter how complex the issues are, if the researcher follows a line of enquiry in a
systematic way, without losing sight of the main objectives. Gathering and analysing all the
facts will ultimately lead to better decision making.

The role of market research
In the last 20 years or so market research has become a much more widespread activity.
Organisations – in the private sector, the public sector and the not-for-profit sector – rely on
research to inform and improve their planning and decision making.
Market research enables organisations to understand the needs and opinions of their
customers and other stakeholders. Armed with this knowledge they are able to make better
quality decisions and provide better products and better services.
Thus, research influences what is provided and the way it is provided. It reduces uncertainty
and monitors performance. A management team which possesses accurate information
relating to the marketplace will be in a strong position to make the best decisions in an
increasingly competitive world.
Decision-makers need data to reduce uncertainty and risk when planning for the future and
to monitor business performance. Market researchers provide the data that help them to do
this.
Page 277

Types of data collected
Data can be either primary (collected at first hand from a sample of respondents), or
secondary (collected from previous surveys, other published facts and opinions, or from
experts). Secondary research is also known as desk research, because it can be carried out
from one's desk.
More importantly for research practice and analysis, data can be either quantitative or
qualitative.
Quantitative data usually deal with numbers and typically provide the decision maker with
information about how many customers, competitors etc act in a certain way. Quantitative data
can, for example, tell the researcher what people need or consume, or where, when and how
people buy goods or consumer services.
Qualitative data tell us why consumers think/buy or act the way they do. Qualitative data are
used in consumer insight (eg understanding what makes consumers prefer one brand to
another), media awareness (eg how much of an advertisement is noticed by the public), new
product development studies and for many other reasons.
Qualitative research has as its specific purpose the uncovering and understanding of thought
and opinion. It is carried out on relatively small samples and unstructured or semi-structured
techniques, such as individual in depth interviews and group discussions (also known as
focus groups), are used.

Conservatism
This approach simply involves estimating outcomes in a conservative manner in order to
provide a built-in safety factor.
However, the method fails to consider explicitly a range of outcomes and, by concentrating
only on conservative figures, may also fail to consider the expected or most likely outcomes.
Conservatism is associated with risk aversion and prudence (in the general sense of the
word). In spite of its shortcomings it is probably the most widely used method in practice.

Page 278

Worst/most likely/best outcome estimates
A more scientific version of conservatism is to measure the most likely outcome from a
decision, and the worst and best possible outcomes. This will show the full range of possible
outcomes from a decision, and might help managers to reject certain alternatives because the
worst possible outcome might involve an unacceptable amount of loss. This requires the
preparation of pay-off tables.

Pay-off tables
Pay-off tables identify and record all possible outcomes (or pay-offs) in situations where
the action taken affects the outcomes.

Example: worst/best possible outcomes
Omelette Ltd is trying to set the sales price for one of its products. Three prices are under
consideration, and expected sales volumes and costs are as follows.

Price per unit

RWF 4,000

RWF 4,300

RWF 4,400

Best possible

16,000

14,000

12,500

Most likely

14,000

12,500

12,000

Worst possible

10,000

8,000

6,000

Expected sales volume (units)

Fixed costs are RWF20,000,000 and variable costs of sales are RWF2,000 per unit.
Which price should be chosen?

Page 279

Solution
Here we need to prepare a pay-off table showing pay-offs (contribution) dependant on
different levels of demand and different selling prices.

Price per unit

RWFk4

RWFk4.30

RWFk4.40

Contribution per unit

RWFk2

RWFk2.30

RWFk2.40

Total contribution towards fixed costs

RWF k

RWF k

RWF k

Best possible

32,000

32,200

30,000

Most likely

28,000

28,750

28,800

Worst possible

20,000

18,400

14,400

a) The highest contribution based on most likely sales volume would be at a price of
RWF4,400 but arguably a price of RWF4,300 would be much better than RWF4,400,
since the most likely profit is almost as good, the worst possible profit is not as bad,
and the best possible profit is better.
b) However, only a price of RWFk4 guarantees that the company would not make a
loss, even if the worst possible outcome occurs. (Fixed costs of RWF20,000,000
would just be covered.) A risk averse management might therefore prefer a price of
RWF4,000 to either of the other two prices.

Page 280

PROBABILITIES AND EXPECTED VALUES
Expected values indicate what an outcome is likely to be in the long term with repetition.
Fortunately, many business transactions do occur over and over again.
Although the outcome of a decision may not be certain, there is some likelihood that
probabilities could be assigned to the various possible outcomes from an analysis of previous
experience.

Expected values
Where probabilities are assigned to different outcomes we can evaluate the worth of a
decision as the expected value, or weighted average, of these outcomes. The principle is that
when there are a number of alternative decisions, each with a range of possible outcomes, the
optimum decision will be the one which gives the highest expected value.

Example: expected values
Suppose a manager has to choose between mutually exclusive options A and B, and the
probable outcomes of each option are as follows.

Option A
Probability

Option B
Profit

Probability

RWF ‘000

Profit
RWF ‘000

0.8

5,000

0.1

(2,000)

0.2

6,000

0.2

5,000

0.6

7,000

0.1

8,000

Page 281

The expected value (EV) of profit of each option would be measured as follows.

_____________Option A____________
Prob

_____________Option B____________

Profit

EV of Prob
profit

Profit

EV of
profit

RWF
‘000

RWF
‘000

RWF
‘000

RWF
‘000

0.8

x

5,000

=

4,000

0.1

x

(2,000)

=

(200)

0.2

x

6,000

=

1,200

0.2

x

5,000

=

1,000

EV

=

5,200

0.6

x

7,000

=

4,200

0.1

x

8,000

=

800

EV

=

5,800

In this example, since it offers a higher EV of profit, option B would be selected in
preference to A, unless further risk analysis is carried out.

Page 282

Example
A manager has to choose between mutually exclusive options C and D and the probable
outcomes of each option are as follows.
Option C
Probability

Option D

Cost

Probability

RWF ‘000

Cost
RWF‘000

0.29

15,000

0.03

14,000

0.54

20,000

0.30

17,000

0.17

30,000

0.35

21,000

0.32

24,000

Both options will produce an income of RWF30,000,000. Which should be chosen?

Answer
Option C. Do the workings yourself in the way illustrated above. Note that the probabilities
are for costs not profits.

Page 283

Limitations of expected values
The preference for B over A on the basis of expected value is marred by the fact that A's
worst possible outcome is a profit of RWF5,000, whereas B might incur a loss of RWF2,000
(although there is a 70% chance that profits would be RWF7,000 or more, which would be
more than the best profits from option A).
Since the decision must be made once only between A and B, the expected value of profit
(which is merely a weighted average of all possible outcomes) has severe limitations as a
decision rule by which to judge preference. The expected value will almost never actually
occur.
Expected values are used to support a risk-neutral attitude. A risk-neutral decision maker
will ignore any variability in the range of possible outcomes and be concerned only with the
expected value of outcomes.
Expected values are more valuable as a guide to decision making where they refer to
outcomes which will occur many times over. Examples would include the probability that so
many customers per day will buy a can of baked beans, the probability that a customer
services assistant will receive so many phone calls per hour, and so on.

Page 284

DECISION RULES
The 'play it safe' basis for decision making is referred to as the maximin basis. This is short
for 'maximise the minimum achievable profit'.
A basis for making decisions by looking for the best outcome is known as the maximax
basis, short for 'maximise the maximum achievable profit'.
The ‘opportunity loss’ basis for decision making is known as minimax regret.

The maximin decision rule
The maximin decision rule suggests that a decision maker should select the alternative that
offers the least unattractive of the worst outcomes. This would mean choosing the alternative
that maximises the minimum profits.

Suppose a businessman is trying to decide which of three mutually exclusive projects to
undertake. Each of the projects could lead to varying net profit under three possible scenarios.

Profits
Project

Scenarios

D

E

F

I

100

80

60

II

90

120

85

III

(20)

10

85

The maximin decision rule suggests that he should select the 'smallest worst result' that could
happen. This is the decision criterion that managers should 'play safe' and either minimise
their losses or costs, or else go for the decision which gives the higher minimum profits. If he
selects project D the worst result is a loss of 20. The worst results for E and F are profits of
Page 285

10 and 60 respectively. The best worst outcome is 60 and project F would therefore be
selected (because this is a better 'worst possible' than either D or E).
Criticisms of maximin

a) It is defensive and conservative, being a safety first principle of avoiding the worst
outcomes without taking into account opportunities for maximising profits.
b) It ignores the probability of each different outcome taking place.

Maximax
The maximax criterion looks at the best possible results. Maximax means 'maximise the
maximum profit'.
Using the information above, the maximum profit for D is 100, for E is 120 and for F is 85.
Project E would be chosen if the maximax rule is followed.

Criticisms of maximax

a) It ignores probabilities.
b) It is over-optimistic.

Example
A company is considering which one of three alternative courses of action, A, B and C to
take. The profit or loss from each choice depends on which one of four economic
circumstances, I, II, III or IV will apply. The possible profits and losses, in millions of francs,
are given in the following payoff table. Losses are shown as negative figures.

Page 286

Action

Circumstance

A

B

C

I

70

60

70

II

–10

20

–5

III

80

0

50

IV

60

100

115

Required
State which action would be selected using each of the maximax and maximin criteria.

Answer
a) The best possible outcomes are as follows.
A (circumstance III):

80

B (circumstance IV):

100

C (circumstance IV):

115

As 115 is the highest of these three figures, action C would be chosen using the
maximax criterion.

b) The worst possible outcomes are as follows.
A (circumstance II):

–10

B (circumstance III):

0

C (circumstance II):

–5

The best of these figures is 0 (neither a profit nor a loss), so action B would be chosen
using the maximin criterion.

Page 287

Minimax regret rule
The minimax regret rule aims to minimise the regret from making the wrong decision.
Regret is the opportunity lost through making the wrong decision.

We first consider the extreme to which we might come to regret an action we had chosen.
Regret for any
combination of action
and circumstances

=

Profit for best action in
those circumstances

–

Profit for the action
actually chosen in
those
circumstances

The minimax regret decision rule is that the decision option selected should be the one which
minimises the maximum potential regret for any of the possible outcomes.
Using the example above, a table of regrets can be compiled as follows.
Project
D
I
Scenario

0

II

30***

III
Maximum regret
* 100 – 80 ** 100 – 60

E
20*

F
40**

0

35

105

75

0

105

75

40

*** 120 – 90

The lowest of maximum regrets is 40 with project F so project F would be selected if the
minimax regret rule is used.

Page 288

Contribution tables
Questions requiring application of the decision rules often incorporate a number of
variables, each with a range of possible values. For example these variables might be:
•
•

Unit price and associated level of demand
Unit variable cost

Each variable might have, for example, three possible values.
Before being asked to use the decision rules, exam questions could ask you to work out
contribution for each of the possible outcomes. (Alternatively profit figures could be
required if you are given information about fixed costs.)
The number of possible outcomes = number of values of variable 1 × number of values of
variable 2 × number of values of variable 3 etc
So, for example, if there are two variables, each with three possible values, there are 3 × 3 =
9 outcomes.
Perhaps the easiest way to see how to draw up contribution tables is to look at an example.

Example: contribution tables and the decision rules
Suppose the budgeted demand for product X will be 11,500 units if the price is Rwf10,000
8,500 units if the price is Rwf12,000 and 5,000 units if the price is rwf14,000. Variable costs
are estimated at either Rwf4000, 5,000, or6,000 per unit. A decision needs to be made on the
price to be charged.
Here is a contribution table showing the budgeted contribution for each of the nine possible
outcomes.

Page 289

Price

Variable cost

Unit
contribution

Total
contribution

RWF ‘000

RWF ‘000

RWF ‘000

RWF m

11,500

10

4

6

69.0

11,500

10

5

5

57.5

11,500

10

6

4

46.0

8,500

12

4

8

68.0

8,500

12

5

7

59.5

8,500

12

6

6

51.0

5,000

14

4

10

50.0

5,000

14

5

9

45.0

5,000

14

6

8

40.0

Demand

Once the table has been drawn up, the decision rules can be applied.

Solution
Maximin
We need to maximise the minimum contribution.
Demand/price

Minimum contribution

Units/RWF ‘000

RWF 000

11,500/10

46,000

8,500/12

51,000

5,000/14

40,000

Set a price of RWF12,000.
Page 290

Maximax
We need to maximise the maximum contribution.
Demand/price

Maximum contribution

Units/RWF ‘000

RWF‘000

11,50010

69,000

8,000/12

68,000

5,000/14

50,000

Set a price of RWF10,000.

Minimax regret
We need to minimise the maximum regret (lost contribution) of making the wrong decision.
Variable cost

Price RWF ‘000

RWF ‘000

10

12

14

4

–

1,000

19,000

5

2,000

–

14,500

6

5,000

–

11,000

Minimax regret

5,000

1,000

19,000

Minimax regret strategy (price of RWF12,000) is that which minimises the maximum regret
(rwf1,000,000).

Page 291

Sample working
At a variable cost of RWF4,000, the best strategy would be a price of RWF10,000. Choosing
a price of RWF12,000 would mean lost contribution of RWF69m – RWF68m, while
choosing a price of RWF14,000 would mean lost contribution of RWFm69 – 50.

Page 292

DECISION TREES
Decision trees are diagrams which illustrate the choices and possible outcomes of a decision.
Rollback analysis evaluates the EV of each decision option. You have to work from right to
left and calculate Evs at each outcome point.

A probability problem such as ‘what is the probability of throwing a six with one throw of a
die? Is fairly straightforward and can be solved using the basic principles of probability.
More complex probability questions, although solvable using the basic principles, require a
clear logical approach to ensure that all possible choices and outcomes of a decision are taken
into consideration.
Decision trees are a useful means of interpreting such probability problems.

A decision tree is a pictorial method of showing a sequence of interrelated decisions and
their expected outcomes. Decision trees can incorporate both the probabilities of, and values
of, expected outcomes, and are used in decision-making

Exactly how does the use of a decision tree permit a clear and logical approach?
•

All the possible choices that can be made are shown as branches on the tree.

•

All the possible outcomes of each choice are shown as subsidiary branches on
the tree.

Page 293

Constructing a decision tree.
There are two stages in preparing a decision tree.
•

Drawing the tree itself to show all the choices and outcomes

•

Putting in the numbers (the probabilities, outcome values and EVs)

Every decision tree starts from a decision point with the decision options that are currently
being considered.
a) It helps to identify the decision point, and any subsequent decision points in the tree,
with a symbol. Here, we shall use a square shape.
b) There should be a line, or branch, for each option or alternative

It is conventional to draw decision trees from left to right ,and so a decision tree will start
as follows.

The square is the decision point, and A, B, C, and D represent four alternatives from which
a choice must be made (such as buy a new machine with cash, hire a machine, continue to use
existing machine, raise a loan to buy a machine).
If the outcome from any choice is certain, the branch of the decision tree for that
alternative is complete.
If the outcome of a particular choice is uncertain, the various possible outcomes must be
shown.

Page 294

We show the various possible outcomes on a decision tree by inserting an outcome point on
the branch of the tree. Each possible outcome is then shown as a subsidiary branch, coming
out form the outcome point. The probability of each outcome occurring should be written on
the branch of the tree which represents that outcome.
To distinguish decision points from outcome points, a circle will be used as the symbol for
an outcome point.

In the example above, there are two choices facing the decision-maker, A and B. The
outcome if A is chosen is known with certainly, but if B is chosen, there are two possible
outcomes, high sales (0.6 probability) or low sales (0.4 probability).
When several outcomes are possible, it is usually simpler to show two or more stage of
outcome points on the decision tree.

Example: Several possible outcomes
A company can choose to launch a new product XYZ or not. If the product is launched,
expected sales and expected unit costs might be as follows.

Sales

Units costs

Units

Probability

RWF

Probability

10,000

0.8

6

0.7

15,000

0.2

8

0.3

Page 295

a) The decision tree could be drawn as follows

b) The layout shown above will usually be easier to use than the alternative way of
drawing the tree, which is as follows.

Page 296

Sometimes, a decision taken now will lead to other decisions to be taken in the future.
When this situation arises, the decision tree can be drawn as a two –stage tree, as follows.

In this tree, either a choice between A and B or else a choice between C and D will be made,
depending on the outcome which occurs after choosing X.
The decision tree should be in chronological order from left to right. When there are twostage decision trees, the first decision in time should be drawn on the left.

Example: A decision tree
Beethoven has a new wonder product, the vylin, of which it expects great things. At the
moment the company has two courses of action open to it, to test market the product or
abandon it.
If the company test markets it, the cost will be RWF100,000 and the market response could
be positive or negative with probabilities of 0.060 and 0.40.
If the response is positive the company could either abandon the product or market if full
scale.
If it markets the vylin full scale, the outcome might be low, medium or high demand, and the
respective net gains/(losses) would be (200) , 200 or 1,000 in units of RWF1,000 (the result
could range from a net loss of RWF200,000 to a gain of RWF1,000,000). These outcomes
have probabilities of 0.20, 0.50 and 0.30 respectively.
If the result of the test marketing is negative and the company goes ahead and markets the
product estimated losses would be RWF600,000.
Page 297

If , at any point, the company abandon the product, there would be a net gain of RWF50,000
from the sale of scrap 0. All the financial values have been discounted to the present.
Required
a) Draw a decision tree
b) Include figures for cost, loss or profit on the appropriate branches of the tree.

Solution
The starting point for the tree is to establish what decision has to be made now. What are
the options?
a) To test market
b) To abandon

The outcome of the ‘abandon’ option is known with certainty. There are two possible
outcomes of the option to test market, positive response and negative response.
Depending on the outcome of the test marketing, another decision will then be made, to
abandon the product or to go ahead.

Page 298

Exam Focus Point
In an examination, remember to draw decision trees (and all diagrams) neatly, using a pen
and ruler. Remember also to label decision points and branches as clearly as possible.

Evaluating the decision with a decision tree
Rollback analysis evaluates the V or each decision option. You have to work from right to
left and calculate EVs at each outcome point.
The EV of each decision option can be evaluated, using the decision tree to help with keeping
the logic on track. The basic rules are as follows.
a) We start on the right hand side of the tree and work back towards the left hand side
and the current decision under consideration . This is sometimes known as the
‘rollback’ technique or ‘rollback analysis’
b) Working from right to left, we calculate the EV of revenue, cost contribution or
profit at each outcome point on the tree

In the above example, the right-hand-most outcome point is pint E, and EV is as follows.
Profit

Probability

x

p

RWF ’000

px
RWF’000

High

1,000

0.3

300

Medium

200

0.5

100

Low

(200)

0.2

(40)

EV

360

Page 299

This is the EV of the decision to market the product if the test shows positive response. It
may help you to write the EV on the decision tree itself, at the appropriate outcome point
(point E).
a) At decision point C, the choice is as follows.
(i)

Market, EV = +360 (the EV at point E)

(ii)

Abandon, value = + 50

The choice would be to market the product, and so the V at decision point C is +360

b) At decision point D, the choice is as follows.
(i)

Market, value = -600

(ii)

Abandon , value =+ 50

The choice would be to abandon, and so the EV at decision point D is +50

The second stage decisions have therefore been made. If the original decision is to test
market, the company will market the product if the test shows positive customer response,
and will abandon the product if the test results are negative.
The evaluation of the decision tree is completed as follows.
a) Calculate the EV at outcome point B.
0.6 x 360 (Ev at C)
+

0.4 x

50

=

216 + 20 = 236

(EV at D)

b) Compare the options at point A, which are as follows
(i)

Test: EV =EV at B minus test marketing cost = 236 -100=136

(ii)

Abandon: Value = 50

The choice would be to test market the product, because it has a higher EV of profit
Page 300

Question
Consider the following diagram

If a decision maker wished to maximise the value of the outcome, which options should
be selected?
A

Option 2 and option 7

B

Option 3

C

Option 1 and option 4

D

Option 2, option 6 and option 8

Answer
The correct answer is A.

The various outcomes must be evaluated using expected values.
EV at point B: (0.75x 10,000) + (0.25x8,000) = 9,500
EV at point D: (0.6 x 20,000) + (0.4 x (-4,000) = 10,400
EV at point C: choice between 10,400 and 11,000
EV at point A : Choice between B (9,500), C (10,400 or 11,000) and choice 3 (9,000).
Page 301

If we are trying to maximise the figure, option 2 and the option 7 are chosen to give 11,000.
Evaluating decisions by using decision trees has a number of limitations.
a) The time value of money may not be taken into account.
b) Decision trees are not very suitable for use in complex situations.
c) The outcome with the highest EV may have the greatest risks attached to it. Managers
may be reluctant to take risks which may lead to loses.
d) The probabilities associated with different branches of he ‘tree’ are likely to be
estimates, and possibly unreliable or inaccurate.

Page 302

THE VALUE OF INFORMATION
Perfect information is guaranteed to predict the future with 100% accuracy. Imperfect
information is better than no information at all but could be wrong in its prediction of the
future.
The value of perfect information is the difference between the EV of profit with perfect
information and the EV of profit without perfect information.

Perfect information removes all doubt and uncertainty from a decision, and enables
managers to make decisions with complete confidence that they have selected the optimum
course of action.

The value of perfect information.
Step 1

If we do not have perfect information and we must choose between two or more
decision options we would select the decision option which offers the highest EV
of profit. This option will not be the best decision under all circumstances. There
will be some probability that what was really the best option will not have been
selected, given the way actual events turn out.

Step 2

With perfect information, the best decision option will always be selected. The
profits from the decision will depend on the future circumstances which are
predicted by the information nevertheless, the EV of profit with perfect
information should be higher than the EV of profit without the information.

Step 3

The value of perfect information is the difference between these two EVs

Page 303

Example : the value of perfect information
The management of Ivor Ore must choose whether to go ahead with either of two mutually
exclusive projects, A and B. The expected profits are as follows.
Profit if there is strong demand

Profit/(loss) if there is weak demand

Option A

RWF4,000

RWF(1,000)

Option B

RWF1,500

RWF500

Probability of demand 0.3

0.7

Required
Ascertain what the decision would be, based on expected values, if no information about
demand were available.
Calculate the value of perfect information about demand.

Solution
Step 1

If there was no information to help with the decision, the project with the higher
EV of profit would be selected.
Probability

Project A

Project B

Profit

EV

Profit

EV

RWF

RWF

RWF

RWF

0.3

4,000

1,200

1,500

450

0.7

(1,000)

(700)

500

350

500
Page 304

800

Project B would be selected
This is clearly the better option if demand turns out to be weak. However, if
demand were to turn out to be strong, project A would be more profitable. There is
a 30% chance that this could happen.

Step 2

Perfect information will indicate for certain whether demand will be weak or
strong. If demand is forecast ‘weak’ project B would be selected. If demand is
forecast as ‘strong’ , project A would be selected, and perfect information would
improve the profit from RWF1,500, which would have been earned by selecting B,
to RWF4,000

Forecast
demand

Project
Probability

chosen

Profit

EV of profit

RWF

RWF

Weak

0.7

B

500

350

Strong

0.3

A

4,000

1,200

EV of profit with perfect information

1,550

Step 3
RWF
EV of profit without perfect information (i.e. if project B is always 800
chosen)
EV of profit with perfect information

1,550
750

Page 305

Provided that the information does not cost more than RWF750 to collect, it would be worth
having.

Perfect information and decision trees
When the option exists to obtain information, the decision can be shown, like any other
decision, in the form of a decision tree, as follows. We will suppose, for illustration, that the
cost of obtaining perfect information is RWF400.

Page 306

The decision would be to obtain perfect information, since the EV of profit is RWF4,050 RWF400 = RWF3,650.
You should check carefully that you understand the logic of this decision and that you can
identify how the EVs at outcome boxes 1, 2, 3 and 4 have been calculated.

The value of imperfect information
There is one serious drawback to the technique we have just looked at: in practice, useful
information is never perfect unless the person providing it is the sole source of the
uncertainty. Market research findings or information from pilot tests and so on are likely to
be reasonably accurate, but they can still be wrong: they provide imperfect information. It is
possible, however, to arrive at an assessment of how much it would be worth paying for
such imperfect information, given that we have a rough indication of how right or
wrong it is likely to be.
Suppose we are considering the sex and hair colour of people in a given group or population
consisting of 70% men and 30% women. We have established the probabilities of hair
colourings as follows:

Men

Women

Brown

0.60

0.35

Blonde

0.35

0.55

Red

0.05

0.10

This shows, for example, that 5% of men in such a sample have red hair. These probabilities
of sex and hair colouring might be referred to as prior probabilities.
Posterior probabilities consider the situation in reverse or retrospect, so that we can ask the
question: ‘Given that a person taken at random from the population is brown-haired what is
the probability that the person is male (or female)?’

Page 307

The information can be presented in a table. Let’s suppose that the population consists of
1,000 people.
Male

Female

Total

Brown

420 (W3)

105 (W4)

525 (W5)

Blonde

245

165

410

Red

35

30

65

700 (W1)

300

(W2)

1,000

Workings
1

1,000 x 70%

2

1,000 – 700

3

700 x 60% (the other two values in the column being calculated in a similar way)

4

300 x 35% (the other two values in the column being calculated in a similar way)

5

420 +105 (the other two values in the column being calculated in a similar way)

∴

P (Person selected is a male, given that that person is brown-haired) = 420/525 = 0.8

Example: The value of imperfect information
Suppose that the Small Oil Company (SOC) is trying to decide whether or not to drill on a
particular site. The chief engineer has assessed the probability that there will be oil, based on
vast experience, as 20% and the probability that there won’t be oil as 80%.
It is possible for the SOC to hire a firm of international consultants to carry out a complete
survey of the site. SOC has used the firm many times before and has estimated that if there
really is oil, there is a 95% chance that the report will be favourable, but if there is no oil,
there is only a 10% chance that the report will indicate there is oil.
Page 308

Required
Determine whether drilling should occur.
Solution
Read the information given carefully. We are given three sets of probabilities.
a) The probability that there will be oil (0.2) or there will not be (0.8). These outcomes
are mutually exclusive.
b) The probability that, If there is oil, the report will say there is oil (0.95) or say there is
no oil (0.05)
c) The probability that, if there is no oil, the report will say there is oil (0.1) or say there
is no oil (0.9).

Both (b) and (c) describe conditional events, since the existence of oil or otherwise influences
the chances of the survey report being correct.
SOC, meanwhile faces a number of choices which we can show as a decision tree.

We must now calculate the probabilities of the following outcomes.
•

The information will say ‘oil’ or ‘no oil’

•

The information will be right or wrong if it says ‘oil’

•

The information will be right or wrong if it says ‘no oil’

Page 309

If you check the information given in the problem, you will find that these probabilities are
not given.
a) We are told that the engineer has assessed that there is a 20% chance of oil and an
80% chance of no oil (ignoring information entirely). These are the prior
probabilities of future possible outcomes.
b) The probabilities that there will be oil or no oil once the information has been
obtained are “posterior” probabilities.

Step 1
We can tabulate the various probabilities as percentages.
Actual outcome
Oil
Survey

Oil

19

Result:

No oil

1

Total

20

No Oil
(w2)

(w1)

8

Total
(w3)

27

72

73

80

100

(w4)

Workings
1

The engineer estimates 20% probability of oil and 80% of no oil.

2.

If there is oil, i.e. in 20 cases out of 100, the survey will say so in 95% of these
cases, i.e. in 20 x0.95 =19 cases. The 1 below the 19 is obtained by subtraction.

3.

In the 80 per 100 cases where there is in fact no oil, the survey will wrongly say that
there is oil 10% of the time; i.e. 80x 0.10 = 8 cases. The 72 below the 8 is obtained
by subtraction.

4.

The horizontal totals are given by addition.

Page 310

Step 2
We can now provide all the probabilities needed to complete the tree.
P (survey will say there is oil)

=

27/100

=

0.27

P (survey will say there is no oil)

=

73/100

=

0.73

If survey says oil P (there is oil)

=

19/27

=

0.704

P (there is no oil)

=

8/27

=

0.296 (or 1-0.704)

If survey says no oil P (there is oil)

=

1/73

=

0.014

P (there is no oil)

=

72/73

=

0.986 (or 1-0.014)

Step 3
We can now go on to complete the decision tree. Let us make the following
assumptions.
(In an exam question such information would have been given to you from the start)
•

The cost of drilling is RWF10m.

•

The value of the benefits if oil is found is RWF70m, giving a net ‘profit’ of
RWF60m

•

The cost of obtaining information from the consultants would be RWF3m.

An assumption is made that the decision maker will take whichever decision the
information indicates is the best. If the information says ‘oil’, the company will drill
and if the information says ‘no oil’ it will not drill.

Page 311

*The information is ‘no oil’@ 4, so the company won’t drill, regardless of whether there
really is oil or not.

Step 4
We can now perform rollback analysis.
RWFm
EV at point 2 =

0.704 x RWF60m

42.24

0.296x (RWF10m)

(2.96)
+39.28

RWFm
EV at point 2 =

0.27xRWF39.28m

10.61

0.73xRWF0

0.00
+10.61

Page 312

Step 5
There are three choices

EV

a) Do not obtain information and do not drill
b) Do not obtain information and drill
c) Obtain information first, decide about drilling later

RWFO
+RWF4million
(RWF(10.61m – 3m)
+7.61million

The decision should be to obtain the information from a survey first.

Step 6
The value of the imperfect information is the difference between (b) and (c) , Rwf3.61
million.

Page 313

BLANK

Page 314

SENSITIVITY ANALYSIS
Sensitivity analysis can be used in any situation so long as the relationships between the key
variables can be established. Typically this involves changing the value of a variable and
seeing how the results are affected.

Approaches to sensitivity analysis
Sensitivity analysis is a term used to describe any technique whereby decision options are
tested for their vulnerability to changes in any 'variable' such as expected sales volume, sales
price per unit, material costs, or labour costs.

Here are three useful approaches to sensitivity analysis.
a) To estimate by how much costs and revenues would need to differ from their
estimated values before the decision would change.
b) To estimate whether a decision would change if estimated costs were x%
higher than estimated, or estimated revenues y% lower than estimated.
c) To estimate by how much costs and/or revenues would need to differ from their
estimated values before the decision maker would be indifferent between two
options.

The essence of the approach, therefore, is to carry out the calculations with one set of values
for the variables and then substitute other possible values for the variables to see how this
affects the overall outcome.
a) From your studies of information technology you may recognise this as what if
analysis that can be carried out using a spreadsheet.
b) From your studies of linear programming you may remember that sensitivity
analysis can be carried out to determine over which ranges the various
constraints have an impact on the optimum solution.
c) Flexible budgeting can also be a form of sensitivity analysis.

Page 315

Example: sensitivity analysis
SVT Ltd has estimated the following sales and profits for a new product which it may launch
on to the market.
RWF
Sales
(2,000 units)
Variable costs: materials

RWF
4,000

2,000

labour

1,000
3,000

Contribution

1,000

Less incremental fixed costs

800

Profit

200

Required
Analyse the sensitivity of the project.

Solution
a) If incremental fixed costs are more than 25% above estimate, the project would make
a loss.
b) If unit costs of materials are more than 10% above estimate, the project would make a
loss.
c) Similarly, the project would be sensitive to an increase in unit labour costs of more
than RWF200, which is 20% above estimate, or else to a drop in the unit selling price
of more than 5%.
d) The margin of safety, given a breakeven point of 1,600 units, is (400/2,000) × 100% =
20%.
Management would then be able to judge more clearly whether the product is likely to be
profitable. The items to which profitability is most sensitive in this example are the selling
price (5%) and material costs (10%). Sensitivity analysis can help to concentrate
management attention on the most important factors.
Page 316

SIMULATION MODELS
Simulation models can be used to deal with decision problems involving a number of
uncertain variables. Random numbers are used to assign values to the variables.

One of the chief problems encountered in decision making is the uncertainty of the future.
Where only a few factors are involved, probability analysis and expected value calculations
can be used to find the most likely outcome of a decision. Often, however, in real life, there
are so many uncertain variables that this approach does not give a true impression of
possible variations in outcome.
To get an idea of what will happen in real life one possibility is to use a simulation model in
which the values and the variables are selected at random. Obviously this is a situation
ideally suited to a computer (large volume of data, random number generation).
The term 'simulation' model is often used more specifically to refer to modelling which
makes use of random numbers. This is the 'Monte Carlo' method of simulation. In the
business environment it can, for example, be used to examine inventory, queuing, scheduling
and forecasting problems.
Random numbers are allocated to each possible value of the uncertain variable in proportion
to the probabilities, so that a probability of 0.1 gets 10% of the total numbers to be assigned.
These random numbers are used to assign values to the variables.

Exam Focus Point
You will not be required to develop a simulation model in your exam. The following
example is provided so that you can understand how simulation models are developed.

Page 317

Example: simulation and spreadsheets
A supermarket sells a product for which the daily demand varies. An analysis of daily
demand over a period of about a year shows the following probability distribution.
Demand per day

Probability

Units
35
36
37
38
39
40

0.10
0.20
0.25
0.30
0.08
0.07
1.00

To develop a simulation model in which one of the variables is daily demand, we would
assign a group of numbers to each value for daily demand. The probabilities are stated to
two decimal places, and so there must be 100 random numbers in total, 00 – 99 (we use 00-99
rather than 1-100 so that we can use two-digit random numbers.) Random numbers are
assigned in proportion to the probabilities, so that a probability of 0.1 gets 10% of the total
numbers to be assigned, that is 10 numbers: 0, 1, 2, 3, 4, 5, 6, 7, 8 and 9.
The assignments would therefore be as follows.

Demand per day

Probability

Numbers assigned

0.10
0.20
0.25
0.30
0.08
0.07

00 – 09
10 – 29
30 – 54
55 – 84
85 – 92
93 – 99

Units
35
36
37
38
39
40

Page 318

When the simulation model is run, random numbers will be generated to derive values for
daily demand. For example, if the model is used to simulate demand over a ten day period,
the random numbers generated might be as follows.
19007174604721296802
The model would then assign values to the demand per day as follows.
Day

Random number

1
2
3
4
5
6
7
8
9
10

19
00
71
74
60
47
21
29
68
02

Demand
Units
36
35
38
38
38
37
36
36
38
35

You might notice that on none of the ten days is the demand 39 or 40 units, because the
random numbers generated did not include any value in the range 85 – 99. When a simulation
model is used, there must be a long enough run to give a good representation of the system
and all its potential variations.

Uses of simulation
In the supermarket example above, the supermarket would use the information to minimise
stock holding without risking running out of the product. This will reduce costs but avoid lost
sales and profit.
A supermarket can also use this technique to estimate queues with predicted length of waiting
time and so determine the number of staff required.

Page 319

CHAPTER ROUNDUP
• An example of a risky situation is one in which we can say that there is a 70% probability
that returns from a project will be in excess of RWF100m but a 30% probability that
returns will be less than RWF100m. If we cannot predict an outcome or assign
probabilities, we are faced with an uncertain situation.
• People may be risk seekers, risk neutral or risk averse.
• Management accounting directs its attention towards the future and the future is
uncertain. For this reason a number of methods of taking uncertainty into consideration
have evolved.
• Market research can be used to reduce uncertainty.
• Expected values indicate what an outcome is likely to be in the long term with repetition.
Fortunately, many business transactions do occur over and over again.
• The 'play it safe' basis for decision making is referred to as the maximin basis. This is
short for 'maximise the minimum achievable profit'.
• A basis for making decisions by looking for the best outcome is known as the maximax
basis, short for 'maximise the maximum achievable profit'.
• The ‘opportunity loss’ basis for decision making is known as minimax regret.
• Decision trees are diagrams which illustrate the choices and possible outcomes of a
decision.
• Rollback analysis evaluates the EV of each decision option. You have to work from right
to left and calculate EVs at each outcome point.
• Perfect information is guaranteed to predict the future with 100% accuracy. Imperfect
information is better than no information at all but could be wrong in its prediction of the
future.
• The value of perfect information is the difference between the EV of profit with perfect
information and the EV of profit without perfect information.
• Sensitivity analysis can be used in any situation so long as the relationships between the
key variables can be established. Typically this involves changing the value of a variable
and seeing how the results are affected.
• Simulation models can be used to deal with decision problems involving a number of
uncertain variables. Random numbers are used to assign values to the variables.

Page 320

STUDY UNIT 12
Objectives Of Budgetary Control
Contents

Page

Objectives …………………………………………………….…………..…….

323

The Planning And Control Cycle ……….…………………….………..……..

327

Objectives Of Budgetary Systems …………………………………...………

333

Behavioural Implications Of Budgeting ……………………………………

335

Setting The Difficulty Level Of A Budget …………………..………………

339

Participation In Budgeting ………………….…………………………………

341

Mutually Exclusive Projects With Unequal Lives ………………….………

345

Page 321

EXAM GUIDE
The topics covered in this chapter may form the discussion part of a budget question or may
form an entire narrative question. Much of the material is common sense and you should
always try to relate it to your own experience.

Page 322

OBJECTIVES
Corporate objectives concern the firm as a whole. Unit objectives are specific to individual
units, divisions or functions of an organisation.

Corporate objectives are set as part of the corporate planning process which is concerned
with the selection of strategies which will achieve the corporate objectives of the
organisation.

Corporate objectives versus unit objectives
Corporate objectives should relate to the key factors for business success.
•
•
•
•
•
•

Profitability
Market share
Growth
Cash flow
Return on capital employed
Risk

•
•
•
•
•

Customer satisfaction
Quality
Industrial relations
Added value
Earnings per share

Page 323

Unit objectives, on the other hand, are specific to individual business units, divisions or
functions of an organisation.
Types
Commercial

Examples
• Increase the number of customers by x% (an objective of a sales
department)
• Reduce the number of rejects by 50% (an objective of a production
department)
• Produce monthly reports more quickly, within 5 working days of the
end of each month (an objective of the finance & management
accounting departments)

Public sector

• Introduce x% more places at nursery schools (an objective of a
district education department)
• Respond more quickly to calls (an objective of a local police station,
fire department or even a telephone-banking help-line)

General

• Resources (e.g. cheaper raw materials, lower borrowing costs, 'topquality’ accountants)
• Market (e.g. market share, market standing)
• Employee development (e.g. training, promotion, safety)
• Innovation in products or processes
• Productivity (the amount of output from resource inputs)
• Technology

Primary and secondary objectives
Primary corporate objectives are supported by secondary objectives, for example for
product development or market share. In practice there may be a trade off between different
objectives.
An organisation has many objectives. It has been argued that there is a limit to the number
of objectives that a manager can pursue effectively. Too many and the manager cannot give
adequate attention to each and/or the focus may inadvertently be placed on minor ones. Some
objectives are more important than others. It has therefore been suggested that there should
Page 324

be one primary corporate objective (restricted by certain constraints on corporate activity)
and other secondary objectives. These are strategic objectives which should combine to
ensure the achievement of the primary corporate objective.
a) For example, if a company sets itself a primary objective of growth in profits,
it will then have to develop strategies by which this primary objective can be
achieved.
b) Secondary objectives might then be concerned with sales growth, continual
technological innovation, customer service, product quality, efficient resource
management (e.g. labour productivity) or reducing the company's reliance on
debt capital.

Conflicting objectives
Corporate objectives may conflict with divisional objectives in large organisations. A danger
is that the organisation will divide into a number of self-interested segments, each acting at
times against the wishes and interests of other segments. Decisions might be taken by a
divisional manager in the best interests of his own part of the business, but possibly against
the interests of the organisation as a whole. The setting of objectives is very much a political
process: objectives are formulated following bargaining by the various interested parties
whose requirements may conflict. Such conflict may be resolved via prioritisation,
compromise, negotiation and satisficing (satisfy and suffice).
a) Prioritisation is where certain goals get priority over others. This is usually
determined by senior managers but there can be quite complicated systems to rank
goals and strategies according to certain criteria.
b) Negotiation is the bargaining process that occurs at each stage of the budgeting
process. This allows full participation to take place by all budget holders. Any
revisions to the budget must be after giving full consideration to arguments for
including any of the budgeted items.
c) Compromise is the central aspect of any process of negotiation where there is
disagreement. It can be seen as positive where both parties win something but also
negative where both parties give something away.
d) Satisficing occurs when a satisfactory and sufficient solution rather than an optimum
solution is found. Organisations may not aim to maximise performance in one area if
Page 325

this leads to poor performance elsewhere. Rather they will accept satisfactory, if not
excellent performance in a number of areas.
Goal congruence exists when managers working in their best interests also act in harmony
with the goals of the organisation as a whole. This is not easy to achieve and a budgetary
control system needs to be designed to evoke the required behaviour.

Page 326

THE PLANNING AND CONTROL CYCLE
The planning and control cycle has seven steps.
•
•
•
•
•
•
•

Step 1. Identify objectives
Step 2. Identify potential strategies
Step 3. Evaluate strategies
Step 4. Choose alternative courses of action
Step 5. Implement the long-term plan
Step 6. Measure actual results and compare with the plan
Step 7. Respond to divergences from the plan

The diagram below represents the planning and control cycle. The first five steps cover the
planning process. Planning involves making choices between alternatives and is primarily a
decision-making activity. The last two steps cover the control process, which involves
measuring and correcting actual performance to ensure that the alternatives that are chosen
and the plans for implementing them are carried out.

Page 327

Step 1

Identify objectives
Objectives establish the direction in which the management of the organisation
wishes it to be heading. They answer the question: 'where do we want to be?'

Step 2

Identify potential strategies
Once an organisation has decided 'where it wants to be', the next step is to
identify a range of possible courses of action or strategies that might enable the
organisation to get there. The organisation must therefore carry out an
information-gathering exercise to ensure that it has a full understanding of
where it is now. This is known as a 'position audit' or 'strategic analysis' and
involves looking both inwards and outwards.
a) The organisation must gather information from all of its internal parts
to find out what resources it possesses: what its manufacturing capacity
and capability are, what is the state of its technical know-how, how well it
is able to market itself, how much cash it has in the bank and so on.
b) It must also gather information externally so that it can assess its
position in the environment. Just as it has assessed its own strengths and
weaknesses, it must do likewise for its competitors (threats). Current and
potential markets must be analysed to identify possible new
opportunities. The 'state of the world' must be considered. Is it in
recession or is it booming? What is likely to happen in the future? This
part of the analysis is known as SWOT analysis – Strengths, Weaknesses,
Opportunities and Threats.
Having carried out a strategic analysis, alternative strategies can be identified. An
organisation might decide to be the lowest cost producer in the industry, perhaps
by withdrawing from some markets or developing new products for sale in
existing markets. This may involve internal development or a joint venture.

Step 3

Evaluate strategies
The strategies must then be evaluated in terms of suitability, feasibility and
acceptability. Management should select those strategies that have the greatest
potential for achieving the organisation's objectives.

Page 328

Step 4

Choose alternative courses of action
The next step in the process is to collect the chosen strategies together and coordinate them into a long-term financial plan. Typically this would show the
following.
•
•
•
•
•

Step 5

Projected cash flows
Capital expenditure plans
Projected long-term profits
Balance sheet forecasts
A description of the long-term objectives and strategies in words

Implement the long-term plan
The long-term plan should then be broken down into smaller parts. It is
unlikely that the different parts will fall conveniently into successive time
periods. Strategy A may take two and a half years, while Strategy B may take five
months, but not start until year three of the plan. It is usual, however, to break
down the plan as a whole into equal time periods (usually one year). The resulting
short-term plan is called a budget.

Step 6

Measure actual results and compare with plan
Actual results are recorded and analysed and information about actual results is
fed back to the management concerned, often in the form of accounting reports.
This reported information is feedback (see section named ‘Feedback’ below).

Step 7

Respond to divergences from plan
By comparing actual and planned results, management can then do one of three
things, depending on how they see the situation.
a) They can take control action. By identifying what has gone wrong, and
then finding out why, corrective measures can be taken.
b) They can decide to do nothing. This could be the decision when actual
results are going better than planned, or when poor results were caused by
something which is unlikely to happen again in the future.

Page 329

c) They can alter the plan or target if actual results are different from the
plan or target, and there is nothing that management can do (or nothing,
Level

Detail

Corporate plans

•
•
•
•
•

Operational
plans

•
•
•
•
•
•
•

Focused on overall performance
Environmental influence
Set plans and targets for units and departments
Sometimes qualitative (e.g. a programme to change the culture
of the organisation)
Aggregate
Based on objectives about 'what' to achieve
Specific (e.g. acceptable number of 'rings' before a phone is
answered)
Little immediate environmental influence
Likely to be quantitative
Detailed specifications
Based on 'how' something is achieved
Short time horizons

perhaps, that they want to do) to correct the situation.

Control
Consider how the activities of planning and control are inter-related.
a) Plans set the targets.
b) Control involves two main processes.
(i)

Measure actual results against the plan.

(ii)

Take action to adjust actual performance to achieve the plan or to change the
plan altogether.

Control is therefore impossible without planning.

Page 330

The essence of control is the measurement of results and comparing them with the original
plan. Any deviation from plan indicates that control action is required to make the results
conform more closely with plan.

Feedback
Feedback occurs when the results (outputs) of a system are used to control it, by adjusting
the input or behaviour of the system.

A business organisation uses feedback for control.
a) Negative feedback indicates that results or activities must be brought back on course,
as they are deviating from the plan.
b) Positive feedback results in control action continuing the current course. You would
normally assume that positive feedback means that results are going according to plan
and that no corrective action is necessary: but it is best to be sure that the control
system itself is not picking up the wrong information.
c) Feedforward control is control based on forecast results: in other words if the
forecast is bad, control action is taken well in advance of actual results.

There are two types of feedback.
a) Single loop feedback is control, like a thermostat, which regulates the output of a
system. For example, if sales targets are not reached, control action will be taken to
ensure that targets will be reached soon. The plan or target itself is not changed, even
though the resources needed to achieve it might have to be reviewed.
b) Double loop feedback is of a different order. It is information used to change the
plan itself. For example, if sales targets are not reached, the company may need to
change the plan.

Page 331

Control at different levels
Budgetary control occurs at the lower levels of the performance hierarchy.
Control at the lower-levels of the performance hierarchy, such as standard costing, and
budgetary control has the following features.
• Exercised externally by management or, in the case of empowered teams, by the
staff themselves
• Immediate or rapid feedback
• Single loop feedback (i.e. little authority to change plans or targets)
Control does also occur at the higher-levels of the hierarchy, however, and has the
following characteristics.
•
•
•
•

Exercised by external stakeholders (e.g. shareholders)
Exercised by the market
Double loop feedback (i.e. relatively free to change targets)
Often feed forward elements

Page 332

OBJECTIVES OF BUDGETARY SYSTEMS
Here are the objectives of a budgetary planning and control system.
•
•
•
•
•
•
•

Ensure the achievement of the organisation's objectives
Compel planning
Communicate ideas and plans
Coordinate activities
Provide a framework for responsibility accounting
Establish a system of control
Motivate employees to improve their performance

A budgetary planning and control system is essentially a system for ensuring
communication, coordination and control within an organisation. Communication,
coordination and control are general objectives: more information is provided by an
inspection of the specific objectives of a budgetary planning and control system.

Objective

Comment

Ensure the
achievement of the
organisation's
objectives

Objectives are set for the organisation as a whole and for individual
departments and operations within the organisation. Quantified
expressions of these objectives are then drawn up as targets to be
achieved within the timescale of the budget plan.

Compel planning

This is probably the most important feature of a budgetary planning
and control system. Planning forces management to look ahead, to
set out detailed plans for achieving the targets for each department,
operation and (ideally) each manager and to anticipate problems. It
thus prevents management from relying on ad hoc or uncoordinated
planning which may be detrimental to the performance of the
organisation.

Communicate
ideas and plans

A formal system is necessary to ensure that each person affected by
the plans is aware of what he or she is supposed to be doing.
Communication might be one-way, with managers giving orders to
subordinates, or there might be a two-way dialogue and exchange
of ideas.

Coordinate
activities

The activities of different departments or sub-units of the
organisation need to be coordinated to ensure maximum
integration of effort towards common goals. This concept of
Page 333

Objective

Comment
coordination implies, for example, that the purchasing department
should base its budget on production requirements and that the
production budget should in turn be based on sales expectations.
Although straightforward in concept, coordination is remarkably
difficult to achieve, and there is often 'sub-optimality' and conflict
between departmental plans in the budget so that the efforts of each
department are not fully integrated into a combined plan to achieve
the company's best targets.

Provide a
framework for
responsibility
accounting

Budgetary planning and control systems require that managers of
budget centres are made responsible for the achievement of budget
targets for the operations under their personal control.

Establish a system A budget is a yardstick against which actual performance is
measured and assessed. Control over actual performance is provided
of control
by the comparisons of actual results against the budget plan.
Departures from budget can then be investigated and the reasons for
the departures can be divided into controllable and uncontrollable
factors.
Motivate
employees to
improve their
performance

The interest and commitment of employees can be retained via a
system of feedback of actual results, which lets them know how
well or badly they are performing. The identification of controllable
reasons for departures from budget with managers responsible
provides an incentive for improving future performance.
Some argue that motivation can only come from within one’s self;
encouragement and discouragement are the external stimuli

Exam Focus Point
An exam question could well ask you to explain a number of these objectives in the context
of a particular scenario such as a not-for-profit organisation.

Page 334

BEHAVIOURAL IMPLICATIONS OF BUDGETING
Used correctly, a budgetary control system can motivate but it can also produce undesirable
negative reactions.

The purpose of a budgetary control system is to assist management in planning and
controlling the resources of their organisation by providing appropriate control information.
The information will only be valuable, however, if it is interpreted correctly and used
purposefully by managers and employees.
The correct use of control information therefore depends not only on the content of the
information itself, but also on the behaviour of its recipients. This is because control in
business is exercised by people. Their attitude to control information will colour their views
on what they should do with it and a number of behavioural problems can arise.
a) The managers who set the budget or standards are often not the managers who are
then made responsible for achieving budget targets.
b) The goals of the organisation as a whole, as expressed in a budget, may not
coincide with the personal aspirations of individual managers.
c) Control is applied at different stages by different people. A supervisor might get
weekly control reports, and act on them; his superior might get monthly control
reports, and decide to take different control action. Different managers can get in each
others' way, and resent the interference from others.

Motivation
Motivation is what makes people behave in the way that they do. It comes from individual
attitudes, or group attitudes. Individuals will be motivated by personal desires and interests.
These may be in line with the objectives of the organisation, and some people 'live for their
jobs'. Other individuals see their job as a chore, and their motivations will be unrelated to the
objectives of the organisation they work for.
It is therefore vital that the goals of management and the employees harmonise with the goals
of the organisation as a whole. This is known as goal congruence. Although obtaining goal
congruence is essentially a behavioural problem, it is possible to design and run a
Page 335

budgetary control system which will go some way towards ensuring that goal
congruence is achieved. Managers and employees must therefore be favourably disposed
towards the budgetary control system so that it can operate efficiently.
The management accountant should therefore try to ensure that employees have positive
attitudes towards setting budgets, implementing budgets (that is, putting the organisation's
plans into practice) and feedback of results (control information).

Poor attitudes when setting budgets
Poor attitudes or hostile behaviour towards the budgetary control system can begin at the
planning stage. If managers are involved in preparing a budget the following may
happen.
a) Managers may complain that they are too busy to spend much time on budgeting.
b) They may build 'slack' into their expenditure estimates.
c) They may argue that formalising a budget plan on paper is too restricting and that
managers should be allowed flexibility in the decisions they take.
d) They may set budgets for their budget centre and not coordinate their own plans with
those of other budget centres.
e) They may base future plans on past results, instead of using the opportunity for
formalised planning to look at alternative options and new ideas.

On the other hand, managers may not be involved in the budgeting process.
Organisational goals may not be communicated to them and they might have their budget
decided for them by senior management or administrative decision. It is hard for people to
be motivated to achieve targets set by someone else.

Page 336

Poor attitudes when putting plans into action

Poor attitudes can also arise when a budget is implemented.
a) Managers might put in only just enough effort to achieve budget targets, without
trying to beat targets.
b) A formal budget might encourage rigidity and discourage flexibility.
c) Short-term planning in a budget can draw attention away from the longer-term
consequences of decisions.
d) There might be minimal cooperation and communication between managers.
e) Managers will often try to make sure that they spend up to their full budget
allowance, and do not overspend, so that they will not be accused of having asked
for too much spending allowance in the first place.

Poor attitudes and the use of control information
The attitude of managers towards the accounting control information they receive might
reduce the information's effectiveness.
a) Management accounting control reports could well be seen as having a relatively low
priority in the list of management tasks. Managers might take the view that they have
more pressing jobs on hand than looking at routine control reports.
b) Managers might resent control information; they may see it as part of a system of
trying to find fault with their work. This resentment is likely to be particularly
strong when budgets or standards are imposed on managers without allowing them to
participate in the budget-setting process.
c) If budgets are seen as pressure devices to push managers into doing better, control
reports will be resented.
d) Managers may not understand the information in the control reports, because they
are unfamiliar with accounting terminology or principles.
e) Managers might have a false sense of what their objectives should be. A production
manager might consider it more important to maintain quality standards regardless of
cost. He would then dismiss adverse expenditure variances as inevitable and
unavoidable.
Page 337

f) If there are flaws in the system of recording actual costs, managers will dismiss
control information as unreliable.
g) Control information might be received weeks after the end of the period to which
it relates, in which case managers might regard it as out-of-date and no longer useful.
h) Managers might be held responsible for variances outside their control.

It is therefore obvious that accountants and senior management should try to implement
systems that are acceptable to budget holders and which produce positive effects.

Pay as a motivator
Many researchers agree that pay can be an important motivator, when there is a formal
link between higher pay (or other rewards, such as promotion) and achieving budget targets.
Individuals are likely to work harder to achieve budget if they know that they will be
rewarded for their successful efforts. There are, however, problems with using pay as an
incentive.
a) A serious problem that can arise is that formal reward and performance evaluation
systems can encourage dysfunctional behaviour. Many investigations have noted
the tendency of managers to pad their budgets either in anticipation of cuts by
superiors or to make the subsequent variances more favourable. And there are
numerous examples of managers making decisions in response to performance
indices, even though the decisions are contrary to the wider purposes of the
organisation.
b) The targets must be challenging but fair, otherwise individuals will become
dissatisfied. Pay can be a de-motivator as well as a motivator!

Page 338

SETTING THE DIFFICULTY LEVEL OF A BUDGET
‘Aspirations’ budgets can be used as targets to motivate higher levels of performance but a
budget for planning and decision making should be based on reasonable expectations.

Budgets can motivate managers to achieve a high level of performance. But how difficult
should targets be? And how might people react to targets of differing degrees of difficulty in
achievement?
a) There is likely to be a de-motivating effect where an ideal standard of performance
is set, because adverse efficiency variances will always be reported.
b) A low standard of efficiency is also de-motivating, because there is no sense of
achievement in attaining the required standards. If the budgeted level of attainment is
too 'loose', targets will be achieved easily, and there will be no impetus for employees
to try harder to do better than this.
c) A budgeted level of attainment could be the same as the level that has been achieved
in the past. Arguably, this level will be too low. It might encourage budgetary slack.

Academics have argued that each individual has a personal 'aspiration level'. This is a level
of performance in a task with which the individual is familiar, which the individual
undertakes for himself to reach.
Individual aspirations might be much higher or much lower than the organisation's
aspirations, however. The solution might therefore be to have two budgets.
a) A budget for planning and decision making based on reasonable expectations
b) A budget for motivational purposes, with more difficult targets of performance

These two budgets might be called an 'expectations budget' and an 'aspirations budget'
respectively.

Page 339

BLANK

Page 340

PARTICIPATION IN BUDGETING
A budget can be set from the top down (imposed budget) or from the bottom up
(participatory budget). Many writers refer to a third style, the negotiated budget.

Participation
It has been argued that participation in the budgeting process will improve motivation and
so will improve the quality of budget decisions and the efforts of individuals to achieve their
budget targets (although obviously this will depend on the personality of the individual, the
nature of the task (narrowly defined or flexible) and the organisational culture).
There are basically two ways in which a budget can be set: from the top down (imposed
budget) or from the bottom up (participatory budget).

Imposed style of budgeting (top-down budgeting)
In this approach to budgeting, top management prepare a budget with little or no input
from operating personnel which is then imposed upon the employees who have to work to
the budgeted figures.
The times when imposed budgets are effective are as follows.
•
•
•
•
•

In newly-formed organisations
In very small businesses
During periods of economic hardship
When operational managers lack budgeting skills
When the organisation's different units require precise coordination

Page 341

There are, of course, advantages and disadvantages to this style of setting budgets.
Advantages
•
•
•
•
•

Strategic plans are likely to be incorporated into planned activities
They enhance the coordination between the plans and objectives of divisions
They use senior management's awareness of total resource availability
They decrease the input from inexperienced or uninformed lower-level
employees
They decrease the period of time taken to draw up the budgets

Disadvantages
•

Dissatisfaction, defensiveness and low morale amongst employees
•
The feeling of team spirit may disappear
•
The acceptance of organisational goals and objectives could be limited
•
The feeling of the budget as a punitive device could arise
•
Unachievable budgets for overseas divisions could result if consideration is not
given to local operating and political environments
•
Lower-level management initiative may be stifled

Participative style of budgeting (bottom-up budgeting)
In this approach to budgeting, budgets are developed by lower-level managers who then
submit the budgets to their superiors. The budgets are based on the lower-level managers'
perceptions of what is achievable and the associated necessary resources.
Participative budgets are effective in the following circumstances.
•
•
•
•
•

In well-established organisations
In very large businesses
During periods of economic affluence
When operational managers have strong budgeting skills
When the organisation's different units act autonomously

Page 342

The advantages of participative budgets are as follows.
•
•
•
•
•
•
•
•

They are based on information from employees most familiar with their unit of
operation
Knowledge spread among several levels of management is pulled together
Morale and motivation are improved
They increase operational managers' commitment to organisational objectives
In general they are more realistic
Co-ordination between units is improved
Specific resource requirements are included
Senior managers' overview is mixed with operational level details

There are, on the other hand, a number of disadvantages of participative budgets.
•
•
•
•
•
•

They often consume more time
Changes implemented by senior management may cause dissatisfaction
Budgets may be unachievable if managers' are not qualified to participate
They may cause managers to introduce budgetary slack
They can support 'empire building' by subordinates
An earlier start to the budgeting process could be required

Negotiated style of budgeting
At the two extremes, budgets can be dictated from above or simply emerge from below but,
in practice, different levels of management often agree budgets by a process of negotiation. In
the imposed budget approach, operational managers will try to negotiate with senior
managers the budget targets which they consider to be unreasonable or unrealistic. Likewise
senior management usually review and revise budgets presented to them under a participative
approach through a process of negotiation with lower level managers. Final budgets are
therefore most likely to lie between what top management would really like and what
junior managers believe is feasible. The budgeting process is hence a bargaining process
and it is this bargaining which is of vital importance, determining whether the budget is an
effective management tool or simply a clerical device.
Page 343

BLANK

Page 344

MUTUALLY EXCLUSIVE PROJECTS WITH
UNEQUAL LIVES
All of the discounted cash flow examples that we have seen so far have involved a choice
between projects with equal lives. However, if manager are deciding between projects with
different time spans a direct comparison of the NPV generated by each project would
not be valid.
For example if an organisation decides to invest in a project with a shorter life it may then
have the opportunity to invest in a new project in the future sooner than if a longer term
project is accepted. This should be taken into account in the analysis in order to be able to
make direct comparisons between projects with unequal lives.

Annualised equivalents are used to enable a comparison made between the net present
values of projects with different durations. However, this method cannot be used when
inflation is a factor. Another method, the lowest common multiple method, is used
instead.

Example: annualised equivalents
An organisation has the opportunity to invest in either Project G or Project H. The forecast
from the cash flows from the projects are as follows:
Project G

Project H

RWF m

RWF m

(200)

(143)

Year 1

90

100

Year 2

120

80

Year 3

50

-

Capital cost
Cash inflows:

The company’s cost of capital is 12%. Which project should be accepted?
Page 345

Solution
Project G
Year

Cash flow

PV factor

PV of cash flow

RWF ‘000

12%

RWF ‘000

0

(200,000)

1.000

(200,000)

1

90,000

0.893

80,370

2

120,000

0.797

95,640

3

50,000

0.712

35,600
NPV = 11,610

Project H
Year

Cash flow

PV factor

PV of cash flow

RWF ‘000

12%

RWF ‘000

0

(143,000)

1.000

(143,000)

1

100,000

0.893

89,300

2

80,000

0.797

63,790
NPV = 10,060

These NPV’s cannot be compared directly because they each relate to a different number of
years. In order to make a comparison we must convert each NPV to an annualised
equivalent cost. In other words, we convert the project’s NPV into an equivalent annual
annuity over its expected life. We do this by using cumulative discount factors.
Page 346

NPV at 12%
Cumulative 12% discount factor
Annualised equivalent

Project G

Project H

RWF 11,610,000

RWF10,060,000

÷ 2.402

÷ 1.69

RWF4,833,470

RWF5,952,665

Project H is offering an equivalent annual annuity of RWF5,952,665 which is higher than that
offered by project G, therefore project H is preferable.

Exam Focus Point
When inflation is a factor, LCM must be used rather than annualised equivalents.

Page 347

Example: Lowest Common Multiple (LCM)
Where asset replacement includes inflation you would not be able to use annualised
equivalent costs. The correct method, lowest common multiple, is the one to use. The key
points when using the lowest common multiple method are:
a) Calculate cash flows including inflated values for both alternatives
b) Use the lowest common multiple to establish a common time period and base asset
lives on that

Fred is considering the replacement of a caravan he lets out for hire. He is planning to retire
in six years time and is therefore only concerned with that period of time, but cannot decide
whether it is better to replace the caravan every two years or every three years.
The following data have been estimated (all values at today’s price levels):

Purchase cost and trade-in values
RWF ‘000
Cost of a new caravan
Trade-in value of caravan:

20,000
after two years

10,000

after three years

5,000

Annual costs and revenues
Per year
RWF ‘000
Caravan running costs

10,000

Lettings charged to customers, that is revenue for Fred

20,000

Page 348

Caravan servicing and repair costs
Caravan servicing and repairs costs depend on the age of the caravan. In the following table,
year 1 represents the cost in the first year of the caravan ownership; year 2 represents the cost
in the second year of ownership, and so on:
RWF ‘000
Year 1

500

Year 2

2,500

Year 3

4,000

Inflation
New caravan costs and trade in values are expected to increase by 5% per year. Caravan
running costs and lettings are expected to increase by 7% per year. Caravan servicing and
repair costs are expected to increase by 10% per year.

Required
Advise Fred on the optimum replacement cycle for his caravan and state the net present value
of the opportunity cost of making the wrong decision. Use a discount rate of 12% per year.
All workings and assumptions should be shown. Ignore taxation.

Page 349

Solution
In this example you need to consider a six-year time horizon, six being the lowest common
multiple of two and three.
Projected cash flows – 2 year trade in
Year 0

Year 1

Year 2

Year 3

Year 4

Year 5 Year 6

RWF
‘000

RWF
‘000

RWF
‘000

RWF
‘000

RWF
‘000

RWF
‘000

RWF
‘000

Caravan cost
(+5% pa)

(20,000)

Trade in value (+5%
pa)

(22,050)

(24,310)

11,025

12,155

13,401

Annual costs and
revenues (net of
costs) (+7%pa)

10,700

11,449

12,250

13,108

14,02
6

15,007

Servicing and repair
(+10% pa)
______

(550)

(3,025)

(666)

(3,660)

(805)

(4,429)

Net cash flow

(20,000)

10,150

(2,601)

11,584

(2,707)

13,22
1

23,979

Discount at 12%

x 1.000

x 0.893

x 0.797

x 0.712

x 0.636

x
0.567

x 0.507

PV of cash flow

(20,000)

9,064

(2,073)

8,248

(1,722)

7,496

12,157

NPV of cash flow = RWFk 13,170

Projected cash flows – 2 year trade in
Page 350

Year 0

Year 1

Year 2

Year 3

Year 4

Year 5

Year 6

RWF
‘000

RWF
‘000

RWF
‘000

RWF
‘000

RWF
‘000

RWF
‘000

RWF
‘000

Caravan cost
(20,000)

(23,153)

(+5% pa)
Trade in value
(+5% pa)

5,788

6,700

Annual costs and
revenues (net of
costs) (+7%pa)

10,700

11,449

12,250

13,108

14,026

15,007

Servicing and
repair (+10% pa)

(550)

(3,025)

(5,324)

(732)

(4,026)

(7,088)

Net cash flow

(20,000)

10,150

8,424

(10,439)

12,376

10,000

14,621

Discount at 12%

x 1.000

x 0.893

x 0.797

x 0.712

x 0.636

x 0.567

x 0.507

PV of cash flow

(20,000)

9,064

6,714

(7,433)

7,871

5,670

7,413

NPV of cash flow = RWF k 9,299
Assumptions: inflation applies from year 0 to all costs and revenues, which are stated at their
values in year 0 in the question.
Based on NPVs of the two alternative replacement cycles, that with the higher positive NPV
is the two-year replacement cycle and so this should be chosen as the optimum replacement
cycle.

Page 351

Not all mutually exclusive investments need to be considered over the same level of time. It
very much depends on what the organisation intends to do once the shorter-life project ends.
If the organisation has to invest in similar assets again at that point, the projects should be
compared over equal time periods. Investment in manufacturing equipment for a product
that will be made for more years than the life of an asset is an example.

If the organisation does not have to invest in similar assets when the asset’s life ends,
however, the approach we have described is not needed. If the investments are alternative
advertising campaigns for a short-life product such as a commemorative item, the
investments will be one-offs and so can be compared over different lives.

Page 352

CHAPTER ROUNDUP
•

Corporate objectives concern the firm as a whole. Unit objectives are specific to
individual units , divisions or functions of an organisation.

•

Primary corporate objectives are supported by secondary objectives, for example
for product development or market share. In practice there may be a trade off between
different objectives.

•

The planning and control cycle has seven steps.
–
–
–
–
–
–
–

Step 1. Identify objectives
Step 2. Identify potential strategies
Step 3. Evaluate strategies – SWOT analysis
Step 4. Choose alternative courses of action
Step 5. Implement the long-term plan
Step 6. Measure actual results and compare with the plan
Step 7. Respond to divergences from the plan

•

Planning and control occurs at all levels of the performance hierarchy to different
degrees.

•

Budgetary control occurs at the lower levels of the performance hierarchy.

•

Here are the objectives of a budgetary planning and control system.
–
–
–
–
–
–
–

Ensure the achievement of the organisation's objectives
Compel planning
Communicate ideas and plans
Coordinate activities
Provide a framework for responsibility accounting
Establish a system of control
Motivate employees to improve their performance

•

Used correctly, a budgetary control system can motivate but it can also produce
undesirable negative reactions.

•

‘Aspirations’ budgets can be used as targets to motivate higher levels of
performance but a budget for planning and decision making should be based on
reasonable expectations.

Page 353

•

A budget can be set from the top down (imposed budget) or from the bottom up
(participatory budget). Many writers refer to a third style, the negotiated budget.

Page 354

STUDY UNIT 13
Budgetary Systems
Contents

Page

Traditional Budgetary Systems …………………………………………….

357

Fixed And Flexible Budgets ……….…………………………..……..……..

361

Zero Based Budgeting Systems …………………………………………….

363

Activity Based Budgeting …………………………………………...………

369

Rolling Budgets …………………………………………...…………………

373

Beyond Budgeting ……………………………………………………………

377

Information Used In Budget Systems ………………….……..……………

381

Changing Budgetary Systems ………………….……………..……………

383

Budget Systems And Uncertainty ………………….……..…..……………

385

Page 355

EXAM GUIDE
The examiner expects you to be aware of the problems of traditional budgetary systems and
why organisations may be reluctant to change to more appropriate systems.

Page 356

TRADITIONAL BUDGETARY SYSTEMS
A budget is a quantified plan of action for a forthcoming accounting period.
A budget can be set from the top down (imposed budget) or from the bottom up
(participatory budget).

Budget preparation
You will have covered budget preparation in your earlier studies and will not be required to
prepare sales, production, materials etc budgets in this exam.
The following are the key points of budget preparation to remind you.
Point

Detail

Long-term plan

The starting point, this will show what the budget has to achieve
(the introduction of new production, the required return, and so on)
and outline how it is to be done. It will also contain general
guidelines on allowable price increases such as wage rates. The longterm policy needs to be communicated to all managers responsible
for preparing budgets so that they are aware of the context within
which they are budgeting and how their area of responsibility is
expected to contribute.

Limiting factor

The factor that limits the scale of operations, this is usually sales
demand, but it may be production capacity where demand is high.
Budgeting cannot proceed until the budget for the limiting factor has
been prepared, since this affects all the other budgets.

Budget manual

Prepared to assist functional managers, this will show how figures
and forecasts are to be arrived at and give any other information that
is to apply across the organisation. It is likely to include proformas
showing how the information is to be presented. If budgeting is done
with spreadsheets, layouts and computations may be pre-programmed,
requiring only the entry of the figures. It may include a flow diagram
showing how individual budgets are interlinked and specify deadlines
by which first drafts must be prepared.

Sales budget

This contains information on the expected volume of sales (based
on estimates or market research), the sales mix, and selling prices.
The total revenues indicated will be used to compile the cash budget,
Page 357

although this information needs to be adjusted to allow for the
expected timing of receipts. The volume of sales indicates the level of
production required and the extent of spending on distribution and
administration.
Production capacity

The level of sales anticipated is matched against opening inventory
and desired closing inventory to establish the level of production.
From this can be calculated the need for materials (again allowing for
opening and closing inventory), labour and machine hours. In other
words production budgeting is done in terms of physical resources
initially and costed afterwards. At this stage, too, it is likely that
needs for new capital expenditure will be identified. This information
will be used in preparing the capital budget.

Functional budgets

Budgets for other areas of the organisation like distribution and
administration take the anticipated sales level as their point of
reference. Vehicle costs, carriage/distribution costs, stationery, IT and
communication costs, and above all staff costs feature in these
budgets.

Discretionary costs

Training and R&D are known as 'discretionary costs' and have
special features.

Consolidation
coordination

and This can begin once all parts of the organisation have submitted their
individual budgets. It is most unlikely that all of the budgets will be
in line with each other at the first attempt. Areas of incompatibility
must be identified and the budgets modified in consultation with
individual managers. Spreadsheets are invaluable at this stage, both
for the consolidation itself and to allow changes to be made quickly
and accurately.

Cash budget

This can only be prepared at this stage because it needs to take
account of all of the plans of the organisation and translate them
into expected cash flows. Cash must be available when it is needed to
enable the plans to be carried out. Overdraft facilities may need to be
negotiated in advance, or some activities may need to be deferred
until cash has been collected.

Master budget

The final stage, once all of the necessary modifications have been
made, is to prepare a summary of all of the budgets in the form of a
master budget, which generally comprises a budgeted income
statement, a budgeted balance sheet and a budgeted cash flow
statement.

Page 358

Incremental budgeting
The traditional approach to budgeting, known as incremental budgeting, bases the budget
on the current year's results plus an extra amount for estimated growth or inflation next year.
It encourages slack and wasteful spending to creep into budgets.
Incremental budgeting is so called because it is concerned mainly with the increments in
costs and revenues which will occur in the coming period.
Incremental budgeting is a reasonable procedure if current operations are as effective,
efficient and economical as they can be. It is also appropriate for budgeting for costs such as
staff salaries, which may be estimated on the basis of current salaries plus an increment for
inflation and are hence administratively fairly easy to prepare.
In general, however, it is an inefficient form of budgeting as it encourages slack and
wasteful spending to creep into budgets. Past inefficiencies are perpetuated because cost
levels are rarely subjected to close scrutiny.

Question
Can incremental budgeting be used to budget for rent? What about for advertising
expenditure?

Answer
Incremental budgeting is appropriate for budgeting for rent, which may be estimated on the
basis of current rent plus an increment for the annual rent increase. Advertising expenditure,
on the other hand, is not so easily quantifiable and is more discretionary in nature. Using
incremental budgeting for advertising expenditure could allow slack and wasteful spending to
creep into the budget.

Incremental budgeting in the public sector
The traditional approach to budgeting in the public sector has been incremental and this has
resulted in existing patterns of public expenditure being locked in. For instance, the public
spending round in the UK established an annual cycle of year-on-year incremental bids by
departments rather than an analysis of outputs and efficiency. How is the annual government
budgeting process carried out in Rwanda?

Page 359

We will look at public sector objectives and performance measurement in more detail in
Study Unit 20.

Page 360

FIXED AND FLEXIBLE BUDGETS
Fixed budgets remain unchanged regardless of the level of activity; flexible budgets are
designed to flex with the level of activity.

Fixed budgets
A fixed budget is a budget which is designed to remain unchanged regardless of the volume
of output or sales achieved.

The master budget prepared before the beginning of the budget period is known as the fixed
budget. The term 'fixed' means the following.
a) The budget is prepared on the basis of an estimated volume of production and an
estimated volume of sales, but no plans are made for the event that actual volumes of
production and sales may differ from budgeted volumes.
b) When actual volumes of production and sales during a control period (month or four
weeks or quarter) are achieved, a fixed budget is not adjusted (in retrospect) to the
new levels of activity.
The major purpose of a fixed budget is at the planning stage, when it seeks to define the
broad objectives of the organisation.

Flexible budgets
A flexible budget is a budget which, by recognising different cost behaviour patterns, is
designed to change as volumes of output change.

Flexible budgets may be used in one of two ways.
a) At the planning stage. For example, suppose that a company expects to sell 10,000
units of output during the next year. A master budget (the fixed budget) would be
prepared on the basis of these expected volumes. However, if the company thinks that
output and sales might be as low as 8,000 units or as high as 12,000 units, it may

Page 361

prepare contingency flexible budgets, at volumes of, say 8,000, 9,000, 11,000 and
12,000 units and then assess the possible outcomes.
b) Retrospectively. At the end of each month (control period) or year, the results that
should have been achieved given the actual circumstances (the flexible budget) can be
compared with the actual results. As we shall see, flexible budgets are an essential
factor in budgetary control.
The preparation and use of flexible budgets were looked at in more detail in Chapters 9 & 10
Linear Programming.

Page 362

ZERO BASED BUDGETARY SYSTEMS
The principle behind zero based budgeting (ZBB) is that the budget for each cost centre
should be made from 'scratch' or zero. Every item of expenditure must be justified in its
entirety in order to be included in the next year's budget.
ZBB, in theory, rejects the assumption inherent in incremental budgeting; that this year's
activities will continue at the same level or volume next year, and that next year's budget can
be based on this year's costs plus an extra amount, perhaps for expansion and inflation.

Zero based budgeting involves preparing a budget for each cost centre from a zero base.
Every item of expenditure has then to be justified in its entirety in order to be included in the
next year's budget.

In reality, however, managers do not have to budget from zero, but can start from their
current level of expenditure and work downwards, asking what would happen if any
particular aspect of current expenditure and current operations were removed from the
budget. In this way, every aspect of the budget is examined in terms of its cost and the
benefits it provides and the selection of better alternatives is encouraged.

Implementing zero based budgeting
There is a three-step approach to ZBB.
•
•
•

Define decision units
Evaluate and rank packages
Allocate resources

The implementation of ZBB involves a number of steps but of greater importance is the
development of a questioning attitude by all those involved in the budgetary process.
Existing practices and expenditures must be challenged and searching questions asked.
•
•

Does the activity need to be carried out?
What would be the consequences if the activity was not carried out?
Page 363

•
•
•
•

Is the current level of provision current?
Are there alternative ways of providing the function?
How much should the activity cost?
Is the expenditure worth the benefits achieved?

The basic approach of ZBB has three steps.
Step 1

Define decision packages, comprehensive descriptions of specific organisational
activities which management can use to evaluate the activities and rank them in
order of priority against other activities. There are two types.
a) Mutually exclusive packages contain alternative methods of getting the
same job done. The best option among the packages must be selected by
comparing costs and benefits and the other packages are then discarded.
b) Incremental packages divide one aspect of an activity into different
levels of effort. The 'base' package will describe the minimum amount of
work that must be done to carry out the activity and the other packages
describe what additional work could be done, at what cost and for what
benefits.
Suppose that a cost centre manager is preparing a budget for maintenance costs.
He might first consider two mutually exclusive packages.
•

Package A might be to keep a maintenance team of two men per shift for
two shifts each day at a cost of Rwf6,000,000 per annum

•

Package B might be to obtain a maintenance service from an outside
contractor at a cost of Rwf5,000,000

A cost-benefit analysis will be conducted because the quicker repairs obtainable
from an in-house maintenance service might justify its extra cost. If we now
suppose that package A is preferred, the budget analysis must be completed by
describing the incremental variations in this chosen alternative.
a) The 'base' package would describe the minimum requirement for the
maintenance work. This might be to pay for one man per shift for two
shifts each day at a cost of RWF3,000,000.
b) Incremental package 1 might be to pay for two men on the early shift and
one man on the late shift, at a cost of RWF4,500,000. The extra cost of
RWF1,500,000 would need to be justified, for example by savings in lost
production time, or by more efficient machinery.
Page 364

c) Incremental package 2 might be the original preference, for two men on
each shift at a cost of RWF6,000,000. The cost-benefit analysis would
compare its advantages, if any, over incremental package 1; and so on.
Step 2

Evaluate and rank each activity (decision package) on the basis of its benefit
to the organisation. This can be a lengthy process. Minimum work requirements
(those that are essential to get a job done) will be given high priority and so too
will work which meets legal obligations. In the accounting department these
would be minimum requirements to operate the payroll, purchase ledger and sales
ledger systems, and to maintain and publish a set of accounts.

Step 3

Allocate resources in the budget according to the funds available and the
evaluation and ranking of the competing packages.

Question
What might the base and incremental packages for a personnel department cover?

Answer
The base package might cover the recruitment and dismissal of staff. Incremental packages
might cover training, pension administration, trade union liaison, staff welfare and so on.

The advantages and limitations of implementing ZBB
The advantages of zero based budgeting are as follows.
•
•
•
•
•
•
•

It is possible to identify and remove inefficient or obsolete operations.
It forces employees to avoid wasteful expenditure.
It can increase motivation.
It responds to changes in the business environment.
ZBB documentation provides an in-depth appraisal of an organisation's
operations.
It challenges the status quo.
In summary, ZBB should result in a more efficient allocation of resources.
Page 365

The major disadvantage of zero based budgeting is the volume of extra paperwork created.
The assumptions about costs and benefits in each package must be continually updated and
new packages developed as soon as new activities emerge.
The following problems might also occur.
a) Short-term benefits might be emphasised to the detriment of long-term benefits.
b) It might give the impression that all decisions have to be made in the budget.
Management must be able to meet unforeseen opportunities and threats at all times,
however, and must not feel restricted from carrying out new ideas simply because
they were not approved by a decision package, cost benefit analysis and the ranking
process.
c) It may call for management skills both in constructing decision packages and in the
ranking process which the organisation does not possess. Managers may have to be
trained in ZBB techniques.
d) The organisation's information systems may not be capable of providing suitable
information.
e) The ranking process can be difficult. Managers face three common problems.
(i)

A large number of packages may have to be ranked.

(ii)

It can be difficult to rank packages which appear to be equally vital, for legal or
operational reasons.

(iii) It is difficult to rank activities which have qualitative rather than quantitative
benefits – such as spending on staff welfare and working conditions.

In summary, perhaps the most serious drawback to ZBB is that it requires a lot of
management time and paperwork. One way of obtaining the benefits of ZBB but of
overcoming the drawbacks is to apply it selectively on a rolling basis throughout the
organisation. This year finance, next year marketing, the year after personnel and so on. In
this way all activities will be thoroughly scrutinised over a period of time.

Page 366

Using zero based budgeting
ZBB is particularly useful for budgeting for discretionary costs and for rationalisation
purposes.
ZBB is not particularly suitable for direct manufacturing costs, which are usually budgeted
using standard costing, work study and other management planning and control techniques. It
is best applied to support expenses, that is expenditure incurred in departments which exist
to support the essential production function. These support areas include marketing, finance,
quality control, HR/personnel, IT/data processing, sales and distribution. In many
organisations, these expenses make up a large proportion of the total expenditure. These
activities are less easily quantifiable by conventional methods and are more discretionary in
nature.
ZBB can also be successfully applied to service industries and non-profit-making
organisations such as local and central government departments, educational establishments,
hospitals and so on, and in any organisation where alternative levels of provision for each
activity are possible and where the costs and benefits are separately identifiable.
ZBB can also be used to make rationalisation decisions. 'Rationalisation' is often a
euphemism for cutting back on production and activity levels and cutting costs. The need for
service departments to operate above a minimum service level, or the need for having a
particular department at all, can be questioned and ZBB can be used to make rationalisation
decisions when an organisation is forced to make spending cuts.

Page 367

BLANK

Page 368

ACTIVITY BASED BUDGETING
At its simplest, activity based budgeting (ABB) is merely the use of costs determined using
ABC as a basis for preparing budgets.

Activity based budgeting involves defining the activities that underlie the financial figures
in each function and using the level of activity to decide how much resource should be
allocated, how well it is being managed and to explain variances from budget.

Implementing ABC (see Chapter 1) leads to the realisation that the business as a whole
needs to be managed with far more reference to the behaviour of activities and cost drivers
identified. For example, traditional budgeting may make managers 'responsible' for activities
which are driven by factors beyond their control: the personnel department cost of setting up
new employee records is driven by the number of new employees required by managers other
than the personnel manager.

Principles of ABB
ABB involves defining the activities that underlie the financial figures in each function and
using the level of activity to decide how much resource should be allocated, how well it is
being managed and to explain variances from budget.
ABB is therefore based on the following principles.
a) It is activities which drive costs and the aim is to control the causes (drivers) of
costs rather than the costs themselves, with the result that in the long term, costs will
be better managed and better understood.
b) Not all activities add value and so activities must be examined and split up according
to their ability to add value.
c) Most departmental activities are driven by demands and decisions beyond the
immediate control of the manager responsible for the department's budget.

Page 369

d) Traditional financial measures of performance are unable to fulfil the objective of
continuous improvement. Additional measures which focus on drivers of costs, the
quality of activities undertaken, the responsiveness to change and so on are needed.

Example: ABB
A stores department has two main activities, receiving deliveries of raw materials from
suppliers into stores and issuing raw materials to production departments. Two major cost
drivers, the number of deliveries of raw materials and the number of production runs, have
been identified. Although the majority of the costs of the department can be attributed to
these activities, there is a small balance, termed 'department running costs', which includes
general administration costs, part of the department manager's salary and so on.
Based on activity levels expected in the next control period, the following cost driver
volumes have been budgeted.
250 deliveries of raw materials
120 production runs
On the basis of budgeted departmental costs and the cost analysis, the following budget has
been drawn up for the next control period.

Total

Cost
Salaries – management
Salaries – store workers
Salaries – administration
Consumables
Information technology costs
Other costs
Activity volumes
Cost per unit of cost driver

RWF m
25
27
15
11
14
19
111

Costs
attributable
to receiving
deliveries

Costs
attributable
to issuing
materials

running
costs

RWF m

RWFm

RWF m

8
13
4
3
5
10
43
250
RWF k 172

12
12
5
5
8
6
48
120
RWF k 400

Page 370

Dept

5
2
6
3
1
3
20
RWFk

20,000
Points to note
a) The apportionment of cost will be subjective to a certain extent. The objective of the
exercise is that the resource has to be justified as supporting one or more of the
activities. Costs cannot be hidden.
b) The cost driver rates of RWF172,000 and RWF400,000 can be used to calculate product
costs using ABC.
c) Identifying activities and their costs helps to focus attention on those activities which
add value and those that do not.
d) The budget has highlighted the cost of the two activities.
e) In larger more complex organisations, the apportionment % or fractions will be given
by senior management; viz HR or IT departmental costs – bearing in mind that each
of these uses the other’s services.

Benefits of ABB
Some writers treat ABB as a complete philosophy in itself and attribute to it all the good
features of strategic management accounting, zero base budgeting, total quality management
and other ideas. For example, the following claims have been made.
a) Different activity levels will provide a foundation for the 'base' package and
incremental packages of ZBB.
b) It will ensure that the organisation's overall strategy and any actual or likely changes
in that strategy will be taken into account, because it attempts to manage the business
as the sum of its interrelated parts.
c) Critical success factors will be identified and performance measures devised to
monitor progress towards them. (A critical success factor is an activity in which a
business must perform well if it is to succeed).
d) Because concentration is focused on the whole of an activity, not just its separate
parts, there is more likelihood of getting it right first time. For example what is the
use of being able to produce goods in time for their despatch date if the budget
provides insufficient resources for the distribution manager who has to deliver them?
Page 371

BLANK

Page 372

ROLLING BUDGETS
Rolling budgets (continuous budgets) are budgets which are continuously being updated by
adding a further period (say a month or a quarter) and dropping the earliest period.

Dynamic conditions
Actual conditions may differ from those anticipated when the budget was drawn up for
a number of reasons.
a) Organisational changes may occur.
(i)

A change in structure, from a functional basis, say, to a process-based one

(ii)

New agreements with the workforce about flexible working or safety
procedures

(iii) The reallocation of responsibilities following, say, the removal of tiers of
middle management and the 'empowerment' of workers further down the line

b) Action may be needed to combat an initiative by a competitor.
c) New technology may be introduced to improve productivity, reduce labour
requirements or enhance quality.
d) Environmental conditions may change: there may be a general boom or a recession,
an event affecting supply or demand, or a change in government or government
policy.
e) The level of inflation may be higher or lower than that anticipated.
f) The level of activities may be different from the levels planned.

Any of these changes may make the original budget quite inappropriate, either in terms of
the numbers expected, or the way in which responsibility for achieving them is divided, or
both.
Page 373

If management needs the chance to revise their plans, they may decide to introduce a system
of rolling budgets.
A rolling budget is a budget which is continuously updated by adding a further accounting
period (a month or quarter) when the earlier accounting period has expired.

Rolling budgets are an attempt to prepare targets and plans which are more realistic and
certain, particularly with a regard to price levels, by shortening the period between
preparing budgets.
Instead of preparing a periodic budget annually for the full budget period, there would be
budgets every one, two, three or four months (three to six, or even twelve budgets each
year). Each of these budgets would plan for the next twelve months so that the current
budget is extended by an extra period as the current period ends: hence the name rolling
budgets.
Suppose, for example, that a rolling budget is prepared every three months. The first three
months of the budget period would be planned in great detail, and the remaining nine months
in lesser detail, because of the greater uncertainty about the longer-term future. If a first
continuous budget is prepared for January to March in detail and April to December in less
detail, a new budget will be prepared towards the end of March, planning April to June in
detail and July to March in less detail. Four rolling budgets would be prepared every 12
months on this 3 and 9 month basis, requiring, inevitably, greater administrative effort.

The advantages and disadvantages of rolling budgets
The advantages are as follows.
a) They reduce the element of uncertainty in budgeting because they concentrate
detailed planning and control on short-term prospects where the degree of uncertainty
is much smaller.
b) They force managers to reassess the budget regularly, and to produce budgets which
are up to date in the light of current events and expectations.
c) Planning and control will be based on a recent plan which is likely to be far more
realistic than a fixed annual budget made many months ago.
d) Realistic budgets are likely to have a better motivational influence on managers.
Page 374

e) There is always a budget which extends for several months ahead. For example, if
rolling budgets are prepared quarterly there will always be a budget extending for the
next 9 to 12 months. This is not the case when fixed annual budgets are used.
The disadvantages of rolling budgets can be a deterrent to using them.
a) They involve more time, effort and money in budget preparation.
b) Frequent budgeting might have an off-putting effect on managers who doubt the
value of preparing one budget after another at regular intervals.
c) Revisions to the budget might involve revisions to standard costs too, which in turn
would involve revisions to stock valuations. This could replace a large
administrative effort from the accounts department every time a rolling budget is
prepared.

Continuous budgets or updated annual budgets
If the expected changes are not likely to be continuous there is a strong argument that routine
updating of the budget is unnecessary. Instead the annual budget could be updated
whenever changes become foreseeable, so that a budget might be updated once or twice,
and perhaps more often, during the course of the year.
When a fixed budget is updated, a 'rolling' budget would probably not be prepared. If a
budget is updated in month 8 of the year, the updated budget would relate to months 8 – 12. It
would not be extended to month 7 of the following year.

Page 375

BLANK

Page 376

BEYOND BUDGETING
Beyond Budgeting is a model that proposes that traditional budgeting should be abandoned.
Adaptive management processes should be used rather than fixed annual budgets.

Criticisms of budgeting
In our discussion of the budgetary planning process we have come across many difficulties
with budgets and criticisms of how they are used in organisations.
The Beyond Budgeting Round Table (BBRT), an independent research collaborative,
proposes that budgeting, as most organisations practise it, should be abandoned. Their
website at www.bbrt.org lists the following ten criticisms of budgeting as put forward by
Hope and Fraser Beyond Budgeting, 1st edition, Harvard Business School Press, 2003.

a) Budgets are time consuming and expensive. Even with the support of computer
models it is estimated that the budgeting process uses up to 20 to 30 per cent of senior
executives’ and financial managers’ time.
b) Budgets provide poor value to users. Although surveys have shown that some
managers feel that budgets give them control, a large majority of financial directors
wish to reform the budgetary process because they feel that finance staff spend too
much time on 'lower value added activities'.
c) Budgets fail to focus on shareholder value. Most budgets are set on an incremental
basis as an acceptable target agreed between the manager and the manager’s superior.
Managers may be rewarded for achieving their short term budgets and will not look to
the longer term or take risks, for fear of affecting their own short term results.
d) Budgets are too rigid and prevent fast response. Although most organisations do
update and revise their budgets at regular intervals as the budget period proceeds the
process is often too slow compared with the pace at which the external environment is
changing.
e) Budgets protect rather than reduce costs. Once a manager has an authorised budget
he can spend that amount of resource without further authorisation. A ‘use it or lose
it’ mentality often develops so that managers will incur cost unnecessarily. This
Page 377

happens especially towards the end of the budget period in the expectation that
managers will not be permitted to carry forward any unused resource into the budget
for next period.
f) Budgets stifle product and strategy innovation. The focus on achieving the budget
discourages managers from taking risks in case this has adverse effects on their short
term performance. Managers do not have the freedom to respond to changing
customer needs in a fast changing market because the activity they would need to
undertake is not authorised in their budget.
g) Budgets focus on sales targets rather than customer satisfaction. The achievement
of short term sales forecasts becomes the focus of most organisations. However this
does not necessarily result in customer satisfaction. The customer may be sold
something inappropriate to their needs, as in recent years in the UK financial
services industry. Alternatively if a manager has already met the sales target for a
particular period they might try to delay sales to the next period, in order to give
themselves a ‘head start’ towards achieving the target for the next period.
Furthermore, there is an incentive towards the end of a period, if a manager feels that
the sales target is not going to be achieved for the period, to delay sales until the next
period, and thus again have a head start towards achieving the target for the next
period. All of these actions, focusing on sales targets rather than customer
satisfaction, can have a detrimental effect on the organisation in the longer term.
h) Budgets are divorced from strategy. Most organisations monitor the monthly results
against the short term budget for the month. What is needed, instead, is a system of
monitoring the longer term progress against the organisation’s strategy.
i) Budgets reinforce a dependency culture. The process of planning and budgeting
within a framework devolved from senior management perpetuates a culture of
dependency. Traditional budgeting systems, operated on a centralised basis, do not
encourage a culture of personal responsibility.
j) Budgets lead to unethical behaviour. For example building slack into the budget in
order to create an easier target for achievement.

Page 378

Beyond Budgeting concepts
Two fundamental concepts underlie the Beyond Budgeting approach.
a) Use adaptive management processes rather than the more rigid annual budget.
Traditional annual plans tie managers to predetermined actions which are not
responsive to current situations. Managers should instead be planning on a more
adaptive, rolling basis but with the focus on cash forecasting rather than purely on
cost control. Performance is monitored against world-class benchmarks, competitors
and previous periods.
b) Move towards devolved networks rather than centralised hierarchies. The
emphasis is on encouraging a culture of personal responsibility by delegating decision
making and performance accountability to line managers.

Page 379

BLANK

Page 380

INFORMATION USED IN BUDGET SYSTEMS
Information used in budget systems will come from a wide variety of sources.
Past data may be used as a starting point for the preparation of budgets but other information
from a wide variety of sources will also be used. Each function of the organisation will be
required to estimate revenue and expenditure for the budget period. For example, marketing,
personnel and research and development.

Sales budget information
As we have seen, for many organisations, the principal budget factor is sales volume. The
sales budget is therefore often the primary budget from which the majority of the other
budgets are derived. Before the sales budget can be prepared a sales forecast has to be made.
Sales forecasting is complex and difficult and involves the use of information from a variety
of sources.
•
•
•
•
•
•

Past sales patterns
The economic environment
Results of market research
Anticipated advertising
Competition
Changing consumer taste

•
•
•
•
•

New legislation
Distribution
Pricing policies and discounts offered
Legislation
Environmental factors

Production budget information
Sources of information for the production budget will include:
a) Labour costs including idle time, overtime and standard output rates per hour.
b) Raw material costs including allowances for losses during production.
c) Machine hours including expected idle time and expected output rates per machine
hour.
Page 381

d) Production required by the sales department to meet their sales targets/budgets
Apart from (d), this information will come from the production department and a large part of
the traditional work of cost accounting involves ascribing costs to the physical information
produced.

Page 382

CHANGING BUDGETARY SYSTEMS
An organisation which decides to change its budgetary practices will face a number of
difficulties.
The business environment has become increasingly complex, uncertain and dynamic and
organisations need to be able to adapt quickly to changing conditions. It has been argued that
traditional budgets are too rigid and prevent fast response to changing conditions.
However, an organisation which decides to change its type of budget used, or budgetary
system, will face a number of difficulties.

a) Resistance by employees. Employees will be familiar with the current system and
may have built in slack so will not easily accept new targets. New control systems that
threaten to alter existing power relationships may be thwarted by those affected.
b) Loss of control. Senior management may take time to adapt to the new system and
understand the implications of results.
c) Training. In order for the new budget to operate effectively, everyone within the
organisation will need to be fully trained. This is time-consuming and expensive.
d) Costs of implementation. Any new system or process requires careful
implementation which will have cost implications.
e) Lack of accounting information. The organisation may not have the systems in
place to obtain and analyse the necessary information.

Page 383

BLANK

Page 384

BUDGET SYSTEMS AND UNCERTAINTY
Uncertainty can be allowed for in budgeting by means of flexible budgeting, rolling
budgets, probabilistic budgeting and sensitivity analysis.

Causes of uncertainty in the budgeting process include:
a) Customers. They may decide to buy less than forecast, or they may buy more.
b) Products/services. In the modern business environment, organisations need to
respond to customers' rapidly changing requirements.
c) Inflation and movements in interest and exchange rates.
d) Volatility in the cost of materials.
e) Competitors. They may steal some of an organisation's expected customers, or some
competitors' customers may change their buying allegiance.
f) Employees. They may not work as hard as was hoped, or they may work harder.
g) Machines. They may break down unexpectedly.
h) There may be political unrest (terrorist activity), social unrest (public transport
strikes) or minor or major natural disasters (storms, floods).

Rolling budgets are a way of trying to reduce the element of uncertainty in the plan. There
are other planning methods which try to analyse the uncertainty such as probabilistic
budgeting (where probabilities are assigned to different conditions – see Chapter 14) and
sensitivity analysis. These methods are suitable when the degree of uncertainty is
quantifiable from the start of the budget period and actual results are not expected to go
outside the range of these expectations.

Page 385

CHAPTER ROUNDUP
•

A budget is a quantified plan of action for a forthcoming accounting period.

•

A budget can be set from the top down (imposed budget) or from the bottom up
(participatory budget).

•

The traditional approach to budgeting, known as incremental budgeting, bases the
budget on the current year's results plus an extra amount for estimated growth or
inflation next year. It encourages slack and wasteful spending to creep into budgets.

•

Fixed budgets remain unchanged regardless of the level of activity; flexible budgets
are designed to flex with the level of activity

•

The principle behind zero based budgeting (ZBB) is that the budget for each cost
centre should be made from 'scratch' or zero. Every item of expenditure must be
justified in its entirety in order to be included in the next year's budget.

•

There is a three-step approach to ZBB.
–
–
–

Define decision units
Evaluate and rank packages
Allocate resources

•

ZBB is particularly useful for budgeting for discretionary costs and for rationalisation
purposes.

•

At its simplest, activity based budgeting (ABB) is merely the use of costs
determined using ABC as a basis for preparing budgets.

•

Rolling budgets (continuous budgets) are budgets which are continuously updated
by adding a further period (say a month or a quarter) and deducting the earliest period.

•

Information used in budget systems will come from a wide variety of sources.

•

An organisation which decides to change its budgetary practices will face a number
of difficulties.

•

Uncertainty can be allowed for in budgeting by means of flexible budgeting, rolling
budgets, probabilistic budgeting and sensitivity analysis.

Page 386

STUDY UNIT 14
Quantitative Analysis In Budgeting
Contents

Page

Analysing Fixed And Variable Costs ………………………………..…….

389

Forecasting Techniques ……….………………………..……..……..……..

399

Time Series ……………………………………………….………………….

401

Learning Curves ………………………………………….................………

421

Applying Expected Values ………………………………………….....……

435

Using Spreadsheets In Budgeting ………………….………………………

439

Page 387

EXAM GUIDE
The quantitative techniques covered in this chapter are likely to form the calculation part of a
budgeting question.

Page 388

ANALYSING FIXED AND VARIABLE COSTS
Two important quantitative methods the management accountant can use to analyse fixed and
variable cost elements from total cost data are the high-low and regression methods.

The high-low method
You will have encountered the high-low method in your earlier studies. It is used to identify
the fixed and variable elements of costs that are semi-variable. Read through the knowledge
brought forward and do the question below to jog your memory.

Knowledge brought forward from earlier studies
Follow the steps below.
Step 1

Review records of costs in previous periods.
•
•

Select the period with the highest activity level
Select the period with the lowest activity level

Step 2

If inflation makes it difficult to compare costs, adjust by indexing up or down.

Step 3

Determine the following.
•
•
•
•

Step 4

Total cost at high activity level
Total costs at low activity level
Total units at high activity level
Total units at low activity level

Calculate the following.
Total cost at high activity level _ total cost at low activity level
_
Total units at high activity level total units at low activity level

= variable cost per unit (v)

Page 389

Step 5

The fixed costs can be determined as follows. (Total cost at high activity level ) –
(total units at high activity level × variable cost per unit)

Example
A department in a large organisation wishes to develop a method of predicting its total costs
in a period. The following data have been recorded.
Month

Activity level (X)

Cost

units

RWF ‘000

January

1,600

28,200

February

2,300

29,600

March

1,900

28,800

April

1,800

28,600

May

1,500

28,000

June

1,700

28,400

The total cost model for a period could be represented by what equation?

Answer
The highest activity level is in February and the lowest in May.
Total cost at highest activity level =

RWF29,600,000

Total cost at lowest activity level =

RWF28,000,000

Total units at highest activity level =

2,300

Total units at lowest activity level =

1,500

Page 390

Variable cost per unit =

29,600,000−28,000,000 1,600,000
=
2,300−1,500
800

Fixed costs =

29,600,000 – (2,300 × 2,000)

Total costs =

25,000,000 + 2x

=

= RWF2,000

RWF25,000,000

The usefulness of the high-low method
The high-low method is a simple and easy to use method of estimating fixed and variable
costs. However there are a number of problems with it.
a) The method ignores all cost information apart from at the highest and lowest volumes
of activity and these may not be representative of costs at all levels of activity.
b) Inaccurate cost estimates may be produced as a result of the assumption of a constant
relationship between costs and volume of activity.
c) Estimates are based on historical information and conditions may have changed.

Linear regression analysis
Knowledge brought forward from earlier studies
Linear relationships
•

A linear relationship can be expressed in the form of an equation which has the general
form y = a + bx
where

y is the dependent variable, depending for its value on the value of x
x is the independent variable, whose value helps to determine the value of y
a is a constant, a fixed amount
b is a constant, being the coefficient of x (that is, the number by which the
value of x should be multiplied to derive the value of y)
Page 391

•

•

If there is a linear relationship between total costs and level of activity, y = total costs,
x = level of activity, a = fixed cost (the cost when there is no activity level) and b =
variable cost per unit.

•

The graph of a linear equation is a straight line and is determined by two things, the
gradient (or slope) of the straight line and the point at which the straight line crosses
the y axis (the intercept).
–

Gradient = b in the equation y = a + bx = (y2 – y1)/(x2 – x1) where (x1, y1), (x2,
y2) are two points on the straight line

–

Intercept = a in the equation y = a + bx

Linear regression analysis, also known as the 'least squares technique', is a
statistical method of estimating costs using historical data from a number of previous
accounting periods.

If y = a + bx, b =

nΣxy − Σ xΣy
Σy bΣx
−
and a =
n
n
nΣx2 − ( Σx ) 2

where n is the number of pair of data for x and y.

Exam Focus Point
Note that you don't need to learn these formulae, as they are provided in the exam, but it
would be very easy to make a mistake when copying them down so always double check
back to the exam paper. Make sure you are confident using these formulae quickly and
accurately.

Example: linear regression analysis
The transport department of NCC Ltd operates a large fleet of vehicles. These vehicles are
used by the various departments of the NCC Ltd. Each month a statement is prepared for the
transport department comparing actual results with budget. One of the items in the transport
department's monthly statement is the cost of vehicle maintenance. This maintenance is
Page 392

carried out by the employees of the department. To facilitate control, the transport manager
has asked that future statements should show vehicle maintenance costs analysed into fixed
and variable costs.
Data from the six months from January to June inclusive are given below.
Vehicle maintenance cost

Vehicle running hours

RWF ‘000
January

13,600

2,100

February

15,800

2,800

March

14,500

2,200

April

16,200

3,000

May

14,900

2,600

June

15,000

2,500

Required
Analyse the vehicle maintenance costs into fixed and variable costs, based on the data given,
utilising the least squares method.

Page 393

Solution
If y = a + bx, where y represents costs and x represents running hours (since costs depend on
running hours) then b= (nΣxy – ΣxΣy)/ (nΣx2 – (Σx)2), when n is the number of pairs of data,
which is 6 in this problem.
x
'000 hrs

y

xy

x2

Rwf m

2.1

13.6

28.56

4.41

2.8

15.8

44.24

7.84

2.2

14.5

31.90

4.84

3.0

16.2

48.60

9.00

2.6

14.9

38.74

6.76

2.5

15.0

37.50

6.25

15.2

90.0

229.54

39.10

Variable cost per hour, b

=

(6 x 229.54) − (15.2 x90.0)
2
(6 x39.10) − 15.2

= (1,377.24 – 1,368)/(234.6 – 231.04) = 9.24/3.56 = RWF2.60
Fixed costs (in RWF m), a
Rwf8,400,000

= (Σy/n) – (bΣx/n) = (90/6) – (2.6(15.2)/6) = 8.41 approx, say

Page 394

The conditions suited to the use of linear regression analysis

The conditions which should apply if linear regression analysis is to be used to estimate costs
are as follows.
a) A linear cost function should be assumed. This assumption can be tested by
measures of reliability, such as the correlation coefficient and the coefficient of
determination (which ought to be reasonably close to 1).
b) When calculating a line of best fit, there will be a range of values for x. In Question 1,
the line y = 28 + 2.6x was predicted from data with output values ranging from x = 16
to x = 24. Depending on the degree of correlation between x and y, we might safely
use the estimated line of best fit to forecast values for y, provided that the value of x
remains within the range 16 to 24. We would be on less safe ground if we used the
equation to predict a value for y when x = 10, or 30, or any other value outside the
range 16 to 24, because we would have to assume that costs behave in the same
way outside the range of x values used to establish the line in the first place.

Interpolation means using a line of best fit to predict a value within the two extreme points
of the observed range.
Extrapolation means using a line of best fit to predict a value outside the two extreme
points.

c) The historical data for cost and output should be adjusted to a common price level
(to overcome cost differences caused by inflation) and the historical data should also
be representative of current technology, current efficiency levels and current
operations (products made).
d) As far as possible, historical data should be accurately recorded so that variable
costs are properly matched against the items produced or sold, and fixed costs are
properly matched against the time period to which they relate. For example, if a
factory rental is RWF120,000 per annum, and if data is gathered monthly, these costs
should be charged RWF10,000 to each month instead of RWF120,000 in full to a
single month.
e) Management should either be confident that conditions which have existed in the
past will continue into the future or amend the estimates of cost produced by the
linear regression analysis to allow for expected changes in the future.

Page 395

f) As with any forecasting process, the amount of data available is very important.
Even if correlation is high, if we have fewer than about ten pairs of data, we must
regard any forecast as being somewhat unreliable.
g) It must be assumed that the value of one variable, y, can be predicted or estimated
from the value of one other variable, x.

Scatter diagrams
Scatter diagrams can be used to estimate the fixed and variable components of costs.
By this method of cost estimation, cost and activity data are plotted on a graph. A 'line of
best fit' is then drawn. This line should be drawn through the middle of the plotted points as
closely as possible so that the distance of points above the line are equal to distances below
the line. Where necessary costs should be adjusted to the same indexed price level to allow
for inflation.

The fixed cost is the intercept of the line of best fit on the vertical axis. Suppose the fixed cost
is RWF500 and that one of the plotted points (which is very close to the line or actually on it)
represents output of 100 units and total cost of RWF550. The variable cost of 100 units is
therefore calculated as RWF(550 – 500) = RWF50 and so the variable cost per unit is
Rwf0.50. The equation of the line of best fit is therefore approximately
Page 396

y = 500 + 0.5x.
If the company to which thede data relate wanted to forecast total costs when output is 90
units, a forecast based on the equation would be 500 + (0.5 × 90) = RWF545. Alternatively
the forecast could be read directly from the graph using the line of best fit.
The disadvantage of the scatter diagram method is that the cost line is drawn by visual
judgement and so is a subjective approximation.

Page 397

BLANK

Page 398

FORECASTING TECHNIQUES
Forecasting techniques include estimates based on judgement and experience, simple
average growth models and time series.

Numerous techniques have been developed for using past data as the basis for forecasting
future values. These techniques range from simple arithmetic and visual methods to advanced
computer-based statistical systems. With all techniques, however, there is the presumption
that the past will provide guidance to the future. Before using any extrapolation
techniques, the past data must therefore be critically examined to assess their
appropriateness for the intended purpose. The following checks should be made.
a) The time period should be long enough to include any periodically paid costs but
short enough to ensure that averaging of variations in the level of activity has not
occurred.
b) The data should be examined to ensure that any non-activity level factors affecting
costs were roughly the same in the past as those forecast for the future. Such factors
might include changes in technology, changes in efficiency, changes in production
methods, changes in resource costs, strikes, weather conditions and so on. Changes to
the past data are frequently necessary.
c) The methods of data collection and the accounting policies used should not
introduce bias. Examples might include depreciation policies and the treatment of byproducts.
d) Appropriate choices of dependent and independent variables must be made.

The sales budget is frequently the first budget prepared since ‘Sales’ is usually the principal
budget factor, but before the sales budget can be prepared a sales forecast has to be made.
Sales forecasting is complex and difficult and involves the consideration of a number of
factors.
Management can use a number of forecasting methods, often combining them to reduce the
level of uncertainty.
a) Sales personnel can be asked to provide estimates. Such estimates are based on
judgement and experience.
b) Market research can be used (especially for new products or services).
Page 399

c) Simple average growth models can be used.
d) Time series can be used to produce forecasts.
e) Mathematical models can be set up so that repetitive computer simulations can be
run which permit managers to review the results that would be obtained in various
circumstances.

Simple average growth models
A growth rate can be estimated from an analysis of the growth in, for example sales, over
the past few years.
Year

Sales revenue
RWF ‘000

20X1

150,000

20X2

192,000

20X3

206,000

20X4

245,000

20X5

262,350

Sales have risen from RWF150m in 20X1 to RWF262.35m in 20X5. The increase represents
four years growth. (Check that you can see that there are four years growth, and not five
years growth, in the table.) The average growth rate, g, may be calculated as follows.
Sales in 20X1 × (1 + g)4 = Sales in 20X5
(1+ g) 4

=

Sales in 20X5
Sales in 20X

=

RWF262.350 m
Rwf150m

= 1.749
1+g =
g

4

1.749 = 1.15

= 0.15, ie 15%
Page 400

TIME SERIES
A time series is a series of figures or values recorded over time.

The following are examples of time series.
•
•
•

Output at a factory each day for the last month
Monthly sales over the last two years
The Retail Prices Index each month for the last ten years

A graph of a time series is called a historigram.

(Note the letters 'ri'; this is not the same as a histogram.) For example, consider the following
time series.

Year

20X0

20X1

Sales (RWF m)

20

21

20X2
24

20X3
23

20X4

20X5

20X6

27

30

28

The historigram is as follows.
Sales
m
30

20

10

20X0 20X1 20X2 20X3 20X4 20X5 20X6

Page 401

The horizontal axis is always chosen to represent time, and the vertical axis represents the
values of the data recorded.

Regression and forecasting
Regression can be used to find a trend line, such as the trend in sales over a number of
periods.
The same regression techniques as those considered earlier in the chapter can be used to
calculate a regression line (a trend line) for a time series. A time series is simply a series
of figures or values recorded over time (such as total annual costs for the last ten years). The
determination of a trend line is particularly useful in forecasting.
The years (or days or months) become the x variables in the regression formulae by
numbering them from 0 upwards.

Example: Regression and forecasting

Sales of product B over the seven year period from 20X1 to 20X7 were as follows.
Year

20X1

Sales of B ('000 22
units)

20X2

20X3

20X4

20X5

20X6

20X7

25

24

26

29

28

30

There is high correlation between time and the volume of sales.

Required
Calculate the trend line of sales, and forecast sales in 20X8 and 20X9.

Page 402

Solution
Workings
Year

x

y

xy

x2

20X1

0

22

0

0

20X2

1

25

25

1

20X3

2

24

48

4

20X4

3

26

78

9

20X5

4

29

116

16

20X6

5

28

140

25

180

36

Σxy = 587

Σx2 = 91

20X7

6
Σx = 21

n

30
Σy = 184

=7

Where y = a + bx
b

= ((7 × 587) – (21 × 184))/((7 × 91) – (21 × 21)) = 245/196 = 1.25

a

= (184/7) – ((1.25 × 21)/7) = 22.5357, say 22.5

y

= 22.5 + 1.25x where x = 0 in 20X1, x = 1 in 20X2 and so on.

Using this trend line, predicted sales in 20X8 (year 7) would be 22.5 + (1.25 × 7) = 31.25 =
31,250 units.
Similarly, for 20X9 (year 8) predicted sales would be 22.5 + (1.25 × 8) = 32.50 = 32,500
units.

Page 403

The components of time series
A time series has four components: a trend, seasonal variations, cyclical variations and
random variations.
There are several components of a time series which it may be necessary to identify.
a) A trend
b) Seasonal variations or fluctuations
c) Cycles, or cyclical variations
d) Non-recurring, random variations. These may be caused by unforeseen
circumstances such as a change in government, a war, technological change or a fire.

The trend
The trend is the underlying long-term movement over time in values of data recorded.
In the following examples of time series, there are three types of trend.

Output per
labour hour

Number of
Cost per unit

employees

Units

RWF

20X4

30

1.00

100

20X5

24

1.08

103

20X6

26

1.20

96

20X7

22

1.15

102

20X8

21

1.18

103

20X9

17

1.25

98

(A)

(B)

(C)

Page 404

a) In time series (A) there is a downward trend in the output per labour hour. Output
per labour hour did not fall every year, because it went up between 20X5 and 20X6,
but the long-term movement is clearly a downward one.
b) In time series (B) there is an upward trend in the cost per unit. Although unit costs
went down in 20X7 from a higher level in 20X6, the basic movement over time is one
of rising costs.
c) In time series (C) there is no clear movement up or down, and the number of
employees remained fairly constant. The trend is therefore a static, or level one.

Seasonal variations
Seasonal variations are short-term fluctuations in recorded values, due to different
circumstances which affect results at different times of the year, on different days of the
week, at different times of day, or whatever.

Here are two examples of seasonal variations.
a) Sales of ice cream will be higher in summer than in winter.
b) The telephone network may be heavily used at certain times of the day (such as
mid-morning and mid-afternoon) and much less used at other times (such as in the
middle of the night).

'Seasonal' is a term which may appear to refer to the seasons of the year, but its meaning in
time series analysis is somewhat broader, as the examples given above show.

Page 405

Example: A trend and seasonal variations
The number of customers served by a company of travel agents over a four year period is
shown in the following historigram.

Customers
(000s)
10

Trend

5

0
1
Quarters

2

3

4

20X0

1

2

3

4

1

20X1

2

3

20X2

4

1

2

3

4

20X3

In this example, there would appear to be large seasonal fluctuations in demand, but there is
also a basic upward trend.

Cyclical variations
Cyclical variations are medium-term changes in results caused by circumstances which
repeat in cycles. In business, cyclical variations are commonly associated with economic
cycles, successive booms and slumps in the economy. Economic cycles may last a few years.
Cyclical variations are longer term than seasonal variations.

Summarising the components
In practice a time series could incorporate all of the four features we have been looking at
and, to make reasonably accurate forecasts, the four features often have to be isolated. We
Page 406

can begin the process of isolating each feature by summarising the components of a time
series as follows.
The actual time series, Y= T + S + C + R
where Y = the actual time series

C = the cyclical component

T = the trend series

R = the random component

S = the seasonal component

Though you should be aware of the cyclical component, it is unlikely that you will be
expected to carry out any calculation connected with isolating it. The mathematical models
which we will use therefore exclude any reference to C.
We will begin by looking at how to find the trend in a time series.

Moving averages
Trend values can be determined by a process of moving averages.
Look at these monthly sales figures.

Sales
(RWFm)

August

September

October

November

December

0.02

0.04

0.04

3.20

14.60

It looks as though the business is expanding rapidly – and so it is, in a way. But when you
know that the business is a Christmas card manufacturer, then you see immediately that the
January sales will no doubt slump right back down again.
It is obvious that the business will do better in the Christmas season than at any other time –
that is the seasonal variation. Using the monthly figures, how can we tell whether or not the
business is doing well overall – whether there is a rising sales trend over time other than the
short-term rise over Christmas?
Page 407

One possibility is to compare figures with the equivalent figures of a year ago. However,
many things can happen over a year to make such a comparison misleading – new products
might now be manufactured and prices will probably have changed.
In fact, there are a number of ways of overcoming this problem of distinguishing trend from
seasonal variations. One such method is called moving averages. This method attempts to
remove seasonal (or cyclical) variations from a time series by a process of averaging so
as to leave a set of figures representing the trend.
A moving average is an average of the results of a fixed number of periods. Since it is an
average of several time periods, it is related to the mid-point of the overall period.

Exam Focus Point
You will not be required to carry out a time series analysis from raw data but you do need to
be able to explain the approach and discuss its use.

Example: Moving averages
Year

Sales
Units

20X0

390

20X1

380

20X2

460

20X3

450

20X4

470

20X5

440

20X6

500

Page 408

Required
Take a moving average of the annual sales over a period of three years.

Solution
a) Average sales in the three year period
20X0 – 20X2 were (390 + 380 + 460)/3 = 1,230/3 = 410.
This average relates to the middle year of the period, 20X1.

b) Similarly, average sales in the three year period
20X1 – 20X3 were (380 + 460 + 450)/3 = 1,290/3 = 430.
This average relates to the middle year of the period, 20X2.

c) The average sales can also be found for the periods 20X2 – 20X4, 20X3 – 20X5 and
Moving total of

Moving average of

3 years sales

3 years sales (÷ 3)

Year

Sales

20X0

390

20X1

380

1,230

410

20X2

460

1,290

430

20X3

450

1,380

460

20X4

470

1,360

453

20X5

440

1,410

470

20X6

500
20X4 – 20X6, to give the following.
Page 409

Note the following points.
(i)

The moving average series has five figures relating to the years 20X1 to 20X5.
The original series had seven figures for the years from 20X0 to 20X6.

(ii)

There is an upward trend in sales, which is more noticeable from the series of
moving averages than from the original series of actual sales each year.

Moving averages of an even number of results

In the previous example, moving averages were taken of the results in an odd number of time
periods, and the average then related to the mid-point of the overall period.
If a moving average of results was taken in an even number of time periods, the basic
technique would be the same, but the mid-point of the overall period would not relate to a
single period. For example, suppose an average were taken of the following four results.

Spring

120

Summer

90

Autumn

180

Winter

70

Average = 115

The average would relate to the mid-point of the period, between summer and autumn.
The trend line average figures need to relate to a particular time period; otherwise, seasonal
variations cannot be calculated. To overcome this difficulty, we take a moving average of
the moving average. An example will illustrate this technique.

Page 410

Example: Moving averages over an even number of periods
Calculate a moving average trend line of the following results.
Year

Quarter

Volume of sales
'000 units

20X5

20X6

1

600

2

840

3

420

4

720

1

640

2

860

3

420

4

740

Solution
A moving average of four will be used, since the volume of sales would appear to depend on
the season of the year, and each year has four quarterly results. The moving average of four
does not relate to any specific period of time; therefore a second moving average of two will
be calculated on the first moving averages.

Page 411

Year

20X5

20X6

Quarter

Moving

Moving

Mid-point of

total of 4

average of 4

2 moving

Actual volume quarters'

quarters'

averages

of sales

sales

sales

Trend line

'000 units

'000 units

'000 units

'000 units

(Y)

(B)

(B ÷ 4)

(T)

1

600

2

840

3

420

2,580

645.0

650.00

4

720

2,620

655.0

657.50

1

640

2,640

660.0

660.00

2

860

2,640

660.0

662.50

3

420

2,660

665.0

4

740

By taking a mid-point (a moving average of two) of the original moving averages, we can
relate the results to specific quarters (from the third quarter of 20X5 to the second quarter of
20X6).

Finding the seasonal variations
Seasonal variations can be estimated using the additive model or the proportional
(multiplicative) model.
Once a trend has been established we can find the seasonal variations.
Page 412

The additive model
The additive model for time series analysis is Y = T + S + R.
We can therefore write Y – T = S + R. In other words, if we deduct the trend series from the
actual series, we will be left with the seasonal and residual components of the time series. If
we assume that the random component is relatively small, and hence negligible, the seasonal
component can be found as S = Y – T, the de-trended series.

The actual and trend sales for the example above are set out below. The difference between
the actual results for any one quarter (Y) and the trend figure for that quarter (T) will be the
seasonal variation for that quarter.

Year

Quarter

Actual

20X5

1

600

2

840

3

420

650.00

–230.00

4

720

657.50

62.50

1

640

660.00

–20.00

2

860

662.50

197.50

3

420

4

740

20X6

Trend

Seasonal
variation

Suppose that seasonal variations for the third and fourth quarters of 20X6 and the first and
second quarters of 20X7 are –248.75, 62.50, –13.75 and 212.50 respectively. The variation
between the actual result for a particular quarter and the trend line average is not the same
from year to year, but an average of these variations can be taken.

Page 413

Q1

Q2

20X5

Q3

Q4

–230.00

62.50

–248.75

62.50

20X6

–20.00

197.50

20X7

–13.75

212.50

Total

–33.75

410.00

–478.75

125.00

Average (÷ 2)

–16.875

205.00

–239.375

62.50

Variations around the basic trend line should cancel each other out, and add up to zero. At the
moment, they do not. We therefore spread the total of the variations (11.25) across the four
quarters (11.25 ÷ 4) so that the final total of the variations sum to zero.

Estimated quarterly variations
Adjustment to reduce variations to
0
Final estimates of quarterly
variations

Q1

Q2

Q3

– 16.8750

205.0000 –239.3750

Q4

Total

62.5000

11.250

–2.8125

–2.8125

–2.8125 –2.8125

–19.6875

202.1875

–242.1875 59.6875

These might be rounded as follows Ql: –20

Q2: 202

Q3:-242

Q4: 60

–11.250

0

Total: 0

The proportional model
The method of estimating the seasonal variations in the above example was to use the
differences between the trend and actual data. This model assumes that the components of
the series are independent of each other, so that an increasing trend does not affect the
seasonal variations and make them increase as well, for example.
Page 414

The alternative is to use the proportional model whereby each actual figure is expressed as a
proportion of the trend. Sometimes this method is called the multiplicative model.
The proportional (multiplicative) model summarises a time series as Y = T × S × R.

The trend component will be the same whichever model is used but the values of the
seasonal and random components will vary according to the model being applied.
The example above can be reworked on this alternative basis. The trend is calculated in
exactly the same way as before but we need a different approach for the seasonal variations.
The proportional model is Y = T × S × R and, just as we calculated S = Y – T for the additive
model above, we can calculate S = Y/T for the proportional model.

Year

20X5

20X6

Quarter

Actual

Trend

Seasonal ratio

(Y)

(T)

(Y/T)

1

600

2

840

3

420

650.00

0.646

4

720

657.50

1.095

1

640

660.00

0.970

2

860

662.50

1.298

3

420

4

740

Suppose that seasonal variations for the next four quarters are 0.628, 1.092, 0.980 and 1.309
respectively. The summary of the seasonal variations expressed in proportional terms is
therefore as follows.

Page 415

Q1

Q2

Q3

Q4

Ratio

Ratio

Ratio

Ratio

20X5

0.646

1.095

0.628

1.092

20X6

0.970

1.298

20X7

0.980

1.309

Total

1.950

2.607

1.274

2.187

Average

0.975

1.3035

0.637

1.0935

Instead of summing to zero, as with the additive approach, the averages should sum (in this
case) to 4.0, 1.0 for each of the four quarters. They actually sum to 4.009 so 0.00225 has to
be deducted from each one.

Q1

Q2

Q3

Q4

Average

0.97500

1.30350

0.63700

1.09350

Adjustment

–0.00225

–0.00225

–0.00225

–0.00225

Final estimate

0.97275

1.30125

0.63475

1.09125

Rounded

0.97

1.30

0.64

1.09

Note that the proportional model is better than the additive model when the trend is
increasing or decreasing over time. In such circumstances, seasonal variations are likely to
be increasing or decreasing too. The additive model simply adds absolute and unchanging
seasonal variations to the trend figures whereas the proportional model, by multiplying
increasing or decreasing trend values by a constant seasonal variation factor, takes account of
changing seasonal variations.

Page 416

Time series analysis and forecasting
Forecasts can be made by calculating a trend line (using moving averages or linear
regression), using the trend line to forecast future trend line values, and adjusting these values
by the average seasonal variation applicable to the future period.

By extrapolating a trend and then adjusting for seasonal variations, forecasts of future values
can be made.

Forecasts of future values should be made as follows.
a) Find a trend line using moving averages or using linear regression analysis
b) Use the trend line to forecast future trend line values.
c) Adjust these values by the average seasonal variation applicable to the future period,
to determine the forecast for that period. With the additive model, add (or subtract for
negative variations) the variation. With the multiplicative model, multiply the trend
value by the variation proportion.

Extending a trend line outside the range of known data, in this case forecasting the future
from a trend line based on historical data, is known as extrapolation.

Page 417

Example: Forecasting
The sales (in Rwf m) of swimwear by a large store for each period of three months and trend
values found using moving averages are as follows.

Quarter

20X4

20X5

20X6

20X7

Actual

Trend

Actual

Trend

Actual

Trend

Actual

Trend

RWF
m

RWF
m

RWF
m

RWF
m

RWF
m

RWF m

RWF m

RWF m

20

40

40

57

First

8

Second

30

30

50

45

62

Third

60

31

80

50

92

20

35

40

54

Fourth

24

Using the additive model, seasonal variations have been determined as follows.

Quarter 1

Quarter 2

Quarter 3

Quarter 4

–RWF k18,250

+RWF k2,750

+RWF k29,750

–RWF k14,250

Required
Predict sales for the last quarter of 20X7 and the first quarter of 20X8, stating any
assumptions.

Page 418

Solution
We might guess that the trend line is rising steadily, by (57 – 40)/4 = 4.25 per quarter in the
period 1st quarter 20X6 to 1st quarter 20X7 (57 being the prediction in 1st quarter 20X7 and
40 the prediction in 1st quarter 20X6). Since the trend may be levelling off a little, a quarterly
increase of +4 in the trend will be assumed.
Seasonal
Trend

variation

Forecast

1st quarter

20X7

57

4th quarter

20X7 (+ (3 × 4))

69

–14.25

54.75

1st quarter

20X8 (+ (4 × 4))

73

–18.25

54.75

Rounding to the nearest million francs, the forecast sales are Rwf m55 for each of the two
quarters.
Note that you could actually plot the trend line figures on a graph, extrapolate the trend line
into the future and read off forecasts from the graph using the extrapolated trend line.
If we had been using the proportional model, with an average variation for (for example)
quarter 4 of 0.8, our prediction for the fourth quarter of 20X7 would have been 69 × 0.8 =
55.2, say RWF55,000,000

Forecasting problems
Errors can be expected in forecasting due to unforeseen changes. This is more likely to
happen the further into the future the forecast is for, and the smaller the quantity of data on
which the forecast is based.
All forecasts are subject to error, but the likely errors vary from case to case.
•

The further into the future the forecast is for, the more unreliable it is likely to be
Page 419

•
•
•
•

The less data available on which to base the forecast, the less reliable the forecast
The historic pattern of trend and seasonal variations may not continue into the
future
Random variations may upset the pattern of trend and seasonal variation
Extrapolation of the trend line is done by judgment and can introduce errors

There are a number of changes that also may make it difficult to forecast future events.

Type of change

Examples

Political and
economic changes

Changes in interest rates, exchange rates or inflation can mean that
future sales and costs are difficult to forecast.

Environmental
changes

The opening of new roads or the railway might have a considerable
impact on some companies' markets.

Technological
changes

These may mean that the past is not a reliable indication of likely
future events. For example new faster machinery may make it
difficult to use current output levels as the basis for forecasting future
production output.

Technological
advances

Advanced manufacturing technology is changing the cost structure of
many firms. Improved education and the increase in the standards of
living are impacting on the skills of the labour force. This causes
forecasting difficulties because of the resulting changes in cost
behaviour patterns, breakeven points and so on.

Social changes

Alterations in taste, fashion and the social acceptability of products
can cause forecasting difficulties.

Management should have reasonable confidence in their estimates and forecasts. The
assumptions on which the forecasts/estimates are based should be properly understood and
the methods used to make a forecast or estimate should be in keeping with the nature,
quantity and reliability of the data on which the forecast or estimate will be based. There is no
point in using a 'sophisticated' technique with unreliable data; on the other hand, if there are a
lot of accurate data about historical costs, it would be a waste of the data to use the scatter
diagram method for cost estimating.
Page 420

LEARNING CURVES
Learning curve theory may be useful for forecasting production time and labour costs in
certain circumstances, although the method has many limitations.

Whenever an individual starts a job which is fairly repetitive in nature, and provided that his
speed of working is not dictated to him by the speed of machinery (as it would be on a
production line), he is likely to become more confident and knowledgeable about the work
as he gains experience, to become more efficient, and to do the work more quickly.
Eventually, however, when he has acquired enough experience, there will be nothing more
for him to learn and so the learning process will stop.

Learning curve theory applies to situations where the work force as a whole improves in
efficiency with experience. The learning effect or learning curve effect describes the
speeding up of a job with repeated performance.

Where does learning curve theory apply?
Labour time should be expected to get shorter, with experience, in the production of items
which exhibit any or all of the following features.
•
•
•

Made largely by labour effort (rather than by a highly mechanised process)
Brand new or relatively short-lived (learning process does not continue
indefinitely)
Complex and made in small quantities for special orders

Page 421

The learning rate: cumulative average time
In learning theory the cumulative average time per unit produced is assumed to decrease
by a constant percentage every time total output of the product doubles.
For instance, where an 80% learning effect occurs, the cumulative average time required per
unit of output is reduced to 80% of the previous cumulative average time when output is
doubled.
a) By cumulative average time, we mean the average time per unit for all units
produced so far, back to and including the first unit made.
b) The doubling of output is an important feature of the learning curve measurement.

Don't worry if this sounds quite hard to grasp in words, because it is hard to grasp (until
you've learned it!). It is best explained by a numerical example.

Example: an 80% learning curve
The first unit of output of a new product requires 100 hours. An 80% learning curve applies.
The production times would be as follows.

Number of units
produced
1

Cumulative avg time
required per unit
100.0

Total time
required
100.0
(× 1)

Incremental time for
additional units

2*

(80%)

80.0

(× 2)

160.0

60.0

(for 1 extra unit)

4*

(80%)

64.0

(× 4)

256.0

96.0

(for 2 extra units)

8*

(80%)

51.2

(× 8)

409.6

153.6

(for 4 extra units)

* Output is being doubled each time.
This effect can be shown on a graph, as a learning curve either for unit times or cumulative
total times or costs.
Page 422

Example: the learning curve
Kivu Sailcraft Ltd has designed a new type of sailing boat, for which the cost of the first boat
to be produced has been estimated as follows:
Rwf m
Materials

5,000

Labour (800 hrs × RWF5,000 per hr)

4,000

Overhead (150% of labour cost)

6,000
15,000

Profit mark-up (20%)

3,000

Sales price

18,000

It is planned to sell all the yachts at full cost plus 20%. An 80% learning curve is expected to
apply to the production work. The management accountant has been asked to provide cost
information so that decisions can be made on what price to charge.
a) What is the separate cost of a second yacht?
Page 423

b) What would be the cost per unit for a third and a fourth yacht, if they are ordered
separately later on?
c) If they were all ordered now, could Kivu Sailcraft Ltd quote a single unit price for
four yachts and eight yachts.

Solution

Number
of
yachts

Cumulative
average
time per
yacht

Total time
for all
yachts to
date

Incremental
time for
additional
yachts

Hours

Hours

Hours

1000.0

1000

1
2

(× 80%)

800.0

(× 2)

1600

(1600 – 1000)

600

4

(× 80%)

640.0

(× 4)

2560

(2560-1600)

960

8

(× 80%)

512.0

(× 8)

4096

(4096-2560)

1536

a) Separate cost of a second yacht
RWF ‘000
Materials

4,000

Labour (4600 hrs × RWF250)

150

Overhead (150% of labour cost)

225

Total cost

4,375

Page 424

b) Cost of the third and fourth yachts
RWF ‘000
Materials cost for two yachts

8,000

Labour (960 hours × RWF250)

240

Overhead (150% of labour cost)

360

Total cost

8,600

Cost per yacht (÷2)

4,300

c) A price for the first four yachts together and for the first eight yachts together
First four yachts

Materials
Labour

RWF
‘000

RWF
‘000

16,000

32,000

(2,560 hrs)

640

Overhead (150% of
labour cost)
Total cost
Profit (20%)

(4,096hrs)

1,536

17,600

34,560
6,912

21,120
(÷4)

1,024

960

93,520

Total sales price
Price per yacht

First eight yachts

5,280

41,472
(÷8)

5,184

This assumes that Kivu Sailcraft is happy to pass on the efficiency savings to the
customer in the form of a lower price.

Page 425

A formula for the learning curve
The formula for the learning curve is y = axb, where b, the learning coefficient or learning
index, is defined as (log of the learning rate/log of 2).

The formula for the learning curve is y = axb
where

y is the average cost per batch
x is the total number of batches produced
a is the cost of the first batch
b is the learning factor (logLR/log2)
LR is the learning rate as a decimal

Logarithms and the value of b
When y = axb in learning curve theory, the value of b = log of the learning rate/log of 2.
The learning rate is expressed as a proportion, so that for an 80% learning curve, the learning
rate is 0.8, and for a 90% learning curve it is 0.9, and so on.
For an 80% learning curve, b = log 0.8/log 2.
Using the button on your calculator marked ‘log’ or the Log function in a spread-sheet such
as MS Excel or IBM Lotus 1-2-3

-0.0969
b=

-0.3219

b=

0.3010

=LOG(0.8,10
)
=LOG(2,10)

Page 426

=+K3/K4

Example: using the formula
Suppose, for example, that an 80% learning curve applies to production of item ABC. To date
(the end of June) 230 units of ABC have been produced. Budgeted production for July is 55
units.
The cost of the very first unit of ABC, in January, was RWF120.
Note: Values are RWF ‘000

Required
Calculate the budgeted total labour cost for July.

Solution
To solve this problem, we need to calculate three things.
a) The cumulative total labour cost so far to produce 230 units of ABC.
b) The cumulative total labour cost to produce 285 units of ABC, that is adding on the
extra 55 units for July.
c) The extra cost of production of 55 units of ABC in July, as the difference between (b)
and (a).

Calculation (a)
y = axb and we know that for 230 cumulative units, a = Rwf120 (cost of first unit), x = 230
(cumulative units) and b = –0.322 (80% learning curve) and so y = 120 × (230–0.322)
=RWF20.83.
So when x = 230 units, the cumulative average cost per unit is Rwf20.83.

Calculation (b)
Now we do the same sort of calculation for x = 285.
Page 427

If x = 285, y = 120 × (285–0.322) = RWF19.44
So when X = 285 units, the cumulative average cost per unit is RWF19.44.

Calculation (c)
Cumulative units

Average cost per unit
RWF

Total cost
RWF

230

20.83

4,790.90

285

19.44

5,540.40

Incremental cost for 55 units

749.50

Average cost per unit, between 230 and 285 units = 749.50/55 = RWF13.63 per unit approx.

Example: Learning curves and standard costs
A company needs to calculate a new standard cost for one of its products. When the product
was introduced, the standard variable cost of the first unit was as follows.

Cost per unit
RWF‘000
Direct material

10 kg @ RWF2400 per kg

24

Direct labour

12 hours @RWF2250 per hour

27

Variable overhead

12 hours @ RWF2500 per hour

30

Total

81
Page 428

During the following year, a 90% learning curve was observed. The cumulative production at
the end of the third quarter was 50 units and the budgeted production for the fourth quarter is
10 units.

Required
a) What is the standard cost per unit for the fourth quarter assuming that the 90%
learning curve still applies?
b) What is the standard cost per unit for the fourth quarter assuming the learning curve
had reached a steady state ie peak efficiency was reached after the 50th unit was
produced?

Solution
a) y = axb and for 60 cumulative units a = 12 hours (time for first unit), x = 60
(cumulative units) and b = -0.152 (90% learning curve) and so y = 10 x (60-0.152) =
5.37 hours.
For 50 cumulative units y = 12 x (50-0.152) = 6.62 hours.

Cumulative units

Average time per unit

Total time

50

6.62

331.0

60

6.44

386.4

Incremental time for 10 units

55.4

Page 429

The
standard
time
per
The standard cost per unit is:

unit

is

therefore

55.4/10

=

5.54

hours

Cost per unit
RWF ‘000
Direct material

10 kg @ RWF2400 per kg

24.00

Direct labour

5.54 hours @ RWF2250 per
hour

12.47

Variable overhead

5.54 hours @ 2500 per hour

13.85

Total

50.32

b) A steady state is reached after the 50th unit so we need the time taken to produce the
50th unit.
For 49 cumulative units a = 12 hours (time for first unit), x = 49 (cumulative units) and
b = -0.152 (90% learning curve) and so y = 12 x (49-0.152) = 5.535 hours.

Cumulative units

Average time per unit

Total time

49

6.6415

325.4335

50

6.6211

331.055

Incremental time for 50th unit

5.6215

Page 430

The standard cost per unit is:
Cost per unit
RWF‘000
Direct material

10 kg @ RWF2400 per kg

24

Direct labour

5.6215 hours @
RWF9000 per hour

2250

12.65

Variable overhead

5.6215 hours @ RWF2500
per hour

14.05

Total

50.7

The practical application of learning curve theory
What costs are affected by the learning curve?
a) Direct labour time and costs
b) Variable overhead costs, if they vary with direct labour hours worked.
c) Materials costs are usually unaffected by learning among the workforce, although it
is conceivable that materials handling might improve, and so wastage costs be
reduced.
d) Fixed overhead expenditure should be unaffected by the learning curve (although
in an organisation that uses absorption costing, if fewer hours are worked in
producing a unit of output, and the factory operates at full capacity, the fixed
overheads recovered or absorbed per unit in the cost of the output will decline as
more and more units are made).

Page 431

The relevance of learning curve effects in management accounting
Learning curve theory can be used to:
a) Calculate the marginal (incremental) cost of making extra units of a product.
b) Quote selling prices for a contract, where prices are calculated at cost plus a
percentage mark-up for profit. An awareness of the learning curve can make all the
difference between winning contracts and losing them, or between making profits and
selling at a loss-making price.
c) Prepare realistic production budgets and more efficient production schedules.
d) Prepare realistic standard costs for cost control purposes.

Considerations to bear in mind include:
a) Sales projections, advertising expenditure and delivery date commitments.
Identifying a learning curve effect should allow an organisation to plan its advertising
and delivery schedules to coincide with expected production schedules. Production
capacity obviously affects sales capacity and sales projections.
b) Budgeting with standard costs. Companies that use standard costing for much of
their production output cannot apply standard times to output where a learning effect
is taking place. This problem can be overcome in practice by:
(i)

Establishing standard times for output, once the learning effect has worn off or
become insignificant, and

(ii)

Introducing a 'launch cost' budget for the product for the duration of the learning
period.

c) Budgetary control. When learning is still taking place, it would be unreasonable to
compare actual times with the standard times that ought eventually to be achieved
when the learning effect wears off. Allowance should be made accordingly when
interpreting labour efficiency variances.
d) Cash budgets. Since the learning effect reduces unit variable costs as more units are
produced, it should be allowed for in cash flow projections.

Page 432

e) Work scheduling and overtime decisions. To take full advantage of the learning
effect, idle production time should be avoided and work scheduling/overtime
decisions should pay regard to the expected learning effect.
f) Pay. Where the workforce is paid a productivity bonus, the time needed to learn a
new production process should be allowed for in calculating the bonus for a period.
g) Recruiting new labour. When a company plans to take on new labour to help with
increasing production, the learning curve assumption will have to be reviewed.
h) Market share. The significance of the learning curve is that by increasing its share of
the market, a company can benefit from shop-floor, managerial and technological
'learning' to achieve economies of scale.

Limitations of learning curve theory
a) The learning curve phenomenon is not always present.
b) It assumes stable conditions at work which will enable learning to take place. This
is not always practicable, for example because of labour turnover.
c) It must also assume a certain degree of motivation amongst employees.
d) Breaks between repeating the production of an item must not be too long, or workers
will 'forget' and the learning process will have to begin all over again.
e) It might be difficult to obtain accurate data to decide what the learning curve is.
f) Workers might not agree to a gradual reduction in production times per unit.
g) Production techniques might change, or product design alterations might be made,
so that it takes a long time for a 'standard' production method to emerge, to which a
learning effect will apply.

Page 433

BLANK

Page 434

APPLYING EXPECTED VALUES
Expected values can be used in budgeting to determine the best combination of expected
profit and risk.

Probabilistic budgeting assigns probabilities to different conditions (most likely, worst
possible, best possible) to derive an Expected Value (EV) of profit.

A company, for example might make the following estimates of profitability for a given
budget strategy under consideration.
Profit/(loss)

Probability

RWF m
Worst possible outcome

(220)

0.3

Most likely outcome

300

0.6

Best possible outcome

770

0.1

Profit

Expected
value

The EV of profit would be calculated as follows.

Probability

RWFm

RWF m

Worst possible

0.3

(220)

(66)

Most likely

0.6

300

180

Best possible

0.1

770

77

EV of profits

191
Page 435

Example: a probabilistic budget
PIB has recently developed a new product, and is planning a marketing strategy for it. A
choice must be made between selling the product at a unit price of either RWF15k or
RWF17k.
Estimated sales volumes are as follows.

At price of RWF9,000 per unit

At price of RWF11,000 per unit

Sales volume

Sales volume

Probability

Units

Probability

Units

20,000

0.1

8,000

0.1

30,000

0.6

16,000

0.3

40,000

0.3

20,000

0.3

24,000

0.3

a) Sales promotion costs would be RWF5,000,000 at a price of RWF9,000 and
RWF12,000,000 at a price of RWF11,000.
b) Material costs are RWF6,000 per unit.
c) Labour and variable production overhead costs will be RWF1,250 per unit up to
30,000 units and RWF1,375per unit for additional units.
d) Fixed production costs will be RWF38,000,000.
The management of PIB wish to allow for the risk of each pricing decision before choosing
RWF9,000 or RWF11,000 as the selling price.

Page 436

Required
Determine which sales price would be preferred if the management selected the alternative
which did the following.
a) Minimised the worst possible outcome of profit
b) Maximised the best possible outcome of profit

Solution
The unit contribution will be as follows.
Price per unit
RWF9,000

RWF 11,000

Up to 30,000 units

RWF1,750

RWF3,750

Above 30,000 units

RWF1,625

N/A

Sales price RWF9,000

Units of
sale
'000

Total

Fixed

EV of

Unit contb'n

contb'n

costs

Profit

RWF ‘000

RWF m

RWF m

RWFm

Probability

profit
RWF m

20

1.75

35.00

43

(8.00)

0.1

(0.8)

30

1.75

52.50

43

9.50

0.6

5.7

68.75

43

25.75

0.3

7.725

40

30 @ RWF
1,750
10 @
RWF1,625

Page 437

12.625
Sales price RWF11,000

Units of
sale
'000

Total

Fixed

EV of

Unit contb'n

contb'n

costs

Profit

RWF ‘000

RWF m

RWF m

RWFm

Probability

profit
RWF m

8

3.75

30

50

(20)

0.1

(2.0)

16

3.75

60

50

10

0.3

3.0

20

3.75

75

50

25

0.3

12.0

24

3.75

90

50

40

0.3

20.5

a) The price which minimises the worst possible outcome is RWF9,000(with a worstpossible loss of RWF8,000,000).
b) The price which maximises the best possible outcome is RWF11,000 (with a bestpossible profit RWF46,000,000).

Page 438

USING SPREADSHEETS IN BUDGETING
Spreadsheet packages can be used to build business models to assist the forecasting and
planning process. They are particularly useful for 'what if?' analysis.
A spreadsheet is a type of general purpose software package with many business
applications, not just accounting ones. It can be used to build a model, in which data is
presented in these rows and columns, and it is up to the model builder to determine what
data or information should be presented in it, how it should be presented and how the data
should be manipulated by the spreadsheet program. The most widely used spreadsheet
packages are IBM Lotus 1-2-3 and Microsoft Excel.

The idea behind a spreadsheet is that the model builder should construct a model as follows.
a) Identify what data you have and what you want to report.
b) Then decide what data go into each row and column and by inserting text (for
example, column headings and row identifications).
c) Specify how the numerical data in the model should be derived. Numerical data
might be derived using one of the following methods.
•

Insertion into the model via keyboard input.

•

Calculation from other data in the model by means of formulae specified
within the model itself. The model builder must insert these formulae into the
spreadsheet model when it is first constructed.

•

Retrieval from data on a disk file from another computer application
program or module.

Page 439

The advantages of spreadsheets
The uses of spreadsheets are really only limited by your imagination, and by the number of
rows and columns in the spreadsheet, but some of the more common accounting
applications are listed below.

•
•
•
•
•
•

Balance sheets
•
Cash flow analysis/forecasting
•
General ledger
•
Inventory records
•
Job cost estimates
•
Market share analysis and planning

Profit projections
Profit statements
Project budgeting and control
Sales projections and records
Tax estimation

The great value of spreadsheets derives from their simple format of rows, columns and
worksheets of data, and the ability of the data users to have direct access themselves to their
spreadsheet model via their own PC. For example, an accountant can construct a cash flow
model with a spreadsheet package on the PC on his desk: he can create the model, input the
data, manipulate the data and read or print the output direct. He will also have fairly
instant access to the model whenever it is needed, in just the time it takes to load the model
into his PC. Spreadsheets therefore bring computer modelling within the everyday reach of
data users.
Also, by linking different models, when one spreadsheet is updated say for Sales, another
such as Raw material stocks can be updated as well – automatically.

The disadvantages of spreadsheets
Spreadsheets have disadvantages if they are not properly used.
a) A minor error in the design of a model at any point can affect the validity of data
throughout the spreadsheet. Such errors can be very difficult to trace.
b) Even if it is properly designed in the first place, it is very easy to corrupt a model by
accidentally changing a cell or inputting data in the wrong place. This can be
minimised by “protecting” cells containing formulae or fixed data
Page 440

c) It is possible to become over-dependent on them, so that simple one-off tasks that
can be done in seconds with a pen and paper are done on a spreadsheet instead.
d) The possibility for experimentation with data is so great that it is possible to lose sight
of the original intention of the spreadsheet.
e) Spreadsheets cannot take account of qualitative factors since these are invariably
difficult to quantify. Decisions should not be made on the basis of quantitative
information alone.

In summary, spreadsheets should be seen as a tool in planning and decision making. The
user must make the decision.

'What if' analysis
Once a model has been constructed the consequences of changes in any of the variables may
be tested by asking 'what if' questions, a form of sensitivity analysis. For example, a
spreadsheet may be used to develop a cash flow model, such as that shown below.

A

B

C

D

1
2

Sales

Month 1
1,000

Month 2
1,200

Month 3
1,440

3
4

Cost of sales
Gross profit

(650)
350

(780)
420

(936)
504

5
6

Receipts:

7

Current month

600

720

864

8
9

Previous month

–
–

400
–

480
–

600
(650)

1,120
(780)

1,344
(936)

340
(50)

408
290

290

698

10
11

Payments

12
13

Balance b/f

(50)
–

14

Balance c/f

(50)
Page 441

Typical 'what if' questions for sensitivity analysis
a) What if the cost of sales is 68% of sales revenue, not 65%?
b) What if payment from debtors is received as 40% in the month of sale, 50% one
month in arrears and 10% two months in arrears, instead of 60% in the month of sale
and 40% one month in arrears?
c) What if sales growth is only 15% per month, instead of 20% per month?
Using the spreadsheet model, the answers to such questions can be obtained simply and
quickly, using the editing facility in the program. The information obtained should provide
management with a better understanding of what the cash flow position in the future might
be, and what factors are critical to ensuring that the cash position remains reasonable. For
example, it might be found that the cost of sales must remain less than 67% of sales value to
achieve a satisfactory cash position.

Page 442

CHAPTER ROUNDUP
•

Two important quantitative methods the management accountant can use to analyse
fixed and variable cost elements from total cost data are the high-low and regression
methods.

•

Scatter diagrams can be used to estimate the fixed and variable components of costs.

•

Forecasting techniques include estimates based on judgement and experience,
simple average growth models and time series.

•

A time series is a series of figures or values recorded over time.

•

Regression can be used to find a trend line, such as the trend in sales over a number
of periods.

•

A time series has four components: a trend, seasonal variations, cyclical variations
and random variations.

•

Trend values can be determined by a process of moving averages.

•

Seasonal variations can be estimated using the additive model or the proportional
(multiplicative) model.

•

Forecasts can be made by calculating a trend line (using moving averages or linear
regression), using the trend line to forecast future trend line values, and adjusting
these values by the average seasonal variation applicable to the future period.

•

Errors can be expected in forecasting due to unforeseen changes. This is more likely
to happen the further into the future the forecast is for, and the smaller the quantity of
data on which the forecast is based.

•

Learning curve theory may be useful for forecasting production time and labour
costs in certain circumstances, although the method has many limitations.

•

The formula for the learning curve is y = axb, where b, the learning coefficient or
learning index, is defined as (log of the learning rate/log of 2).

•

Expected values can be used in budgeting to determine the best combination of
expected profit and risk.

•

Spreadsheet packages can be used to build business models to assist the forecasting
and planning process. They are particularly useful for 'what if?' analysis.

Page 443

BLANK

Page 444

STUDY UNIT 15
Budgeting And Standard Costing
Contents

Page

The Use Of Standard Costs ………………………………………….…….

447

Deriving Standards ……….………………………..………….……..……..

451

Budgets And Standards Compared …………………….………………….

457

Allowing For Waste And Idle Time ……………………….............………

459

Flexible Budgets …………………………………………………….....……

463

The Principle Of Controllability ………………….….……………………

475

Page 445

EXAM GUIDE
The contents of this chapter are likely to be examined in conjunction with variance analysis,
covered in the next chapter.

Page 446

THE USE OF STANDARD COSTS
A standard cost is an estimated unit cost built up of standards for each cost element
(standard resource price and standard resource usage).
Standard costing is principally used to value inventories and cost production and to act as a
control device.

What is a standard cost?
A standard cost is an estimated unit cost.

The standard cost of product 12345 is set out below on a standard cost card.

STANDARD COST CARD
Product: the Splodget, No 12345
Direct materials
A
B
C
Others

Cost per
RWF
1,500.00
2,250.00
3,000.00

Unit
Kg
Kg
Litre

Requirement
Units
RWF
6.00
9,000.00
2.00
4,500.00
1.00
3,000.00
2,000.00
18,500.00

Direct labour
Grade I
Grade II

1,000.00
1,620.00

Hour
Hour

3.00
5.00

3,000.00
8,100.00
11,100.00

Variable production overheads
Fixed production overheads
Standard full cost of production

1,000.00
3,000.00

Hour
Hour

8.00
8.00

8,000.00
24,000.00
32,000.00
61,600.00

Page 447

Notice how it is built up from standards for each cost element: standard quantities of
materials at standard prices, standard quantities of labour time at standard rates and so on. It
is therefore determined by management's estimates of the following.
•

The expected prices of materials, labour and expenses

•

Efficiency levels in the use of materials and labour

•

Budgeted overhead costs and budgeted volumes of activity

We will see how management arrives at these estimates later in the chapter.
But why should management want to prepare standard costs? Obviously to assist with
standard costing, but what is the point of standard costing?

The uses of standard costing
Standard costing has two principal uses.
•

To value inventories and cost production for cost accounting purposes. It is an
alternative method of valuation to methods such as FIFO and LIFO which you will
have covered in your earlier studies.

•

To act as a control device by establishing standards (expected costs) and comparing
actual costs with the expected costs, thus highlighting areas of the organisation which
may be out of control.

It can also be used in the following circumstances.
a) To assist in setting budgets and evaluating managerial performance.
b) To enable the principle of 'management by exception' to be practised. A standard
cost, when established, is an average expected unit cost. Because it is only an average,
actual results will vary to some extent above and below the average. Only significant
differences between actual and standard should be reported.
c) To provide a prediction of future costs to be used in decision-making.
Page 448

d) To motivate staff and management by the provision of challenging targets.
e) To provide guidance on possible ways of improving efficiency.

Although the various uses of standard costing should not be overlooked, we will be
concentrating on the control aspect.

Standard costing as a control technique
Standard costing involves the establishment of predetermined estimates of the costs or units
of products or services, the collection of actual costs and units and the comparison of the
actual costs/units with the predetermined estimates. The predetermined costs/units are known
as standard costs or units and the difference between standard and actual is known as a
variance. The process by which the total difference between standard and actual results is
analysed in known as variance analysis.

Where standard costing should be used
Standard costing is most suited to mass production and repetitive assembly work.

Although standard costing can be used in a variety of costing situations (batch and mass
production, process manufacture, jobbing manufacture (where there is standardisation of
parts) and service industries (if a realistic cost unit can be established)), the greatest benefit
from its use can be gained if there is a degree of repetition in the production process so that
average or expected usage of resources can be determined. It is therefore most suited to mass
production and repetitive assembly work and less suited to organisations which produce to
customer demand and requirements.
But even in cases such as production of “one offs”, the method of production, or even design,
can often be measured “by unit”, standard costing can be useful. A unit could be an hour, a
sq. metre of steel or metre of cable
Page 449

BLANK

Page 450

DERIVING STANDARDS
The responsibility for deriving standard costs should be shared between managers able to
provide the necessary information about levels of expected efficiency, prices and overhead
costs.

Setting standards for materials costs
Direct materials costs per unit of raw material will be estimated by the purchasing department
from their knowledge of the following.
•
•
•
•
•

Purchase contracts already agreed
Pricing discussions with regular suppliers
The forecast movement of prices in the market
The availability of bulk purchase discounts
The quality of material required by the production departments

The standard cost ought to include an allowance for bulk purchase discounts, if these are
available on all or some of the purchases, and it may have to be a weighted average price of
the differing prices charged for the same product by alternative suppliers.
A decision must also be taken as to how to deal with price inflation. Suppose that a material
costs RWF10,000 per kilogram at the moment, and during the course of the next 12 months,
it is expected to go up in price by 20% to RWF12,000 per kilogram. What standard price
should be selected?

a) If the current price of RWF10,000 per kilogram were used in the standard, the
reported price variance would become adverse as soon as prices go up, which might
be very early in the year. If prices go up gradually rather than in one big jump, it
would be difficult to select an appropriate time for revising the standard.
b) If an estimated mid-year price of, say, RWF11,000 per kilogram were used, price
variances should be favourable in the first half of the year and adverse in the second
half, again assuming that prices go up gradually. Management could only really check
that in any month, the price variance did not become excessively adverse (or
Page 451

favourable) and that the price variance switched from being favourable to adverse
around month six or seven and not sooner.

Standard costing is therefore more difficult in times of inflation but it is still worthwhile.
•
•
•

Usage and efficiency variances will still be meaningful
Inflation is measurable: there is no reason why its effects cannot be removed
Standard costs can be revised, so long as this is not done too frequently

Setting standards for labour costs
Direct labour rates per hour will be set by reference to the payroll and to any agreements on
pay rises with trade union representatives of the employees. A separate hourly rate or weekly
wage will be set for each different labour grade/type of employee and an average hourly rate
will be applied for each grade (even though individual rates of pay may vary according to age
and experience).
Similar problems to those which arise when setting material standards in times of high
inflation can be met when setting labour standards.

Setting standards for material usage and labour efficiency
To estimate the materials required to make each product (material usage) and also the labour
hours required (labour efficiency), technical specifications must be prepared for each product
by production experts (either in the production department or the work study department).

Setting standards for overheads
When standard costs are fully absorbed costs (standard costs can be used in both marginal
and absorption costing systems), the absorption rate of fixed production overheads will be
Page 452

predetermined and based on budgeted fixed production overhead and planned production
volume.
Production volume will depend on two factors.
a) Production capacity (or 'volume capacity') measured perhaps in standard hours of
output (a standard hour being the amount of work achievable at standard efficiency
levels in an hour), which in turn reflects direct production labour hours.
b) Efficiency of working, by labour or machines, allowing for rest time and contingency
allowances.

Suppose that a department has a work force of ten men, each of whom works a 36 hour week
to make standard units, and each unit has a standard time of two hours to make. The expected
efficiency of the work-force is 125%.
a) Budgeted capacity, in direct labour hours, would be 10 × 36 = 360 production hours
per week.
b) Budgeted efficiency is 125% so that the work-force should take only 1 hour of actual
production time to produce 1.25 standard hours of output.
c) This means in our example that budgeted output is 360 production hours × 125% =
450 standard hours of output per week. At 2 standard hours per unit, this represents
production activity or volume of 225 units of output per week.

Example
ABC carries out routine office work in a sales order processing department, and all tasks in
the department have been given standard times. There are 40 clerks in the department who
work on average 140 hours per month each. The efficiency ratio of the department is 110%.

Required
Calculate the budgeted output in the department.

Page 453

Solution
Capacity

= 40 × 140 = 5,600 hours per month

Efficiency

= 110%

Budgeted output = 5,600 × 110% = 6,160 standard hours of work per month.

Setting standards for sales price and margin
The standard selling price will depend on a number of factors including the following.
•
•

Anticipated market demand
Competing products

•
•

Manufacturing costs
Inflation estimates

The standard sales margin is the difference between the standard cost and the standard
selling price.

The following problems can occur when setting standards.
a) Deciding how to incorporate inflation into planned unit costs
b) Agreeing on a performance standard (attainable or ideal)
c) Deciding on the quality of materials to be used (a better quality of material will cost
more, but perhaps reduce material wastage)
d) Estimating materials prices where seasonal price variations or bulk purchase
discounts may be significant
e) Finding sufficient time to construct standards as standard setting can be time
consuming
f) Incurring the cost of setting up and maintaining a system for establishing standards

Page 454

Types of standard
There are four types of standard:
•
•
•
•

ideal,
attainable,
current and
basic.

These can have an impact on employee motivation.
How demanding should a standard be?
Should the standard represent perfect performance, easily attainable performance or
achievable target?

An ideal standard is a standard which can be attained under perfect operating conditions: no
wastage, no inefficiencies, no idle time, no breakdowns
An attainable standard is a standard which can be attained if production is carried out
efficiently, machines are properly operated and/or materials are properly used. Some
allowance is made for wastage and inefficiencies
A current standard is standard based on current working conditions (current wastage,
current inefficiencies)
A basic standard is a long-term standard which remains unchanged over the years and is
used to show trends

The different types of standard have a number of advantages and disadvantages.
a) Ideal standards can be seen as long-term targets but are not very useful for day-today control purposes.
b) Ideal standards cannot be achieved. If such standards are used for budgeting, an
allowance will have to be included to make the budget realistic and attainable.
c) Attainable standards can be used for product costing, cost control, stock (raw
materials,WIP & finished goods) valuation, estimating and as a basis for budgeting.
Page 455

d) Current standards or attainable standards provide the best basis for budgeting,
because they represent an achievable level of productivity.
e) Current standards do not attempt to improve on current levels of efficiency.
f) Current standards are useful during periods when inflation is high. They can be set
on a month by month basis.
g) Basic standards are used to show changes in efficiency or performance over a long
period of time. They are perhaps the least useful and least common type of standard in
use.

The impact on employee behaviour of the type of standard set
The type of standard set can have an impact on the behaviour of the employees trying to
achieve those standards.
Type of
standard

Impact

Ideal

Some say that they provide employees with an incentive to be more
efficient even though it is highly unlikely that the standard will be
achieved. Others argue that they are likely to have an unfavourable effect
on employee motivation because the differences between standards and
actual results will always be adverse. The employees may feel that the
goals are unattainable and so they will not work so hard.

Attainable

Might be an incentive to work harder as they provide a realistic but
challenging target of efficiency.

Current

Will not motivate employees to do anything more than they are
currently doing.

Basic

May have an unfavourable impact on the motivation of employees. Over
time they will discover that they are easily able to achieve the standards.
They may become bored and lose interest in what they are doing if they
have nothing to aim for.

Page 456

BUDGETS AND STANDARDS COMPARED
Budgets and standards are very similar and interrelated, but there are important differences
between them.
You will recall from previous chapters that a budget is a quantified monetary plan for a
future period, which managers will try to achieve. Its major function lies in communicating
plans and coordinating activities within an organisation.
On the other hand, a standard is a carefully predetermined quantity target which can be
achieved under certain conditions.
Budgets and standards are similar in the following ways.
a) They both involve looking to the future and forecasting what is likely to happen
given a certain set of circumstances.
b) They are both used for control purposes. A budget aids control by setting financial
targets or limits for a forthcoming period. Actual achievements or expenditures are
then compared with the budgets and action is taken to correct any variances where
necessary. A standard also achieves control by comparison of actual results against a
predetermined target.

As well as being similar, budgets and standards are interrelated. For example, a standard
unit production cost can act as the basis for a production cost budget. The unit cost is
multiplied by the budgeted activity level to arrive at the budgeted expenditure on production
costs.
There are, however, important differences between budgets and standards.
Budgets

Standards

Gives planned total aggregate
costs for a function or cost
centre

Shows the unit resource usage for a single task, for
example the standard labour hours for a single unit of
production

Can be prepared for all
functions, even where output
cannot be measured

Limited to situations where repetitive actions are
performed and output can be measured in more than
monetary terms

Expressed in money terms

Need not be expressed in money terms. For example a
standard rate of output does not need a financial value
put on it
Page 457

BLANK

Page 458

ALLOWING FOR WASTE AND IDLE TIME
In the exam you may be asked to deal with a situation in which not all resources are actually
used to make saleable products.

Wastage
The amount of raw material used to meet the budgeted production level might be less than the
amount of raw material contained in the finished products for a number of reasons.
•
•
•

Evaporation
Spillage
Natural wastage (such as the skin of fruit used to make fruit juice)

This wastage can be built into an attainable materials standard and adjustments can be
made to materials budgets.
If the wastage occurs before production commences, the materials purchases budget must
be adjusted. If the wastage occurs during production, the materials usage budget must be
adjusted.

Example: wastage in budgets
The production quantities required for each of the first four periods of the year are as follows.

Production budget – units
Period 1

Period 2

Period 3

Period 4

10,204

13,010

14,796

12,755

Page 459

From the standard cost card it is determined that each unit of production requires 2 kg of raw
material X. However we also know that the production process has a normal loss of 20% of
the materials input into the process.
This means that although each unit of product requires 2 kg of material X, this represents
only 80% of the actual amount required. 25% (100/80) more than 2 kg per unit must be input
into the process. The amount of material X required for each unit is therefore:
2 kg × 100/80 = 2.5 kg
The amount of normal loss can be calculated separately as:
2 kg × 20/80 = 0.5 kg
The materials usage budget can now be prepared.

Materials usage budget

Quantity of production

Period 1

Period 2

Period 3

Period 4

10,204

13,010

14,796

12,755

32,525 kg

36,990 kg

31,888 kg

Materials usage (quantity × 2.5 25,510 kg
kg)

Example
A business requires 15,400 units of production in a period and each unit uses 5 kg of raw
materials. The production process has a normal loss of 10% during the production process.
What is the total amount of the raw material required for the period?

Kg

Solution
Kg required for production

5 × 15,400

Additional for normal loss

77,000 × 10/90

Required usage

(15,400 × 5 × 100/90)

Page 460

77,000
8,556
85,556

Idle time
A workforce that is expected to work at a particular level of efficiency may not always be
able to achieve this. Idle time may be caused by machine breakdowns or not having enough
work to give to employees, perhaps because of bottlenecks in production or a shortage of
orders for customers.
Idle time can again be built into an attainable labour hours standard and adjustments can
be made to the labour budget.

Example: idle time and standards
A machine has running costs of RWF60,000 per hour and typically incurs 5% non-productive
time. To get 60 minutes of output (a standard hour) would take 60 ÷ (100 – 5)% = 63.16
minutes. The standard cost of production or cost of idle time is therefore RWF63,160 per
hour.

Example: idle time and the labour usage budget
The standard cost card for the Stephenson shows that the standard time for production of one
unit is 1 grade A labour hour. However due to necessary break times only 80% of the time
paid is productive, that is there is 20% idle time.
To calculate the number of hours of labour required, again the starting point will be the
production budget showing the number of units to be produced in each period. However the
number of hours that must be paid in total in order to produce one unit is:
1 hour × 100/80 = 1.25 hours
The idle time per product can be calculated as 1 hour ×

Page 461

20
= 0.25 hours.
80

Production budget

Quantity of production

Period 1

Period 2

Period 3

Period 4

10,204

13,010

14,796

12,755

Period 1

Period 2

Period 3

Period 4

12,755 hrs

16,263 hrs

18,495 hrs

15,944 hrs

Labour usage budget – hours

Labour hours

Example
A product requires 10 labour hours for each unit. However 10% of working hours are idle
time. For how long must an employee be paid in order to produce 20 units?

Solution
Standard time

20 units × 10 hours

Additional time

200 × 10/90

Total time required

20 × 10 × 100/90

Page 462

200
22
222

FLEXIBLE BUDGETS
Comparison of a fixed budget with the actual results for a different level of activity is of little
use for control purposes. Flexible budgets should be used to show what cost and revenues
should have been for the actual level of activity.

A flexible budget is a budget which, by recognising different cost behaviour patterns, is
designed to change as volume of activity changes.
If you previously studied F2, you will be familiar with this material.

Preparing a flexible budget
Step 1

The first step in the preparation of a flexible budget is the determination of cost
behaviour patterns, which means deciding whether costs are fixed, variable
or semi-variable.

Step 2

The second step in the preparation of a flexible budget is to calculate the budget
cost allowance for each cost item.
Budget cost allowance = budgeted fixed cost* + (number of units × variable
cost per unit)**
* nil for variable cost
** nil for fixed cost
Semi-variable costs therefore need splitting into their fixed and variable
components so that the budget cost allowance can be calculated. One method for
splitting semi-variable costs is the high-low method, which we covered in Study
Unit 14.

Page 463

Example: preparing a flexible budget
a) Prepare a budget for 20X6 for the direct labour costs and overhead expenses of a
production department flexed at the activity levels of 80%, 90% and 100%, using the
information listed below.
(i)

The direct labour hourly rate is expected to be RWF 375.

(ii)

100% activity represents 60,000 direct labour hours.

(iii) Variable costs
Indirect labour
Consumable supplies
Canteen and other
welfare services

RWF75 per direct labour hour
RWF37.5 per direct labour hour
6% of direct and indirect labour costs

(iv) Semi-variable costs are expected to relate to the direct labour hours in the same
manner as for the last five years.

Year

Direct labour
hours

20X1
20X2
20X3
20X4
20X5

(v)

Semi-variable
costs
RWF ‘000
64,000
2,080
59,000
1,980
53,000
1,860
49,000
1,780
40,000 (estimate)
1,600 (estimate)

Fixed costs
RWF ‘000
1,800
1,000
400
1,500
2,500

Depreciation
Maintenance
Insurance
Rates
Management salaries

(vi) Inflation is to be ignored.

Page 464

b) Calculate the budget cost allowance (ie expected expenditure) for 20X6 assuming that
57,000 direct labour hours are worked.

Solution
a)

Direct labour

80% level

90% level

100% level

48,000 hrs

54,000 hrs

60,000 hrs

RWF ‘000

RWF '000

RWF '000

18,000

20,250

22,500

3,600

4,050

4,500

Other variable costs
Indirect labour
Consumable supplies

1,800

2,025

2,250

Canteen etc

1,296

1,458

1,620

Total variable costs (Rwf515 per hour)

24,696

24,696

24,696

Semi-variable costs (W)

1,760

1,880

2,000

Depreciation

1,800

1,800

1,800

Maintenance

1,000

1,000

1,000

400

400

400

1,500

1,500

1,500

2,500

2,500

Fixed costs

Insurance
Rates
Management salaries
Budgeted costs

2,500
33,656

Page 465

36,863

40,070

Working
Using the high/low method:
RWF ‘000
Total cost of 64,000 hours

2,080

Total cost of 40,000 hours

1,600

Variable cost of 24,000 hours

480

Variable cost per hour (RWF480,000/24,000)

0.02

RWF‘000
Total cost of 64,000 hours

2,080
1,2800

Variable cost of 64,000 hours (× RWF0.020)
Fixed costs

800

Semi-variable costs are calculated as follows.
RWF ‘000
60,000 hours

(60,000 × RWF20) + RWF800,000

=

2,000

54,000 hours

(54,000 RWF20) + RWF800,000

=

1,880

48,000 hours

(48,000 × RWF20) + RWF800,000

=

1,760

Page 466

b) The budget cost allowance for 57,000 direct labour hours of work would be as
follows.
RWF ‘000
Variable costs

(57,000 × RWF k5.145)

Semi-variable costs

(RWFk800+ (57,000 × RWF k0.020))

Fixed costs

29,326.5
1,940.0

7,200
384,66.5

Flexible budgets and performance management
Budgetary control involves drawing up budgets for the areas of responsibility for individual
managers (for example production managers, purchasing managers and so on) and of
regularly comparing actual results against expected results. The differences between actual
results and expected results are called variances and these are used to provide a guideline for
control action by individual managers.
Note that individual managers are held responsible for investigating differences between
budgeted and actual results, and are then expected to take corrective action or amend the plan
in the light of actual events.
The wrong approach to budgetary control is to compare actual results against a fixed budget.
Suppose that a company manufactures a single product, Z. Budgeted results and actual results
for June 20X2 are shown below.

Page 467

Production and sales of the cloud (units)

Budget

Actual
results

2,000

3,000

Variance

RWF ‘000

RWF ‘000

RWF ‘000

Sales revenue (a)

20,000

30,000

10,000 (F)

Direct materials

6,000

8,500

2,500 (A)

Direct labour

4,000

4,500

500 (A)

Maintenance

1,000

1,400

400 (A)

Depreciation

2,000

2,200

200 (A)

Rent and rates

1,500

1,600

100 (A)

Other costs

3,600

5,000

1,400 (A)

Total costs (b)

18,100

23,200

5,100

Profit (a) – (b)

1,900

6,800

4,900 (F)

a) Here the variances are meaningless for control purposes. Costs were higher than
budget because the output volume was also higher; variable costs would be expected
to increase above the costs budgeted in the fixed budget. There is no information to
show whether control action is needed for any aspect of costs or revenue.
b) For control purposes, it is necessary to know the following.
(i)

Were actual costs higher than they should have been to produce and sell 3,000
Zs?

(ii)

Was actual revenue satisfactory from the sale of 3,000 Zs?

Page 468

The correct approach to budgetary control is as follows.
a) Identify fixed and variable costs
b) Produce a flexible budget using marginal costing techniques

Let's suppose that we have the following estimates of cost behaviour for the company.
a) Direct materials, direct labour and maintenance costs are variable.
b) Rent and rates and depreciation are fixed costs.
c) Other costs consist of fixed costs of RWF1,600,000 plus a variable cost of RWF1,000
per unit made and sold.

Now that the cost behaviour patterns are known, a budget cost allowance can be calculated
for each item of expenditure. This allowance is shown in a flexible budget as the expected
expenditure on each item for the relevant level of activity. The budget cost allowances are
calculated as follows - RWF values in thousands.

a) Variable cost allowances
eg material cost allowance

b) Fixed cost allowances

= original budgets × (3,000 units/2,000 units)
= RWF6,000 × 3/2 = RWF9,000

= as original budget

c) Semi-fixed cost allowances = original budgeted fixed costs
+ (3,000 units × variable cost per unit)
eg other cost allowances

= RWF1,600 + (3,000 × RWF1) = RWF4,600

The budgetary control analysis should be as follows.

Page 469

Fixed
budget

Flexible
budget

Actual
results

Budget
variance

(a)

(b)

(c)

(b) – (c)

2,000

3,000

3,000

RWF
‘000

RWF
‘000

RWF
‘000

RWF ‘000

20,000

30,000

30,000

0

Direct materials

6,000

9,000

8,500

500 (F)

Direct labour

4,000

6,000

4,500

1,500 (F)

Maintenance

1,000

1,500

1,400

100 (F)

3,600

4,600

5,000

400 (A)

2,000

2,000

2,200

200 (A)

Production and sales (units)

Sales revenue
Variable costs

Semi-variable costs
Other costs
Fixed costs
Depreciation
Rent and rates

1,500

1,500

1,600

100 (A)

Total costs

18,100

24,600

23,200

1,400 (F)

Profit

1,900

5,400

6,800

1,400 (F)

Note. (F) denotes a favourable variance and (A) an adverse or unfavourable variance.
We can analyse the above as follows again RWF in thousands.
a) In selling 3,000 units the expected profit should have been, not the fixed budget profit
of RWF1,900, but the flexible budget profit of RWF5,400. Instead, actual profit was
RWF6,800 ie RWF1,400 more than we should have expected. One of the reasons for
the improvement is that, given output and sales of 3,000 units, costs were lower
than expected (and sales revenue exactly as expected).
Page 470

Rwf ‘000
Direct materials cost variance

500 (F)

Direct labour cost variance

1,500 (F)

Maintenance cost variance

100 (F)

Other costs variance

400 (A)

Fixed cost variances
Depreciation

200 (A)

Rent and rates

100 (A)
1,400 (F)

b) Another reason for the improvement in profit above the fixed budget profit is the sales
volume (3,000 Zs were sold instead of 2,000).

RWF ‘000
Sales revenue increased by

RWF ‘000
10,000

Variable costs increased by:
Direct materials

3,000

Direct labour

2,000

Maintenance

500

Variable element of other costs

1,000

Fixed costs are unchanged

6,500

Profit increased by

3,500

Profit was therefore increased by RWF 3,500,000 because sales volumes increased.
Page 471

c) A full variance analysis statement would be as follows.
RWF ‘000
Fixed budget profit

RWF ‘000
1,900

Variances
Sales volume

3,500 (F)

Direct materials cost

500 (F)

Direct labour cost

1,500 (F)

Maintenance cost

100 (F)

Other costs

400 (A)

Depreciation

200 (A)

Rent and rates

100 (A)
4,900 (F)

Actual profit

6,800

If management believes that any of these variances are large enough to justify it, they will
investigate the reasons for them to see whether any corrective action is necessary or whether
the plan needs amending in the light of actual events.

Page 472

Factors to consider when preparing flexible budgets
The mechanics of flexible budgeting are, in theory, fairly straightforward but in practice there
are a number of points to consider before figures are simply flexed.
a) Splitting mixed costs is not always straightforward.
b) Fixed costs may behave in a step-line fashion as activity levels increase/decrease.
c) Account must be taken of the assumptions upon which the original fixed budget was
based. Such assumptions might include the constraint posed by limiting factors, the
rate of inflation, judgements about future uncertainty, the demand for the
organisation's products and so on.
d) 'Flexing … can incorporate changes for any factor which differs from that which
applied when the budget was prepared, for example different states of the economy.
In this way, flexing is saying "If I knew then what I know now, what budget would I
set?" It is a useful concept but can lead to some concern, if taken to extremes, because
managers can be confused and frustrated if faced throughout the year with a possibly
moving target.'
(Mike Tayles, ACCA Students Newsletter, December 1998)

The need for flexible budgets
We have seen that flexible budgets may be prepared in order to plan for variations in the level
of activity above or below the level set in the fixed budget. It has been suggested, however,
that since many cost items in modern industry are fixed costs, the value of flexible budgets in
planning is dwindling.
a) In manufacturing industries, especially in Europe or North America,, plant costs
(depreciation, rent and so on) are a very large proportion of total costs, and these tend
to be fixed costs.
b) Wage costs also tend to be fixed, because employees are generally guaranteed a basic
wage for a working week of an agreed number of hours.
c) With the growth of service industries, labour (wages or fixed salaries) and overheads
will account for most of the costs of a business, and direct materials will be a
relatively small proportion of total costs.
Page 473

Flexible budgets are nevertheless necessary, and even if they are not used at the planning
stage, they must be used for budgetary control variance analysis.

Page 474

THE PRINCIPLE OF CONTROLLABILITY
The principle of controllability is that managers of responsibility centres should only be
held accountable for costs over which they have some influence.

Budget centres
Budgetary control is based around a system of budget centres. Each budget centre will have
its own budget and a manager will be responsible for managing the budget centre and
ensuring that the budget is met.
The selection of budget centres in an organisation is therefore a key first step in setting up a
control system. What should the budget centres be? What income, expenditure and/or capital
employment plans should each budget centre prepare? And how will measures of
performance for each budget centre be made?

Page 475

A well-organised system of control should have the following features.

Feature

Explanation

A hierarchy of
budget centres

If the organisation is quite large a hierarchy is needed. Subsidiary
companies, departments and work sections might be budget
centres. Budgets of each section would then be consolidated into
a departmental budget, departmental budgets in turn would be
consolidated into the subsidiary's budget, and the budgets of each
subsidiary would be combined into a master budget for the
group as a whole.

Clearly identified
responsibilities for
achieving budget
targets

Individual managers should be made responsible for achieving
the budget targets of a particular budget centre.

Responsibilities for
revenues, costs and
capital employed

Budget centres should be organised so that all the revenues earned
by an organisation, all the costs it incurs, and all the capital it
employs are made the responsibility of someone within the
organisation, at an appropriate level of authority in the
management hierarchy.

Budgetary control and budget centres are therefore part of the overall system of
responsibility accounting within an organisation.

Responsibility accounting is a system of accounting that segregates revenue and costs into
areas of personal responsibility in order to monitor and assess the performance of each part of
an organisation.

Controllable costs
Controllable costs are items of expenditure which can be directly influenced by a given
manager within a given time span.

Page 476

Care must be taken to distinguish between controllable costs and uncontrollable costs in
variance reporting. The controllability principle is that managers of responsibility centres
should only be held accountable for costs over which they have some influence. From a
motivation point of view this is important because it can be very demoralising for managers
who feel that their performance is being judged on the basis of something over which they
have no influence. It is also important from a control point of view in that control reports
should ensure that information on costs is reported to the manager who is able to take action
to control them.
Responsibility accounting attempts to associate costs, revenues, assets and liabilities with
the managers most capable of controlling them. As a system of accounting, it therefore
distinguishes between controllable and uncontrollable costs.
Most variable costs within a department are thought to be controllable in the short term
because managers can influence the efficiency with which resources are used, even if they
cannot do anything to raise or lower price levels.
A cost which is not controllable by a junior manager might be controllable by a senior
manager. For example, there may be high direct labour costs in a department caused by
excessive overtime working. The junior manager may feel obliged to continue with the
overtime to meet production schedules, but his senior may be able to reduce costs by hiring
extra full-time staff, thereby reducing the requirements for overtime.
A cost which is not controllable by a manager in one department may be controllable by
a manager in another department. For example, an increase in material costs may be
caused by buying at prices higher than expected (controllable by the purchasing department)
or by excessive wastage (controllable by the production department) or by a faulty machine
producing rejects (controllable by the maintenance department).
Some costs are non-controllable, such as increases in expenditure items due to inflation.
Other costs are controllable, but in the long term rather than the short term. For
example, production costs might be reduced by the introduction of new machinery and
technology, but in the short term, management must attempt to do the best they can with the
resources and machinery at their disposal.

Page 477

The controllability of fixed costs
It is often assumed that all fixed costs are non-controllable in the short run. This is not so.
a) Committed fixed costs are those costs arising from the possession of plant,
equipment, buildings and an administration department to support the long-term
needs of the business. These costs (depreciation, rent, administration salaries) are
largely non-controllable in the short term because they have been committed by
longer-term decisions affecting longer-term needs. When a company decides to cut
production drastically, the long-term committed fixed costs will be reduced, but only
after redundancy terms have been settled and assets sold.
b) Discretionary fixed costs, such as advertising and research and development costs,
are incurred as a result of a top management decision, but could be raised or lowered
at fairly short notice (irrespective of the actual volume of production and sales).

Controllability and apportioned costs
Managers should only be held accountable for costs over which they have some
influence. This may seem quite straightforward in theory, but it is not always so easy in
practice to distinguish controllable from uncontrollable costs. Apportioned overhead costs
provide a good example.
Suppose that a manager of a production department in a manufacturing company is made
responsible for the costs of his department. These costs include directly attributable overhead
items such as the costs of indirect labour employed and indirect materials consumed in the
department. The department's overhead costs also include an apportionment of costs from other
cost centres, such as rent and rates for the building it shares with other departments and a share
of the costs of the maintenance department.
Should the production manager be held accountable for any of these apportioned costs?
a) Managers should not be held accountable for costs over which they have no control.
In this example, apportioned rent and rates costs would not be controllable by the
production department manager.
b) Managers should be held accountable for costs over which they have some influence.
In this example, it is the responsibility of the maintenance department manager to
keep maintenance costs within budget. But their costs will be partly variable and
partly fixed, and the variable cost element will depend on the volume of demand for
Page 478

their services. If the production department's staff treat their equipment badly we
might expect higher repair costs, and the production department manager should
therefore be made accountable for the repair costs that his department makes the
maintenance department incur on its behalf.
c) Charging the production department with some of the costs of the maintenance
department prevents the production department from viewing the maintenance
services as 'free services'. Over-use would be discouraged and the production manager
is more likely to question the activities of the maintenance department possibly
resulting in a reduction in maintenance costs or the provision of more efficient
maintenance services.

Controllability and dual responsibility
Quite often a particular cost might be the responsibility of two or more managers. For
example, raw materials costs might be the responsibility of the purchasing manager (prices)
and the production manager (usage). A reporting system must allocate responsibility
appropriately. The purchasing manager must be responsible for any increase in raw
materials prices whereas the production manager should be responsible for any increase in
raw materials usage.
This where Standard Cost analysis can be useful. It can show price and usage variances
separately.

You can see that there are no clear cut rules as to which costs are controllable and which are
not. Each situation and cost must be reviewed separately and a decision taken according to
the control value of the information and its behavioural impact.

Page 479

CHAPTER ROUNDUP
•

A standard cost is an estimated unit cost built up of standards for each cost element
(standard resource price and standard resource usage)

•

Standard costing is principally used to value inventories and cost production and to
act as a control device.

•

Standard costing is most suited to mass production and repetitive assembly work.

•

The responsibility for deriving standard costs should be shared between managers
able to provide the necessary information about levels of expected efficiency,
prices and overhead costs.

•

There are four types of standard: ideal, attainable, current and basic. These can
have an impact on employee motivation.

•

Budgets and standards are very similar and interrelated, but there are important
differences between them.

•

Comparison of a fixed budget with the actual results for a different level of activity is
of little use for control purposes. Flexible budgets should be used to show what cost
and revenues should have been for the actual level of activity.

•

The principle of controllability is that managers of responsibility centres should only
be held accountable for costs over which they have some influence.

•

Controllable costs are items of expenditure which can be directly influenced by a
given manager within a given time span.

Page 480

STUDY UNIT 16
Variance Analysis
Contents

Page

Basic Variances ……………………………………..………………..…….

483

The Reasons For Variances ……….………………………..………..……..

489

Labour Variances And The Learning Curve …………………………….

491

Idle Time And Waste ………………………………………….................…

495

Operating Statements …………………………………………..….....……

503

ABC And Variance Analysis ………………….……………………..……

511

Investigating Variances ………………….…………………………..……

515

Materials Mix And Yield Variances ………………….……………..……

523

Page 481

EXAM GUIDE
The variance calculations set in this paper are likely to be the more complicated variances
and you will be required to explain them and evaluate performance.

Page 482

BASIC VARIANCES
Knowledge brought forward from earlier studies
•

A variance is the difference between an actual result and an expected result.

•

Variance analysis is the process by which the total difference between standard and
actual results is analysed.

•

When actual results are better than expected results, we have a favourable variance
(F). If actual results are worse than expected results, we have an adverse variance
(A).

•

The selling price variance measures the effect on expected profit of a selling price
different from the standard selling price. It is calculated as the difference between
what the sales revenue should have been for the actual quantity sold, and what it was.

•

The sales volume variance measures the increase or decrease in expected profit as a
result of the sales volume being higher or lower than budgeted. It is calculated as the
difference between the budgeted sales volume and the actual sales volume multiplied
by the standard profit per unit.

•

The material total variance is the difference between what the output actually cost
and what it should have cost, in terms of material(s). It can be divided into the
following two sub-variances.

•

The material price variance is the difference between what the material did cost and
what it should have cost.

•

The material usage variance is the difference between the standard cost of the material
that should have been used and the standard cost of the material that was used.

•

The labour total variance is the difference between what the output should have cost
and what it did cost, in terms of labour. It can be divided into two sub-variances.

•

The labour rate variance is the difference between what the labour did cost and what
it should have cost.

•

The labour efficiency variance is the difference between the standard cost of the
hours that should have been worked and the standard cost of the hours that were
worked.

Page 483

•

The variable production overhead total variance is the difference between what
the output should have cost and what it did cost, in terms of variable production
overhead. It can be divided into two sub-variances.

•

The variable production overhead expenditure variance is the difference between
the amount of variable production overhead that should have been incurred in the
actual hours actively worked, and the actual amount of variable production overhead
incurred.

•

The variable production overhead efficiency variance is the difference between the
standard cost of the hours that should have been worked for the number of units
actually produced, and the standard cost of the actual number of hours worked.

•

Fixed production overhead total variance is the difference between fixed
production overhead incurred and fixed production overhead absorbed. In other
words, it is the under– or over-absorbed fixed production overhead.

•

Fixed production overhead expenditure variance is the difference between the
budgeted fixed production overhead expenditure and actual fixed production overhead
expenditure.

•

Fixed production overhead volume variance is the difference between actual and
budgeted production/volume multiplied by the standard absorption rate per unit.

•

Fixed production overhead volume efficiency variance is the difference between the
number of hours that actual production should have taken, and the number of hours
actually taken (that is, worked) multiplied by the standard absorption rate per hour.

•

Fixed production overhead volume capacity variance is the difference between
budgeted hours of work and the actual hours worked, multiplied by the standard
absorption rate per hour.

Example
A company produces and sells one product only, the Thing, the standard cost for one unit
being as follows.

Direct material A – 10 kilograms at RWF20 per kg
Direct material B – 5 litres at RWF6 per litre
Page 484

RWF
200
30

Direct wages – 5 hours at RWF6per hour
Fixed production overhead
Total standard cost

30
50
310

The fixed overhead included in the standard cost is based on an expected monthly output of
900 units. Fixed production overhead is absorbed on the basis of direct labour hours.

During April the actual results were as follows.
Production

800 units

Material A

7,800 kg used, costing RWF159,900

Material B

4,300 litres used, costing RWF23,650

Direct wages

4,200 hours worked for RWF24,150

Fixed production overhead

RWF47,000

Required
a) Calculate price and usage variances for each material.
b) Calculate labour rate and efficiency variances.
c) Calculate fixed production overhead expenditure and volume variances and then
subdivide the volume variance.

Solution
a) Price variance – A
7,800 kgs should have cost (× RWF20)

RWF
156,000

but did cost
Price variance

159,900
3,900 (A)
Page 485

Usage variance – A
800 units should have used (× 10 kgs)
but did use
Usage variance in kgs
× standard cost per kilogram

8,000 kgs
7,800 kgs
200 kgs (F)
× RWF20

Usage variance

RWF4,000 (F)

Price variance – B
4,300 litres should have cost (× RWF6)

RWF
25,800

but did cost
Price variance

23,650
2,150 (F)

Usage variance – B
RWF
4,000 l

800 units should have used (× 5 l)
but did use
Usage variance in litres
× standard cost per litre

4,300 l
300 (A)
× RWF6

Usage variance

RWF1,800 (A)

b) Labour rate variance

4,200 hours should have cost (× Rwf6)
but did cost
Rate variance

Page 486

RWF
25,200
24,150
1,050 (F)

Labour efficiency variance
800 units should have taken (× 5 hrs)
but did take
Efficiency variance in hours
× standard rate per hour

4,000 hrs
4,200 hrs
200 hrs (A)
× RWF6

Efficiency variance

RWF1,200 (A)

c) Fixed overhead expenditure variance
Budgeted expenditure (RWF50 × 900)

RWF
45,000

Actual expenditure
Expenditure variance

47,000
2,000 (A)

Fixed overhead volume variance
Budgeted production at standard rate (900 × RWF50)

RWF
45,000

Actual production at standard rate (800 × RWF50)

40,000

Volume variance

5,000 (A)

Fixed overhead volume efficiency variance

800 units should have taken (× 5 hrs)
but did take

4,000 hrs
4,200 hrs

Volume efficiency variance in hours
× standard absorption rate per hour

200 hrs
× RWF10

Volume efficiency variance

RWF2,000 (A)

Page 487

Fixed overhead volume capacity variance
Budgeted hours

4,500 hrs

Actual hours

4,200 hrs

Volume capacity variance in hours

300 hrs (A)

× standard absorption rate per hour (RWF50 ÷ 5)

× RWF10

RWF3,000 (A)

Exam Focus Point
You have to be very happy with basic variance calculations so it is essential to do more
practice if you struggled with this question.

Page 488

THE REASONS FOR VARIANCES
Knowledge brought forward from Formation 2 Management
Accounting
In an examination question you should review the information given and use your
imagination and common sense to suggest possible reasons for variances.
Variance

Favourable

Adverse

Material price

Unforeseen discounts received
Greater care taken in purchasing
Change in material standard

Price increase
Careless purchasing
Change in material standard

Material usage

Material used of higher quality than
standard
More effective use made of material
Errors in allocating material to jobs

Defective material
Excessive waste
Theft
Stricter quality control
Errors in allocating material to jobs

Labour rate

Use of workers at a rate of pay lower
than standard

Wage rate increase

Idle time

Possible if idle time has been built into
the budget

Machine breakdown
Non-availability of material
Illness or injury to worker

Labour
efficiency

Output produced more quickly than
expected, because of work motivation,
better quality of equipment or
materials, better learning rate
Errors in allocating time to jobs

Lost time in excess of standard
allowed
Output lower than standard set
because of lack of training,
sub-standard material etc
Errors in allocating time to jobs

Overhead
expenditure

Savings in costs incurred
More economical use of services

Increase in cost of services
Excessive use of services
Change in type of services used

Overhead
volume

Production or level of activity greater
than budgeted.

Production or level of activity less
than budgeted

Fixed overhead
capacity

Production or level of activity greater
than budgeted

Production or level of activity less
than budgeted

Selling price

Unplanned price increase

Unplanned price reduction

Sales volume

Additional demand

Unexpected fall in demand
Production difficulties
Page 489

BLANK

Page 490

LABOUR VARIANCES AND THE LEARNING
CURVE
Care must be taken when interpreting labour variances where the learning curve has been
used in the budget process.

In Chapter 14 we looked at how the learning curve can be used for forecasting production
time and labour costs in certain circumstances where the workforce as a whole improves in
efficiency with experience.
Companies that use standard costing for much of their production output cannot apply
standard times to output where a learning effect is taking place. This problem can be
overcome in practice by:
a) Establishing standard times for output, once the learning effect has worn off or
become insignificant; and
b) Introducing a ‘launch cost’ budget for the product for the duration of the learning
period.

When learning is still taking place, it would be unreasonable to compare actual times with the
standard times that ought eventually to be achieved so allowances must be made when
interpreting labour efficiency variances. Standard costs should reflect the point that has been
reached on the learning curve. When learning has become insignificant, standards set on the
basis of this 'steady state' will be different to when learning was taking place. If the learning
rate has been wrongly calculated, this must be allowed for in the variance calculations.

Example: labour variances and the learning curve
A new product has been introduced for which an 80% learning curve is expected to apply.
The standard labour information has been based on estimates of the time needed to produce
the first unit which is 200 hours at RWF5 per hour.
The first four units took 700 hours to produce at a cost of RWF37,500.

Page 491

Required
a) The original labour rate and efficiency variances.
b) The labour rate and efficiency variances which take into account the learning effect.

Solution
a) Labour rate variance
RWF
700 hours should have cost (× RWF5)
but did cost

35,000
37,500

Labour rate variance

2,500

(A)

Labour efficiency variance
Four units should have taken (× 200 hrs)
but did take
Efficiency variance in hours

800

hours

700

hours

100

hours (F)

× standard rate per hour

× RWF5

Efficiency variance

RWF500

Page 492

(F)

b) Incorporating the learning curve effect
Average standard time per unit for 4 units
= 200 × 0.8 × 0.8
= 128 hours
Total expected time for 4 units
= 128 × 4
= 512 hours

Labour efficiency variance
Four units should have taken
but did take

× standard rate per hour

512

hours

700

hours

188

hours (A)

× RWF5
RWF940

(A)

The labour rate variance does not change but a favourable labour efficiency variance is now
adverse once the learning effect has been incorporated.

Page 493

BLANK

Page 494

IDLE TIME AND WASTE
In the previous chapter we looked at the meaning of waste and idle time and how they can be
allowed for in standards and budgets. We now need to calculate their effect on variances.

Idle time variance
Idle time may be caused by machine breakdowns or not having work to give to employees,
perhaps because of bottlenecks in production or a shortage of orders from customers. When it
occurs, the labour force is still paid wages for time at work, but no actual work is done. Such
time is unproductive and therefore inefficient. In variance analysis, idle time is an adverse
efficiency variance.

The idle time variance is the number of hours that labour were idle valued at the standard
rate per hour.

The idle time variance is shown as a separate part of the total labour efficiency variance. The
remaining efficiency variance will then relate only to the productivity of the labour force
during the hours spent actively working.
In Chapter 15 we discussed how budgets can be prepared which incorporate expected idle
time. An adverse variance will only result from idle time in excess of what was expected.

Example: Labour variances with idle time
During period 5, 1,500 units of product X were made and the cost of grade Z labour was
RWF17,500 for 3,080 hours. A unit of product X is expected to use 2 hours of grade Z labour
at a standard cost of RWF5 per labour hour. During the period, however, there was a shortage
of customer orders and 100 hours were recorded as idle time.

Page 495

Required
Calculate the following variances.
a) The labour total variance
b) The labour rate variance
c) The idle time variance
d) The labour efficiency variance

Solution
a) The labour total variance
RWF
1,500 units of product X should have cost (×RWF10)
but did cost

15,000
17,500

Labour total variance

2,500 (A)

Actual cost is greater than standard cost. The variance is therefore adverse.

b) The labour rate variance. This is a comparison of what the hours paid should have
cost and what they did cost.
RWF
3,080 hours of grade Z labour should have cost (× RWF5)

15,400
17,500

but did cost

2,100 (A)

Labour rate variance

Actual cost is greater than standard cost. The variance is therefore adverse.
Page 496

c) The idle time variance. This is the hours of idle time, valued at the standard rate per
hour.
Idle time variance = 100 hours (A) × RWF5 = RWF500 (A)

d) The labour efficiency variance. This considers the hours actively worked (the
difference between hours paid for and idle time hours) and is calculated by taking the
amount of output produced and comparing the time it should have taken to make
them, with the actual time spent actively making them (3,080 – 100 = 2,980 hours). The
variance in hours is valued at the standard rate per labour hour.
1,500 units of product X should take (× 2 hrs)
but did take (3,080 – 100)

3,000 hrs
2,980 hrs

Labour efficiency variance in hours

20 hrs (F)

× standard rate per hour

×RWF5

Labour efficiency variance

RWF100 (F)

e) Summary
RWF
Labour rate variance

2,100 (A)

Idle time variance

500 (A)

Labour efficiency variance

100 (F)

Labour total variance

2,500 (A)

Remember that, if idle time is recorded, the actual hours used in the efficiency variance
calculation are the hours worked and not the hours paid for.

Page 497

Example: Idle time in the budget
Bruno’s budget for April includes total budgeted machine time of 5,000 hours, and budgeted
output of 18,525 units. Due to inevitable delays for set-ups, idle time of 5% is allowed. Total
budgeted costs for the month are RWF44,460. In practice in April, actual machine hours
were 6,000, of which 800 were idle hours.

Required
a) Calculate the idle time variance.
b) Suggest why this variance has arisen and what could be done to control excess idle
time.

Solution
a)
Hours
Actual idle time

800

Standard idle time

5% of 6,000

Excess idle time

300
500

Cost of labour per hour (44,460/5,000)*100/95

RWF9.36

Variance 500 × RWF9.36 =

RWF4,680

b) There is an adverse variance due to excess idle time. Idle time of 5% is expected
for machine set-up delays and this has presumably taken longer than usually
expected. This could be due to faulty machinery or problems with staff trained
to do the set-ups. Management needs to investigate why the idle time was
excessive and take action to prevent re-occurrence.

Page 498

Wastage and material variances
In the same way as idle time, a certain amount of expected wastage may be built into the
material usage standard. A variance therefore needs to be calculated comparing the actual
results with a standard that has been adjusted for expected wastage.

Example
a)

Capella has prepared standard material specifications for each of products A and
B as follows.
(i)

Each finished unit of product A and product B contains 2 units and 6
units of component X respectively.

(ii)

The standard input requirements for both products must also allow for
losses during processing of 10% of the units of component X.

(iii) The standard purchase price for component X is RWF8 per unit.

Customer demand for period 2 for products A and B is budgeted at 2,280 units and
2,925 units respectively. It is budgeted that returns from customers of products A and B
requiring free replacement will be 5% and 2.5% respectively of goods delivered to
customers. No stocks of raw material, work-in-progress or finished goods are planned.

Required
(i)

Calculate the material purchase budget for period 2 (units and RWF) for
component X.

(ii)

Comment on the usefulness of standard specifications in the compilation of the
material budget for Capella rather than using the following actual information for
period 1.
Product A

Product B

Sales to customers (units)

2,500

2,750

Purchases of component X (units)

6,250

19,250

Page 499

Solution
a) (i) Losses and returns
The figure of two units of X required for product A represents (1 – 0.1) = 0.9 (or
90%) of the requirement, because of losses. Likewise the demand figure of 2,280
represents 95% of the production needs, because of returns.
Material purchases budget for period 2
A
Demand (units)

B
2,280

Returns (as decimal)

2,925

0.05

0.025

Demand allowing for returns (eg 2,925 × 1/(1 – 0.025))
(units)
2,400
Free replacements (units)

3,000

120

75

A

B

Standard input of X (units)

2

6

Losses (as decimal)

0.1

0.1

Standard input allowing for losses (eg 2 × 1/(1 –
0.1)) (units)
2.2222

Standard units of component X required (eg 3,000 × 6)
Losses in process (balance)

Page 500

6.6667

A

B

Units

Units

4,800

18,000

533

2,000

Actual units of component X required (eg 2,400 × 2.2222)

5,333

20,000

RWF

(ii)

Cost for product A (5,333 × Rwf8)

42,664

Cost for product B (20,000 × Rwf8)

160,000

Total cost

202,664

Usefulness of standard specifications
With sales of 2,500 units of product A the standard input figure suggests that
only 5,000 units of component X need be purchased. In fact 6,250 units were
required. Likewise for product B: the standard suggests that 16,500 units of X
would have been needed but actual needs were 19,250 units.
It could be argued that the current standard is misleading, and could lead to
under-purchasing of component X, and inability to meet demand or
additional emergency purchasing, at above average cost, later in the period. It
might be better to revise the standard to reflect the actual figures 6,250/2,500 =
2.5 units for product A and 19,250/2,750 = 7 units for product B.
On the other hand it could be argued that such a revision sends the wrong
signals to management. The losses in process and returns are undesirable, and it
may be possible to reduce them or eliminate them entirely. From this point of
view the current standards have been set at a level that will give rise to
variances: this will continually draw management attention to inefficiencies,
and give management a target to aim at.
The period 2 budget prepared for part (a)(i) illustrates this: effectively it sets
management a period 2 target of reducing requirements for component X from
2.5 units to 2.2 units (product A), and from 7 units to 6.6 units (product B).
The standard specifications are thus quite in keeping with the principles of Total
Quality Management.

Page 501

BLANK

Page 502

OPERATING STATEMENTS
Knowledge brought forward from earlier studies
•

An operating statement is a regular report for management which compares actual
costs and revenues with budgeted figures and shows variances.

•

There are several ways in which an operating statement may be presented. Perhaps the
most common format is one which reconciles budgeted profit to actual profit. Sales
variances are reported first, and the total of the budgeted profit and the two sales
variances results in a figure for 'actual sales minus the standard cost of sales'. The cost
variances are then reported, and an actual profit calculated.

Example
A company manufactures one product, and the entire product is sold as soon as it is produced.
There are no opening or closing inventories and work in progress is negligible. The company
operates a standard costing system and analysis of variances is made every month. The
standard cost card for the product, a widget, is as follows.

STANDARD COST CARD – WIDGET
All RWFvalues in thousands

RWF

Direct materials

0.5 kilos at RWF4 per kilo

2.00

Direct wages

2 hours at RWF2.00 per hour

4.00

Variable overheads

2 hours at RWF0.30 per hour

0.60

Fixed overhead

2 hours at RWF3.70 per hour

7.40

Standard cost

14.00

Standard profit

6.00

Standing selling price

20.00

Page 503

Budgeted output for January was 5,100 units. Actual results for January were as follows.
Production of 4,850 units was sold for RWF95,600
Materials consumed in production amounted to 2,300 kilos at a total cost of RWF9,800
Labour hours paid for amounted to 8,500 hours at a cost of RWF16,800
Actual operating hours amounted to 8,000 hours
Variable overheads amounted to RWF2,600
Fixed overheads amounted to RWF42,300

Required
Calculate all variances and prepare an operating statement for January.

Page 504

Answer
AGAIN RWF values in thousands RWF
a) 2,300 kg of material should cost (× Rwf4)
but did cost

9,200
9,800
Material price variance

b) 4,850 Widgets should use (× 0.5 kgs)
but did use

600 (A)

2,425 kg
2,300 kg

Material usage variance in kgs

125 kg (F)

× standard cost per kg

× RWF4

Material usage variance in RWF

RWF 500 (F)

c) 8,500 hours of labour should cost (× RWF2)
but did cost

17,000
16,800

Labour rate variance

200 (F)

d) 4,850 Widgets should take (× 2 hrs)
but did take (active hours)

9,700 hrs
8,000 hrs

Labour efficiency variance in hours

1,700 hrs (F)

× standard cost per hour

× RWF2

Labour efficiency variance

RWF3,400 (F)

Page 505

e) Idle time variance 500 hours (A) × RWF2

f) 8,000 hours incurring variable o/hd expenditure should cost

RWF1,000 (A)

2,400

(× RWF 0.30)
but did cost

2,600

Variable overhead expenditure variance

200 (A)

g) Variable overhead efficiency variance is the same as the
labour efficiency variance:
1,700 hours (F) × RWF0.30 per hour

h) Budgeted fixed overhead (5,100 units × 2 hrs × RWF3.70)
Actual fixed overhead

RWF 510 (F)

37,740
42,300

Fixed overhead expenditure variance

i) Actual production at standard rate (4,850 units × RWF7.40)
Budgeted production at standard rate (5,100 units × RWF7.40)
Fixed overhead volume variance

4,560 (A)

35,890
37,740
1,850 (A)

j) 4,850 Widgets should have sold for (× RWF20)
but did sell for

97,000
95,600

Selling price variance

1,400 (A)

k) Budgeted sales volume

5,100 units
Page 506

Actual sales volume

4,850 units

Sales volume variance in units

250 units

× standard profit per unit

× RWF6 (A)

Sales volume variance

RWF1,500 (A)

RWF

RWF

Budgeted profit (5,100 units × RWF6 profit)

30,600

Selling price variance

1,400

(A)

Sales volume variance

1,500

(A)
2,900 (A)

Actual sales (RWF95,600) less the standard cost of sales (4,850 × RWF14)

OPERATING STATEMENT FOR JANUARY
RWF

RWF

Budgeted profit

RWF
30,600

Sales variances: price

1,400 (A)

volume

1,500 (A)
2,900
(A)

Actual sales minus the standard cost of
sales

Page 507

27,700

27,700

Cost variances
(F)

(A)

RWF

RWF

Material price

RWF

600

Material usage

500

Labour rate

200

Labour efficiency

3,400

Labour idle time

1,000

Variable overhead expenditure

200

Variable overhead efficiency

510

Fixed overhead expenditure

4,560

Fixed overhead volume

1,850
4,610

8,210

Actual profit for January

3,600 (A)
24,100

Check
RWF
Sales

RWF
95,600

Materials

9,800

Labour

16,800

Variable overhead

2,600

Fixed overhead

42,300
71,500

Actual profit

24,100

Page 508

Operating statements in a marginal cost environment
Knowledge brought forward from earlier studies
•

There are two main differences between the variances calculated in an absorption
costing system and the variances calculated in a marginal costing system. In a
marginal costing system the only fixed overhead variance is an expenditure variance
and the sales volume variance is valued at standard contribution margin, not standard
profit margin.

Example
Returning to the question above, now assume that the company operates a marginal costing
system.

Required
Recalculate any variances necessary and produce an operating statement.
Solution
a) There is no fixed overhead volume variance.
b) The standard contribution per unit is RWF(20 – 6.60) = RWF13.40, therefore the
sales volume variance of 250 units (A) is valued at (× RWF13.40) = RWF3,350 (A).

The other variances are unchanged, therefore an operating statement might appear as follows.

Page 509

OPERATING STATEMENT FOR JANUARY
RWF

RWF

Budgeted profit
Budgeted fixed production costs

30,600
37,740

Budgeted contribution
Sales variances: volume
price

68,340
3,350 (A)
1,400 (A)

RWF

4,750 (A)
Actual sales (RWF95,600) minus the standard
variable cost of sales (4,850 × RWF6.60)
Variable cost variances
Material price
Material usage
Labour rate
Labour efficiency
Labour idle time
Variable overhead expenditure
Variable overhead efficiency

63,590
(F)

(A)

RWF

RWF
600

RWF

500
200
3,400
1,000
200
510
4,610

1,800
2,810 (F)
66,400

Actual contribution
Budgeted fixed production overhead
Expenditure variance
Actual fixed production overhead

37,740
4,560 (A)
42,300
24,100

Actual profit

Note. The profit here is the same as the profit calculated by standard absorption costing
because there were no changes in stock levels. Absorption costing and marginal costing do
not always produce an identical profit figure.

Page 510

ABC AND VARIANCE ANALYSIS
Within an ABC system, efficiency variances for longer-term variable overheads are the
difference between the level of activity that should have been needed and the actual activity
level, valued at the standard rate per activity.
All overheads within an ABC system are treated as variable costs, varying either with
production levels in the short term or with some other activity. The traditional method of
calculating fixed overhead variances is therefore not taken. The calculation of ABC overhead
variances is either the same as the traditional approach for variable overheads (if the
overhead varies with production level) or extremely similar (if it varies with some other
activity).

Approach for longer-term variable overheads
Expenditure variances are the difference between what expenditure should have been for
the actual level of activity and actual expenditure.
Efficiency variances are the difference between the level of activity that should have been
needed and the actual activity level, valued at the standard rate per activity.

Example: Simple ABC overhead variance analysis
The following information relates to B’s ordering activity during control period 2.

Budget
Output
Activity level
Total cost of activity

10,000 units
2,000 orders
RWF90,000,000

Output
Activity level
Total cost of activity

10,500 units
1,800 orders
RWF84,000,000

Actual

Page 511

Required
Calculate the overhead expenditure and efficiency variances relating to the ordering activity.

Solution
Expenditure variance
This is the difference between how much 1,800 orders should have cost and how much they
did cost.
Each order should cost RWF90,000,000/2,000 = RWF45,000. This is the cost driver rate.

RWF ‘000
1,800 orders should have cost (× RWF45,000)

81,000

but did cost

84,000

Expenditure variance

3,000 (A)

Efficiency variance
This is the difference between what the level of activity should have been for the output of
10,500 units, and what it was, valued at the standard rate per order (the cost driver rate).
Each unit of output should use 2,000/10,000 = 0.2 of an order

Activity level for 10,500 units should have been (× 0.2)

2,100 orders

but was

1,800 orders

Variance in orders

300 orders (F)

Page 512

× standard rate per order

× RWF45,000

Efficiency variance

RWF13,500,000 (F)

Usefulness of this analysis
Given the lack of relevant management information provided by traditionally-analysed fixed
overhead variances, the results using ABC analysis are of more use. It is clear in the
example above that one reason why the cost of the ordering activity was greater than it should
have been given the level of production was because the cost per order was above budget.
The main difference was because it took more orders than planned given the actual level
of production, however. The analysis has highlighted the efficiency of the ordering process
for investigation.

Page 513

BLANK

Page 514

INVESTIGATING VARIANCES
Materiality, controllability, the type of standard being used, variance trend,
interdependence and costs should be taken into account when deciding on the significance
of a variance.

The decision whether or not to investigate
Before management decide whether or not to investigate the reasons for the occurrence of a
particular variance, there are a number of factors which should be considered in assessing the
significance of the variance.

Materiality
Because a standard cost is really only an average expected cost, small variations between
actual and standard are bound to occur and are unlikely to be significant. Obtaining an
'explanation' of the reasons why they occurred is likely to be time consuming and irritating
for the manager concerned. The explanation will often be 'chance', which is not, in any case,
particularly helpful. For such variations further investigation is not worthwhile since such
variances are not controllable.

Controllability
This must also influence the decision about whether to investigate. If there is a general
worldwide increase in the price of a raw material there is nothing that can be done internally
to control the effect of this. If a central decision is made to award all employees a 10%
increase in salary, staff costs in division A will increase by this amount and the variance is
not controllable by division A's manager. Uncontrollable variances call for a change in the
plan, not an investigation into the past.

The type of standard being used
The efficiency variance reported in any control period, whether for materials or labour, will
depend on the efficiency level set. If, for example, an ideal standard is used, variances will
always be adverse. A similar problem arises if average price levels are used as standards. If
Page 515

inflation exists, favourable price variances are likely to be reported at the beginning of a
period, to be offset by adverse price variances later in the period.

Variance trend
Although small variations in a single period are unlikely to be significant, small variations
that occur consistently may need more attention. Variance trend is probably more important
that a single set of variances for one accounting period. The trend provides an indication of
whether the variance is fluctuating within acceptable control limits or becoming out of
control.
a) If, say, an efficiency variance is RWF1,000,000 adverse in month 1, the obvious
conclusion is that the process is out of control and that corrective action must be
taken. This may be correct, but what if the same variance is RWF1,000,000 adverse
every month? The trend indicates that the process is not out of control and may be
the standard has been wrongly set.
b) Suppose, though, that the same variance is consistently RWF1,000,000 adverse for
each of the first six months of the year but that production has steadily fallen from
100 units in month 1 to 65 units by month 6. The variance trend in absolute terms is
constant, but relative to the number of units produced, efficiency has got steadily
worse.

Individual variances should therefore not be looked at in isolation; variances should be
scrutinised for a number of successive periods if their full significance is to be appreciated.

Interdependence between variances
Individual variances should not be looked at in isolation. One variance might be inter-related
with another, and much of it might have occurred only because the other variance occurred
too. When two variances are interdependent (interrelated) one will usually be adverse
and the other one favourable.

Page 516

Here are some examples.
Interrelated
variances
Materials price
and usage

Explanation
If cheaper materials are purchased for a job in order to obtain a
favourable price variance, materials wastage might be higher and an
adverse usage variance may occur.
If the cheaper materials are more difficult to handle, there might be
an adverse labour efficiency variance too.
If more expensive materials are purchased, the price variance will
be adverse but the usage variance might be favourable if the
material is easier to use or of a higher quality.

Labour rate and If employees are paid higher rates for experience and skill, using a
highly skilled team might lead to an adverse rate variance and a
efficiency
favourable efficiency variance (experienced staff are less likely to
waste material, for example).
In contrast, a favourable rate variance might indicate a
larger-than-expected proportion of inexperienced workers, which
could result in an adverse labour efficiency variance, and perhaps
poor materials handling and high rates of rejects too (and hence
an adverse materials usage variance).
Selling price and A reduction in the selling price might stimulate bigger sales demand,
so that an adverse selling price variance might be counterbalanced
sales volume
by a favourable sales volume variance.
Similarly, a price rise would give a favourable price variance, but
possibly cause an adverse sales volume variance.
Costs of investigation
The costs of an investigation should be weighed against the benefits of correcting the cause
of a variance.

Exam Focus Point
When asked to provide a commentary on variances you have calculated, make sure that you
interpret your calculations rather than simply detail them.

Page 517

Variance investigation models
The rule-of-thumb and statistical significance variance investigation models and/or
statistical control charts can be used to determine whether a variance should be investigated.

Rule-of-thumb model
This involves deciding a limit and if the size of a variance is within the limit, it should be
considered immaterial. Only if it exceeds the limit is it considered materially significant, and
worthy of investigation.
In practice many managers believe that this approach to deciding which variances to
investigate is perfectly adequate. However, it has a number of drawbacks.
a) Should variances be investigated if they exceed 10% of standard? Or 5%? Or 15%?
b) Should a different fixed percentage be applied to favourable and unfavourable
variances?
c) Suppose that the fixed percentage is, say, 10% and an important category of
expenditure has in the past been very closely controlled so that adverse variances have
never exceeded, say, 2% of standard. Now if adverse variances suddenly shoot up to,
say, 8% or 9% of standard, there might well be serious excess expenditures
incurred that ought to be controlled, but with the fixed percentage limit at 10%, the
variances would not be 'flagged' for investigation.
d) Unimportant categories of low-cost expenditures might be loosely controlled, with
variances commonly exceeding 10% in both a favourable and adverse direction.
These would be regularly – and unnecessarily – flagged for investigation.
e) Where actual expenditures have normal and expected wide fluctuations from period
to period, but the 'standard' is a fixed expenditure amount, variances will be flagged
for investigation unnecessarily often.
f) There is no attempt to consider the costs and potential benefits of investigating
variances (except insofar as the pre-set percentage is of 'material significance').
g) The past history of variances in previous periods is ignored. For example, if the
pre-set percentage limit is set at 10% and an item of expenditure has regularly
exceeded the standard by, say, 6% per month for a number of months in a row, in all
probability there is a situation that ought to warrant control action. Using the pre-set
Page 518

percentage rule, however, the variance would never be flagged for investigation in
spite of the cumulative adverse variances.
Some of the difficulties can be overcome by varying the pre-set percentage from account
to account (for example 5% for direct labour efficiency, 2% for rent and rates, 10% for
salesmen's expenditure, 15% for postage costs, 5% for direct materials price, 3% for direct
materials usage and so on). On the other hand, some difficulties, if they are significant, can
only be overcome with a different cost-variance investigation model.

Statistical significance model
Historical data are used to calculate both a standard as an expected average and the
expected standard deviation around this average when the process is under control. An incontrol process (process being material usage, fixed overhead expenditure and so on) is one
in which any resulting variance is simply due to random fluctuations around the expected
outcome. An out-of-control process, on the other hand, is one in which corrective action
can be taken to remedy any variance.
By assuming that variances that occur are normally distributed around this average, a
variance will be investigated if it is more than a distance from the expected average than
the estimated normal distribution suggests is likely if the process is in control. (Note that
such a variance would be deemed significant.)

The statistical significance rule has two principal advantages over the rule of thumb
approach.
a) Important costs that normally vary by only a small amount from standard will be
signalled for investigation if variances increase significantly.
b) Costs that usually fluctuate by large amounts will not be signalled for investigation
unless variances are extremely large.

The main disadvantage of the statistical significance rule is the problem of assessing
standard deviations in expenditure.

Page 519

Statistical control charts
By marking variances and control limits on a control chart, investigation is signalled not
only when a particular variance exceeds the control limit (since it would be non-random
and worth investigating) but also when the trend of variances shows a progressively
worsening movement in actual results (even though the variance in any single control period
has not yet overstepped the control limit).
The x control chart is based on the principle of the statistical significance model. For each
cost item, a chart is kept of monthly variances and tolerance limits are set at 1, 2 or 3
standard deviations.

In this example, variances do not exceed the tolerance limits in any month, but the chart
shows a worsening of variances over time, and so management might decide that an
investigation is warranted, perhaps when it exceeds an inner warning limit.
Using a cusum chart, the cumulative sum of variances over a long period of time is
plotted. If the variances are not significant, these 'sums' will simply fluctuate in a random
way above and below the average to give a total or cumulative sum of zero. But if significant
variances occur, the cumulative sum will start to develop a positive or negative drift, and
when it exceeds a set tolerance limit, the situation must be investigated.

Page 520

The advantage of the multiple period approach over the single period approach is that trends
are detectable earlier, and control action would be introduced sooner than might have been
the case if only current-period variances were investigated.

Possible control action
Measurement errors and out of date standards, as well as efficient/inefficient operations
and random fluctuations, can cause differences between standard and actual performance.
There are few basic reasons why variances occur and the control action which may be
taken will depend on the reason why the variance occurred.

Measurement errors
In exam questions there is generally no question of the information that you are given being
wrong. In practice on the factory floor, however, it may be extremely difficult to establish
that 1,000 units of product A used 32,000 kg of raw material X. Scales may be misread, the
pilfering or wastage of materials may go unrecorded, items may be wrongly classified (as
material X3, say, when in reality material X8 was used), or employees may make 'cosmetic'
adjustments to their records to make their own performance look better than it really was. An
investigation may show that control action is required to improve the accuracy of the
recording system so that measurement errors do not occur.

Out of date standards
Price standards are likely to become out of date quickly when frequent changes to the costs of
material, power, labour and so on occur, or in periods of high inflation. In such
circumstances an investigation of variances is likely to highlight a general change in
market prices rather than efficiencies or inefficiencies in acquiring resources. Standards may
also be out of date where operations are subject to technological development or if learning
Page 521

curve effects have not been taken into account. Investigation of this type of variance will
provide information about the inaccuracy of the standard and highlight the need to
frequently review and update standards.
Efficient or inefficient operations
Spoilage and better quality material/more highly skilled labour than standard are all likely to
affect the efficiency of operations and hence cause variances. Investigation of variances in
this category should highlight the cause of the inefficiency or efficiency and will lead to
control action to eliminate the inefficiency being repeated or action to compound the
benefits of the efficiency. For example, stricter supervision may be required to reduce
wastage levels and the need for overtime working. The purchasing department could be
encouraged to continue using suppliers of good quality materials.

Random or chance fluctuations
A standard is an average figure and so actual results are likely to deviate unpredictably
within the predictable range. As long as the variance falls within this range, it will be
classified as a random or chance fluctuation and control action will not be necessary.

Page 522

MATERIALS MIX AND YIELD VARIANCES
The materials usage variance can be subdivided into a materials mix variance and a
materials yield variance when more than one material is used in the product.
Manufacturing processes often require that a number of different materials are combined to
make a unit of finished product. When a product requires two or more raw materials in its
make-up, it is often possible to sub-analyse the materials usage variance into a materials
mix and a materials yield variance.
Adding a greater proportion of one material (therefore a smaller proportion of a different
material) might make the materials mix cheaper or more expensive. For example the
standard mix of materials for a product might consist of the following.
RWF
(2/3) 2 kg of material A at RWF1.00 per kg

2.00

(1/3) 1 kg of material B at RWF0.50 per kg

0.50
2.50

It may be possible to change the mix so that one kilogram of material A is used and two
kilograms of material B. The new mix would be cheaper.
RWF
(1/3) 1 kg of material A

1

(2/3) 2 kg of material B

1
2

By changing the proportions in the mix, the efficiency of the combined material usage may
change. In our example, in making the proportions of A and B cheaper, at 1:2, the product
may now require more than three kilograms of input for its manufacture, and the new
materials requirement per unit of product might be 3.6 kilograms.

Page 523

RWF
(1/3) 1.2 kg of material A at RWF1.00 per kg

1.20

(2/3) 2.4 kg of material B at RWF0.50 per kg

1.20
2.40

In establishing a materials usage standard, management may therefore have to balance the
cost of a particular mix of materials with the efficiency of the yield of the mix.
Once the standard has been established it may be possible for management to exercise control
over the materials used in production by calculating and reviewing mix and yield variances.

A mix variance occurs when the materials are not mixed or blended in standard proportions
and it is a measure of whether the actual mix is cheaper or more expensive than the standard
mix.
A yield variance arises because there is a difference between what the input should have
been for the output achieved and the actual input.

Calculating the variances
The mix variance is calculated as the difference between the actual total quantity used in
the standard mix and the actual quantities used in the actual mix, valued at standard
costs.
The yield variance is calculated as the difference between the standard input for what
was actually output, and the actual total quantity input (in the standard mix), valued at
standard costs.

Page 524

When to calculate the mix and yield variance
A mix variance and yield variance are only appropriate in the following situations.
a) Where proportions of materials in a mix are changeable and controllable
b) Where the usage variance of individual materials is of limited value because of
the variability of the mix, and a combined yield variance for all the materials
together is more helpful for control

It would be totally inappropriate to calculate a mix variance where the materials in the 'mix'
are discrete items. A chair, for example, might consist of wood, covering material, stuffing
and glue. These materials are separate components, and it would not be possible to think in
terms of controlling the proportions of each material in the final product. The usage of each
material must be controlled separately.

Example: Materials usage, mix and yield variances
A company manufactures a chemical, Dynamite, using two compounds Flash and Bang. The
standard materials usage and cost of one unit of Dynamite are as follows.
RWF
Flash

5 kg at RWF2 per kg

10

Bang

10 kg at RWF3 per kg

30
40

In a particular period, 80 units of Dynamite were produced from 500 kg of Flash and 730 kg
of Bang.

Required
Calculate the materials usage, mix and yield variances.
Page 525

Solution
a) Usage variance

Std usage for
actual output

Actual
usage

Variance

Standard
cost per
kg
RWF

Variance

kgs

kgs

kgs

RWF

Flash

400

500

100 (A)

2

200 (A)

Bang

800

730

70 (F)

3

210 (F)

1,200

1,230

30 (A)

10 (F)

The total usage variance of RWF10 (F) can be analysed into a mix variance and a yield
variance.

b) Mix variance
To calculate the mix variance, it is first necessary to decide how the total quantity of
materials used (500 kg + 730 kg) should have been divided between Flash and Bang. In
other words, we need to calculate the standard mix of the actual quantity of
materials used.

Page 526

kg
Total quantity used (500 + 730)

1,230
kg

Standard mix of actual use: 1/3 Flash

410

2/3 Bang

820
1,230

The differences between what should have been used in the mix (as calculated above)
and what was actually used is the mix variance (in kg) which should be converted into
money values at standard cost.

Actual
quantity
standard mix

Actual
quantity
actual mix

Standard
Variance
kgs

cost per
kg
RWF

Variance

kgs

kgs

Flash

410

500*

90 (A)

2

180 (A)

Bang

820

730

90 (F)

3

270 (F)

1,230

1,230

–

RWF

90 (F)

* When actual use exceeds standard use the variance is always adverse.
Note that the total mix variance in quantity is zero. This must always be the case
since the expected mix is based on the total quantity actually used and hence the
difference between the total expected and total actual is zero.
The favourable money variance is due to the greater use in the mix of the relatively
cheap material, Flash.

Page 527

c) Yield variance
The yield variance can be calculated in total or for each individual material input.
In total
Each unit of output (Dynamite)
requires

5 kg

of Flash, costing

RWF10

10 kg

of Bang, costing

RWF30

15 kg

RWF40

1,230 kg should have yielded (÷ 15 kg)

82 units of Dynamite

but did yield

80 units of Dynamite

Yield variance in units

2 units (A)

× standard cost per unit of output

× RWF40

Yield variance in RWF

RWF80 (A)

The adverse yield variance is due to the output from the input being less than standard.
For individual materials
This is calculated as the difference between what the usage should have been for the
output actually achieved and the actual usage in the standard mix, converted into money
values at standard cost.
Standard
quantity
standard mix

Actual
quantity
standard mix

Standard
Variance

Variance

kgs

kgs

Flash

400

410

10 (A)

2

20 (A)

Bang

800

820

20 (A)

3

60 (A)

1,200

1,230

30 (A)

Page 528

kgs

cost per
kg
RWF

RWF

80 (A)

Exam Focus Point
With all variance calculations, it is vital that you do not simply learn formulae. You must
have a thorough understanding of what your calculations are showing. This is especially true
of the variances we will look at in this section and in the next chapter.

Example: Losses, mix and yield
C and S Ltd makes product T42 in a continuous process, for which standard and actual
quantities in month 10 were as follows.

Quantity

Standard
Price
per kg

Value

kg

RWF

RWF

Material P

40,000

2.50

Material Q

20,000

4.00

60,000

Quantity

Actual
Price
per kg

Std cost of
actual usage

kg

RWF

RWF

100,000

34,000

2.50

85,000

80,000

22,000

4.00

88,000

180,000

56,000

173,000

Losses occur at an even rate during the processing operation and are expected to be 10% of
materials input. Actual output during the month was 53,000 kgs.

Required
Calculate total usage, mix and yield variances.

Page 529

Solution
Usage variance
Output of 53,000 kgs should have used input of 53,000/90% = 58,889 kgs.
∴Standard input should have been as follows.
Kg
P

2/3 × 58,889

=

39,259

Q

1/3 × 58,889

=

19,630
58,889

P

Q

53,000 kg of T42 should need

39,259 kg

19,630 kg

but did need

34,000 kg

22,000 kg

Usage variance in kg
× standard price per kg
Usage variance in RWF
Total usage variance

5,259 kg (F)
× RWF2.50
RWF13,148 (F)
RWF3,668 (F)

Page 530

2,370 kg (A)
×

RWF4

RwfRWF9,480 (A)

Yield variance
Each kg of T42 requires (1 × 100/90) kg of input costing RWF3.33 (RWF180,000/(60 ,000 ×
90%))
56,000 kg should have yielded (÷ 100/90)

50,400 kg

but did yield

53,000 kg

Yield variance in kgs

2,600 kg (F)

× standard cost per kg of T42

× RWF3.33

Yield variance in RWF

RWF8,667 (F)

Mix variance
Total quantity used

56,000.00 kg

Standard mix for actual use: 2/3 P

37,333.33 kg

1/3 Q

18,666.67 kg
56,000.00 kg

P

Q

Mix should have been

37,333.33 kg

18,666.67 kg

but was

34,000.00 kg

22,000.00 kg

Mix variance in kg

3,333.33 kg (F)

× standard cost per kg

× RWF2.50

× RWF4.00

Mix variance in RWF

RWF8,333.00 (F)

RWF13,333.00 (A)

Page 531

3,333.33 kg (A)

Total mix variance

RWF5,000 (A)

(Note that there is a difference between the sum of the mix and yield variances and the usage
variance due to rounding.)

The issues involved in changing the mix
The materials mix variance indicates the cost of a change in the mix of materials and the
yield variance indicates the productivity of the manufacturing process. A change in the mix
can have wider implications. For example, rising raw material prices may cause pressure to
change the mix of materials. Even if the yield is not affected by the change in the mix, the
quality of the final product may change. This can have an adverse effect on sales if
customers do not accept the change in quality. The production manager’s performance may
be measured by mix and yield variances but these performance measures may fail to
indicate problems with falling quality and the impact on other areas of the business. Quality
targets may also be needed.

Alternative methods of controlling production processes
In a modern manufacturing environment with an emphasis on quality management, using mix
and yield variances for control purposes may not be possible or may be inadequate. Other
control methods could be more useful.
•
•
•
•
•

Rates of wastage
Average cost of input calculations
Percentage of deliveries on time
Customer satisfaction ratings
Yield percentage calculations or output to input conversion rates

We will be considering performance measures in more detail in Study Unit 18.

Page 532

CHAPTER ROUNDUP
•

Care must be taken when interpreting labour variances where the learning curve has
been used in the budget process.

•

Within an ABC system, efficiency variances for longer-term variable overheads
are the difference between the level of activity that should have been needed and the
actual activity level, valued at the standard rate per activity.

•

Materiality, controllability, the type of standard being used, variance trend,
interdependence and costs should be taken into account when deciding on the
significance of a variance.

•

The rule-of-thumb and statistical significance variance investigation models and/or
statistical control charts can be used to determine whether a variance should be
investigated.

•

Measurement errors and out of date standards, as well as efficient/inefficient
operations and random fluctuations, can cause differences between standard and
actual performance.

•

The materials usage variance can be subdivided into a materials mix variance and a
materials yield variance when more than one material is used in the product.

Page 533

BLANK

Page 534

STUDY UNIT 17
Behavioural Aspects Of Standard Costing
Contents

Page

Planning And Operational Variances……………………..………...…….

537

Behavioural Aspects Of Standard Costing ……….……………………..

555

Page 535

EXAM GUIDE
Planning and operational variances are highly examinable and behavioural aspects of
standard costing may form the discussion part of a question.

Page 536

PLANNING AND OPERATIONAL VARIANCES
A planning and operational approach to variance analysis divides the total variance into those
variances which have arisen because of inaccurate planning or faulty standards (planning
variances) and those variances which have been caused by adverse or favourable operational
performance, compared with a standard which has been revised in hindsight (operational
variances).

So far in this text we have been looking at variances which are calculated using what we will
call the conventional approach to variance analysis, whereby an actual cost is compared with
an original standard cost. In this section of the chapter we will be examining planning and
operational variances. They are not really alternatives to the conventional approach, they
merely provide a much more detailed analysis.
Basically, the planning and operational approach attempts to divide a total variance (which
has been calculated conventionally) into a group of variances which have arisen because of
inaccurate planning or faulty standards (planning variances) and a group of variances
which have been caused by adverse or favourable operational performance (operational
variances, surprisingly enough!).
Planning and operational variances may seem confusing if you do not have a really good
grasp of the conventional approach and so, before you go any further, make sure that you
understand everything that we covered so far in this Text. Go back over any areas you are
unsure about.
Only when you are happy that you have mastered the basics should you begin on this
section.

A planning variance (or revision variance) compares an original standard with a revised
standard that should or would have been used if planners had known in advance what was
going to happen.
An operational variance (or operating variance) compares an actual result with the revised
standard.

Planning and operational variances are based on the principle that variances ought to be
reported by taking as the main starting point, not the original standard, but a standard
which can be seen, in hindsight, to be the optimum that should have been achievable.
Page 537

Exponents of this approach argue that the monetary value of variances ought to be a realistic
reflection of what the causes of the variances have cost the organisation. In other words they
should show the cash (and profit) gained or lost as a consequence of operating results being
different to what should have been achieved. Variances can be valued in this way by
comparing actual results with a realistic standard or budget. Such variances are called
operational variances.
Planning variances arise because the original standard and revised more realistic
standards are different and have nothing to do with operational performance. In most cases, it
is unlikely that anything could be done about planning variances: they are not controllable by
operational managers but by senior management.

In other words the cause of a total variance might be one or both of:
•

Adverse or favourable operational performance (operational variance)

•

Inaccurate planning, or faulty standards (planning variance)

Calculating total planning and operational variances
We will begin by looking at how to split a total cost variance into its planning and operational
components.

Example: Total cost planning and operational variances
At the beginning of 20X0, WB set a standard marginal cost for its major product of RWF25
per unit. The standard cost is recalculated once each year. Actual production costs during
August 20X0 were RWF304,000, when 8,000 units were made.
With the benefit of hindsight, the management of WB realises that a more realistic standard
cost for current conditions would be RWF40 per unit. The planned standard cost of RWF25 is
unrealistically low.

Required
Calculate the planning and operational variances.
Page 538

Solution
With the benefit of hindsight, the realistic standard should have been RWF40. The
variance caused by favourable or adverse operating performance should be calculated by
comparing actual results against this realistic standard.
RWF
Revised standard cost of actual production (8,000 × RWF40)

320,000

Actual cost

304,000

Total operational variance

16,000 (F)

The variance is favourable because the actual cost was lower than would have been expected
using the revised basis.
The planning variance reveals the extent to which the original standard was at fault.

Revised standard cost

8,000 units × RWF40 per unit

320,000

Original standard cost

8,000 units ×RWF25 per unit

200,000

Planning variance

120,000 (A)

It is an adverse variance because the original standard was too optimistic, overestimating the
expected profits by understating the standard cost. More simply, it is adverse because the
revised cost is much higher than the original cost.
RWF
Planning variance

120,000 (A)

Operational variance

16,000 (F)

Total

104,000 (A)

Page 539

If traditional variance analysis had been used, the total cost variance would have been the
same, but all the 'blame' would appear to lie on actual results and operating inefficiencies
(rather than some being due to faulty planning).

RWF
Standard cost of 8,000 units (× RWF25)

200,000

Actual cost of 8,000 units

304,000

Total cost variance

104,000 (A)

Example
Suppose a budget is prepared which includes a raw materials cost per unit of product of
RWF2,000 (2 kg of copper at RWF1,000 per kg). Due to a rise in world prices for copper
during the year, the average market price of copper rises to RWF1,500 per kg. During the
year, 1,000 units were produced at a cost of RWF3,250,000 for 2,200 kg of copper.
What are the planning and operational variances?

Solution
Operational variance
RWF ‘000
Actual cost (for 1,000 units)

3,250

Revised standard cost (for 1,000 units) (2,000 kg × Rwf1.50k)

3,000

Total operational variance

250 (A)

Page 540

Planning variance
RWF ‘000
Revised standard cost (1,000 × 2 kg × Rwf1,500)

3,000

Original standard cost (1,000 × 2 kg × Rwf1,000)

2,000

Total planning variance

1,000 (A)

Operational price and usage variances
So far we have only considered planning and operational variances in total, without carrying
out the usual two-way split. In the question above, for instance, we identified a total
operational variance for materials of RWF250,000 without considering whether this
operational variance could be split between a usage variance and a price variance.
This is not a problem so long as you retain your grasp of knowledge you already possess.
You know that a price variance measures the difference between the actual amount of money
paid and the amount of money that should have been paid for that quantity of materials (or
whatever).

Thus, in our example:
RWF ‘000
Actual price of actual materials (2,200 kg)

3,250

Revised standard price of actual materials (RWF1,500 × 2,200 kg)

3,300

Operational price variance

50 (F)

Page 541

The variance is favourable because the materials were purchased more cheaply than would
have been expected.
Similarly, a usage variance measures the difference between the actual physical quantity of
materials used or hours taken and the quantities that should have been used or taken for the
actual volume of production. Those physical differences are then converted into money
values by applying the appropriate standard cost.
In our example we are calculating operational variances, so we are not interested in
planning errors. This means that the appropriate standard cost is the revised standard cost
of RWF1.50k.

Actual quantity should have been
but was
Operational usage variance in kgs
× revised standard cost per kg in RWF ‘000
Operational usage variance in RWF ‘000

2,000 kgs
2,200 kgs
200 kgs (A)
× 1,500
300,000 (A)

The two variances of course reconcile to the total variance as previously calculated.
RWF
Operational price variance

50,000 (F)

Operational usage variance

(300,000) (A)

Total operational variance

250,000 (A)

Operational variances for labour and overheads
Precisely the same argument applies to the calculation of operational variances for labour and
overheads, and the examples already given should be sufficient to enable you to do the next
question.
Page 542

Question
A new product requires three hours of team labour per unit at a standard rate of RWF6,000
per hour. In a particular month the budget is to produce 500 units. Actual results were as
follows.
Team Hours worked

1,700

Production

540 units

Wages cost

RWF10,500,000

Within minutes of production starting it was realised that the job was extremely messy and
the labour force could therefore claim an extra RWF 250 per hour in 'dirty money'.

Required
Calculate planning and operational variances in as much detail as possible.

Answer
Keep calm and calculate the total variance in the normal way to begin with. Then you will
understand what it is that you have to analyse. Next follow through the workings shown
above, substituting the figures in the exercise for those in the example.
Total labour variance
RWF ‘000
540 units should have cost (× 3 hrs × Rwf6,000)
but did cost

9,720
10,500
780 (A)

Page 543

Planning variance
RWF ‘000
Revised standard cost (540 × 3 hrs × RWF6,250)

10,125

Original standard cost (540 × 3 hrs × RWF6,000)

9,720
405 (A)

Operational rate variance
RWF ‘000
Actual cost of actual units

10,500

Revised cost of actual units (1,700 × RWF6,250)

10,625
125 (F)

Operational efficiency variance

540 units should have taken (× 3 hrs)

1,620 hrs

but did take

1,700 hrs

Operational efficiency variance in hours

80 hrs

× revised standard rate per hour RWF ‘000
Operational efficiency variance in RWF ‘000

Page 544

× 6.250
500 (A)

Planning and operational sales variances
Our final calculations in this section deal with planning and operational sales variances.

Example: Planning and operational sales variances
Dimsek budgeted to make and sell 400 units of its product, the role, in the four-week period
no 8, as follows.
RWF
‘000
Budgeted sales (100 units per week)

40,000

Variable costs (400 units × RWF60,000)

24,000

Contribution

16,000

Fixed costs

10,000

Profit

6,000

At the beginning of the second week, production came to a halt because stocks of raw
materials ran out and a new supply was not received until the beginning of week 3. As a
consequence, the company lost one week's production and sales. Actual results in period 8
were as follows.
RWF‘00
0
Sales (320 units)

32,000

Variable costs (320 units × RWF60,000)

19,200

Contribution

12,800

Fixed costs

10,000

Page 545

Actual profit

2,800

In retrospect, it is decided that the optimum budget, given the loss of production facilities in
the third week, would have been to sell only 300 units in the period.

Required
Calculate appropriate planning and operational variances.

Solution
The planning variance compares the revised budget with the original budget.
Revised sales volume, given materials shortage

300 units

Original budgeted sales volume

400 units

Planning variance in units of sales

100 units(A)

× standard contribution RWF ‘000 per unit
Planning variance in RWF ‘000

× 40
4,000 (A)

Arguably, running out of raw materials is an operational error and so the loss of sales
volume and contribution from the materials shortage is an opportunity cost that could have
been avoided with better purchasing arrangements. The operational variances are variances
calculated in the usual way, except that actual results are compared with the revised standard
or budget. There is a sales volume variance which is an operational variance, as follows.

Page 546

Actual sales volume

320 units

Revised sales volume

300 units

Operational sales volume variance in units

20 units (F)

(possibly due to production efficiency or marketing efficiency)
× standard contribution per unit in RWF ‘000

40

RWF ‘000

800 (F)

These variances can be used as control information to reconcile budgeted and actual profit.
RWF ‘000

RWF ‘000

Operating statement, period 8
Budgeted profit

6,000

Planning variance

4,000 (A)

Operational variance – sales volume

800 (F)
3,200 (A)

Actual profit in period 8

2,800

You will have noticed that in this example sales volume variances were valued at
contribution forgone, and there were no fixed cost volume variances. This is because
contribution forgone, in terms of lost revenue or extra expenditure incurred, is the nearest
equivalent to opportunity cost which is readily available to management accountants (who
assume linearity of costs and revenues within a relevant range of activity).

Page 547

Example
KSO budgeted to sell 10,000 units of a new product during 20X0. The budgeted sales price
was RWF10,000 per unit, and the variable cost RWF3,000 per unit.
Although actual sales in 20X0 were 10,000 units and variable costs of sales were
RWF30,000,000, sales revenue was only Rwf5,000 per unit. With the benefit of hindsight, it
is realised that the budgeted sales price of RWF10,000 was hopelessly optimistic, and a price
of RWF4,500 per unit would have been much more realistic.

Required
Calculate planning and operational variances.
Solution
The only variances are selling price variances.

Planning (selling price) variance
Total
RWF ‘000
Revised budget (10,000 × Rwf4,500)

45,000

Original budget (10,000 × Rwf10,000)

100,000

Planning variance

55,000 (A)

The original variance was too optimistic and so the planning variance is an adverse variance.
Operational (selling price) variance
RWF‘000
Actual sales (10,000 × Rwf5,000)

50,000

Revised sales (10,000 × Rwf4,500)

45,000

Page 548

Operational (selling price) variance

5,000 (F)

The total difference between budgeted and actual profit of RWF50,000,000 (A) is therefore
analysed as follows.
RWF ‘000
Operational variance (selling price)

5,000 (F)

Planning variance

55,000 (A)
50,000 (A)

Question
PG budgeted sales for 20X8 were 5,000 units. The standard contribution is RWF9,600 per
unit. A recession in 20X8 meant that the market for PG's products declined by 5%. PG's
market share also fell by 3%. Actual sales were 4,500 units.

Required
Calculate planning and operational variances for sales volume.

Answer
Planning variance
Units
Original budgeted sales

5,000

Revised budget sales (–5%)

4,750
250 A

@ Contribution per unit of RWF9,600

RWF2,400,000

Page 549

Operational variance
Units
Revised budget sales

4,750

Actual sales

4,500
250 A

@ Contribution per unit of RWF9,.600

RWF2,400,000

The fall in market size is uncontrollable by the management of PG and therefore results in a
planning variance. The fall in market share is controllable and forms part of the
operational variance.

Exam Focus Point
The examiner is very keen that candidates should understand the practical problems involved
with these variances and do not just concentrate on the mechanics of the calculations.

Calculating a revised budget
The syllabus requires you to be able to calculate a revised budget, which could involve
revising standards for sales, materials and/or labour so that only operational variances are
highlighted when actual results are compared to the revised budget.

Page 550

Example: revised budget
A company produces Widgets and Splodgets which are fairly standardised products. The
following information relates to period 1.
The standard selling price of Widgets is RWF50,000 each and Splodgets RWF100,000 each.
In period 1, there was a special promotion on Splodgets with a 5% discount being offered. All
units produced are sold and no inventory is held.
To produce a Widget they use 5 kg of X and in period 1, their plans were based on a cost of
X of RWF3,000 per kg. Due to market movements the actual price changed and if they had
purchased efficiently the cost would have been RWF4,500 per kg. Production of Widgets was
2,000 units.
A Splodget uses raw material Z but again the price of this can change rapidly. It was thought
that Z would cost RWF30,000 per tonne but in fact they only paid RWF25,000 per tonne and
if they had purchased correctly the cost would have been less as it was freely available at
only RWF23,000 per tonne. It usually takes 1.5 tonnes of Z to produce 1 Splodget and 500
Splodgets are usually produced.
Each Widget takes 3 hours to produce and each Splodget 2 hours. A labour team is paid
Rwf5,000 per hour. At the start of period 1, management negotiated a job security package
with the workforce in exchange for a promised 5% increase in efficiency – that is, that the
workers would increase output per hour by 5%.
Fixed overheads are usually RWF12,000,000 every period and variable overheads are
RWF3,000 per labour hour.

Required
Produce the original budget and a revised budget allowing for controllable factors in a
suitable format.

Page 551

Solution
Original budget for Period 1
RWF
‘000
Sales revenue ((2,000 × RWF50,000) + (500 ×RWF100,000))

150,000

Material costs X (2,000 × 5kg × RWF3,000)

30,000

Material costs Z (500 × RWF30,000 × 1.5)

22,500

Labour costs ((2,000 × 3 × RWF5,000) + ( 500 × 2 × RWF5,000))

35,000

Variable overheads ((2,000 × 3 × RWF3,000) + ( 500 × 2 × RWF3,000))

21,000

Fixed overheads

12,000

Profit

29,500

Revised budget for Period 1
RWF
’000
Sales revenue ((2,000 × RWF50,000) + (500 × RWF95,000))

147,500

Material costs X (2,000 × 5kg × RWF4,500)

45,000

Material costs Z (500 × RWF23,000 × 1.5)

17,250

Labour costs ((2,000 × 3 × RWF5,000 ) + ( 500 × 2 × RWF5,000)) × 0.95

33,250

Variable overheads ((2,000 × 3 × RWF3,000) + ( 500 × 2 × RWF3,000)) × 0.95

19,950

Fixed overheads

12,000

Profit

20,050
Page 552

When should budget revisions be allowed?
A budget revision should be allowed if something has happened which is beyond the control
of the organisation or individual manager and which makes the original budget unsuitable for
use in performance management.
Any adjustment should be approved by senior management who should look at the issues
involved objectively and independently. Operational issues are the issues that a budget is
attempting to control so they should not be subject to revision. However, it can be very
difficult to establish what is due to operational problems (controllable) and what is due to
planning (uncontrollable).

The value of planning and operational variances
Advantages of a system of planning and operational variances
•

The analysis highlights those variances which are controllable and those which are
non-controllable.

•

Managers' acceptance of the use of variances for performance measurement, and
their motivation, is likely to increase if they know they will not be held responsible
for poor planning and faulty standard setting.

•

The planning and standard-setting processes should improve; standards should be
more accurate, relevant and appropriate.

•

Operational variances will provide a 'fairer' reflection of actual performance.

The limitations of planning and operational variances, which must be overcome if they
are to be applied in practice.
•

It is difficult to decide in hindsight what the realistic standard should have been.

•

It may become too easy to justify all the variances as being due to bad planning,
so no operational variances will be highlighted.

•

Establishing realistic revised standards and analysing the total variance into planning
and operational variances can be a time consuming task, even if a spreadsheet
package is devised.
Page 553

•

Even though the intention is to provide more meaningful information, managers may
be resistant to the very idea of variances and refuse to see the virtues of the approach.
Careful presentation and explanation will be required until managers are used to the
concepts.

Page 554

BEHAVIOURAL ASPECTS OF STANDARD
COSTING
The role of standards and variances in the modern business environment is open to
question.

Standard costing and new technology
Standard costing has traditionally been associated with labour-intensive operations, but it can
be applied to capital-intensive production too.
It is quite possible that with advanced manufacturing technology variable overheads are
incurred in relation to machine time rather than labour time, and standard costs should reflect
this where appropriate.
With computer aided design/computer aided manufacture (CADCAM) systems, the
planning of manufacturing requirements can be computerised, so that standard costs can be
constructed by computer, saving administrative time and expense while providing far more
accurate standards.

Total quality management (TQM)
In the context of TQM, quality means getting it right first time and keeping ahead of the
game.
Total quality management (TQM) is the process of applying a zero defects philosophy to
the management of all resources and relationships within an organisation as a means of
developing and sustaining a culture of continuous improvement which focuses on meeting
customers' expectations.

Page 555

Mark Lee Inman listed 'eight requirements of quality' in an ACCA Students' Newsletter
article, which could be seen as the characteristics of total quality management
programmes.
a) Organisation wide there must be acceptance that the only thing that matters is the
customer.
b) There should be recognition of the all-pervasive nature of the customer-supplier
relationship, including internal customers; passing sub-standard material to another
division is not satisfactory
c) Instead of relying on inspection to a predefined level of quality, the cause of the
defect in the first place should be prevented.
d) Each employee or group of employees must be personally responsible for defect-free
production or service in their domain.
e) There should be a move away from 'acceptable' quality levels. Any level of defects
must be unacceptable.
f) All departments should try obsessively to get things right first time; this applies to
misdirected phone calls and typing errors as much as to production.
g) Quality certification programmes should be introduced.
h) The cost of poor quality, i.e. correcting mistakes as well as losing customers, should
be emphasised; good quality generates savings.

Standard costing and TQM
Standard costing concentrates on quantity and ignores other factors contributing to
effectiveness. In a total quality environment, however, quantity is not an issue; quality is.
Effectiveness in such an environment therefore centres on high quality output (produced as a
result of high quality input and the elimination of non-value adding activities) and the cost of
failing to achieve the required level of effectiveness is measured not in variances, but in terms
of internal and external failure costs, neither of which would be identified by a traditional
standard costing analysis.
Standard costing systems might measure, say, labour efficiency in terms of individual tasks
and level of output. In a total quality environment, labour is more likely to be viewed as a
number of multi-task teams who are responsible for the completion of a part of the
Page 556

production process. The effectiveness of such a team is more appropriately measured in terms
of re-working required, returns from customers, defects identified in subsequent stages of
production and so on.
Traditional feedback control would seek to eliminate an adverse material price variance by
requiring managers to source cheaper, possibly lower quality supplies. This may run counter
to the aim of maximising quality of output.

Can standard costing and TQM co-exist?
Arguably, there is little point in running both a total quality management programme and a
standard costing system simultaneously.
a) Predetermined standards are at odds with the philosophy of continual improvement
inherent in a total quality management programme.
b) Continual improvements are likely to alter methods of working, prices, quantities of
inputs and so on, whereas standard costing is most appropriate in a stable,
standardised and repetitive environment.
c) Material standard costs often incorporate a planned level of scrap. This is at odds with
the TQM aim of zero defects and there is no motivation to 'get it right first time'.
d) Attainable standards, which make some allowance for wastage and inefficiencies are
commonly set. The use of such standards conflicts with the elimination of waste
which is such a vital ingredient of a TQM programme.
e) Standard costing control systems make individual managers responsible for the
variances relating to their part of the organisation's activities. A TQM programme, on
the other hand, aims to make all personnel aware of, and responsible for, the
importance of supplying the customer with a quality product.

Page 557

Standard costing and new philosophy
It has been argued that traditional variance analysis is unhelpful and potentially misleading
in the modern organisation, and can make managers focus their attention on the wrong issues,
for example - over-producing and stockpiling finished goods, because higher production
volumes mean that overheads are spread over more units. Here are two examples.
a) Efficiency variance. Adverse efficiency variances should be avoided, which means
that managers should try to prevent idle time and to keep up production. In a TQM
environment using just-in-time manufacturing, action to eliminate idle time could
result in the manufacture of unwanted products that must be held in store and might
eventually be scrapped. Efficiency variances could focus management attention on the
wrong problems.
b) Materials price variance. In a JIT environment, the key issues with materials
purchasing are supplier reliability, materials quality and delivery in small order
quantities. Purchasing managers shouldn't be shopping around every month looking
for the cheapest price. Many JIT systems depend on long-term contractual links with
suppliers, which means that material price variances are not relevant for control
purposes.

The role of standards and variances in the modern business environment is viewed as
follows by George Brown (a previous ACCA examiner).
'The rate of change in product type and design due to technological improvement,
customer requirements and increased competition has led to rapid change in how
businesses operate. The need to respond to customer demands for speedy availability of
products, shortening product life cycles and higher quality standards has contributed to
a number of changes in the way businesses operate…just-in-time systems…total
quality programmes…..greater emphasis on the value chain…..accurate product costing
and pricing information……improved speed and flexibility of information
availability…' ('Standard costing – a status check')
Standard costing, on the other hand, is most appropriate in a stable, standardised and
repetitive environment and one of the main objectives of standard costing is to ensure that
processes conform to standards, that they do not vary and that variances are eliminated. This
may seem restrictive and inhibiting in the business environment of the twenty first
century. (In fact, in the article referred to above, George Brown attempts to show that
concerns about the restrictive and inhibiting nature of standard costing have been raised since
it was first used and that efforts have continuously been made (such as planning and
Page 558

operating variances) to redesign standards and variances to maintain their relevance in an
environment of change.)

Other problems with using standard costing in today's environment
a) Variance analysis concentrates on only a narrow range of costs, and does not give
sufficient attention to issues such as quality and customer satisfaction.
b) Standard costing places too much emphasis on direct labour costs. Direct labour is
only a small proportion of costs in the modern manufacturing environment and so this
emphasis is not appropriate.
c) Many of the variances in a standard costing system focus on the control of short-term
variable costs. In most modern manufacturing environments, the majority of costs,
including direct labour costs, tend to be fixed in the short run.
d) The use of standard costing relies on the existence of repetitive operations and
relatively homogeneous output. Nowadays many organisations are forced continually
to respond to customers’ changing requirements, with the result that output and
operations are not so repetitive.
e) Standard costing systems were developed when the business environment was more
stable and less prone to change. The current business environment is more dynamic
and it is not possible to assume stable conditions.
f) Standard costing systems assume that performance to standard is acceptable.
Today’s business environment is more focused on continuous improvement.
g) Most standard costing systems produce control statements weekly or monthly. The
modern manager needs much more prompt control information in order to function
efficiently in a dynamic business environment.

Page 559

The role in modern business of standards and variances
Two surveys ((Puxty and Lyall (1989) and Drury et al (1993)) have confirmed the continued
wide use of standard costing systems. Drury et al, for instance, showed that 76% of the
responding organisations operated a standard costing system.
•

Planning. Even in a TQM environment, budgets will still need to be quantified. For
example, the planned level of prevention and appraisal costs needs to be determined.
Standards, such as returns of a particular product should not exceed 1% of deliveries
during a budget period, can be set.

•

Control. Cost and mix changes from plan will still be relevant in many processing
situations.

•

Decision making. Existing standards can be used as the starting point in the
construction of a cost for a new product.

•

Performance measurement. If the product mix is relatively stable, performance
measurement may be enhanced by the use of a system of planning and operational
variances.

•

Product pricing. Target costs may be compared with current standards, and the
resulting 'cost gap' investigated with a view to reducing it or eliminating it using
techniques such as value engineering.

•

Improvement and change. Variance trends can be monitored over time.

•

Accounting valuations. Although the operation of a JIT system in conjunction with
backflush accounting will reduce the need for standard costs and variance analysis,
standards may be used to value residual inventory and the transfers to cost of sales
account.

Page 560

CHAPTER ROUNDUP
•

A planning and operational approach to variance analysis divides the total variance
into those variances which have arisen because of inaccurate planning or faulty
standards (planning variances) and those variances which have been caused by
adverse or favourable operational performance, compared with a standard which has
been revised in hindsight (operational variances).

•

The role of standards and variances in the modern business environment is open to
question.

•

In the context of TQM, quality means getting it right first time and improving
continuously.

Page 561

BLANK

Page 562

STUDY UNIT 18
Divisional Performance And Transfer Pricing Issues
Contents

Page

Divisional Structure And Performance Measures ……………………….

567

Measuring Performance ……….………………………..….……….……..

577

When Transfer Pricing Is Required ……………………………………..

579

The ‘General Rule’ ………………………………………………..……….

583

The Use Of Market Price ………………………………………….........…

585

Cost-Based Approaches To Transfer Pricing ……………………..……..

589

Fixed Costs And Transfer Pricing ………………….…………….....……

595

Standard Cost Versus Actual Cost ………………….……………..….…

597

Cost-Based Approaches With No External Market ……………….……

599

Opportunity Costs And Transfer Prices …………………………….……

603

Page 563

Transfer Pricing When Intermediate Products Are In Short Supply …

605

Transfer Pricing And A Range Of Limiting Factors ……………….……

617

Shadow Price And Transfer Prices ………………………………….……

621

Negotiated Transfer Prices ………………………………………….……

623

Multinational Transfer Pricing ……………………………….…….……

625

Page 564

EXAM GUIDE
The topics in this chapter provide plenty of material for an exam question. Indeed, you may
find a full question testing your knowledge of transfer pricing. Expect written or calculation
questions.

Page 565

BLANK

Page 566

DIVISIONAL STRUCTURE AND PERFORMANCE
MEASURES
In this section we look at three performance measures relevant in a divisionalised structure.
These are Return on Investment, Residual Income and Economic Value Added .
Return on Investment (ROI) and Residual Income (RI) were discussed previously when
we considered the scope of strategic performance measures in the private sector. In this
chapter we will just pick out the salient features that apply to their use in divisionalised
structures.

Divisional performance: return on investment (ROI)
Return on investment (ROI) is a form of ROCE and is calculated as:
Profit before Interest and Tax x 100
Operations Management Capital Employed

ROI is normally applied to investment centres or profit centres. These normally reflect the
existing organisation structure of the business.

Evaluation of ROI
You may like to consider the following factors when evaluating the use of ROI as a divisional
performance measure.
a) Comparisons. It permits comparisons to be drawn between investment centres that
differ in their absolute size.
b) Aggregation. ROI is a very convenient method of measuring the performance for a
division or company as an entire unit.
c) Using an identical target return. This may not be suitable for many divisions or
investment centres as it makes no allowance for the different risk of each investment
centre.
Page 567

d) Misleading impression of improved performance. If an investment centre
maintains the same annual profit, and keeps the same assets without a policy of
regular non-current asset replacement, its ROI will increase year by year as the
assets get older. This can give a false impression of improving 'real' performance
over time.
e) Valuation and classification of assets. Many of the criticisms of ROI arise from the
valuation of assets used in the denominator. Refer back to Chapter 12 for a full
explanation of the problems in measuring asset values. Chapter 12 also refers to the
tricky decision of when to classify expenditure as assets.
f) Short-term perspective. Since managers will be judged on the basis of the ROI that
their centre earns each year, they are likely to be motivated into taking those
decisions, which increase their centre's short-term ROI. So, in the short term, a
desire to increase ROI might lead to projects being taken on without due regard to
their risk
g) Lack of goal congruence. An investment might be desirable from the group's point
of view, but would not be in the individual investment centre's 'best interest' to
undertake. Furthermore, any decisions which benefit the company in the long term
but which reduce the ROI in the immediate short term would reflect badly on the
manager's reported performance.

Divisional performance: residual income (RI)
Residual income is a measure of the centre's profits after deducting a notional or imputed
interest cost.
Its use highlights the finance charge associated with funding.

Page 568

Evaluation of RI
You may like to consider the following factors when evaluating the use of RI. Think about
how it compares to ROI as a possible divisional performance measure.
a) Usefulness in decision-making. Residual income increases in the following
circumstances.
(i)

Investments earning above the cost of capital are undertaken

(ii)

Investments earning below the cost of capital are eliminated

Thus it leads managers to make the correct investment decision to benefit the company
as a whole.
b) Flexibility compared to ROI since a different cost of capital can be applied to
investments with different risk characteristics.
c) Does not allow comparisons between investment centres. RI cannot be used to
make comparisons between investment centres as it is an absolute measure of
performance.
d) Difficulty in deciding on an appropriate and accurate measure of the capital
employed. As we discussed above, there can be some difficulty in knowing what
values to place on assets.
e) Does not relate the size of a centre's income to the size of the investment, other
than indirectly through the interest charge.

Divisional performance: Economic Value Added (EVA®)
EVA ® is an alternative absolute performance measure. It is similar to RI and is calculated as
follows.
EVA = net operating profit after tax (NOPAT) less capital charge
where the capital charge = weighted average cost of capital × net assets
® Economic value added (EVA) is a registered trade mark owned by Stern Stewart & Co. It
is a specific type of residual income (RI) calculated as follows.
EVA = net operating profit after tax (NOPAT) less capital charge
Page 569

where the capital charge = weighted average cost of capital x net assets
You can see from the formula that the calculation of EVA is very similar to the calculation of
RI.
EVA and RI are similar because both result in an absolute figure, which is calculated by
subtracting an imputed interest charge from the profit earned by the investment centre.
However there are differences as follows.
a) The profit figures are calculated differently. EVA is based on an 'economic profit'
which is derived by making a series of adjustments to the accounting profit.
b) The notional capital charges use different bases for net assets. The replacement cost
of net assets is usually used in the calculation of EVA.

The calculation of EVA is different from RI because the net assets used as the basis of the
imputed interest charge are usually valued at their replacement cost and are increased by
any costs that have been capitalised (see below).
There are also differences in the way that NOPAT is calculated compared with the profit
figure that is used for RI, as follows.

a) Costs which would normally be treated as expenses, but which are considered within
an EVA calculation as investments building for the future, are added back to
NOPAT to derive a figure for 'economic profit'. These costs are included instead as
assets in the figure for net assets employed, i.e. as investments for the future. Costs
treated in this way include items such as goodwill, research and development
expenditure and advertising costs.
b) Adjustments are sometimes made to the depreciation charge, whereby accounting
depreciation is added back to the profit figures, and economic depreciation is
subtracted instead to arrive at NOPAT. Economic depreciation is a charge for the fall
in asset value due to wear and tear or obsolescence.
c) Any lease charges are excluded from NOPAT and added in as a part of capital
employed.

Page 570

Another point to note about the calculation of NOPAT, which is the same as the calculation
of the profit figure for RI, is that interest is excluded from NOPAT because interest costs are
taken into account in the capital charge.
Example: calculating EVA
An investment centre has reported operating profits of RWF21 million. This was after
charging RWF4 million for the development and launch costs of a new product that is
expected to generate profits for four years. Taxation is paid at the rate of 25% of the
operating profit.
The company has a risk adjusted weighted average cost of capital of 12% per annum and is
paying interest at 9% per annum on a substantial long term loan.
The investment centre’s non-current asset value is RWF50 million and the net current assets
have a value of RWF22 million. The replacement cost of the non-current assets is estimated
to be RWF64 million.

Required
Calculate the investment centre’s EVA for the period.

Solution
Calculation of NOPAT
Operating profit
Add back development costs
Less one year’s amortisation of development costs (RWF4 million/4)
Taxation at 30%
NOPAT

Page 571

RWF m
21
4
(1)
24
(8)
16

Calculation of economic value of net assets

Replacement cost of net assets (RWF22 million + RWF64 million)
Add back investment in new product to benefit future

RWF
million
86
3
89

Economic value of net assets

Calculation of EVA
The capital charge is based on the weighted average cost of capital, which takes account of
the cost of share capital as well as the cost of loan capital. Therefore the correct interest rate
is 12%.
RWF
million
NOPAT

16.00

Capital charge (12% × RWF89 million)

(10.68)

EVA

5.32

Example: calculating EVA
Read the article referred to on the previous page and then try the question here which uses
that method. This is how EVA was tested in the December 2007 exam.

Page 572

Question
B division of Z Ltd has operating profits and assets as below:
RWF m
156

Gross profit

8
5
15
128
38

Less: Non-cash expenses
Amortisation of goodwill
Interest @ 10%
Profit before tax
Tax @ 30%

90

Net profit

350
150
500

Total equity
Long-term debt

Z Ltd has a target capital structure of 25% debt/75% equity. The cost of equity is estimated at
15%. The capital employed at the start of the period amounted to RWF450,000,000. The
division had non-capitalised leases of RWF20 million throughout the period. Goodwill
previously written off against reserves in acquisitions in previous years amounted to RWF40
million.

Required
Calculate EVA® for B division and comment on your results.

Answer
EVA
RWF m

NOPAT
Net profit
Add back:
Non-cash expenses
Amortisation of goodwill
Interest (net of 30% tax) 15 × 0.7

8
5
10.5
Page 573

RWF m
90

23.5
113.5

Assets
At start of period
Non-capitalised leases
Amortised goodwill

450
20
40
510

WACC
Equity 15% × 75%
Debt (10% × 0.7) × 25%
WACC

0.1125
0.0175
0.13
113.5

EVA NOPAT
Capital charge
13% × RWF510 m

66.3
47.2

RI
Net profit
Capital charge
13% × RWF 500m

90
65
25

The EVA for B division is RWF47.2m, higher than its RI. This is despite the higher net asset
value and is caused by treating expenses, such as amortisation, in line with economic, not
accountancy, principles. The business is creating value as its return (however calculated) is
greater than the group's WACC. The division's ROI is 18% vs WACC of 13% (based on
target not actual capital structure).
Its "economic" ROI is 22.3%.

Page 574

Evaluation of EVA
The advantages of EVA include the following.
a) Real wealth for shareholders. Maximisation of EVA will create real wealth for the
shareholders.
b) Less distortion by accounting policies. The adjustments within the calculation of
EVA mean that the measure is based on figures that are closer to cash flows than
accounting profits.
c) An absolute value. The EVA measure is an absolute value, which is easily
understood by non-financial managers.
d) Treatment of certain costs as investments thereby encouraging expenditure. If
management are assessed using performance measures based on traditional
accounting policies they may be unwilling to invest in areas such as advertising and
development for the future because such costs will immediately reduce the current
year’s accounting profit. EVA recognises such costs as investments for the future
and thus they do not immediately reduce the EVA in the year of expenditure.

EVA does have some drawbacks.

a) Focus on short-term performance. It is still a relatively short-term measure,
which can encourage managers to focus on short term performance.
b) Dependency on historical data. EVA is based on historical accounts, which may be
of limited use as a guide to the future. In practice, the influences of accounting
policies on the starting profit figure may not be completely negated by the
adjustments made to it in the EVA model.
c) Number of adjustments needed to measure EVA. Making the necessary
adjustments can be problematic as sometimes a large number of adjustments are
required.
d) Comparison of like with like. Investment centres, which are larger in size, may have
larger EVA figures for this reason. Allowance for relative size must be made when
comparing the relative performance of investment centres.

Page 575

BLANK

Page 576

MEASURING PERFORMANCE
One of the problems of measuring managerial performance is segregating managerial
performance from the economic performance of their department or division.

Managerial performance
The distinction between the manager’s performance and that of the division is very important.
C. T. Horngren provides a good illustration .
'The most skilful divisional manager is often put in charge of the sickest division in an
attempt to change its fortunes. Such an attempt may take years, not months.
Furthermore the manager's efforts may merely result in bringing the division up to a
minimum acceptable ROI. The division may continue to be a poor profit performer in
comparison with other divisions. If top management relied solely on the absolute ROI
to judge management, the skilful manager would be foolish to accept such a troubleshooting assignment.'
It is difficult to devise performance measures that relate specifically to a manager to judge his
or her performance as a manager. It is possible to calculate statistics to assess the manager as
an employee like any other employee (days absent, professional qualifications obtained,
personability and so on), but this is not the point. As soon as the issue of ability as a
manager arises it is necessary to consider him or her in relation to his/her area of
responsibility. If we want to know how good a manager is at marketing the only information
there is to go on is the marketing performance of their division (which may or may not be
traceable to his/her own efforts).
In summary then, managers should only be assessed on results within their control.
Divisional performance should be based on total economic performance to provide an
assessment of the measure of the worth of the division to the organisation.

Page 577

Profit statement
A possible profit statement for a division might look as follows:
RWF
Sales revenue

X

Variable costs

(X)

Contribution

X

Controllable fixed costs

(X)

Controllable profit

X

Non-controllable fixed costs

(X)

Divisional profit

X

Contribution should be an acceptable measure of managerial performance unless it contains
imposed transfers and transfer prices.
Controllable profit may be a more appropriate measure of managerial performance where
managers can make decisions about equipment rental or labour costs. It is more acceptable
when managers are free to secure services either in house or from third parties. Deprecation is
likely to be included and this will only be controllable to the extent that managers control
investment decisions.
Divisional profit is unlikely to be an acceptable managerial measure. It is suitable for
assessing the economic performance of the divisions provided the allocation of fixed costs is
reasonable.

Page 578

WHEN TRANSFER PRICING IS REQUIRED
It is necessary for control purposes that some record of the market in inter-divisional goods
or services should be kept. One way of doing this is through the accounting system. Interdivisional work can be given a cost or a charge: a transfer price.

Introduction to transfer pricing
Where there are transfers of goods or services between divisions of an organisation, the
transfers could be made 'free' or 'as a favour' to the division receiving the benefit. For
example, if a garage and car showroom has two divisions, one for car repairs and servicing
and the other for car sales, the servicing division will be required to service cars before they
are sold and delivered to customers. There is no requirement for this service work to be
charged for: the servicing division could do its work for the car sales division without making
any record of the work done.
Unless the cost or value of such work is recorded, however, management cannot keep a
proper check on the amount of resources (such as labour time) being used up on new car
servicing. It is necessary for control purposes that some record of the inter-divisional
services should be kept, and one way of doing this is through the accounting system. Interdivisional work can be given a cost or charge: a transfer price.

A transfer price is the price at which goods or services are transferred from one department
to another, or from one member of a group to another.

Criteria for design of a transfer pricing policy
Transfer prices are a way of promoting divisional autonomy, ideally without prejudicing
divisional performance measurement or discouraging overall corporate profit
maximisation.

Page 579

Divisional autonomy
Transfer prices are particularly appropriate for profit centres because if one profit centre
does work for another the size of the transfer price will affect the costs of one profit centre
and the revenues of another.
However, a danger with profit centre accounting is that the business organisation will divide
into a number of self-interested segments, each acting at times against the wishes and
interests of other segments. A profit centre manager might take decisions in the best interests
of his own part of the business, but against the best interests of other profit centres and
possibly the organisation as a whole.
A task of head office is therefore to try to prevent dysfunctional decision making by
individual profit centres. To do this, it must reserve some power and authority for itself and
so profit centres cannot be allowed to make entirely autonomous decisions.
Just how much authority head office decides to keep for itself will vary according to
individual circumstances. A balance ought to be kept between divisional autonomy to
provide incentives and motivation, and retaining centralised authority to ensure that the
organisation's profit centres are all working towards the same target, the benefit of the
organisation as a whole (in other words, retaining goal congruence among the organisation's
separate divisions).

Divisional profit maximisation
Profit centre managers tend to put their own profit performance above everything else. Since
profit centre performance is measured according to the profit they earn, no profit centre will
want to do work for another and incur costs without being paid for it. Consequently, profit
centre managers are likely to dispute the size of transfer prices with each other, or disagree
about whether one profit centre should do work for another or not. Transfer prices affect
behaviour and decisions by profit centre managers.

Corporate profit maximisation
When there are disagreements about how much work should be transferred between
divisions, and how many sales the division should make to the external market, there is
presumably a profit-maximising level of output and sales for the organisation as a whole.
However, unless each profit centre also maximises its own profit at this same level of output,
there will be inter-divisional disagreements about output levels and the profit-maximising
output will not be achieved.
Page 580

The ideal solution
Ideally a transfer price should be set at a level that overcomes these problems.
a) The transfer price should provide an 'artificial' selling price that enables the
transferring division to earn a return for its efforts, and the receiving division to
incur a cost for benefits received.
b) The transfer price should be set at a level that enables profit centre performance to be
measured 'commercially' (that is, it should be a fair commercial price).
c) The transfer price, if possible, should encourage profit centre managers to agree on
the amount of goods and services to be transferred, which will also be at a level that is
consistent with the organisation's aims as a whole such as maximising company
profits.

In practice it is very difficult to achieve all three aims.

Page 581

BLANK

Page 582

THE ‘GENERAL RULE’
We shall see eventually that the ideal transfer price should reflect the opportunity cost of
sale to the supplying division and the opportunity cost to the buying division. However,
this 'general rule' requires extensive qualification, and you will need to work through the rest
of this chapter before we return to it and you fully appreciate what it means. In the meantime,
be content with Horngren's formulation of the problem:
'Is there an all-pervasive rule for transfer pricing that leads toward optimal decisions for the
organisation as a whole? No. Why? Because the three criteria of goal congruence, managerial
effort, and sub-unit autonomy must all be considered simultaneously.'

Page 583

BLANK

Page 584

THE USE OF MARKET PRICE
Transfer prices may be based on market price (or an adjusted market price) where there is
an external market for the item being transferred.

Market price as the transfer price
If an external market price exists for transferred goods, profit centre managers will be aware
of the price they could charge or the price they would have to pay for their goods on the
external market, and so will compare this price with the internal transfer price.

Example: transferring goods at market value
A company has two profit centres, A and B. Centre A sells half of its output on the open
market and transfers the other half to B. Costs and external revenues in an accounting period
are as follows.

External sales
Costs of production

A

B

RWF

RWF

Total
RWF

8,000

24,000

32,000

12,000

10,000

22,000
10,000

Company profit

Required
What are the consequences of setting a transfer price at market value?

Page 585

Solution
If the transfer price is at market price, A would be happy to sell the output to B for
RWF8,000, which is what A would get by selling it externally instead of transferring it.

A

B

RWF

RWF

Total

RWF

RWF

RWF
32,000

Market sales

8,000

24,000

Transfer sales

8,000

–

16,000
Transfer costs
Own costs

24,000

–

8,000

12,000

10,000

Profit

22,000

12,000

18,000

4,000

6,000

10,000

The consequences, therefore, are as follows.
a) A earns the same profit on transfers as on external sales. B must pay a commercial
price for transferred goods, and both divisions will have their profit measured fairly.
b) A will be indifferent about selling externally or transferring goods to B because the
profit is the same on both types of transaction. B can therefore ask for and obtain as
many units as it wants from A.

A market-based transfer price therefore seems to be the ideal transfer price.

Page 586

Adjusted market price
However, internal transfers are often cheaper than external sales, with savings in selling and
administration costs, bad debt risks and possibly transport/delivery costs. It would therefore
seem reasonable for the buying division to expect a discount on the external market price.
The transfer price might be slightly less than market price, so that A and B could share the
cost savings from internal transfers compared with external sales. It should be possible to
reach agreement on this price and on output levels with a minimum of intervention from head
office.

The merits of market value transfer prices
Divisional autonomy
In a decentralised company, divisional managers should have the autonomy to make output,
selling and buying decisions, which appear to be in the best interests of the division's
performance. (If every division optimises its performance, the company as a whole must
inevitably achieve optimal results.) Thus a transferor division should be given the freedom
to sell output on the open market, rather than to transfer it within the company.
'Arm's length' transfer prices, which give profit centre managers the freedom to negotiate
prices with other profit centres as though they were independent companies, will tend to
result in a market-based transfer price.

Corporate profit maximisation
In most cases where the transfer price is at market price, internal transfers should be
expected, because the buying division is likely to benefit from a better quality of service,
greater flexibility, and dependability of supply. Both divisions may benefit from cheaper
costs of administration, selling and transport. A market price as the transfer price would
therefore result in decisions, which would be in the best interests of the company or
group as a whole.

Page 587

Divisional performance measurement
Where a market price exists, but the transfer price is a different amount (say, at standard
cost plus), divisional managers will argue about the volume of internal transfers.
For example, if division X is expected to sell output to division Y at a transfer price of Rwf8
per unit when the open market price is Rwf10, its manager will decide to sell all output on the
open market. The manager of division Y would resent the loss of his cheap supply from X,
and would be reluctant to buy on the open market. A wasteful situation would arise where X
sells on the open market at Rwf10, where Y buys at Rwf10, so that administration, selling
and distribution costs would have been saved if X had sold directly to Y at Rwf10, the market
price.

The disadvantages of market value transfer prices
Market value as a transfer price does have certain disadvantages.
a) The market price may be a temporary one, induced by adverse economic conditions,
or dumping, or the market price might depend on the volume of output supplied to the
external market by the profit centre.
b) A transfer price at market value might, under some circumstances, act as a
disincentive to use up any spare capacity in the divisions. A price based on
incremental cost, in contrast, might provide an incentive to use up the spare resources
in order to provide a marginal contribution to profit.
c) Many products do not have an equivalent market price so that the price of a similar,
but not identical, product might have to be chosen. In such circumstances, the option
to sell or buy on the open market does not really exist.
d) The external market for the transferred item might be imperfect, so that if the
transferring division wanted to sell more externally, it would have to reduce its price.

Page 588

COST-BASED APPROACHES TO TRANSFER
PRICING
Problems arise with the use of cost-based transfer prices because one party or the other is
liable to perceive them as unfair.

Cost-based approaches to transfer pricing are often used in practice, because in practice the
following conditions are common.
a) There is no external market for the product that is being transferred.
b) Alternatively, although there is an external market it is an imperfect one because the
market price is affected by such factors as the amount that the company setting the
transfer price supplies to it, or because there is only a limited external demand.

In either case there will not be a suitable market price upon which to base the transfer price.

Transfer prices based on full cost
Under this approach, the full cost (including fixed overheads absorbed) incurred by the
supplying division in making the 'intermediate' product is charged to the receiving division. If
a full cost plus approach is used a profit margin is also included in this transfer price.
An intermediate product is one that is used as a component of another product, for example
car headlights or food additives.

Example: transfers at full cost (plus)
Consider the example introduced in Section 5.1.1, but with the additional complication of
imperfect intermediate and final markets. A company has 2 profit centres, A and B. Centre A
can only sell half of its maximum output externally because of limited demand. It transfers
the other half of its output to B, which also faces limited demand. Costs and revenues in an
accounting period are as follows.
Page 589

A

B

RWF ‘000

External sales
Costs of production in the division

Total

RWF ‘000

RWF ‘000

8,000

24,000

32,000

12,000

10,000

22,000

Profit

10,000

There are no opening or closing inventories. It does not matter here whether marginal or
absorption costing is used and we shall ignore the question of whether the current output
levels are profit maximising and congruent with the goals of the company as a whole.

Transfer price at full cost only
If the transfer price is at full cost, A in our example would have 'sales' to B of RWF6,000
(costs of RWF12,000 × 50%). This would be a cost to B, as follows.

A
RWF
‘000

B
RWF
‘000

RWF

RWF

Open market sales

8,000

24,000

Transfer sales

6,000

–

14,000

24,000

Total sales, incl.
transfers
Transfer costs
Own costs
Total costs, incl.
transfers
Profit

Company as a
whole
RWF
32,000

6,000
12,000

10,000

22,000

12,000

16,000

2,000

8,000

Page 590

10,000

The transfer sales of A are self-cancelling with the transfer costs of B so that total profits are
unaffected by the transfer items. The transfer price simply spreads the total profit of
RWF10,000 between A and B.
The obvious drawback to the transfer price at cost is that A makes no profit on its work, and
the manager of division A would much prefer to sell output on the open market to earn a
profit, rather than transfer to B, regardless of whether or not transfers to B would be in the
best interests of the company as a whole. Division A needs a profit on its transfers in order to
be motivated to supply B; therefore transfer pricing at cost is inconsistent with the use of a
profit centre accounting system.

Transfer price at full cost plus
If the transfers are at cost plus a margin of, say, 25%, A's sales to B would be RWF7,500
(RWF12,000 × 50% × 1.25).

A
RWF

B
RWF

Total

RWF

RWF

RWF

Open market sales

8,000

24,000

Transfer sales

7,500

–

15,500

24,000
7,500

Transfer costs
Own costs

Profit

32,000

12,000

10,000

22,000

12,000

17,500

3,500

6,500

Page 591

10,000

Compared to a transfer price at cost, A gains some profit at the expense of B. However, A
makes a bigger profit on external sales in this case because the profit mark-up of 25% is less
than the profit mark-up on open market sales. The choice of 25% as a profit mark-up was
arbitrary and unrelated to external market conditions.

Divisional autonomy, divisional performance measurement and corporate profit
maximisation
In the above case the transfer price fails on all three criteria for judgement.
a) Arguably, it does not give A fair revenue or charge B a reasonable cost, and so their
profit performance is distorted. It would certainly be unfair, for example, to compare
A's profit with B's profit.
b) Given this unfairness it is likely that the autonomy of each of the divisional managers
is under threat. If they cannot agree on what is a fair split of the external profit a
decision will have to be imposed from above.
c) It would seem to give A an incentive to sell more goods externally and transfer less to
B. This may or may not be in the best interests of the company as a whole.

Example
Suppose, in the example above, that the cost per unit of A's output is RWF9 in variable costs
and RWF6 in fixed costs. B's own costs are RWF25 including a fixed element of RWF10.
What is the minimum price that B should charge for its products to break even?

Answer
A produces RWF12,000/(RWF9 + RWF6) = 800 units and transfers half of them to B for
RWF6,000. The cost for each unit that B buys is therefore RWF6,000/400 = RWF15. From
B's perspective this is a variable cost. B's costs are as follows.

Page 592

Cost per unit
RWF
Variable cost: transfers from A

15

Own variable costs

15
30

From B's perspective it must charge more than RWF30 per unit to earn a contribution.
However, from the overall perspective, RWF6 of the 'variable' cost of transfers is fixed. The
variable cost is really RWF9 + RWF15 = RWF24, and any price above this will earn a
contribution for the organisation as a whole.

Transfer price at marginal cost
A marginal cost approach entails charging the marginal cost that has been incurred by the
supplying division to the receiving division. As above, we shall suppose that A's cost per unit
is RWF15, of which RWF6 is fixed and RWF9 variable.

Page 593

A
RWF
Market sales
Transfer sales (Rwf6,000 ×
9/15)

B
RWF
8,000
3,600

RWF

11,600
Transfer costs
Own variable costs
Own fixed costs
Total costs and transfers
(Loss)/Profit

–
7,200
4,800

RWF
24,000
–
24,000

3,600
6,000
4,000
12,000
(400)

Company as a
whole
RWF
RWF
32,000

13,200
8,800
13,600
10,400

22,000
10,000

Divisional autonomy, divisional performance measurement and corporate profit
maximisation
a) This result is deeply unsatisfactory for the manager of division A who could make an
additional RWF4,400 (RWF(8,000 – 3,600)) profit if no goods were transferred to
division B.
b) Given that the manager of division A would prefer to transfer externally, head office
are likely to have to insist that internal transfers are made.
c) For the company overall, external transfers only would cause a large fall in profit,
because division B could make no sales at all.

The problem is that with a transfer price at marginal cost the supplying division does not
cover its fixed costs.

Page 594

FIXED COST AND TRANSFER PRICING
Fixed costs in the supplying division can be accounted for in a number of ways to ensure that
it at least breaks even.
There are a number of ways in which this problem could be overcome.

Sharing contribution
Each division can be given a share of the overall contribution earned by the organisation, but
it is probably necessary to decide centrally what the shares should be, undermining divisional
autonomy. Alternatively central management could impose a range within which the transfer
price should fall, and allow divisional managers to negotiate what they felt was a fair price
between themselves.

Two-part charging system
Transfer prices are set at variable cost and once a year there is a transfer of a fixed fee to the
supplying division, representing an allowance for its fixed costs. Care is needed with this
approach. It risks sending the message to the supplying division that it need not control its
fixed costs because the company will subsidise any inefficiencies. On the other hand, if
fixed costs are incurred because spare capacity is kept available for the needs of other
divisions it is reasonable to expect those other divisions to pay a fee if they 'booked' that
capacity in advance but later failed to utilise it. The main problem with this approach once
more is that it is likely to conflict with divisional autonomy.

Page 595

Dual pricing
Be careful not to confuse this term with 'two-part' transfer pricing. Dual pricing means that
two separate transfer prices are used.
a) For the transfer of goods from the supplying division to the receiving division the
transfer price is set at variable cost. This ensures that the receiving division makes
optimal decisions and it leads to corporate profit maximisation.
b) For the purposes of reporting results the transfer price is based on the total costs of
the transferring division, thus avoiding the possibility of reporting a loss.
This method is not widely used in practice.

Addressing organisational structure
One final possibility that may be worth mentioning. Given that the problems are caused by
the divisional structure, might it not be better to address the structure, for example by
merging the two divisions, or ceasing to treat the transferring division as a profit centre.
This may not be practical. Some would argue that the benefits of decentralisation in terms of
motivation outweigh any costs that might arise due to slight inefficiencies.

Page 596

STANDARD COST VERSUS ACTUAL COST
Standard costs should be used for transfer prices to avoid encouraging inefficiency in the
supplying division.
When a transfer price is based on cost, standard cost should be used, not actual cost. A
transfer of actual cost would give no incentive to control costs, because they could all be
passed on. Actual cost-plus transfer prices might even encourage the manager of A to
overspend, because this would increase the divisional profit, even though the company as a
whole (and division B) suffers.
Suppose, for example, that A's costs should have been RWF12,000, but actually were
RWF16,000. Transfers (50% of output) would cost RWF8,000 actual, and the cost plus
transfer price is at a margin of 25% (Rwf8,000 × 125% = Rwf10,000).

A

B

RWF

RWF

RWF

Total
RWF

RWF

Market sales

8,000

24,000

Transfer sales

10,000

–

18,000

24,000

Transfer costs
Own costs

Profit

–

10,000

16,000

10,000

32,000

26,000

16,000

20,000

2,000

4,000

6,000

A's overspending by RWF4,000 has reduced the total profits from RWF10,000 to RWF6,000.
In this example, B must bear much of the cost of A's overspending, which is clearly
unsatisfactory for responsibility accounting. If, however, the transfer price were at standard
cost plus instead of actual cost plus, the transfer sales would have been RWF7,500, regardless
of A's overspending.
Page 597

A

B

RWF

RWF

Total

RWF

RWF

RWF

Market sales

8,000

24,000

Transfer sales

7,500

–

15,500

24,000

Transfer costs
Own costs

–

7,500

16,000

10,000
16,000

Profit/(loss)

(500)

32,000

17,500

26,000

6,500

6,000

The entire cost of the overspending by A of RWF4,000 is now borne by division A itself as a
comparison with the first table of figures in this section will show.

Page 598

COST-BASED APPROACHES WITH NO EXTERNAL
MARKET
With no external market, the transfer price should be set in the range where variable cost in
the supplying division ≤ net marginal revenue in the receiving division.

Unlimited capacity and no external market
So far we have considered the use of cost-based approaches where the following factors
applied.
a) There was a limit on the maximum output of the supplying division.
b) There was a limit to the amount that could be sold in the intermediate market.

We found that a marginal cost based approach led to the best decisions for the organisation
overall, but that this was beset with problems in maintaining divisional autonomy and
measuring divisional performance fairly.
We shall now consider whether this finding changes in different conditions. We shall remove
the limit on output and demand for the final product, but assume that there is no intermediate
market at all.

Example: unlimited capacity and no intermediate market
Motivate Ltd has two profit centres, P and Q. P transfers all its output to Q. The variable cost
of output from P is RWF5,000 per unit, and fixed costs are RWF1,200,000 per month.
Additional processing costs in Q are RWF4,000 per unit for variable costs, plus fixed costs of
RWF800k. Budgeted production is 400 units per month, and the output of Q sells for
RWF15k per unit. The transfer price is to be based on standard full cost plus. From what
range of prices should the transfer price be selected, in order to motivate the managers of
both profit centres to both increase output and reduce costs?
Page 599

Solution
Any transfer price based on standard cost plus will motivate managers to cut costs, because
favourable variances between standard costs and actual costs will be credited to the division's
profits.
Managers of each division will also be willing to increase output above the
budget of 400 units provided that it is profitable to do so; that is:
a) In P, provided that the transfer price exceeds the variable cost of RWF5,000 per unit
b) In Q, provided that the transfer price is less than the difference between the fixed
selling price (RWF15k) and the variable costs in Q itself (RWF4,000). This amount of
RWF11,000 (RWF15k – RWF4k) is sometimes called net marginal revenue

The range of prices is therefore between RWF5,001 and RWF10,999.
Let's do a check. Suppose the transfer price is RWF9,000. With absorption based on the
budgeted output of 400 units what would divisional profits be if output and sales are 400
units or 500 units?
Overheads per unit are RWF1,200,000/400 = RWF3,000, so the full cost of sales is RWF(5k
+ 3k) = RWF8k in division P. In division Q, full cost is RWF(4k + 2k) = RWF6,000, plus
transfer costs of RWF9,000.
At 400 units:

P

Q

Total

RWF ‘000

RWF ‘000

RWF ‘000

–

6,000

6,000

Transfer sales

3,600

–

Transfer costs

–

Sales

Own full cost of sales

(3,200)

Under/over absorbed overhead
Page 600

(3,600)
(2,400)

(5,600)

400

0

400

0

0

0

Profit/(loss)

400

0

400

At 500 units:
P

Q

Total

RWF ‘000

RWF ‘000

RWF ‘000

–

7,500

7,500

Transfer sales

4,500

–

–

Transfer costs

–

Sales

Own full cost of sales

(4,000)

(4,500)
(3,000)

–
(7,000)

500

0

500

Over absorbed overhead (100 × RWF3; 100 ×
RWF2)

300

200

500

Profit/(loss)

800

200

1,000

Increasing output improves the profit performance of both divisions and the company as a
whole, and so decisions on output by the two divisions are likely to be goal congruent.

Summary
To summarise the transfer price should be set in the range where:
Variable
division

cost

in

supplying ≤

Selling price minus variable costs (net
marginal revenue) in the receiving
division

Page 601

In fact, if there is no external market, and if the transferred item is the major product of the
transferring division, there is a strong argument for suggesting that profit centre accounting is
a waste of time.
Profit centres cannot be judged on their commercial performance because there is no way of
gauging what a fair revenue for their work should be. It would be more appropriate, perhaps,
to treat the transferring 'division' as a cost centre, and to judge performance on the basis of
cost variances.

Page 602

OPPORTUNITY COSTS AND TRANSFER PRICES
If a profit-maximising output level has been established, the transfer price should be set such
that there is not a more profitable opportunity for individual divisions. In other words transfer
prices should include opportunity costs of transfer.

The ideal transfer price
Ideally, a transfer price should be set that enables the individual divisions to maximise their
profits at a level of output that maximises profit for the company as a whole. The transfer
price which achieves this is unlikely to be a market-based transfer price (if there is one) and
is also unlikely to be a simple cost plus based price.

An opportunity cost approach
If optimum decisions are to be taken transfer prices should reflect opportunity costs.
a) If profit centre managers are given sufficient autonomy to make their own output and
selling decisions, and at the same time their performance is judged by the company
according to the profits they earn, they will be keenly aware of all the commercial
opportunities.
b) If transfers are made for the good of the company as a whole, the commercial benefits
to the company ought to be shared between the participating divisions.

Transfer prices can therefore be reached by:
a) Recognising the levels of output, external sales and internal transfers that are best for
the company as a whole, and
b) Arriving at a transfer price that ensures that all divisions maximise their profits at this
same level of output. The transfer price should therefore be such that there is not a
more profitable opportunity for individual divisions. This in turn means that the
opportunity costs of transfer should be covered by the transfer price.
Page 603

BLANK

Page 604

TRANSFER PRICING WHEN INTERMEDIATE
PRODUCTS ARE IN SHORT SUPPLY
When an intermediate resource is in short supply and acts as a limiting factor on
production in the supplying division, the cost of transferring an item is the variable cost of
production plus the contribution obtainable from using the scarce resource in its next most
profitable way.

Example: scarce resources
Suppose, for example, that division A is a profit centre that produces three items, X, Y and Z.
Each item has an external market.
X

Y

Z

External market price, per unit

RWF48,000

RWF46,000

RWF40,000

Variable cost of production in division A

RWF33,000

RWF24,000

RWF28,000

3

4

2

Labour hours required per unit in division A

Product Y can be transferred to division B, but the maximum quantity that might be required
for transfer is 300 units of Y.
The maximum external sales are 800 units of X, 500 units of Y and 300 units of Z.
Instead of receiving transfers of product Y from division A, division B could buy similar
units of product Y on the open market at a slightly cheaper price of RWF45,000 per unit.
What should the transfer price be for each unit if the total labour hours available in division A
are 3,800 hours or 5,600 hours?

Page 605

Solution
Hours required to meet maximum demand:
External sales:

Hours

X (3 × 800)

2,400

Y (4 × 500)

2,000

Z (2 × 300)

600
5,000

Transfers of Y (4 × 300)

1,200
6,200

Contribution from external sales:
X

Y

Z

Contribution per unit

RWF15,000

RWF22,000

RWF12,000

Labour hours per unit

3 hrs

4 hrs

2 hrs

RWF5,000

RWF5,500

RWF6,000

Contribution per labour hour
Priority for selling

3rd

Total hours needed

2,400

Page 606

2nd
2,000

1st
600

a) If only 3,800 hours of labour are available, division A would choose, ignoring
transfers to B, to sell:
Hours
300 Z (maximum)

600

500 Y (maximum)

2,000
2,600

400 X (balance)

1,200
3,800

To transfer 300 units of Y to division B would involve forgoing the sale of 400 units of
X because 1,200 hours would be needed to make the transferred units.
Opportunity cost of transferring units of Y, and the appropriate transfer price:

RWF per
unit
Variable cost of making Y

24,000

Opportunity cost (contribution of Rwf5 per hour
available from selling X externally): benefit forgone (4 hours ×
Rwf5k)

20,000

Transfer price for Y

44,000

The transfer price for Y should, in this case, be less than the external market price.

b) If 5,600 hours are available, there is enough time to meet the full demand for
external sales (5,000) and still have 600 hours of spare capacity, before
consideration of transfers. However, 1,200 hours are needed to produce the full
Page 607

amount of Y for transfer (300 units), and so 600 hours need to be devoted to
producing Y for transfer instead of producing X for external sale.
This means that the opportunity cost of transfer is:
(i)

the variable cost of 150 units of Y produced in the 600 'spare' hours
(RWF24k/unit);

(ii)

the variable cost of production of the remaining 150 units of Y (RWF24k per
unit), plus the contribution forgone from the external sales of X that could have
been produced in the 600 hours now devoted to producing Y for transfer (RWF5
per labour hour). An average transfer price per unit could be negotiated for the
transfer of the full 300 units (see below), which works out at RWF34 per unit.

RWF ‘000
150 units × Rwf24k

3,600

150 units × RWf24k

3,600

600 hours × RWF5k per hour

3,000

Total for 300 units

10,200

In both cases, the opportunity cost of receiving transfers for division B is the
price it would have to pay to purchase Y externally –RWF45k per unit. Thus:

Maximum labour
hours in A

Opportunity cost
to A of transfer

Opportunity cost
to B of transfer

RWF ‘000

RWF ‘000

3,800

44

45

5,600

34 (average)

45

In each case any price between the two opportunity costs would be sufficient to persuade B to
order 300 units of Y from division A and for division A to agree to transfer them.
Page 608

Central information
The only way to be sure that a profit-maximising transfer policy will be implemented is to
dictate the policy from the centre. This means that the following information must be
available centrally.
a) A precise breakdown of costs in each division at all levels of output
b) Market information for each market, indicating the level of demand at a range of
prices
c) Perhaps most vitally, knowledge of the likely reaction of divisional managers to
a centrally imposed policy that undermines their autonomy and divisional profits

Why not try to explain in your own words why transfer prices should reflect opportunity
costs?
If you cannot do so, start reading this section again. You probably would not be able to do a
Paper P5 transfer pricing question unless you can give this explanation.

Optimal transfer prices: an extended example
A group of highly integrated divisions wishes to be advised as to how it should set transfer
prices for the following inter-divisional transactions:
a) Division L sells all its output of product LX to Division M. To one kilogram of LX,
Division M adds other direct materials and processes it to produce two kilograms of
product MX which it sells outside the group. The price of MX is influenced by
volume offered and the following cost and revenue data are available:
Division L
The variable costs per kg of LX are RWF4,000 of direct materials and RWF2,000 of
direct labour.

Page 609

The following cost increases are expected at different levels of production per annum:
Direct materials

At 60,000 kg pa increase to RWF5,000 per kg
At 90,000 kg pa increase to RWF5,500 per kg
At 100,000 kg pa increase to RWF6,000 per kg

Direct labour

At 80,000 kg pa increases to RWF2,500 per kg
At 100,000 kg pa increases to RWF3,000 per kg

Fixed overhead

Under 70,000 kg
70,000 – 79,999 kg
80,000 – 89,999 kg
90,000 or more kg

RWF210m pa
RWF 260m pa
RWF280m pa
RWF 310m pa

Division M
To produce one kilogram of product MX, the variable cost incurred for each halfkilogram of LX used is made up of RWF1.50 of other direct materials and RWF3.50
processing cost.
The following cost increases are expected at different levels of production of MX per
annum:
Other direct materials

At 140,000 kg pa increase to RWF1,750k per kg
At 160,000 kg pa increase to RWF2,000k per kg

Processing

At 180,000 kg pa increases to RWF 4,000k per kg

Fixed overhead

Under 120,000 kg
120,000 – 139,999 kg
140,000 – 159,999 kg
160,000 – 199,999 kg
200,000 or more kg

Selling price

Up to 199,999 kg
200,000 or more kg

Page 610

RWF ‘000
250,000 pa
280,000 pa
290,000 pa
320,000 pa
360,000 pa
RWF 16,000k per kg
RWF 15,500k per kg

b) Division N manufactures two products, NA and NB, whose variable production cost
and selling prices per unit are:
NA

Product

NB

RWF

RWF

RWF

Direct materials

4,000

3,000

Direct labour

4,000

6,000

Production cost
Selling price

RWF

8,000

9,000

16,000

25,000

Direct labour is costed at RWF 500 per hour. Fixed overhead is RWF 36,000,000 per
annum and total capacity is 960,000 man-hours per annum.
Division N sells product NA either to Division P or outside the group and Division P
can buy from either source. Product NB is sold only outside the group. When NA and
NB are sold outside the group, variable selling costs of RWF1,000 and RWF2,000 per
unit respectively are incurred.

Required
a) Recommend, with supporting calculations and explanations, the most appropriate
narrow range of transfer price per kg for product LX between the two divisions;
assume that any changes in output are in steps of 10,000 kg of product LX and 20,000
kg of product MX.
b) Recommend, with supporting calculations and explanations:
(i)

The most appropriate transfer price of product NA between Divisions N and
P:
(1)

On the assumption that Division N can just sell all of, but no more
than, its capacity;

(2)

On the assumption that Division N could sell more than its existing
capacity, though the market price stays the same;
Page 611

(ii)

In the case of (i) (2) above, what quantities of NA Division P should buy
from Division N.

Discussion and solution
Part (a) of the question is long and the calculations might easily be confusing. The ratio of 1
kg of LX to 2 kg of MX also complicates the figure-work. It is probably tempting to calculate
unit contribution rather than unit net profit, but in this case it is probably easier to work out
unit full costs, because of the stepped changes in fixed costs.

Step 1

The first step in a solution is to work out what is best for the group as a whole.
MX costs and profits can be calculated on a unit basis first and then total
profitability at each level of output derived.

Quantity of MX ('000 kg)
100

120

140

Division L cost (1 kg LX per 2 kg MX)
RWF RWF RWF
‘000
‘000
‘000
2.00
2.50
2.50
Direct materials
1.00
1.00
1.00
Direct labour
2.10
1.75
1.86
Fixed overhead
5.10
5.25
5.36
Total
Division M costs
Other materials
Processing cost
Fixed overhead
Total
Full unit cost
Sales price
Unit profit
Total
(RWF m)

1.50
3.50
2.50
7.50
12.60
16.00
3.40

profit 340

160

180

200

220

240

RWF
‘000
2.50
1.25
1.75
5.50

RWF
‘000
2.75
1.25
1.72
5.72

RWF
‘000
3.00
1.50
1.55
6.05

RWF
‘000
3.00
1.50
1.41
5.91

RWF
‘000
3.00
1.50
1.29
5.79

1.50
3.50
2.33
7.33
12.58
16.00
3.42

1.75
3.50
2.07
7.32
12.68
16.00
3.32

2.00
3.50
2.00
7.50
13.00
16.00
3.00

2.00
4.00
1.78
7.78
13.50
16.00
2.50

2.00
4.00
1.80
7.80
13.85
15.50
1.65

2.00
4.00
1.64
7.64
13.55
15.50
1.95

2.00
4.00
1.50
7.50
13.29
15.50
2.21

410

465

480

450

330

429

530

Page 612

Below 200,000 kg, profit is maximised at 160,000 kg of MX (RWF480 m) but
this profit figure is exceeded when output rises to 240,000 kg and beyond, by
which time profit is rising by RWF100 m per extra 20,000 kg of MX. Division M
ought to make 240,000 kg or more of MX, up to capacity output.

Step 2

So how do we calculate the ideal transfer price? First of all, the transfer price
must be higher than RWF5.79k per ½ kg of LX, but not more than
RWF(5.79k + 2.21k) = RWF8k per ½ kg of LX. At the lower price, Division L
would make no profit at 240,000 kg of MX and at the higher price, Division M
would make no profit.

Step 3

The selection of a transfer price is further complicated by the changing unit costs
at lower levels of output. The transfer price must give each division an incentive
not to want to restrict output to less than 240,000 kg of MX.

a) Left to its own devices Division M will produce 140,000 kg because its
unit costs are minimised at this level. So Division M will be willing to
offer Division L a transfer price that persuades it to produce 70,000 kg
and this must at least cover Division L's unit costs at this level of
RWF5.36k per half kg.

b) However, overall profit is maximised at the 240,000 kg level, where unit
costs for Division L are RWF430 (RWF5.79k – RWF5.36k) higher.
240,000x

>

140,000(x + 0.43k)

100,000x

>

60,200

>

600

x

Our analysis suggests that unless Division L earns a unit profit of at least
600 francs, the division's manager will need a lot of persuading to increase
output above 70,000 kg (enough for 140,000 kg of MX).

Page 613

c) In the case of Division M, a similar analysis can be applied. Unit costs are
RWF180 per kg lower at 140,000 kg of output, and so if the unit profit at
240,000 kg is RWFy, we want a transfer price where:
240,000y
100,000y
y

>
>
>

140,000(y + 180)
25,200
250

This suggests that unless Division M makes a profit per kg of at least 250
Francs at 240,000 kg of output the division's manager might prefer to halt
production and sales at 140,000 kg.

Step 4

The range of transfer prices per kg of MX is therefore narrower than the
RWF5.79k to RWF8k range we began with. Division L should have a profit of at
least RWF600 per kg of MX at 240,000 kg of output and so the minimum transfer
price should be (RWF5,790 + RWF600) RWF6,390 per kg of MX. Division M
should have a profit of at least RWF 250 and so the maximum transfer price
should be (RWF8,000 – RWF250) RWF7750.

Conclusion for Division L and Division M. The transfer price per kg of MX should be in the
range RWF6,390 – RWF7,750, so that the transfer price range per kg of LX (2:1) is
RWF12,780 – RWF15,500.

Let's now turn our attention to part (b). The situation facing Division N is probably a bit
easier to understand.
a)

If Division N can sell all of its capacity but no more, the opportunity cost of
transferring NA instead of selling it will be the external revenue per unit of NA
less the variable selling costs – i.e. RWF(16,000 – 1,000) = RWF15,000 per unit.

b)

In part (2) of the question, N's existing capacity acts as a constraint on total
output: labour hours become a scarce resource.

Page 614

NA

NB

RWF ‘000

RWF ‘000

Contribution before variable selling costs

8

16

Variable selling costs
Net contribution

1
7

2
14

Labour hours per unit
Contribution per labour hour

8
RWF875

12
RWF1,167

Transferring NA to Division P (rather than making NB) would force Division N to forgo
contribution of Rwf1,167 per labour hour, and the transfer price of NA should reflect this
opportunity cost:
RWF
‘000
Variable cost of making NA

8.0

Opportunity cost of lost contribution on NB (8 hrs × RWF1,167)

9.3

Transfer price of NA, per kg

c)

17.3

Since the external market price of NA is only RWF16,000, Division P should buy
all its supplies externally, and buy nothing from Division N. This would leave
Division N free to make NB exclusively, and earn a contribution of RWF1,167
per labour hour on all its external sales.

Page 615

BLANK

Page 616

TRANSFER PRICING AND A RANGE OF LIMITING
FACTORS
If a supplying division is subject to a range of limiting factors, the optimum production plan
can be derived using a linear programming model.

Example: transfer pricing with a range of limiting factors
LP Ltd has two divisions, division 1 and division 2. Division 1 produces liquid A, all of
which is transferred to division 2, and liquid B which can either be sold externally or
transferred to division 2. Division 2 uses these liquids to produce its powdered products, X
and Y.
Production of liquid A is restricted due to a shortage of skilled labour so that only 4,000 litres
can be produced. Liquid B can also only be produced in limited numbers due to a scarcity of
ingredients. Only 6,000 litres of liquid B can be made. Details of costs and revenues are as
follows.

A

B

X

Y

RWF
‘000

RWF
‘000

RWF
‘000

RWF
‘000

Variable cost (division 1)

4

6

–

–

Variable cost (division 2)

–

–

7

5

Selling price

–

9

30

35

One sachet of powder X requires 1 litre of liquid A and 2 litres of liquid B.
One sachet of powder Y requires 2 litres of liquid A and 2 litres of liquid B.

Page 617

Required
Formulate a linear programming model to determine the optimum production levels and
transfer prices.

Solution
Step 1

Work out the contribution obtained from each product
This needs to take account of the usage of A and B by X and Y.

Step 2

B

X

Y

Variable costs

6

7

5

Liquid A (l litre/2 litres)

–

4

8

Liquid B (2 litres/2 litres)

–

12

12

6

23

25

Selling price

9

30

35

Contribution

3

7

10

Formulate objective function
The objective is to maximise the corporate contribution by producing the
optimum quantities of products B, X and Y. Algebraically this is expressed as
follows.
Maximise 3B + 7X + 10Y

Step 3

Define constraints
The constraints are as follows.
1X + 2Y

≤

4,000 (labour shortage)

B + 2X + 2Y

≤

6,000 (ingredients shortage)

B, X, Y

≥

0

Page 618

In many exam questions you will only be required to formulate the model, not solve it; read
the question carefully!!.

In practice, as you probably remember, where there are more than two variables in the
objective function and more than a few constraints a computer software package is needed.
The output from the model will show how many sachets of X and Y should be produced
and how many litres, if any, of B should be sold externally. The output also provides a
means of calculating the ideal transfer price, because it indicates the shadow price of scarce
resources.

Page 619

BLANK

Page 620

SHADOW PRICE AND TRANSFER PRICES
Shadow prices replace opportunity costs when determining transfer prices if there are
constraints on production.

The shadow price is the maximum extra amount that it would be worth paying to obtain one
extra unit of a scarce resource. To put it another way, the shadow price is the opportunity cost
of the scarce resource, the amount of benefit forgone by not having the availability of the
extra resources.

We already know that an optimal transfer price can be calculated by adding together the
variable cost of the intermediate product and the opportunity cost of making the
transfer. In our example, let us suppose that the shadow price of liquid A is RWF3,000 and
of liquid B, RWF2,000.
A

B

RWF ‘000

RWF ‘000

Variable cost

4

6

Shadow price

3

2

Transfer price

7

8

This solution might be tested by the divisional manager of the supplying division by applying
his own linear programming model attempting to maximise the contribution from external
sales of B (which we shall call B1) and from transfers of A and B.
Maximise 3A + 2B + 3B1
Subject to

A

≤

4,000

B + B1

≤

6,000

A, B, B1

≥

0

Page 621

This would give the same optimum production levels as the original linear programme,
because it is derived from the same information.
For division 2, however, these transfer prices would result in each product yielding a
contribution of nil. In effect this means that the optimal solution must be centrally
imposed, otherwise the manager of division 2 will have no incentive to produce X and Y at
all.

Page 622

NEGOTIATED TRANSFER PRICES
In practice, negotiated transfer prices, market-based transfer prices and full cost-based
transfer prices are the methods normally used.

A transfer price based on opportunity cost is often difficult to identify, for lack of suitable
information about costs and revenues in individual divisions. In this case it is likely that
transfer prices will be set by means of negotiation. The agreed price may be finalised from a
mixture of accounting arithmetic, politics and compromise.
The process of negotiation will be improved if adequate information about each division's
costs and revenues are made available to the other division involved in the negotiation. By
having a free flow of cost and revenue information, it will be easier for divisional managers
to identify opportunities for improving profits, to the benefit of both divisions involved in the
transfer.
A negotiating system that might enable goal congruent plans to be agreed between profit
centres is as follows.
a) Profit centres submit plans for output and sales to head office, as a preliminary step
in preparing the annual budget.
b) Head office reviews these plans, together with any other information it may obtain.
Amendments to divisional plans might be discussed with the divisional managers.
c) Once divisional plans are acceptable to head office and consistent with each other,
head office might let the divisional managers arrange budgeted transfers and transfer
prices.
d) Where divisional plans are inconsistent with each other, head office might try to
establish a plan that would maximise the profits of the company as a whole.
Divisional managers would then be asked to negotiate budgeted transfers and
transfer prices on this basis.
e) If divisional managers fail to agree a transfer price between themselves, a head
office 'arbitration' manager or team would be referred to for an opinion or a
decision.
f)

Divisions finalise their budgets within the framework of agreed transfer prices and
resource constraints.

g) Head office monitors the profit performance of each division.
Page 623

BLANK

Page 624

MULTINATIONAL TRANSFER PRICING
Multinational transfer pricing needs to take account of a range of factors.
•
•
•
•
•
•
•

Exchange rate fluctuations
Anti-dumping legislation
Exchange controls
Taxation in different countries
Competitive pressures
Import tariffs
Repatriation of funds

Globalisation, the rise of the multinational corporation and the fact that more than 60% of
world trade takes place within multinational organisations mean that international
transfer pricing is very important.

Factors to consider when setting multinational transfer prices
The level at which a transfer price should be set is even less clear cut for organisations
operating in a number of countries, when even more factors need to be taken into
consideration. Moreover, the manipulation of profits through the use of transfer pricing is a
common area of confrontation between multinational organisations and host country
governments.

Page 625

Factors to
consider

Explanation

Exchange
rate
fluctuation

The value of a transfer of goods between profit centres in different
countries could depend on fluctuations in the currency exchange rate.

Taxation in
different
countries

If taxation on profits is 20% of profits in Country A and 50% on profits in
Country B, a company will presumably try to 'manipulate' profits (by
means of raising or lowering transfer prices or by invoicing the subsidiary
in the high-tax country for 'services' provided by the subsidiary in the lowtax country) so that profits are maximised for a subsidiary in Country A, by
reducing profits for a subsidiary in Country B.
Some multinationals set up marketing subsidiaries in countries with low
tax rates and transfer products to them at a relatively low transfer price.
When the products are sold to the final customer, a low rate of tax will be
paid on the difference between the two prices.

Import tariffs

Suppose that Country A imposes an import tariff of 20% on the value of
goods imported. A multi-national company has a subsidiary in Country A
which imports goods from a subsidiary in Country B. In such a situation,
the company would minimise costs by keeping the transfer price to a
minimum value.

Exchange
controls

If a country imposes restrictions on the transfer of profits from domestic
subsidiaries to foreign multinationals, the restrictions on the transfer can be
overcome if head office provides some goods or services to the subsidiary
and charges exorbitantly high prices, disguising the 'profits' as sales revenue,
and transferring them from one country to the other. The ethics of such an
approach should, of course, be questioned.

Antidumping
legislation

Governments may take action to protect home industries by preventing
companies from transferring goods cheaply into their countries. They may
do this, for example, by insisting on the use of a fair market value for the
transfer price.

Competitive
pressures

Transfer pricing can be used to enable profit centres to match or undercut
local competitors.

Repatriation
of funds

By inflating transfer prices for goods sold to subsidiaries in countries
where inflation is high, the subsidiaries' profits are reduced and funds
repatriated, thereby saving their value.
Page 626

Transfer prices and tax
Tax authorities obviously recognise the incentive to set transfer prices to minimise taxes
and import tariffs. Many tax authorities have the power to modify transfer prices in
computing tariffs or taxes on profit, although a genuine arms-length market price should
be acceptable.
a) For instance, UK government legislation restricts how far companies can declare their
profits in a low taxation country. Some scope for profit apportionment between
divisions clearly exists, however. HM Revenue and Customs has the power to adjust
the taxable income of the UK party to a cross-border transaction to the figure that
would have resulted if the prices actually used had been between two unrelated
parties ('arm's length' price).
b) And in the USA, multinational organisations must follow an Internal Revenue Service
Code specifying that transfers must be priced at 'arm's length' market values, or at the
values that would be used if the divisions were independent companies. Even with
this rule, companies have some leeway in deciding an appropriate 'arm's length' price.

To meet the multiple objectives of transfer pricing, companies may choose to maintain two
sets of accounting records, one for tax reporting and one for internal management
reporting. The tax authorities may interpret the use of two sets of records as suggestive of
profit manipulation, however.
Double taxation agreements between countries mean that companies pay tax on specific
transactions in one country only. If a company sets an unrealistically low transfer price,
however, the company will pay tax in both countries (double taxation) if it is spotted by the
tax authorities.
Most countries now accept the Organisation for Economic Co-operation and Development
(OECD) 1995 guidelines. These aim to standardise national approaches to transfer pricing
and provide guidance on the application of the 'arm's length' price. This can be determined in
three main ways.
a) Comparable uncontrolled price method is the most widely used and involves
setting the arm's length price on the basis of the externally verified prices of similar
products. In other words, the market price or an approximation to one is used. It can
be difficult to make meaningful comparisons, however, as most international trade is
carried out between related companies.
b) Resale price method involves deducting a percentage from the selling price of the
final product to allow for profit.
Page 627

c) Cost-plus method. This method is used where there is no market price and so the
comparable uncontrolled price method cannot be used.

These methods are of little use in determining arm's length prices for intangible assets such
as a trade name, however, and much of the information required is not in the public domain.
Setting transfer prices is therefore not straightforward.
Many countries are tightening their regulations in response to the OECD guidelines. In
the UK, for example, it used to be up to the tax authorities to detect cases of inappropriate
transfer pricing. Under self-assessment, it is now the duty of the tax payer to provide the
correct information. A penalty of 100% of any tax adjustment is payable for failing to
demonstrate a reasonable attempt at using an arm's length price in a tax return. The taxpayer
may enter into an Advanced Pricing Agreement (APA) with the two tax authorities involved.
This is done in advance to avoid dispute, double taxation and penalties.

Example: arm's length transfer price
Suppose division A produces product B in a country where the income tax rate is 30% and
transfers it to division C, which operates in a country with a 40% rate of income tax. An
import duty equal to 25% of the price of product B is also assessed. The full cost per unit is
RWF290,000, the variable cost RWF160,000.

Required
The tax authorities allow either variable or full cost transfer prices. Determine which should
be chosen.

Page 628

Solution
Effect of transferring at RWF290,000 instead of RWF160,000
RWF
‘000
Income of A is RWFk130 higher and so A pays RWFk130 × 30% more
income tax

(39.0)

Income of C is RWFk130 lower and so C pays RWFk130 × 40% less income
tax

52.0

Import duty is paid by C on an additional RWFk130, and so C pays
RWFk130 × 25% more duty

(32.5)

Net effect (cost) of transferring at RWFk290 instead of RWFk160

(19.5)

The pros and cons of different transfer pricing bases
a) A transfer price at market value is usually encouraged by the tax and customs
authorities of both host and home countries as they will receive a fair share of the
profits made but there are problems with its use.
(i)

Prices for the same product may vary considerably from one country to another.

(ii)

Changes in exchange rates, local taxes and so on can result in large variations in
selling price.

(iii) A division will want to set its prices in relation to the supply and demand
conditions present in the country in question to ensure that it can compete in that
country.
b) A transfer price at full cost is usually acceptable to tax and customs authorities since
it provides some indication that the transfer price approximates to the real cost of
supplying the item and because it indicates that they will therefore receive a fair share
of tax and tariff revenues.
c) Transfer prices at variable cost are unlikely to be acceptable to the tax authorities of the
country in which the supplying division is based as all the profits are allocated to the
receiving division and the supplying division makes a loss equal to the fixed costs
incurred.
Page 629

d) In a multinational organisation, negotiated transfer prices may result in overall suboptimisation because no account is taken of factors such as differences in tax and
tariff rates between countries.

Page 630

CHAPTER ROUNDUP
•

EVA ® is an alternative absolute performance measure. It is similar to RI and is
calculated as follows.
EVA = net operating profit after tax (NOPAT) less capital charge
where the capital charge = weighted average cost of capital × net assets

•

EVA and RI are similar because both result in an absolute figure, which is calculated
by subtracting an imputed interest charge from the profit earned by the investment
centre. However there are differences as follows.
a) The profit figures are calculated differently. EVA is based on an 'economic profit'
which is derived by making a series of adjustments to the accounting profit.
b) The notional capital charges use different bases for net assets. The replacement
cost of net assets is usually used in the calculation of EVA.

•

One of the problems of measuring managerial performance is segregating
managerial performance from the economic performance of their department or
division.

•

It is necessary for control purposes that some record of the market in interdivisional goods or services should be kept. One way of doing this is through the
accounting system. Inter-divisional work can be given a cost or a charge: a transfer
price.

•

Transfer prices are a way of promoting divisional autonomy, ideally without
prejudicing divisional performance measurement or discouraging overall
corporate profit maximisation.

•

Transfer prices may be based on market price (or an adjusted market price) where
there is an external market for the item being transferred.

•

Problems arise with cost-based transfer prices because one party or the other is liable
to perceive them as unfair.

•

Fixed costs in the supplying division can be accounted for in a number of ways to
ensure that it at least breaks even.

Page 631

•

Standard costs should be used for transfer prices to avoid encouraging inefficiency in
the supplying division.

•

With no external market, the transfer price should be set in the range where variable
cost in the supplying division ≤ net marginal revenue in the receiving division.

•

If a profit-maximising output level has been established, the transfer price should be
set such that there is not a more profitable opportunity for individual divisions. In
other words transfer prices should include opportunity costs of transfer.

•

When an intermediate resource is in short supply and acts as a limiting factor on
production in the supplying division, the cost of transferring an item is the variable
cost of production plus the contribution obtainable from using the scarce resource in
its next most profitable way.

•

If a supplying division is subject to a range of limiting factors, the optimum
production plan can be derived using a linear programming model.

•

Shadow prices replace opportunity costs when determining transfer prices if there are
constraints on production.

•

In practice, negotiated transfer prices, market-based transfer prices and full costbased transfer prices are the methods normally used.

•

Multinational transfer pricing needs to take account of a range of factors.
−
−
−
−
−
−
−

Exchange rate fluctuations
Anti-dumping legislation
Taxation in different countries
Competitive pressures
Import tariffs
Repatriation of funds
Exchange controls

Page 632

STUDY UNIT 19
Value Based Management
Contents

Page

Value-based Management Approaches ……………………..………...…….

Page 633

635

EXAM GUIDE
Remember that the main capability required is to be able to identify and assess the impact
of current developments [..] organisational performance. So you must think about how
you would use the information here in a broad discussion of performance management. You
may also be asked to explain a particular approach or framework as part of a larger question.

Exam Focus Point
Exam questions on this syllabus part are likely to require you to draft a report. Make sure that
you keep your answer to the point. Always refer back to the question and what it asks of you.

Page 634

VALUE-BASED MANAGEMENT APPROACHES
The following explanation of value-based management is based on an article in the
McKinsey quarterly 1994, adapted from a book Valuation: Measuring and Managing the
Value of Companies, Second edition by Tom Copeland, Tim Koller and Jack Murrin. 1994
John Wiley and Sons.

VBM aligns an organisation's overall aspirations, analytical techniques, and management
processes with the key drivers of value. So, VBM takes the idea of creating value through
return on future cash flow and embeds this in the organisational culture in its strategy, as well
as making this a performance measure to be used throughout the organisation.

What is value-based management?
Value-based management (VBM) starts with the philosophy that the value of a company is
measured by its discounted future cash flows. Value is created only when companies invest
capital at returns that exceed the cost of that capital.
VBM extends this philosophy by focusing on how companies use the idea of value creation
to make both major strategic and everyday operating decisions. So VBM is an approach to
management that aligns the strategic, operational and management processes to focus
management decision making on what activities create value.

Principles
VBM focuses on better decision making at all levels in an organisation. Hierarchical
command-and-control structures cannot work well, especially in large multi-divisional
organisations. Managers need to use value-based performance measures for making better
decisions. This means that they must manage the statement of financial position (balance
sheet) as well as the income statement, and maintain a balance between long- and short-term
perspectives. This approach to performance measurement is known as the value mindset.

Page 635

The Value mindset
VBM requires companies to move on from only using traditional financial performance
measures, such as earnings or earnings growth as these do not focus enough on value
creation. Companies should set also goals in terms of discounted cash flow value, the most
direct measure of value creation. These targets can then be cascaded down the organisation as
shorter-term, more objective financial performance targets.
However, non-financial goals such as customer satisfaction, product innovation, and
employee satisfaction are also important as these inspire and guide the entire organisation.
The most prosperous companies are usually the ones that combine their financial and nonfinancial goals to have a balanced approach to performance review and measurement.
A value mindset means that senior managers are fully aware that their ultimate financial
objective is maximising value. They have clear rules for deciding when other objectives (such
as employment or environmental goals) outweigh this objective; and that they have a solid
analytical understanding of which performance variables drive the value of the company.

Planning, target setting, performance measurement, and incentive systems need to be linked
to value creation at the different levels of the organisation. Management processes and
systems encourage managers and employees to behave in a way that maximises the value of
the organisation.

a) For the head of a business unit, the objective may be stated as value creation
measured in financial terms.
b) A functional manager's goals could be expressed in terms of customer service.
c) A manufacturing manager might focus on operational measures such as cost per
unit, cycle time, or defect rate.

The focus of VBM should be on the why and how of changing the organisation's
corporate culture. A value-based manager balances an awareness of organisational
behaviour with using valuation as a performance metric and decision-making tool.

Page 636

Case Study
How VBM works
The article explains how VBM works in practice.
'When VBM is working well, an organisation's management processes provide decision
makers at all levels with the right information and incentives to make value-creating
decisions.
Take the manager of a business unit. VBM would provide him or her with the information
to quantify and compare the value of alternative strategies and the incentive to choose the
value-maximising strategy. Such an incentive is created by specific financial targets set by
senior management, by evaluation and compensation systems that reinforce value creation,
and – most importantly – by the strategy review process between manager and superiors. In
addition, the manager's own evaluation would be based on long- and short-term targets that
measure progress toward the overall value creation objective.
Line managers and supervisors can have targets and performance measures that are tailored
to their particular circumstances but driven by the overall strategy.
A production manager might work to targets for cost per unit, quality, and turnaround time.
At the top of the organisation, on the other hand, VBM informs the board of directors and
corporate centre about the value of their strategies and helps them to evaluate mergers,
acquisitions, and divestitures. Value-based management can best be understood as a marriage
between a value creation mindset and the management processes and systems that are
necessary to translate that mindset into action. Taken alone, either element is insufficient.
Taken together, they can have a huge and sustained impact.'

Value drivers
VBM requires that management understand the performance variables that create the value of
the business that is the key value drivers. Management cannot act directly on value, but can
respond to things it can influence such as customer satisfaction, cost, and capital expenditure.

A value driver is any variable that affects the value of the company.

Page 637

Value drivers need to be ranked in terms of their impact on value and responsibility
assigned to individuals who can help the organisation meet its targets.
Value drivers must be matched to the appropriate level of management so that they are
consistent with the decision variables that are directly under the control of line management.
Value drivers are useful at three levels in the organisation.
a) Generic, where operating margins and invested capital are combined to compute
ROIC; (ROIC Return on Invested Capital is the USA equivalent of ROCE, Return on
Capital Employed (UK))
b) Business unit, where variables such as customer mix are particularly relevant;
c) Grass roots, where value drivers are precisely defined and tied to specific decisions
that front-line managers have under their control.

So value drivers are usually cascaded in 'trees' down the organisation so that each layer of
management has clear targets relevant to areas under their control.
These 'trees' are then usually linked into ROIC trees, which are in turn linked into multiperiod cash flows and valuation of the business unit.
It can be difficult to identify key value drivers because it requires an organisation to think
about its processes in a different way and existing reporting systems are often not equipped to
supply the necessary information. Mechanical approaches based on available information and
purely financial measures rarely succeed. What is needed instead is a creative process
involving much trial and error. Nor can value drivers be considered in isolation from each
other. The article suggests that a good way of relating a range of value drivers is to use
scenario analysis. It is a way of assessing the impact of different sets of mutually consistent
assumptions on the value of a company or its business units.

Page 638

Management processes
VBM also requires that managers must establish processes that ensure all line managers
adopt value-based thinking as an improved way of making decisions. VBM must eventually
involve every decision maker in the company.
The article notes that there are four essential management processes that collectively
govern the adoption of VBM. These four processes are linked across the company at the
corporate, business-unit, and functional levels. The four processes which run in order are
expressed below as steps:

Step 1

A company or business unit develops a strategy to maximise value.

Step 2

This strategy translates into short- and long-term performance targets defined in
terms of the key value drivers.

Step 3

Action plans and budgets are drawn up to define the steps that will be taken
over the next year or so to achieve these targets.

Step 4

Finally performance measurement and incentive systems are set up to monitor
performance against targets and to encourage employees to meet their goals.

a) Strategy development
Corporate level. Under VBM, senior management devises a corporate strategy that
explicitly maximises the overall value of the company, including buying and selling
business units as appropriate. This should be built on a thorough understanding of
business-unit strategies.
Business-unit level. Alternative strategies, should be weighed up and the one chosen
with the highest value. The chosen strategy should spell out how the business unit will
achieve a competitive advantage that will permit it to create value. The VBM elements
of the strategy then apply. They include:
(i)

Assessing the results of the valuation and the key assumptions driving the value
of the strategy.

(ii)

Assessing the value of the alternative strategies that were discarded, along
with the reasons for rejecting them.
Page 639

(iii) Looking at resource requirements. Business-unit managers need to focus on the
statement of financial position (balance sheet) and also consider human resource
requirements.
(iv) Summarising the strategic plan projections, by focusing on the key value
drivers. These should be supplemented by an analysis of the Return on Invested
Capital (ROIC) over time and relative to competitors.
(v)

Analysing alternative scenarios to assess the effect of competitive threats or
opportunities.

b) Target setting
The next step, after strategies for maximising value are agreed, is to translate
these into specific targets. In applying VBM to target setting, several general
principles are helpful:
(i)

Base targets on key value drivers. This should cover both financial and nonfinancial targets. The latter serve to prevent manipulation of short-term financial
targets.

(ii)

Tailor the targets to the different levels within an organisation. So that senior
business-unit managers should have targets for overall financial performance and
unit-wide non-financial objectives. Functional managers need functional targets,
such as cost per unit and quality.

(iii) Link short-term targets to long-term ones. The article gives the example of
setting linked performance targets for ten years, three years, and one year. The
ten-year targets express a company's aspirations; the three-year targets define
how much progress it has to make within that time in order to meet its ten-year
aspirations; and the one-year target is a working budget for managers.

The article notes that 'Ideally, targets should be expressed in terms of value, but since
value is always based on long-term future cash flows and depends on an assessment of
the future, short-term targets need a more immediate measure derived from actual
performance over a single year.
Economic profit is a short-term financial performance measure that is tightly linked to
value creation. It is defined as: Economic profit = Invested capital x (Return on
invested capital – Weighted average cost of capital)
Page 640

Economic profit measures the gap between what a company earns during a period and
the minimum it must earn to satisfy its investors. Maximising economic profit over
time will also maximize company value.'

c) Action plans and budgets
Then, management must translate strategy into the specific steps that an organisation
will take to achieve its targets, particularly in the short term through action plans. The
plans must identify the actions that the organisation will take so that it can pursue its
goals in a methodical manner.

d) Performance measurement
Finally performance measurement and incentive systems will track progress in
achieving targets and motivate managers and other employees to achieve them. VBM
may force a company to modify its traditional approach to these systems by linking
performance measures to long-term value creation and strategy. In particular, it shifts
performance measurement from being accounting driven to being management
driven. Key principles include:
(i)

Tailor performance measurement to the business unit. Each business unit
should have its own performance measures which it can influence.

(ii)

Link performance measurement to a unit's short- and long-term targets.
Performance measurement systems are often based almost exclusively on
accounting results.

(iii) Combine financial and operating performance in the measurement. Financial
performance is often reported separately from operating performance, whereas an
integrated report would better serve managers' needs.
(iv) Identify performance measures that serve as early warning indicators. Early
warning indicators might be simple non-financial indicators such as market share
or sales trends. Once performance measurements are an established part of
corporate culture and managers are familiar with them, it is time to revise the
compensation system.

Page 641

The pitfalls of VBM
Case Study
Value-based management is not without some problems however as the article illustrates.
'A few years ago, the chief planning officer of a large company gave us a preview of a
presentation intended for his chief financial officer and board of directors. For about two
hours we listened to details of how each business unit had been valued, complete with cash
flow forecasts, cost of capital, separate capital structures, and the assumptions underlying the
calculations of continuing value. When the time came for us to comment, we had to give the
team A+ for their valuation skills. Their methodology was impeccable. But they deserved an
F for management content. None of the company's significant strategic or operating issues
were on the table. The team had not even talked to any of the operating managers at the group
or business-unit level. Scarcely relevant to the real decision makers, their presentation was a
staff-captured exercise that would have no real impact on how the company was run. Instead
of value-based management, this company simply had value veneering. '.

Page 642

CHAPTER ROUNDUP
•

VBM aligns an organisation's overall aspirations, analytical techniques, and
management processes with the key drivers of value. So, VBM takes the idea of
creating value through return on future cash flow and embeds this in the
organisational culture in its strategy, as well as making this a performance measure to
be used throughout the organisation.

Page 643

BLANK

Page 644

STUDY UNIT 20
Performance Hierarchy
Contents

Page

Mission Statements And Vision ……………………………………..…….

647

Goals And Objectives: An Introduction …………………..………..……..

655

Corporate Objectives ………………………………………..…………….

659

Subsidiary Or Secondary Objectives ……………………….................…

663

Social And Ethical Obligations …………………………………......……

669

The Short Term And Long Term ………………….………………….…

683

The Planning Gap And Strategies To Fill It ………………….…………

685

Operational Performance …………………………………………..……

689

Planning And Control At Different Levels In The Performance ………

691

Hierarchy

Page 645

EXAM GUIDE
In the exam for this paper, you should focus on the implication of mission and objectives for
performance management. In other words, this is not, emphatically, a paper about
strategy as such.

Page 646

MISSION STATEMENTS AND VISION
Vision is oriented towards the future, to give a sense of direction to the organisation. Mission
describes an organisation's basic purpose, what it is trying to accomplish.

Underlying the behaviour and management processes of most organisations are one or two
guiding ideas, which influence the organisation's activities. Management writers typically
analyse these into two categories: vision and mission.
Mission:
Vision:

What is the business for?
Where is the business going?

Case Study
Beyond petroleum
Vision
In the early 1990s, BP articulated its 'vision' as follows:
'With our bold, innovative strategic agenda, BP will be the world's most successful oil
company in the 1990s and beyond.'
In the early 2000s, BP effectively rebranded itself, with an advertising campaign with the
strapline 'beyond petroleum'. Although petrol is the world's principal source of global
warming, BP is trying to promote an 'environmentally conscious' message, by repositioning
itself as an energy company and investing more in alternative or renewable sources of energy
such as solar power.
Arguably, the 'vision' has changed. From the stated desire to be the world's most successful
oil company, BP has moved to a state 'beyond' petroleum. The point is that as a business
progresses, its vision will need to be “updated”.
But what about mission? Mission describes the purpose of the company, in other words why
it exists at all. Many organisations interpret their mission in terms of stakeholders, typically
the owners or shareholders and customers.
BP once described itself as follows:
Page 647

'BP is a family of businesses principally in oil and gas exploration and production, refining
and marketing, chemicals and nutrition. In everything we do we are committed to creating
wealth, always with integrity, to reward the stakeholders in BP – our shareholders, our
employees, our customers and suppliers and the community.'

Case Study
The Times Online reported the result of research by Bain & Company (Bain & Company's
Management Tools 2005 survey) which found that 72% of companies have a vision or
mission statement. However the survey also reported that 88% of companies had them back
in 1993.
The survey concluded that 'They are a fashion staple which is in decline'.

Vision
A vision for the future has three aspects.
a) What the business is now
b) What it could be in an ideal world
c) What the ideal world would be like

A vision gives a general sense of direction to the company. A vision, it is hoped, enables
flexibility to exist in the context of a guiding idea.

Mission
Mission 'describes the organisation's basic function in society, in terms of the products and
services it produces for its clients'.
(Mintzberg)
Page 648

Case Study
The Co-op
The Co-operative movement (The Co-operative Group is the UKs larget mutual business –
owned by its members most of whom are customers) in the UK is a good example of the role
of mission. Its mission is not simply profit. Being owned by suppliers/customers rather than
external shareholders, it has always, since its foundation, had a wider social concern.
“The Co-op” has been criticised by some analysts on the grounds that it is insufficiently
profitable, certainly in comparison with supermarket chains such as Tesco in the UK or
Simba and Nakumatt in Rwanda. The Co-op has explicit social objectives, however. In some
cases it will retain stores which, although too small to be as profitable as a large supermarket,
provide an important social function in the communities which host them.
Of course, the Co-op's performance as a retailer can be improved, but judging it on the
conventional basis of profitability ignores its social objectives.

An expanded definition of mission includes four elements.
Elements of mission
Purpose

Strategy

Detail
Why does the company exist?
•

To create wealth for shareholders?

•

To satisfy the needs of all stakeholders (including employees,
society at large, for example)?

Mission provides the commercial logic for the company, and so
defines the following.
•

Nature of its business

•

Products/services it offers; competitive position

•

The competences and competitive advantages by which it
hopes to prosper, and its way of competing

Policies and standards
of behaviour

The mission needs to be converted into everyday performance. For
example, a firm whose mission covers excellent customer service
must deal with simple matters such as politeness to customers,
speed at which phone calls are answered and so forth.

Values and culture

Values are the basic, perhaps unstated, beliefs of the people who
work in the organisation
Page 649

Mission statements
A mission statement should be brief, flexible and distinctive, and is likely to place an
emphasis on serving the customer.

Although many organisations do not have a clearly defined mission, they are becoming
increasingly common, especially in larger organisations, and are usually set out in the form of
a mission statement. This written declaration of an organisation's central mission is a
useful concept that can:
a) Provide a ready reference point against which to make decisions
b) Help guard against there being different (and possibly misleading) interpretations of
the organisation's stated purpose
c) Help to present a clear image of the organisation for the benefit of customers and the
general public

Most mission statements will address some of the following aspects.
a) The identity of the persons for whom the organisation exists (such as shareholders,
customers and employees)
b) The nature of the firm's business (such as the products it makes or the services it
provides, and the markets it produces for)
c) Ways of competing (such as reliance on quality, innovation, technology and low
prices; commitment to customer care; policy on acquisition versus organic growth;
and geographical spread of its operations)
d) Principles of business (such as commitment to suppliers and staff; social policy, for
example, on non-discrimination or environmental issues)
e) Commitment to customers

Page 650

A number of questions need to be considered when a mission statement is being formulated.
a) Who is to be served and satisfied?
b) What need is to be satisfied?
c) How will this be achieved?

Case Study
The Financial Times reported the result of research by the Digital Equipment Corporation
into a sample of 429 company executives.
•

80% of the sample have a formal mission statement.

•

80% believed mission contributes to profitability.

•

75% believe they have a responsibility to implement the mission statement.

Mission statements might be reproduced in a number of places (at the front of an
organisation's annual report, on publicity material, in the chairman's office, in communal
work areas and so on) as they are used to communicate with those inside and outside the
organisation.
There is no standard format, but they should possess certain characteristics.
a) Brevity – easy to understand and remember
b) Flexibility – to accommodate change
c) Distinctiveness – to make the firm stand out
d) Open-ended – not stated in quantifiable terms

They tend to avoid commercial terms (such as profit) and do not refer to time frames
(some being carved in stone or etched on a plaque!).

Page 651

A mission does not have to be internally oriented. Some of the most effective focus outwards
– on customers and/or competitors. Most mission statements tend to place an emphasis on
serving the customer.

Case Study
a) Private sector organisations (such as Bank of Kigali Ltd, Bralirwa Ltd or Agro-Live)
traditionally seek to make a profit, but increasingly companies try to project other
images too, such as being environmentally friendly, being a good employer, or being a
provider of friendly service. Here's one such example.
The Bank of Kigali: Vision: Bank of Kigali aspires to be the leading provider of most
innovative financial solutions in the region.
And the mission: Our mission is to be the leader in creating value for our stakeholders
by providing the best financial services to businesses and individual customers, through
motivated and professional staff.
Public sector organisations (such as district councils, universities, colleges and
hospitals) provide services and increasingly seek to project quality, value for money,
green issues, concern for staff (equal opportunities) and so on as missions. This is
illustrated by the following examples.
For instance: 'The University of Bradford (in UK) makes knowledge work through
accessible programmes of teaching, learning and research with particular emphasis on
applied and multi-disciplinary areas of study. It aims to support students in developing
the knowledge, understanding and skills that will enable them to fulfil their intellectual
and personal potential, and thereby make a mature and critical contribution to society. It
aims to attract and retain high quality academic staff, actively engaged in teaching and
research.'
(University of Bradford)
Voluntary and community sector organisations cover a wide range of organisations
including charities, trades unions, pressure groups and religious organisations. They
usually exist either to serve a particular need or for the benefit of their membership.
Such organisations do need to raise funds but they will rarely be dedicated to the
pursuit of profit. Their mission statements are likely to reflect the particular interests
they serve (and perhaps the values of their organisation). Here are some examples.
Page 652

'We work to preserve the diversity and abundance of life on earth and the health of
ecological systems by:
(i)

Protecting natural areas and wild populations of plants and animals

(ii)

Promoting sustainable approaches to the UK of renewable natural resources

(iii) Promoting more efficient use of resources and energy and the maximum
reduction of pollution'
(The World Wide Fund for Nature)

b) The following statements were taken from annual reports of the organisations
concerned. Are they mission statements? If so, are they any good?
(i)

Before its succession of mergers, Glaxo, now GlaxoSmithKline, described itself
as 'an integrated research-based group of companies whose corporate purpose is
to create, discover, develop, manufacture and market throughout the world, safe,
effective medicines of the highest quality which will bring benefit to patients
through improved longevity and quality of life, and to society through economic
value.'

(ii)

The British Film Institute claimed 'The BFI is the UK national agency with
responsibility for encouraging and conserving the arts of film and television. Our
aim is to ensure that the many audiences in the UK are offered access to the
widest possible choice of cinema and television, so that their enjoyment is
enhanced through a deeper understanding of the history and potential of these
vital and popular art forms.'

Mission and planning
Although the mission statement might be seen as a set of abstract principles, it can play an
important role in the planning process.
a) Inspires planning. Plans should develop activities and programmes consistent with
the organisation's mission.
b) Screening. Mission also acts as a yardstick by which plans are judged.
Page 653

c) Mission also affects the implementation of a planned strategy, in the culture and
business practices of the firm.

Factors to incorporate in a mission statement
•

The business areas in which the organisation will operate

•

The organisation's reason for existence

•

The stakeholder groups served by the organisation

Page 654

GOALS AND OBJECTIVES: AN INTRODUCTION
Goals and objectives are set out to give flesh to the mission in any particular period.

Definitions
There is much confusion over the terms 'goals' and 'objectives'. Some writers use the terms
interchangeably while others refer to them as two different concepts, unfortunately with no
consistency as to which term refers to which concept.
Here we will use the following definitions/distinctions.
•
•
•

(Shorter-term) objectives are the means by which (longer-term) goals can ultimately
be achieved.
Goals are based on an individual's value system whereas objectives are based on
practical needs.
Goals are therefore more subjective than objectives.

In particular, operational goals can be expressed as quantified (SMART) objectives:
Specific, Measurable, Attainable, Results-oriented, Time-bounded
a) Mission: deliver a quality service
b) Goal: enhance manufacturing quality
c) Objectives: over the next twelve months, reduce the number of defects to 1 part per
million

Non-operational goals or aims cannot be expressed as objectives.
a) A university's goal might be to 'seek truth'. This cannot really be expressed as a
quantified objective. To 'increase truth by 5% this year' does not make a great deal of
sense.
b) Customer satisfaction is a goal, but satisfying customers and ensuring that they
remain satisfied is a continuous process that does not stop when one target has been
reached.
Page 655

In practice, most organisations set themselves quantified objectives in order to enact the
corporate mission.

Features of goals and objectives in organisations
a) Goal congruence. Goals should be consistent with each other.
(i)

Across all departments. There should be horizontal consistency. In other
words, the goals set for different parts of the organisation should be consistent
with each other.

(ii)

At all levels. Objectives should be consistent vertically, in other words at all
levels in the organisation.

(iii) Over time. Objectives should be consistent with each other over time.

b) An objective should identify the beneficiaries as well as the nature and size of the
benefit.

Types of goal, how they are developed and set
Goal

Comment

Ideological goals

These goals focus on the organisation's mission. They are
shared sets of beliefs and values.

Formal goals

These are imposed by a dominant individual or group such as
shareholders. People work to attain these goals as a route to
their personal goals.

Shared personal
goals

Individuals reach a consensus about what they want out of an
organisation (e.g. a group of academics who decide they want
to pursue research).

System goals

Derive from the organisation's existence as an organisation,
independent of mission.
Page 656

Organisations set goals in a number of different ways.
Goals can be set in many different ways: top down; bottom up; imposed; consensus; precedent.

Method

Comment

Top-down

Goals and objectives are structured from 'top to bottom', a cascading
process down the hierarchy, with goals becoming more specific the
'lower' down the hierarchy.

Bottom-up

People in individual departments set their own goals, which eventually
shape the overall goals of the organisation.

By precedent

Some goals are set simply because they have been set before (e.g. last
year's sales targets plus 5%)

By 'diktat'

A few key individuals dictate what the goals should be.

By consensus

Goals and objectives are achieved by a process of discussion amongst
managers – reputedly, Japanese companies employ this approach.

The setting of objectives is very much a political process: objectives are formulated
following bargaining by the various interested parties.
a) Shareholders want profits leading to dividends and capital growth.
b) Employees want salaries and good working conditions.
c) Managers want power.
d) Customers demand quality products and services.

These conflicting requirements make it difficult to maximise the objectives of any one
particular group. The objectives have to change over time, too, to reflect the changing
membership of the groups.

Page 657

BLANK

Page 658

CORPORATE OBJECTIVES
Corporate objectives concern the firm as a whole. Unit objectives are specific to individual
units of an organisation.

Corporate objectives are set as part of the corporate planning process. Basically, the
corporate planning process is concerned with the selection of strategies which will achieve
the corporate objectives of the organisation.

Corporate objectives versus unit objectives
Corporate objectives should relate to the key factors for business success.
a)
b)
c)
d)
e)
f)
g)
h)
i)
j)
k)

Profitability
Market share
Growth
Cash flow
Return on capital employed
Risk
Customer satisfaction
Quality
Industrial relations
Added value
Earnings per share

Similar objectives can be developed for each strategic business unit (SBU). (An SBU is a
part of the company that for all intents and purposes has its own distinct products, markets
and assets.)
Unit objectives, on the other hand, are specific to individual units of an organisation.

Page 659

Types
Commercial

Public sector

General

Examples
•

Increase the number of customers by x% (an objective of a sales
department)

•

Reduce the number of rejects by 50% (an objective of a production
department)

•

Produce monthly reports more quickly, within 5 working days of the
end of each month (an objective of the management accounting
department)

•

Introduce x% more places at nursery schools (an objective of a local
education department)

•

Respond more quickly to calls (an objective of a local police station,
fire department or hospital ambulance service)

•

Resources (e.g. cheaper raw materials, lower borrowing costs, 'topquality college graduates')

•

Market (e.g. market share, market standing)

•

Employee development (e.g. training, promotion, safety)

•

Innovation in products or processes

•

Productivity (the amount of output from resource inputs)

•

Technology

Page 660

Primary and secondary objectives
An organisation has many objectives: even a mission may have multiple parts. It has been
argued that there is a limit to the number of objectives that a manager can pursue
effectively. Too many and the manager cannot give adequate attention to each and/or the
focus may inadvertently be placed on minor ones. Some objectives are more important than
others. It has therefore been suggested that there should be one primary corporate objective
(restricted by certain constraints on corporate activity) and other secondary objectives.
These are strategic objectives which should combine to ensure the achievement of the
primary corporate objective.
a) For example, if a company sets itself a primary objective of growth in profits, it
will then have to develop strategies by which this primary objective can be achieved.
b) Secondary objectives might then be concerned with sales growth, continual
technological innovation, customer service, product quality, efficient resource
management (e.g. labour productivity) or reducing the company's reliance on debt
capital.

Page 661

BLANK

Page 662

SUBSIDIARY OR SECNDARY OBJECTIVES
Primary corporate objectives are supported by secondary objectives, for example for
product development or market share. In practice there may be a trade-off between different
objectives.
Whatever primary objective(s)is (are) set, subsidiary objectives will then be developed
beneath them.

Types of subsidiary objective
Financial
We will be considering these in the next chapter.

Technological
a) A commitment to product design and production methods using current and new
technology
b) A commitment to improve current products through research and development work
c) A commitment to a particular level of quality

Page 663

Product market
Objectives for products and
markets

Comment

Market leadership

Whether the organisation wants to be the market leader, or
number two in the market etc.

Coverage

Whether the product range needs to be expanded

Positioning

Whether there should be an objective to shift position in the
market – e.g. from producing low-cost for the mass market
to higher-cost specialist products

Expansion

Whether there should be a broad objective of 'modernising'
the product range or extending the organisation's markets

Product market objectives are key, as the organisation satisfies its shareholders by operating
in product market areas. Most major product market objectives are set at corporate level.

Others
a) Objectives for the organisation structure are particularly important for growing
organisations.
b) Productivity objectives. When an organisation is keenly aware of a poor profit
record, cost reduction will be a primary consideration. Productivity objectives are
often quantified as targets to reduce unit costs and increase output per employee by
a certain percentage each year.
c) Expansion or consolidation objectives are concerned with the question of whether
there is a need to expand, or whether there is a need to consolidate for a while.

Page 664

Ranking objectives and trade-offs
Where there are multiple objectives a problem of ranking can arise.
a) There is never enough time or resources to achieve all of the desired objectives.
b) There are degrees of accomplishment. For example, if there is an objective to
achieve a 10% annual growth in earnings per share, an achievement of 9% could be
described as a near-success. When it comes to ranking objectives, a target ROI of,
say, 25% might be given greater priority than an EPS growth of 10%.

When there are several key objectives, some might be achieved only at the expense of
others. For example, attempts to achieve a good cash flow or good product quality, or to
improve market share, might call for some sacrifice of short-term profits.

For example, there might be a choice between the following two options.
Option A

15% sales growth, 10% profit growth, a RWF10 million negative cash flow
and reduced product quality and customer satisfaction

Option B

8% sales growth, 5% profit growth, a RWF5 million surplus cash flow, and
maintenance of high product quality/customer satisfaction

If the firm chose option B in preference to option A, it would be trading off sales growth and
profit growth for better cash flow, product quality and customer satisfaction. It may feel that
the long-term effect of reduced quality would negate the benefits under Option A.
One of the tasks of strategic management is to ensure goal congruence. Some objectives may
not be in line with each other, and different stakeholders have different sets of priorities.

Page 665

Departmental plans and objectives
Implementation involves three tasks.
a) Document the responsibilities of divisions, departments and individual managers.
b) Prepare responsibility charts for managers at divisional, departmental and
subordinate levels.
c) Prepare activity schedules for managers at divisional, departmental and subordinate
levels.

Responsibility charts
Responsibility charts can be drawn up for management at all levels in the organisation,
including the board of directors. They show the control points that indicate what needs to be
achieved and how to recognise when things are going wrong. For each manager, a
responsibility chart will have four main elements.
a) The manager's major objective
b) The manager's general programme for achieving that objective
c) Sub-objectives
d) Critical assumptions underlying the objectives and the programme

Example: responsibility charts for marketing director
a) Major objective and general programme: to achieve a targeted level of sales, by
selling existing well-established products, by breaking into new markets and by a new
product launch
b) Sub-objectives: details of the timing of the product launch; details and timing of
promotions, advertising campaigns and so on
c) Critical assumptions: market share, market size, competitors' activity and so on
Page 666

Activity schedules
Successful implementation of corporate plans also means getting activities started and
completed on time. Every manager should have an activity schedule in addition to his
responsibility chart, which identifies what activities he must carry out and the start-up
and completion dates for each activity.
Critical dates might include equipment installation dates and product launch dates. In some
markets, the launch date for a new product or new model can be extremely important, with an
aim to gain maximum exposure for the product at a major trade fair or exhibition. New car
models must be ready for a major motor show, for example. If there is a delay in product
launch there might be a substantial loss of orders which the trade fair could have generated.
Consequently, to ensure co-ordination, the various functional objectives must be
interlocked.
a) Vertically from top to bottom of the business.
b) Horizontally, for example, the objectives of the production function must be linked
with those of sales, warehousing, purchasing, R&D and so on.
c) Over time. Short-term objectives can be regarded as intermediate milestones on the
road towards long-term objectives.

Hierarchy of objectives
The hierarchy of objectives which emerges is this.
Mission
Goals
Objectives
Strategy

Increasing coverage of
aspects of the organisation

Tactics
Operational plans

Page 667

Objectives are normally established within this hierarchical structure. Each level of the
hierarchy derives its objectives from the level above, so that all ultimately are founded in the
organisation's mission. Objectives therefore cascade down the hierarchy so that, for example,
strategies are established to achieve objectives and they, in turn, provide targets for the
purposes of tactical planning.

Page 668

SOCIAL AND ETHICAL OBLIGATIONS
Goals and objectives are often set with stakeholders in mind. For a business, adding value
for shareholders is a prime corporate objective, but other stakeholders need to be satisfied.
There is no agreement as to the extent of the social or ethical responsibilities of a business.

Public opinion and attitudes, and legal and political pressures, mean that organisations can
no longer concentrate solely on financial corporate objectives. Environmental and social
obligations now play a part in shaping an organisation's objectives.

Stakeholder approach
An organisation's stakeholders have a significant impact on its social and ethical obligations.

Stakeholders are groups of people or individuals who have a legitimate interest in the
activities of an organisation. They include customers, employees, the community,
shareholders, suppliers and lenders.

There are three broad types of stakeholder in an organisation.
a) Internal stakeholders (employees, management)
b) Connected stakeholders (shareholders, customers, suppliers, financiers)
c) External stakeholders (the community, government, pressure groups)

The stakeholder approach suggests that corporate objectives are, or should be, shaped and
influenced by those who have sufficient involvement or interest in the organisation's
operational activities.

Page 669

Internal stakeholders: employees and management
Because employees and management are so intimately connected with the company, their
objectives are likely to have a strong influence on how it is run. They are interested in the
following issues.
a) The organisation's continuation and growth. Management and employees have a
special interest in the organisation's continued existence.
b) Managers and employees have individual interests and goals which can be harnessed
to the goals of the organisation.
(i)

Jobs/careers

(iii) Benefits

(v)

(ii)

Money

(iv) Satisfaction

Promotion

For managers and employees, an organisation's social obligations will include the
provision of safe working conditions and anti-discrimination policies.

Connected stakeholders
Increasing shareholder value should assume a core role in the strategic management of a
business. If management performance is measured and rewarded by reference to
changes in shareholder value then shareholders will be happy, because managers are likely
to encourage long-term share price growth.

Connected stakeholder

Interests to defend

Shareholders (corporate • Increase in shareholder wealth, measured by profitability, P/E
strategy)
ratios, market capitalisation, dividends and yield
• Risk
Bankers (cash flows)

• Security of loan

(purchase • Profitable sales
• Payment for goods
Customers
(product • Goods as promised
market strategy)
Suppliers
strategy)

Page 670

• Adherence to loan agreements
• Long-term relationship
• Future benefits

Even though shareholders are deemed to be interested in return on investment and/or capital
appreciation, many want to invest in ethically-sound organisations.

External stakeholders
External stakeholder groups – the government, local authorities, pressure groups, the
community at large, professional bodies – are likely to have quite diverse objectives.

External stakeholder
Government

Interests to defend
• Jobs, training, tax revenues

Interest/pressure groups / charities / 'civil • Pollution
society'

• Rights

• Other

It is external stakeholders in particular who induce social and ethical obligations.

Social responsibility
Why should organisations play an active social role in the society within which they
function?
a) 'The public' is a stakeholder in the business. A business only succeeds because it is
part of a wider society. Giving to charity is one way of enhancing the reputation of
the business.
b) Charitable donations and artistic sponsorship are a useful medium of public
relations and can reflect well on the business.
c) Involving managers and staff in community activities is good work experience.
d) It helps create a value culture in the organisation and a sense of mission, which is
good for motivation.

Page 671

e) In the long term, upholding the community's values, responding constructively to
criticism, contributing towards community well-being might be good for business, as
it promotes the wider environment in which businesses flourish.
f) There is increasing political pressure on businesses to be socially responsible. Such
activities help 'buy off' environmentalists.

Case Study
Arriva plc
Arriva plc (Arriva was acquired by Deutsche Bahn of Germany in 2010) operated bus and
train services in the UK and Europe. It had a revenue of $1.6bn and 33,000 employees. Here
is an extract from its 2005 accounts.
'A community focus
We know that we are an important part of the communities we serve. By the very nature of
our business, we have a responsibility to these communities across the UK and mainland
Europe. For that reason, our community relations activities are at the heart of our
commitment to corporate responsibility. Our Community Relations Committee, which is
chaired by an executive director, Steve Clayton, includes representatives from across the
Group and it continues to work towards the vision we established in 2004: 'As a people
business, we value, encourage and celebrate the contribution our employees and others make
to the communities we serve.' Arriva has worked hard to establish a series of long-term
partnerships giving not only financial, but also practical support. One such partnership is with
Age Concern. As part of this developing partnership, in 2004 we supported Age Concern’s
information technology (IT) project in Hertfordshire to help older people take part in
computer training. A facility was established to help them use IT in a number of areas,
including keeping in touch with family and friends, finding out information such as bus and
train timetables and ordering shopping for home delivery. Arriva’s support has meant that
this project was able to continue throughout 2005. We have joined forces with the Wales
Deaf Rugby Union as official sponsor of the team in another successful partnership. In
addition to supporting the development of deaf rugby in communities right across Arriva’s
rail network, the partnership is also helping us to understand better the needs of people who
are hard of hearing and ensure we offer the best possible service to all our customers.

Page 672

Employees in the Community
Our employees are involved in our Community Relations Programme and vote each year for
their ‘Charity of the Year’ for the UK. This has been running for six years and provides
employees with the opportunity to select a charity that they would like the Group to support
for 12 months. Our employees voted for Cancer Research UK as the 2006 Charity of the Year
for the third year running. In the Netherlands, some of the charities we have supported during
2005 include ‘Doo een Wens’, the Dutch Make a Wish Foundation whose aim is to grant the
wishes of children with life-threatening illnesses around the world; ‘Cool Flevoland’, a youth
panel initiative which aims to improve young people’s interest in politics and social themes,
and ‘Dance4Life’ which has been set up by young people to fight the spread of Aids. Many
of our employees across the UK and mainland Europe make valuable contributions to their
local communities outside of their working life with Arriva. We value this and we recognise
many of their efforts through our Community Action Awards. Employees are encouraged to
tell us about their charitable work. In return, they are put forward for consideration of an
award, which is donated to the organisation they support. In 2005, we presented 57 cash
awards to our employees for their chosen causes.

Business in the Community
We are a national member of Business in the Community (BITC) and we actively support
some of its initiatives. BITC is a charitable organisation which helps businesses contribute to
the social and economic regeneration of local communities. During 2005, Arriva employees
from the north east of England took part in a reading programme with a local primary school,
which selected pupils it felt would benefit from extra support. They listened to the children
read and helped with the reading process. The project proved enjoyable and worthwhile for
both the children and volunteers. We were also named as one of BITC’s top ten overall
performers in the Race for Opportunity awards. This recognises efforts that organisations
make to ensure that their workforce is diverse and that differences are valued and
understood.'

There are three contrasting views about a corporation's responsibilities.
a) If the company creates a social problem, it must fix it (eg Exxon (see below)).
b) The multinational corporation has the resources to fight poverty, illiteracy, malnutrition,
illness and so on. This approach disregards who actually creates the problem.

Page 673

Case Study
Such an approach dates back to Henry Ford, who said 'I do not believe that we should make
such an awful profit on our cars. A reasonable profit is right, but not too much. So it has been
my policy to force the price of the car down as fast as production would permit, and give the
benefits to the users and the labourers, with surprisingly enormous benefits to ourselves.'

c) Companies already discharge their social responsibility, simply by increasing their
profits and thereby contributing more in taxes. If a company was expected to divert
more resources to solve society's problems, this would represent a double tax.

The social audit
Social audits involve five key elements.
a) Recognising a firm's rationale for engaging in socially responsible activity
b) Identification of programmes which are congruent with the mission of the company
c) Determination of objectives and priorities related to this programme
d) Specification of the nature and range of resources required
e) Evaluation of company involvement in such programmes past, present and future

Whether or not a social audit is used depends on the degree to which social responsibility is
part of the corporate philosophy.

Case Study
In the USA, social audits on environmental issues have increased since the Exxon Valdez
catastrophe in 1989 in which millions of gallons of crude oil were released into Alaskan
waters.
Page 674

Conclusion
The importance of corporate social responsibility reporting is evident from facts included in
'How to be good' by Cathy Hayward (Financial Management, October 2002).
a) A Mori poll has found that [92 per cent of the British public believe that multinational
companies should meet the highest human health, animal welfare and environmental
standards wherever they are operating, and] that almost 90 per cent of British people
believe the government should protect the environment, employment conditions and
health even when this conflicts with the interests of multinationals.
b) A survey by the National Union of Students in UK has shown that more than 75 per
cent of student jobseekers would not work for an "ethically unsound" employer.'

Ethics and ethical conduct
Whereas social responsibility deals with the organisation's general stance towards society,
and affects the activities the organisation chooses to do, ethics relates far more to how an
organisation conducts individual transactions.
We previously looked at the ethical issues that might impact on drawing up strategic plans
as well as affecting performance. In this chapter we consider how ethical obligations should
be considered when pursuing corporate objectives.
Organisations are coming under increasing pressure from a number of sources to behave
more ethically.
a)
b)
c)
d)
e)
f)

Government
UK and European legislation
Treaty obligations (such as the Kyoto Summit)
Consumers
Employers
Pressure groups

Page 675

These sources of pressure expect an ethical attitude towards the following.
a)
b)
c)
d)
e)

Stakeholders
Animals
Green issues (such as pollution and the need for re-cycling)
The disadvantaged
Dealings with unethical companies or countries

A clear example of unethical conduct is bribery.
a) In some countries, government officials routinely demand bribes, to supplement their
meagre incomes. For example customs officials demand 'commission' before
releasing documentation enabling goods to move from the warehouse.
b) More serious bribes occur when companies bid for large public sector contracts, and
pay substantial amounts to politicians and key decision makers.

The boundary between dubious ethics and criminality has shifted over the years, particularly
with increased standards of corporate governance. Insider dealing, whereby individuals
benefit from unpublished information which may affect the share price, used to be normal
practice (a perk of working in stock broking); now it is a crime.

Attitudes to corporate ethics
Reidenbach and Robin usefully distinguish between five different attitudes to corporate
ethics. The following is an adapted version of a report in the Financial Times.

a) Amoral organisations
Such organisations are prepared to condone any actions that contribute to the
corporate aims (generally the owner's short-term greed). Getting away with it is the
only criterion for success. Getting caught will be seen as bad luck. In a nutshell, there is
no set of values other than greed. Obviously, this company gets away without a
written code.

Page 676

b) Legalistic organisations
Such organisations obey the letter of the law but not necessarily the spirit of it, if
that conflicts with economic performance. Ethical matters will be ignored until they
become a problem. Frequent problems would lead to a formal code of ethics that says,
in effect, 'Don't do anything to harm the organisation'.

c) Responsive companies
These organisations take the view – perhaps cynically, perhaps not – that there is
something to be gained from ethical behaviour. It might be recognised, for example,
that an enlightened attitude towards staff welfare enabled the company to attract and
retain higher calibre staff. If such a company has a formal code of ethics it will be one
that reflects concern for all stakeholders in the business.

d) Emerging ethical (or 'ethically engaged') organisations
They take an active (rather than a reactive) interest in ethical issues.
'Ethical values in such companies are part of the culture. Codes of ethics are action
documents, and contain statements reflecting core values. A range of ethical support
measures are normally in place, such as ethical review committees; hotlines; ethical
audits; and ethics counsellors or ombudsmen.
Problem solving is approached with an awareness of the ethical consequence of an
action as well as its potential profitability, and pains are taken to uphold corporate
values.'

e) Ethical organisations
These organisations have a 'total ethical profile': a philosophy that informs everything
that the company does and a commitment on the part of everyone to carefully selected
core values.

Page 677

Case Study
The Co-operative Bank (a mutual organisation and part of the Co-operative Group in the
UK), which has a strong record of ethical reporting, publishes a partnership report. This is an
independently-audited ethical and ecological health check that considers how the bank is
meeting its obligations to customers, employees and their families, members, suppliers, local
communities, national and international society and past and future generations.
According to an article in Financial Management ('How to be good,' Cathy Hayward,
October 2002), the ethical and ecological positioning of the Co-operative bank contributed
more than £20 million, or 20 per cent, of its profits in 2001. Almost a third of its current
account customers (the bank's key market) were with the bank primarily because of its ethical
policies, according to a survey.

Corporate codes and corporate culture
Case Study
The following extraxct is from a UK newspaper but is a good illustration:
'Shocking tactics including bribery, fabrication and plagiarism are being used by
unscrupulous drug companies to get their research published in influential medical journals,
according to a damning new report.
Only a week after controversial research on the MMR (MMR = Measles, Mumps and
Rubella) vaccine was discredited by the journal which published it following a 'fatal conflict
of interest', an influential committee has revealed the widespread use of underhand tactics by
researchers.
And the Committee on Public Ethics (COPE) is urging the editors of scientific journals to
sign up to a new code of conduct aimed at preventing conflicts of interest and mistakes in
published research.
The right kind of report appearing in a scientific journal can be worth millions of pounds for
drug companies and manufacturers of medical equipment. In addition, industries such as
tobacco, alcohol and health, and even junk foods, hope to cite studies masquerading as
independent research in support of their products.
Articles in scientific journals can also be used to knock out competitors or quieten down
public concern about controversial issues.
Page 678

In one of the cases cited by the report a journal published a paper on passive smoking in
which the authors failed to declare financial support from the tobacco industry.
On another occasion, a high-ranking government official telephoned an editor in an attempt
to stop an article being written which was critical of some government research.
In one of the most disturbing examples, a team of scientists was forced to withdraw a paper
after refusing to detail how they had been allowed to analyse blood taken from babies.
Tactics have also included bribery. On one occasion an editor was telephoned by a
representative who said she would guarantee to buy 1,000 reprints if the journal would
continue to consider for publication a study that conflicted with a policy the journal had just
introduced. 'And I will buy you a dinner at any restaurant you choose,' added the
representative.
Plagiarism is also seen as a growing problem in scientific papers, especially where Internet
search engines allow researchers to cull information from other papers.
A spectacular example saw authors copy another study almost wholesale, but change the
number of patients, the type of surgery, the regimen of one of the drugs and add data for
another drug.
‘If we don’t have strict ethics, how can we trust medical research?’
The revelations follow the controversy surrounding Dr Andrew Wakefield’s work on the
MMR vaccine, published in the Lancet in 1998. It emerged 10 days ago that Wakefield had
been looking for evidence to support a legal action by parents claiming the vaccine had
harmed their children at the same time.
The research, linking MMR to autism in children, sparked a major controversy and led to
large numbers of parents refusing to allow their children to be treated with the jab.
Lancet editor Dr Richard Horton said the situation represented a 'fatal conflict of interest' and
called into question whether Wakefield’s paper should have been published.
His comments came as the General Medical Council prepared to open an investigation into
the way Wakefield carried out his study.
Last week, Sir Liam Donaldson, England’s chief medical officer, accused Wakefield of
peddling 'poor science'. Wakefield has insisted he has done nothing wrong and says the
science behind his study still stands.
COPE, which represents more than 178 editors around the world, is now publishing a guide
for the editors of science publications. It puts them under an obligation to ensure the accuracy
Page 679

of the material they publish, maintain scientific integrity and ensure business needs do not
compromise intellectual standards.
Editors are also asked to be willing to correct, clarify and retract material or issue apologies
when necessary.
The new code warns them that they can be held responsible for publishing 'unethical'
research. It advises that systems should be in place for managing conflicts of interest relating
to journal editors, their staff, authors and the reviewers who vet papers before they are
published.
The code was drafted by Dr Richard Smith, editor of the British Medical Journal - the
Lancet’s main rival.
He said: 'Like everybody else we are much more interested in other people’s accountability
than we are our own. Editors are peculiarly unaccountable because of their traditions of
editorial freedom, and there are no bodies that attempt to regulate medical and scientific
editors.'
A spokeswoman for the Medical Research Council said: 'We in no way ever approve of such
tactics to get research approved. We expect scientists to abide by the highest ethical
standards in their work.'
Bill O’ Neill, the Scottish secretary of the British Medical Association, said: 'I don’t think
anyone would question the need for strict rules. If we don’t have rigorous ethics then how
can we trust research?'
Source: Scotsman News, March 2004

Many commentators would argue that the introduction of a code of ethics is inadequate on
its own. To be effective a code needs to be accompanied by positive attempts to foster
guiding values, aspirations and patterns of thinking that support ethically sound
behaviour – in short a change of culture.
Increasingly organisations are responding to this challenge by devising ethics training
programmes for the entire workforce, instituting comprehensive procedures for reporting
and investigating ethical concerns within the company, or even setting up an ethics office
or department to supervise the new measures. About half of all major companies now have a
formal code of some kind.

Page 680

Lynne Paine (Harvard Business Review, March – April 1994) suggests that ethical decisions
are becoming more important as, in the US at least, penalties for companies which break the
law are becoming tougher. Paine describes two approaches to the management of ethics in
organisations.
a) A compliance-based approach is primarily designed to ensure that the company and
its personnel act within the letter of the law. Mere compliance is not an adequate
means for addressing the full range of ethical issues that arise every day.
b) An integrity-based approach combines a concern for the law with an emphasis on
managerial responsibility for ethical behaviour. When integrated into the day-to-day
operations of an organisation, such strategies can help prevent damaging ethical
lapses.

It would seem to follow that the imposition of social and ethical responsibilities on
management should come from within the organisation itself, and that the organisation
should issue its own code of conduct for its employees.

Code of conduct
A corporate code typically contains a series of statements setting out the company's
values and explaining how it sees its responsibilities towards stakeholders.

The impact of a corporate code
A code of conduct can set out the company's expectations, and in principle a code may
address many of the problems that the organisations may experience. However, merely
issuing a code is not enough.
a) The commitment of senior management to the code needs to be real, and it needs to
be very clearly communicated to all staff. Staff need to be persuaded that expectations
really have changed.
b) Measures need to be taken to discourage previous behaviours that conflict with the
code.

Page 681

c) Staff need to understand that it is in the organisation's best interests to change
behaviour, and become committed to the same ideals.
d) Some employees – including very able ones – may find it very difficult to buy into a
code that they perceive may limit their own earnings and/or restrict their freedom to
do their job.
e) In addition to a general statement of ethical conduct, more detailed statements
(codes of practice) will be needed to set out formal procedures that must be followed.

Case Study
An extract from Google’s Code of Conduct
'Our informal corporate motto is 'Don't be evil'. We Googlers generally relate those words to
the way we serve our users – as well we should. But being 'a different kind of company'
means more than the products we make and the business we're building; it means making
sure that our core values inform our conduct in all aspects of our lives as Google employees.
The Google Code of Conduct is the code by which we put those values into practice. This
document is meant for public consumption, but its most important audience is within our own
walls. This code isn't merely a set of rules for specific circumstances but an intentionally
expansive statement of principles meant to inform all our actions; we expect all our
employees, temporary workers, consultants, contractors, officers and directors to study these
principles and do their best to apply them to any and all circumstances which may arise.
The core message is simple: being Googlers means striving toward the highest possible
standard of ethical business conduct. This is a matter as much practical as ethical; we hire
great people who work hard to build great products, but our most important asset by far is our
reputation as a company that warrants our users' faith and trust. That trust is the foundation
upon which our success and prosperity rests, and it must be re-earned every day, in every
way, by every one of us.
So please do read this code, and then read it again, and remember that as our company
evolves, The Google Code of Conduct will evolve as well. Our core principles won't change,
but the specifics might, so a year from now, please read it a third time. And always bear in
mind that each of us has a personal responsibility to do everything we can to incorporate
these principles into our work, and our lives.'

Page 682

THE SHORT TERM AND LONG TERM
The S/L trade-off refers to the balance of organisational activities aiming to achieve longterm and short-term objectives when they conflict or where resources are scarce.

Long-term and short-term objectives
Objectives may be long term and short term.
a) For example, a company's primary objective might be to increase its earnings per
share from RWF300 to RWF500 in the next five years. A number of strategies for
achieving the objective might then be selected.
(i)

Increasing profitability in the next twelve months by cutting expenditure

(ii)

Increasing export sales over the next three years

(iii) Developing a successful new product for the domestic market within five years

b) Secondary objectives might then be re-assessed to include the following.
(i)

The objective of improving manpower productivity by 10% within twelve
months.

(ii)

Improving customer service in export markets with the objective of doubling the
number of overseas sales outlets in selected countries within the next three years.

(iii) Investing more in product-market research and development, with the objective
of bringing at least three new products to the market within five years.

Targets cannot be set without an awareness of what is realistic. Quantified targets for
achieving the primary objective, and targets for secondary objectives, must therefore emerge
from a realistic 'position audit'.

Page 683

Trade-offs between short-term and long-term objectives
Just as there may have to be a trade-off between different objectives, so too might there be a
need to make trade-offs between short-term objectives and long-term objectives. This is
referred to as S/L trade-off.

The S/L trade-off refers to the balance of organisational activities aiming to achieve long
term and short-term objectives when they are in conflict or where resources are scarce.

Some decisions involve the sacrifice of longer-term objectives.
a) Postponing or abandoning capital expenditure projects, which would eventually
contribute to growth and profits, in order to protect short term cash flow and profits.
b) Cutting R&D expenditure to save operating costs, and so reducie the prospects for
future product development.
c) Reducing quality control, to save operating costs (but also adversely affect reputation
and goodwill).
d) Reducing the level of customer service, to save operating costs (but sacrifice goodwill).
e) Cutting training costs or recruitment (so the company might be faced with skills
shortages).

Steps that could be taken to control S/L trade-offs, so that the 'ideal' decisions are taken,
include the following.
a) Making short-term targets realistic. If budget targets are unrealistically tough, a
manager will be forced to make S/L trade-offs.
b) Providing sufficient management information to allow managers to see what tradeoffs they are making. Managers must be kept aware of long-term aims as well as
shorter-term (budget) targets.
c) Evaluating managers' performance in terms of contribution to long-term as well as
short-term objectives.

Page 684

THE PLANNING GAP AND STRATEGIES TO FILL
IT
Forecasts based on current performance may reveal a gap between the firm's objectives and
the likely outcomes. New strategies (e.g. market penetration, market development, product
development, diversification, withdrawal) are developed to fill the gap.
Strategic planners need to consider the extent to which new strategies are needed to enable
the organisation to achieve its objectives. One technique whereby this can be done is gap
analysis.

Gap analysis
Gap analysis involves comparing an organisation's ultimate objective (most commonly
expressed in terms of demand, but may be reported in terms of profit, ROCE and so on) and
the expected performance of planned and current projects.

Purpose of gap analysis
a) Determine the organisation's targets for achievement over the planning period
b) Establish what the organisation would be expected to achieve if it 'did nothing' (did
not develop any new strategies, but simply carried on in the current way with the
same products and selling to the same markets)
This difference is the 'gap'. New strategies will then have to be developed which will close
this gap, so that the organisation can expect to achieve its targets over the planning period.
We cover these strategies below.

Page 685

The planning gap is not the gap between the current position of the organisation and the
forecast desired position.
Rather, it's the gap between the forecast position from continuing with current activities, and
the forecast of the desired position.

A forecast based on doing nothing will probably provide an unrealistic estimate of future
performance, but it is useful.
a) The forecast is used to determine the requirement for new strategies and so it must
exclude such strategies.
b) Including the impact of strategies of which the organisation has little or no experience
will produce an even more inaccurate forecast.
c) It reduces the complexity involved in the forecasting exercise.
d) It provides an assessment of what could be achieved without taking on new risk.

Forecasts must cover a period far enough into the future to reveal any significant gap. How
far ahead an organisation needs to plan, however, will depend on the lead time for
corrective action to take effect, which in turn depends on the nature of the organisation's
business and the type of action required.

Closing the gap with product-market strategies
A product-market strategy considers the mix of products and markets. The aim of such
strategies is to close the gap found by gap analysis.
Product-market mix is a short hand term for the products/services a firm sells (or a service
which a public sector organisation provides) and the markets it sells them to.

Page 686

Product-market mix: Ansoff's growth vector matrix
The Ansoff matrix identifies various options.
•
•
•
•

Market penetration: current products, current markets
Market development: current products, new markets
Product development: new products, current markets
Diversification: new products, new markets

All of these can secure growth.
Igor Ansoff drew up a growth vector matrix, describing how a combination of a firm's
activities in current and new markets, with existing and new products, can lead to growth.
Ansoff's original model was a four cell matrix based on product and market.
We look at Ansoff's matrix in more detail in Study Unit 21. All you need to know here is that
the matrix is used to indicate strategies for closing the gap found by gap analysis.

Closing the profit gap and product-market strategy

RWF Profit

The aim of product-market strategies is to close the profit gap that is found by gap analysis.
A mixture of strategies may be needed to do this.

Page 687

It is worth remembering that divestment is a product-market option to close the profit gap, if
the business is creating losses.
A related question is what do you do with spare capacity – go for market penetration, or go
into new markets. Many companies begin exporting into new overseas markets to use surplus
capacity.
The strategies in the Ansoff matrix are not mutually exclusive. A firm can quite legitimately
pursue a penetration strategy in some of its markets, while aiming to enter new markets.

Page 688

OPERATIONAL PERFORMANCE
Operations can make or break strategies. They are directly focused on value-adding
activities.
The Study Guide requires you to 'identify and discuss the characteristics of operational
performance'.
Operations are the day to day activities that are carried out in order to achieve specific targets
and objectives. Here are some examples.

Sector

Operations carried out by …

Fast-food

Staff employed at a McDonald's checkout

Bank

Dealer on the Forex markets

Law

Solicitor finalising the details of a contract for a client

Call centre

People hosting the switch board

Media

TV camera-person, or presenter
Website construction

Manufacturing

Assembly line worker

Construction

Building site operations

These examples show that operations are directly focused on activities which immediately
add value to the customer.
Unlike strategy, which involves taking decisions, operations have the following
characteristics.
a) Customer-facing, in service industries
b) Specialised, as the tasks are closely defined
c) More likely to be routine, but this is not true of all 'operations'
d) Limited in scope
e) Characterised by short time horizons
f) Easier to automate than some management tasks

Page 689

The significance of operations
a) Many operational activities require expert or specialised skills – such as surgery.
b) Operations can be areas of significant risk for a company and its customers.
c) Operations are 'moments of truth' between the firm and its customers. A company's
reputation can be made or broken by the quality of its goods and services, which are
determined by operational quality and consistency.
d) The operational infrastructure comprises the most significant element of cost for
most businesses.
e) The most well-designed strategy can be destroyed by poor implementation at
operational level.
f) Operations and the deployment of operational activities are a key determinant of
organisation structure.

Page 690

PLANNING AND CONTROL AT DIFFERENT
LEVELS IN THE PERFORMANCE HIERARCHY
Planning and control occurs at all levels of the performance hierarchy to different degrees.

Planning
Although it implies a 'top down' approach to management, we could describe a cascade of
goals, objectives and plans down through the layers of the organisation. The plans made at
the higher levels of the performance hierarchy provide a framework within which the plans
at the lower levels must be achieved. The plans at the lower levels are the means by which
the plans at the higher levels are achieved.
It could therefore be argued that without the plans allied directly to the vision and corporate
objective the operational-level and departmental plans have little meaning. Planning could
therefore be deemed as more significant at the higher levels of the performance hierarchy
than the lower levels.
This is not to say that planning at an operational level is not important. It is just that the two
types of planning are different.

Level
Corporate
plans

Operational
plans

Detail
•
•
•
•
•

Focused on overall performance
Environmental influence
Set plans and targets for units and departments
Sometimes qualitative (e.g. a programme to change the culture of the
organisation)
Aggregate

•
•
•
•
•
•
•

Based on objectives about 'what' to achieve
Specific (e.g. acceptable number of 'rings' before a phone is answered)
Little immediate environmental influence
Likely to be quantitative
Detailed specifications
Based on 'how' something is achieved
Short time horizons

Page 691

Control
Consider how the activities of planning and control are inter-related.
a) Plans set the targets.
b) Control involves two main processes.
(i)

Measure actual results against the plan.

(ii)

Take action to adjust actual performance to achieve the plan or to change the plan
altogether.

Control is therefore impossible without planning.
The essence of control is the measurement of results and comparing them with the original
plan. Any deviation from plan indicates that control action is required to make the results
conform more closely with plan.

Feedback
Feedback occurs when the results (outputs) of a system are used to control it, by adjusting
the input or behaviour of the system.

A business organisation uses feedback for control.
a) Negative feedback indicates that results or activities must be brought back on course,
as they are deviating from the plan.
b) Positive feedback results in control action continuing the current course. You would
normally assume that positive feedback means that results are going according to plan
and that no corrective action is necessary: but it is best to be sure that the control
system itself is not picking up the wrong information.
c) Feedforward control is control based on forecast results: in other words if the
forecast is bad, control action is taken well in advance of actual results.

Page 692

There are two types of feedback.
a) Single loop feedback is control, like a thermostat, which regulates the output of a
system. For example, if sales targets are not reached, control action will be taken to
ensure that targets will be reached soon. The plan or target itself is not changed, even
though the resources needed to achieve it might have to be reviewed.
b) Double loop feedback is of a different order. It is information used to change the plan
itself. For example, if sales targets are not reached, the company may need to change
the plan.

Control at different levels
You might think that control can only occur at the lower-levels of the performance hierarchy,
as that is the type of control you have encountered in your studies to date (standard costing,
budgetary control). Such control has the following features.
a) Exercised externally by management or, in the case of empowered teams, by the staff
themselves
b) Immediate or rapid feedback
c) Single loop feedback (i.e. little authority to change plans or targets)

Control does occur at the higher-levels of the hierarchy, however, and has the following
characteristics.
a) Exercised by external stakeholders (e.g. shareholders)
b) Exercised by the market
c) Double loop feedback (i.e. relatively free to change targets)
d) Often feedforward elements

Summary
The best way to envisage the differences is by two case examples.
Page 693

Case Study
a) Call centres
Staff who in call centres are subject to precise controls and targets.
(i)

The longest time a phone should ring before it is answered

(ii)

Speed of dealing with the caller's query

(iii) Rehearsal of a 'script', or use of precise responses or prompts from software
Staff who take too long dealing with queries may be counselled or dismissed.
The targets are precisely and exactly linked to the service provided and provide
rapid feedback. Control and planning is exercised over the process of delivery.
b) Senior management
Senior management initiate the planning process, but their time is planned to a far less
rigid degree than people at operational level.
For example, the Chief Executive of Network Rail in the UK is responsible to
shareholders but, given the nature of the industry and its reliance on UK government
subsidies, must also be accountable to other stakeholders. The market is mainly
concerned with results. Controls over corporate governance – over how the company is
run – are mainly to do with ensuring the transparency and integrity of the governance
process.

Page 694

CHAPTER ROUNDUP
•

Vision is oriented towards the future, to give a sense of direction to the organisation.
Mission describes an organisation's basic purpose, what it is trying to accomplish.

•

A mission statement should be brief, flexible and distinctive, and is likely to place an
emphasis on serving the customer.

•

Goals and objectives are set out to give flesh to the mission in any particular period.

•

Goals can be set in many different ways: top down; bottom up; imposed; consensus;
precedent.

•

Corporate objectives concern the firm as a whole. Unit objectives are specific to
individual units of an organisation.

•

Primary corporate objectives are supported by secondary objectives, for example
for product development or market share. In practice there may be a trade-off between
different objectives.

•

Goals and objectives are often set with stakeholders in mind. For a business, adding
value for shareholders is a prime corporate objective, but other stakeholders need to
be satisfied. There is no agreement as to the extent of the social or ethical
responsibilities of a business.

•

The S/L trade-off refers to the balance of organisational activities aiming to achieve
long-term and short-term objectives when they conflict or where resources are scarce.

•

Forecasts based on current performance may reveal a gap between the firm's
objectives and the likely outcomes. New strategies (eg market penetration, market
development, product development, diversification, withdrawal) are developed to fill
the gap.

•

A product-market strategy considers the mix of products and markets. The aim of
such strategies is to close the gap found by gap analysis.

•

The Ansoff matrix identifies various options.
–
–
–
–

Market penetration: current products, current markets
Market development: current products, new markets
Product development: new products, current markets
Diversification: new products, new markets

Page 695

All of these can secure growth.
•

Operations can make or break strategies. They are directly focused on value-adding
activities.

•

Planning and control occurs at all levels of the performance hierarchy to different
degrees.

Page 696

STUDY UNIT 21
Competitive Strategies
Contents

Page

Strategic Models Used In Planning And Assessing Business ……………

699

Performance

Criticisms Of Performance Indicators …………………….………..……..

Page 697

713

EXAM GUIDE
The use of decision-making techniques to enhance performance is a core area of the syllabus.
This chapter explains three models that could well be tested as a part of a longer question.
You could also be asked to discuss problems in performance measurement, as part of a
question. This is likely to be in response to a scenario given in a question.

Exam Focus Point
In the past, examiners have commented on how students do not answer the question set. A
common fault is candidates writing all they know on a particular topic without applying that
knowledge to the question set. You must make sure that you know the material in this chapter
well so that you can select which models to evaluate when answering a question. This advice
also applies in deciding what problems to discuss when considering a scenario that you are
given to analyse.

Page 698

STRATEGIC MODELS USED IN PLANNING AND
ASSESSING BUSINESS PERFORMANCE
In this section, we review three strategic models that aid the formulation of strategy and the
appraisal of business performance. Each model looks at a different aspect of the business
environment in which a business operates.
a) Porter’s Five forces considers the sources of competition in an industry or sector.
b) The Boston Consulting Group matrix helps management assess products, services
and strategic business units in terms of their market potential. This is measured in
terms of market share and market growth and can therefore suggest the attractiveness
of entering or remaining in an industry or sector.
c) The Ansoff growth vector matrix uses a matrix consisting of new or existing
products and/or markets to generate possible strategies to use to achieve growth.

Remember that the models are useful but they have limitations. Some of these are mentioned
below so bear these in mind when you evaluate their usefulness in performance management
and apply them to particular scenarios.

Porter's Five Forces Model
Porter's Five Forces Model suggests the importance of pressure from five competitive
forces on profit.

a) Threat of new entrants (which will be affected by barriers to entry and expected
reaction from existing firms).
b) Threat of substitutes (which will be determined by the level of innovation of
existing producers, the ability of existing competitors to finance responses to the
threat and the propensity of buyers to substitute).
c) Bargaining power of buyers (which will be linked to the number of buyers).
d) Bargaining power of suppliers (supplier power and the impact on costs being greater
when there are fewer of them).

Page 699

e) Rivalry between existing competitors (the strength of rivalry being determined by
number of competitors, market power, brand identity, producer differences cost
structure and so on).
The threat of new entrants (and barriers to entry to keep them out)
A new entrant into an industry will bring extra capacity and more competition. The strength
of this threat is likely to vary from industry to industry and depends on two things.
a) The strength of the barriers to entry. Barriers to entry discourage new entrants.
b) The likely response of existing competitors to the new entrant.

Barriers to entry
a) Scale economies. High fixed costs often imply a high breakeven point, and a high
breakeven point depends on a large volume of sales. If the market as a whole is not
growing, the new entrant has to capture a large slice of the market from existing
competitors. This is expensive (although Japanese companies have done this in some
cases).
b) Product differentiation. Existing firms in an industry may have built up a good
brand image and strong customer loyalty over a long period of time. A few firms may
promote a large number of brands to crowd out the competition.
c) Capital requirements. When capital investment requirements are high, the barrier
against new entrants will be strong, particularly when the investment would possibly
be high-risk.
d) Switching costs. Switching costs refer to the costs (time, money, convenience) that a
customer would have to incur by switching from one supplier’s products to another’s.
Although it might cost a consumer nothing to switch from one brand of frozen peas
to another, the potential costs for the retailer or distributor might be high.
e) Access to distribution channels. Distribution channels carry a manufacturer’s
products to the end-buyer. New distribution channels are difficult to establish, and
existing distribution channels hard to gain access to.
f) Cost advantages of existing producers, independent of economies of scale include:
(i) Patent rights
(ii) Experience and know-how (the learning curve)
(iii) Government subsidies and regulations
Page 700

(iv) Favoured access to raw materials

Case Study
Japanese firms
A little while ago, it was assumed that, following the success of Japanese firms worldwide in
motor vehicles (Nissan, Honda, Toyota) and consumer electronics (e.g. Sony, JVC,
Matsushita), no Western companies were safe from Japanese competition. Kao (household
goods), Suntory (drinks), Nomura (banking and securities) were seen as successors to firms
such as Procter and Gamble and Heineken.
This has not happened: for example, Japanese pharmaceutical firms, such as Green Cross,
have not achieved the world domination (anticipated in 1982). US and European firms are
still dominant in this industry.
Perhaps cars and consumer electronics are the exception rather than the rule. The reason for
this might be distribution. Normally, outsiders do not find it easy to break into established
distribution patterns. However, distribution channels in cars and consumer electronics offered
outsiders an easy way in.

a) The car industry is vertically integrated, with a network of exclusive dealerships.
Given time and money, the Japanese firms could simply build their own dealerships
and run them as they liked, with the help of local partners. This barrier to entry was
not inherently complex.
b) Consumer electronics
(i)

In the early years, the consumer electronics market was driven by technology, so
innovative firms such as Sony and Matsushita could overcome distribution
weaknesses with innovative products, as they had plenty to invest. This lowered
entry barriers.

(ii)

Falling prices changed the distribution of hifi goods from small specialist shops to
large cut-price outlets. Newcomers to a market are the natural allies of such new
outlets: existing suppliers prefer to shun 'discount’ retailers to protect margins in
their current distribution networks.

Page 701

Japanese firms have not established dominant positions in:
a) Healthcare, where national pharmaceuticals wholesalers are active as 'gatekeepers’
b) Household products, where there are strong supermarket chains
c) Cosmetics, where department stores and specialist shops offer a wide choice.

Entry barriers might be lowered by the impact of change:
a) Changes in the environment
b) Technological changes
c) Novel distribution channels for products or services

The threat from substitute products
A substitute product is a good or service produced by another industry which satisfies the
same customer needs.

Case Study
Supermarkets
The major supermarket chains in the UK are all able to provide substitutes for most of the
products stocked by the other chains. This means that they must keep prices competitive with
each other.
Supermarkets have also expanded into products offered by specialist retailers such as
electrical goods and books. This means these retailers also have substitutes for their products
and must keep their prices linked to those of the supermarkets for equivalent products.

Page 702

The bargaining power of buyers

Customers want better quality products and services at a lower price. Satisfying this want
might force down the profitability of suppliers in the industry. Just how strong the position of
customers will be depends on a number of factors.
a) How much the customer buys
b) How critical the product is to the customer’s own business
c) Switching costs (i.e. the cost of switching supplier)
d) Whether the products are standard items (hence easily copied) or specialised
e) The customer’s own profitability: a customer who makes low profits will be forced to
insist on low prices from suppliers
f) Customer’s ability to bypass the supplier (or take over the supplier)
g) The skills of the customer purchasing staff, or the price-awareness of consumers
h) When product quality is important to the customer, the customer is less likely to be
price-sensitive, and so the industry might be more profitable as a consequence

Case Study
Jewellery
The market for high end jewellery is one where the customer is concerned with quality ahead
of price. Customers do set themselves price limits but they are more concerned with
reputation and the implied worth to the recipient of the jewellery.

The bargaining power of suppliers
Suppliers can exert pressure for higher prices. The ability of suppliers to get higher prices
depends on several factors.
a) Whether there are just one or two dominant suppliers to the industry, able to charge
monopoly or oligopoly prices
Page 703

b) The threat of new entrants or substitute products to the supplier’s industry
c) Whether the suppliers have other customers outside the industry, and do not rely on
the industry for the majority of their sales
d) The importance of the supplier’s product to the customer’s business
e) Whether the supplier has a differentiated product which buyers need to obtain
f) Whether switching costs for customers would be high

Case Study
De Beers and the diamond trade
De Beers established a near monopoly over the supply of diamonds to the diamond trade
from the 1930's the beginning of the 21st century. During the twentieth century, De Beers
sold between 85% and 90% of the diamonds mined worldwide. Diamond dealers
traditionally had to source their rough diamonds from De Beers. Prices were kept high and
supply was rationed. In fact, diamonds are not rare as there are diamond mines in many
countries including Canada and Australia.
In July 2004 De Beers pleaded guilty in a US court to price fixing and had to pay a $10m
fine. One rival, the Lev Leviev Group, decided to invest in its own diamond mining
operations, thereby bypassing De Beers entirely.
Source: various including BBC website

The rivalry amongst current competitors in the industry
The intensity of competitive rivalry within an industry will affect the profitability of the
industry as a whole. Competitive actions might take the form of price competition,
advertising battles, sales promotion campaigns, introducing new products for the market,
improving after sales service or providing guarantees or warranties. Competition can
stimulate demand, expanding the market, or it can leave demand unchanged, in which case
individual competitors will make less money, unless they are able to cut costs.

Page 704

Factors determining the intensity of competition
a) Market growth. Rivalry is intensified when firms are competing for a greater market
share in a total market where growth is slow or stagnant.
b) Cost structure. High fixed costs are a temptation to compete on price, as in the short
run any contribution from sales is better than none at all. A perishable product
produces the same effect.
c) Switching. Suppliers will compete if buyers can switch easily (e.g. Coke vs. Pepsi).
d) Capacity. A supplier might need to achieve a substantial increase in output capacity,
in order to obtain reductions in unit costs.
e) Uncertainty. When one firm is not sure what another is up to, there is a tendency to
respond to the uncertainty by formulating a more competitive strategy.
f) Strategic importance. If success is a prime strategic objective, firms will be likely to
act very competitively to meet their targets.
g) Exit barriers make it difficult for an existing supplier to leave the industry. These
can take many forms.
(i)

Non-current assets with a low break-up value (e.g. there may be no other use
for them, or they may be old)
(ii) The cost of redundancy payments to employees
(iii) If the firm is a division or subsidiary of a larger enterprise, consider the effect of
withdrawal on the other operations within the group
(iv) The reluctance of managers to admit defeat, their loyalty to employees and
their fear for their own jobs
(v) Government pressures on major employers not to shut down operations,
especially when competition comes from foreign producers rather than other
domestic producers

Page 705

Using the five forces model: a caution
The five forces model provides a comprehensive framework for analysing the competitive
environment. However, it must be used with caution. It’s very comprehensiveness can
encourage a feeling of omniscience in those who use it: a sense that all factors have been duly
considered and dealt with. Unfortunately, no one is actually omniscient. Any analysis must
pursue as high a degree of objectivity as possible. If there is too much subjectivity,
unfounded complacence will result.
The creation in the UK of direct motor insurance selling by Direct Line Insurance is a case in
point. Existing motor insurers’ view of the threat from new entrants was that the need to
create a distribution network of local agents and brokers was an effective barrier to entry.
Direct Line’s centralised call-centre approach simply bypassed the barrier.
The effect of subjectivity appears at an early stage in any analysis using the five forces
approach. It is necessary to define with great care just what market or market segment one
is dealing with. For a large organisation, or one operating in a complex environment, this may
be extremely difficult. BPP’s provision of classroom training in accountancy is a good
example. The market for training for potential chartered accountants is subject to
considerable customer bargaining power, since there are a few large firms that predominate.
ACCA and CIMA courses, on the other hand, are more subject to the rivalry of existing
competitors, since, as well as other commercial training providers, universities and local
technical colleges are also sources of competition.
The need for careful analysis is, perhaps, most demanding in the area of substitute products
or services. It takes a particular alertness to discern potential substitutes in the early stages of
their development.

Boston Consulting Group (BCG) Portfolio matrix
The BCG portfolio matrix provides a method of positioning products through their life
cycles in terms of market growth and market share.
High

Star

Question
mark

Cash Cow

Dog

Market
growth

Low

Page 706
High

Market share

Low

a) Stars are products with a high share of a high growth market. In the short term,
items require investment in excess of the cash they generate in order to maintain their
market position, but promise high returns in the future.
b) In due course, however, stars will become cash cows, with a high share of a low
growth (mature) market. They require very little investment and generate high levels
of cash income. The important strategic feature of cash cows is that they are already
generating high cash returns, which can support the stars.
c) Question marks are competitive products with a low share of a high growth
market. They have the potential to become stars but a question mark hangs over their
ability to achieve sufficient market retention to justify further investment.
d) Dogs are products with a low share of a low growth market. They should be allowed
to die, or be killed off.

The matrix must be managed so that an organisation's product range is balanced. Four basic
strategies can be adopted.

a) Build. This involves increasing the market share, even at the expense of short-term
profits. A 'build' strategy might be to turn a question mark into a star. A penetration
pricing policy (covered in the syllabus for Paper F5 Performance Management) and
investment in stabilising quality and brand loyalty may be required.
b) Hold. This involves preserving market share and ensuring that cash cows remain cash
cows. Additional investment in customer retention through competitive pricing and
marketing may be required.
c) Harvest. This involves using funds to promote products which have the potential to
become future stars or to support existing stars.
d) Divest. This involves eliminating dogs and question marks which are under
performing.

Page 707

Using the BCG matrix: cautions
a) The model is probably too simplistic in the four classifications used. Some divisions
or products could fall into more than one category.
b) The market is not always easy to define especially if a company operates in a
specialist market.
c) The model requires the collection of a large amount of data and this can be costly
and time consuming.
d) The model fails to consider the relationship between divisions or any links between
products.

Page 708

Product-market mix: Ansoff’'s growth vector matrix
The Ansoff matrix identifies various options.
•
•
•
•

Market penetration: current products, current markets
Market development: current products, new markets
Product development: new products, current markets
Diversification: new products, new markets

All of these can secure growth.
We mentioned Ansoff’s matrix briefly in Study Unit 20. There we looked at its role in
closing the gap found by gap analysis.
Ansoff drew up a growth vector matrix, describing how a combination of a firm's activities
in current and new markets, with existing and new products, can lead to growth. Ansoff's
original model was a four cell matrix based on product and market, shown as the heart of the
diagram below.

Page 709

Present products and present markets: market penetration
Market penetration. The firm seeks to:
a) Maintain or to increase its share of current markets with current products, e.g.
through competitive pricing, advertising, sales promotion
b) Secure dominance of growth markets
c) Restructure a mature market by driving out competitors
d) Increase usage by existing customers (e.g. ‘airmiles’, loyalty cards)

Present products and new markets: development
a) New geographical areas and export markets
b) Different package sizes for food and other domestic items
c) New distribution channels to attract new customers
d) Differential pricing policies to attract different types of customer and create new
market segments.

New products and present markets: product development
Product development is the launch of new products to existing markets.
a) Advantages
(i)

Product development forces competitors to innovate

(ii)

Newcomers to the market might be discouraged

b) The drawbacks include the expense and the risk.

Page 710

New products and new markets: diversification
Diversification occurs when a company decides to make new products for new markets. It
should have a clear idea about what it expects to gain from diversification.
a) Growth. New products and new markets should be selected which offer prospects for
growth which the existing product-market mix does not.
b) Investing surplus funds not required for other expansion needs. (The funds could be
returned to shareholders.)

Closing the profit gap and product-market strategy
The aim of product-market strategies is to close the profit gap that is found by gap analysis.
A mixture of strategies may be needed to do this.

It is worth remembering that divestment is a product-market option to close the profit gap, if
the business is creating losses.
A related question is what do you do with spare capacity – go for market penetration, or go
into new markets. Many companies begin exporting into new overseas markets to use surplus
capacity.
The strategies in the Ansoff matrix are not mutually exclusive. A firm can quite legitimately
pursue a penetration strategy in some of its markets, while aiming to enter new markets.
Page 711

BLANK

Page 712

CRITICISMS OF PERFORMANCE INDICATORS
Performance measures are open to misinterpretation and manipulation. You need to be
aware of this when you apply these measures.

Non-financial indicators versus financial measures
If performance measurement systems focus entirely on those items that can be expressed in
monetary terms, managers will concentrate on only those variables and ignore other
important variables that cannot be expressed in monetary terms.
For example, pressure from senior management to cut costs and raise productivity will
produce short-term benefits in cost control but, in the long term, managerial performance
and motivation are likely to be affected. Labour turnover will increase and product quality
will fall.
Reductions in cost can easily be measured and recorded in performance reports. Employee
morale cannot. Performance reports should therefore include not only financial measures
but other important variables too, to give an indication of expected future results from
current activity. The wider implications for the organisation of achieving a particular
indicator should always be considered.

Pursuit of detailed operational goals
A danger of indicators measuring operational performance, especially non-financial
indicators, is that managers might be led into pursuing detailed operational goals, becoming
blind to the overall objectives that these goals were meant to attain.

Page 713

Not measuring what is supposed to be measured
Sometimes performance indicators do not actually measure what they are supposed to be
measuring.
For example, suppose that an organisation wished to measure the efficiency of its
production workforce and used profit margin to do so.
Although profit margin is a key measure of efficiency (the efficiency with which sales have
been used to generate profit), the production workforce cannot directly affect the revenue
earned. Use of the indicator should therefore be questioned. Or maybe the organisation
should instead be measuring the workforce's productivity.

Manipulating the way in which performance is measured
Suppose a poster in a doctor's surgery states that the doctor sees 98% of patients punctually.
This sounds impressive. But you need to ask how 'punctually' has been defined. It could be
that punctual means the patient was seen within ten minutes of the appointment time. You
should also consider whether such a statement was based on the experience of all patients, or
whether a sample was used. And if a sample was used, could it be biased? What if the doctor
cut short the appointments of those patients he knew not to be in the sample in order to ensure
those patients in the sample were seen on time.

Page 714

CHAPTER ROUNDUP
•

Porter's Five Forces Model suggests the importance of pressure from five
competitive forces on profit.

•

The BCG portfolio matrix provides a method of positioning products through their
life cycles in terms of market growth and market share.

•

The Ansoff matrix identifies various options.
–
–
–
–

Market penetration: current products, current markets
Market development: current products, new markets
Product development: new products, current markets
Diversification: new products, new markets

All of these can secure growth.
•

Performance measures are open to misinterpretation and manipulation. You need to
be aware of this when you apply these measures.

Page 715

Page 716

STUDY UNIT 22
Performance Measurement Models
Contents

Page

The Balanced Scoreboard …………………………………………..…….

719

The Performance Pyramid …………………..……………………..……..

723

Building Blocks ……………………………………….……..…………….

725

Page 717

EXAM GUIDE
One of the syllabus aims is to be able to advise [...] on strategic business performance
evaluation [...]. So you must think about how you would use the models here in a report to
advise management. Also you must think about the action words used in the study guide so
you may need to ‘evaluate’ in your exam answer.

Exam Focus Point
The examiner may ask you to discuss links between performance measures in an
organisation. The question made clear which model to apply by using key words such as
vertical, horizontal, hierarchy, internal and external aspects.
This represents an easy five marks as the words give a strong hint of the performance
pyramid.

Page 718

THE BALANCED SCORECARD
The balanced scorecard approach to performance measurement focuses on four different
perspectives and uses financial and non-financial indicators.

Knowledge brought forward from earlier studies
The balanced scorecard approach emphasises the need to provide management with a set of
information which covers all relevant areas of performance in an objective and unbiased
fashion. The information provided may be both financial and non-financial and cover areas
such as profitability, customer satisfaction, internal efficiency and innovation.
The balanced scorecard focuses on four different perspectives, as follows.

Performance targets are set once the key areas for improvement have been identified, and
the balanced scorecard is the main monthly report.

The scorecard is 'balanced' as managers are required to think in terms of all four
perspectives, to prevent improvements being made in one area at the expense of another.
An example of how a balanced scorecard might appear is offered below.
Page 719

Read the case study. You should be able to identify the perspectives as they appear here.

Case Study
The fall from grace of Digital Equipment Corporation, in the past second only to IBM in the
world computer rankings, was examined in a Financial Times article. The downfall is blamed
on Digital's failure to keep up with the development of the PC, but also on the company's
culture.
The company was founded on brilliant creativity, but was insufficiently focused on the
bottom line. Outside the finance department, monetary issues were considered vulgar and
organisational structure was chaotic. Costs were not a core part of important decisions – 'if
expenditure was higher than budget, the problem was simply a bad budget'. Ultimately the
low-price world of lean competitors took its toll, leading to huge losses. It was acquired by
Compaq which itself is now part of the HP (Hewlett Packard) group
Page 720

Advantages and disadvantages of the balanced scorecard
Important features of this approach are as follows.
a) It looks at both internal and external matters concerning the organisation.
b) It is related to the key elements of a company's strategy.
c) Financial and non-financial measures are linked together.

As with all techniques, problems can arise when it is applied.

Problem

Explanation

Conflicting
measures

Some measures in the scorecard such as research funding and cost
reduction may naturally conflict. It is often difficult to determine the
balance which will achieve the best results.

Selecting measures

Not only do appropriate measures have to be devised but the number
of measures used must be agreed. Care must be taken that the impact
of the results is not lost in a sea of information.

Expertise

Measurement is only useful if it initiates appropriate action. Nonfinancial managers may have difficulty with the usual profit
measures. With more measures to consider this problem will be
compounded.

Interpretation

Even a financially-trained manager may have difficulty in putting the
figures into an overall perspective.

The scorecard should be used flexibly. The process of deciding what to measure forces a
business to clarify its strategy. For example, a manufacturing company may find that 50% –
60% of costs are represented by bought-in components, so measurements relating to suppliers
could usefully be added to the scorecard. These could include payment terms, lead times, or
quality considerations.

Page 721

Case Study
An oil company (quoted by Kaplan and Norton, Harvard Business Review) ties 60% of its
executives' bonuses to their achievement of ambitious financial targets on ROI, profitability,
cash flow and operating cost, and 40% on indicators of customer satisfaction, retailer
satisfaction, employee satisfaction and environmental responsibility.

Page 722

THE PERFORMANCE PYRAMID
The performance pyramid highlights the links running between an organisation's vision and
its functional objectives.

The performance pyramid derives from the idea that an organisation operates at different
levels, each of which has different concerns which should nevertheless support each other in
achieving business objectives. The pyramid therefore links the overall strategic view of
management with day to day operations.

It includes a range of objectives for both external effectiveness (such as related to customer
satisfaction) and internal efficiency (such as related to productivity), which are achieved
through measures at the various levels.

a) At corporate level, financial and market objectives are set.
b) At strategic business unit level, strategies are developed to achieve these financial
and market objectives.
(i) Customer satisfaction is defined as meeting customer expectations.
(ii) Flexibility indicates responsiveness of the business operating system as a whole.
(iii) Productivity refers to the management of resources such as labour and time.
Page 723

c) These in turn are supported by more specific operational criteria.
(i)

Quality of the product or service, consistency of product and fit for the purpose
for which it is intended
(ii) Delivery of the product or service (the method of distribution, its speed and ease of
management)
(iii) Process time of all processes from cash collection to order processing to recruitment
(iv) Cost, meaning the elimination of all non-value added activities

The pyramid highlights the links running between the vision for the company and
functional objectives. For example, a reduction in process time should lead to increased
productivity and hence improved financial performance.

Page 724

BUILDING BLOCKS
Fitzgerald and Moon's building blocks for dimensions, standards and rewards attempt to
overcome the problems associated with performance measurement of service businesses.

Question
What are the five major characteristics of services that distinguish services from
manufacturing. Can you relate them to the provision of a haircut?

Answer
a) Intangibility. A haircut is intangible in itself, and the performance of the service
comprises many other intangible factors, like the music in the salon, the personality of
the hairdresser.
b) Simultaneity/inseparability. The production and consumption of a haircut are
simultaneous, and so cannot be inspected for quality in advance, nor returned if it is
not what was required.
c) Perishability. Haircuts are perishable, so they cannot be stored. You cannot buy them
in bulk, and the hairdresser cannot do them in advance and keep them in store in case
of heavy demand.
d) Heterogeneity/variability. A haircut is heterogeneous and so the exact service
received will vary each time: not only will A and B cut hair differently, but A will not
consistently deliver the same standard of haircut.
e) No transfer of ownership. A haircut does not become the property of the customer.

Question
Consider how the factors intangibility, simultaneity, perishability, no transfer of ownership
and heterogeneity apply to the various services that you use: public transport, your bank
account, meals at stalls or in restaurants, the mobile ‘phone service, your annual holiday and
so on.
Page 725

Knowledge brought forward from earlier studies
Performance measurement in service businesses has sometimes been perceived as difficult
because of the five factors listed above, but the modern view is that if something is difficult
to measure this is because it has not been clearly enough defined. Fitzgerald et al and
Fitzgerald & Moon provide building blocks for dimensions, standards and rewards for
performance measurement systems in service businesses.

Standards
These are ownership, achievability and equity.
a) To ensure that employees take ownership of standards, they need to participate in
the budget and standard-setting processes. They are then more likely to accept the
standards, feel more motivated as they perceive the standards to be achievable and
morale is improved. The disadvantage to participation is that it offers the opportunity
for the introduction of budgetary slack.
b) Standards need to be set high enough to ensure that there is some sense of
achievement in attaining them, but not so high that there is a demotivating effect
because they are unachievable. It is management's task to find a balance between
what the organisation perceives as achievable and what employees perceive as
achievable.
c) It is vital that equity is seen to occur when applying standards for performance
measurement purposes. The performance of different business units should not
be measured against the same standards if some units have an inherent advantage
unconnected with their own efforts. For example, divisions operating in different
countries should not be assessed against the same standards.

Rewards
The reward structure of the performance measurement system should guide individuals to
work towards standards. Three issues need to be considered if the performance measurement
system is to operate successfully: clarity, motivation and controllability.
a) The organisation's objectives need to be clearly understood by those whose
performance is being appraised i.e. they need to know what goals they are working
towards.

Page 726

b) Individuals should be motivated to work in pursuit of the organisation's strategic
objectives. Goal clarity and participation have been shown to contribute to higher
levels of motivation to achieve targets, providing managers accept those targets.
Bonuses can be used to motivate.
c) Managers should have a certain level of controllability for their areas of
responsibility. For example they should not be held responsible for costs over which
they have no control.

Dimensions
a) Competitive performance, focusing on factors such as sales growth and market
share.
b) Financial performance, concentrating on profitability, capital structure and so on.
c) Quality of service looks at matters like reliability, courtesy and competence.
d) Flexibility is an apt heading for assessing the organisation's ability to deliver at the right
speed, to respond to precise customer specifications, and to cope with fluctuations in
demand.
e) Resource utilisation, not unsurprisingly, considers how efficiently resources are
being utilised. This can be problematic because of the complexity of the inputs to a
service and the outputs from it and because some of the inputs are supplied by the
customer (he or she brings their own hair, for example). Many measures are possible,
however, for example 'number of customers per hairdresser per day/week'.
Performance measures can be devised easily if it is known what activities are
involved in the service.
f) Innovation is assessed in terms of both the innovation process and the success of
individual innovations.

These dimensions can be divided into two sets.
1) The results (measured by financial performance and competitiveness)
2) The determinants (the remainder)

Page 727

Focus on the examination and improvement of the determinants should lead to improvement
in the results.
There is no need to elaborate on competitive performance, financial performance and
quality of service issues, all of which have been covered already. The other three dimensions
deserve more attention.

Flexibility
Flexibility has three aspects.
1) Speed of delivery
Punctuality is vital in some service industries like passenger transport: indeed punctuality is
currently one of the most widely publicised performance measures - such as waiting time at
hospitals out-patients, because organisations like health centres are making a point of it.
Measures public transport might include waiting time in queues, as well as late buses. In
other types of service it may be more a question of timeliness. Does the auditor turn up to do
the annual audit during the appointed week? Is the audit done within the time anticipated by
the partner or does it drag on for weeks? These aspects are all easily measurable in terms of
'days late'. Depending upon the circumstances, 'days late' may also reflect on inability to
cope with fluctuations in demand.

2) Response to customer specifications
The ability of a service organisation to respond to customers' specifications is one of the
criteria by which Fitzgerald et al distinguish between the three different types of service.
Clearly a professional service such as legal advice and assistance must be tailored exactly to
the customer's needs. Performance is partly a matter of customer perception and so customer
attitude surveys may be appropriate. However it is also a matter of the diversity of skills
possessed by the service organisation and so it can be measured in terms of the mix of staff
skills and the amount of time spent on training. In mass service business customisation is
not possible by the very nature of the service.

3) Coping with demand
This is clearly measurable in quantitative terms in a mass service like a railway company
which can ascertain the extent of overcrowding. It can also be very closely monitored in
service shops: customer queuing time can be measured in banks and retailers, for example.
Page 728

Professional services can measure levels of overtime worked: excessive amounts indicate
that the current demand is too great for the organisation to cope with in the long term without
obtaining extra human resources.
Resource utilisation measures
Resource utilisation is usually measured in terms of productivity. The ease with which this
may be measured varies according to the service being delivered.
The main resource of a firm of accountants, for example, is the ”time” of various grades of
staff. The main output of an accountancy firm is chargeable hours.
In a restaurant it is not nearly so straightforward. Inputs are highly diverse: the ingredients
for the meal, the chef's time and expertise, the surroundings and the customers' own likes and
dislikes. A customer attitude survey might show whether or not a customer enjoyed the
food, but it could not ascribe the enjoyment or lack of it to the quality of the ingredients, say,
rather than the skill of the chef.
Here are some of the resource utilisation ratios listed by Fitzgerald et al.

Innovation
In a modern environment in which product quality, product differentiation and continuous
improvement are the order of the day, a company that can find innovative ways of satisfying
customers' needs has an important competitive advantage.
Fitzgerald et al suggest that individual innovations should be measured in terms of whether
they bring about improvements in the other five 'dimensions'.
The innovating process can be measured in terms of how much it costs to develop a new
service, how effective the process is (that is, how innovative is the organisation, if at all?),
and how quickly it can develop new services. In more concrete terms this might translate into
the following.
Page 729

a) The amount of R&D spending and whether (and how quickly) these costs are
recovered from new service sales
b) The proportion of new services to total services provided
c) The time between identification of the need for a new service and making it
available
Now look at the Example below using your knowledge of the model.

Example
A service business has collected some figures relating to its year just ended.

Customer enquiries:

Business won:

Types of services performed:

Employees:

Budget

Actual

New customers

6,000

9,000

Existing customers

4,000

3,000

New customers

2,000

4,000

Existing customers

1,500

1,500

Service A

875

780

Service B

1,575

1,850

Service C

1,050

2,870

Service A

5

4

Service B

10

10

Service C

5

8

Required
Calculate figures that illustrate competitiveness and resource utilisation.

Solution
Competitiveness can only be measured from these figures by looking at how successful the
organisation is at converting enquiries into firm orders.
Page 730

Percentage of enquiries converted into firm orders
Budget

Actual

New customers (W1)

33%

44%

Existing customers (W1)

37.5%

50%

Resource utilisation can be measured by looking at average services performed per
employee.
Budget

Actual

Rise

Service A (W2)

175

195

+11.4%

Service B (W2)

157.5

185

+17.5%

Service C (W2)

210

358.75

+70.8%

Workings
1

For example 2,000/6,000 = 33%

2

For example 875/5 = 175

What comments would you make about these results? How well is the business doing?

Points to consider
There is some debate as to how far the links between the financial results and the
determinants of those results can be precisely identified. Better quality will please
customers, but there is a problem of short-term versus long-term benefits. Quality costs
money now, while the benefits may take a long time to come through.
There is also the question of how much quality is enough: endless improvements that cost a
lot of money, but are not necessarily sought by the customers (who may indeed be unwilling
to pay for them) will harm long-term profitability.
Page 731

Be prepared to think up performance measures for different areas of an organisation's
business. Remember to make the measures relevant to the organisation in question. There is
little point in suggesting measures such as waiting times in queues to assess the quality of the
service provided by an educational establishment.
Question
Suggest two separate performance indicators that could be used to assess each of the
following areas of a fast food chain's operations.
a) Food preparation department
b) Marketing department

Solution
Here are some suggestions.
a) Material usage per product
Wastage levels
Incidences of food poisoning
b) Market share
Sales revenue per employee
Growth in sales revenue

Page 732

CHAPTER ROUNDUP
•

The balanced scorecard approach to performance measurement focuses on four
different perspectives and uses financial and non-financial indicators.

•

The performance pyramid highlights the links running between an organisation's
vision and its functional objectives.

•

Fitzgerald and Moon's building blocks for dimensions, standards and rewards
attempt to overcome the problems associated with performance measurement of
service businesses.

Page 733

BLANK

Page 734

STUDY UNIT 23
Financial Performance Measurement
Contents

Page

The Private Sector: Shareholder Benefits ……………………………….

737

Survival And Growth …………………..…………………………..……..

745

Profitability ………………………………………..……………..……….

749

Gearing ……………………………………………………….................…

771

Liquidity …………………………………......……………………………

779

Short-Run And Long-Run Financial Performance ………………….…

785

Profits And Share Value ………………….………………………………

789

Comparisons Of Accounting Figures ……………………………………

793

Page 735

EXAM GUIDE
This part of the syllabus lends itself to a range of possible question styles. Thus the examiner
could ask for calculations as he has done in the pilot paper. He is likely to ask for
commentary on these measures too, given the syllabus objectives. This part of the syllabus
also covers public sector and non-profit organisations. So he may ask you to compare and
contrast performance measurement in the different types of organisation.

Page 736

THE PRIVATE SECTOR: SHAREHOLDER
BENEFITS
The overriding purpose of a business is to increase long-term owner wealth.
Carefully read the case study below.

Case Study
The statement of prospects below is adapted from the published accounts of a UK printing
company with revenue in 2000 of £110m, operating profits of £16m, and post-tax profit of
£10m.
You may be interested in the order of priorities of the sections/paragraphs.
Note: Wyndeham Group is now owned by Walstead Investments,, a private company, and
their strategy may be different. But Wyndeham Group remains a leading printer. .

Wyndeham Press plc
Group Strategy
1) Our strategy is to build on our position as one of the leading printers of magazines,
brochures etc. offering a complete service for the customer from pre-press and
printing to finishing and despatch. We remain focused on making acquisitions to
assist in achieving our goal as well as developing our existing businesses.

Capital investment – investment criteria and budgeted expenditure
2) As referred to in the Chairman's and the Chief Executive's review, the last year has
been a period of considerable investment for your company. We purchased new
presses, finishing equipment and pre-press equipment, at a total cost of £14.1m. We
plan to invest a further £14m this year on upgrading existing equipment and
expanding capacity by installing additional machines.

Page 737

Funding structure
3) Our closing level of debt is £26.7m of which £23.2m is at fixed rates ranging from
5.9% to 8.1%. The balance is at 1% over base rate and this averaged 6.4% during the
period. Given the high level of operating gearing within a printing business, we
believe our optimal level of debt/ equity is between 50% – 70%.
4) Interest cover has reduced from 14 times to 10.3 times, which is still a very healthy
level, and gearing increased to 56%. Both ratios are well within our targets of a
minimum of 8 times interest cover and a maximum of 70% gearing. The covenants
under our debt facilities require a gearing of less than 85% and debt of less than
twice EBITDA (earnings before interest, tax, depreciation and amortisation).
5) The Group has a progressive dividend policy. Dividend growth will follow earnings
growth and we will maintain dividend cover at our target of 3 times. We believe
this level of cover should generate sufficient retained capital to support the equity
component of our investment programme.

Key performance indicators and benchmarking of performance
6) We benchmark our performance against a peer group of comparable businesses (A, B,
C, D). We aim for top quartile performance compared to this group in the following
categories: operating profit as a percentage of sales, return on capital employed,
profit per employee, proportion of repeat business. We believe we currently rank in
the top quartile of the printing sector on all these criteria.
7) The group achieved a return on capital employed of 29%. In the long-term, our
objective is a steady rise in return on capital employed as a result of acquisitions,
capital expenditure programmes and improvements in efficiency and machine
utilisation.

Risks and sensitivities
8) The commercial risks we face in the coming year are:
(i)

If sterling continues to increase in value, overseas companies will become even
more competitive on the non-magazine work.

(ii)

Whilst we expect a very modest growth in the economy, if economic activity
contracts there will be a resultant decline in demand for our services.
Page 738

Trading prospects
9) Our prospects for the current year are dependent on prices achieved and volume of
work. We believe that volume of work will move ahead this year arising from the
increased capacity generated by the installation of new plant.
10) We remain confident about the prospects for our business.

Why are shareholders important?
In the case example above, the 'statement of prospects' is expressed almost exclusively in
financial terms, with the exception of the 1st paragraph. The 'prospects' are not the prospects
of the business but the prospects for the shareholders who have invested in the company and
to whom the report is aimed.
As we saw in the previous chapters, organisations are likely to have a number of goals,
objectives and targets which, despite managerial effort to attain goal congruence, are at times
likely to conflict. This is often due to the difficulty in satisfying the differing objectives of the
organisation's various stakeholder groups.
But profit-making organisations tend to focus on financial performance in general and on the
interests of shareholders in particular.
The traditional argument for this is that shareholders are the legal owners, the company
belongs to them and so their interests are paramount.

Question
Go back to the case example above. Identify ways in which maximising long-term owner
value was Wyndeham Press plc's objective.

Answer
a) Group strategy, to serve customers, is undertaken with profit in mind
b) Capital investment – generate future profits by raising productivity

Page 739

c) Funding structure – there is generally an optimum mix of debt and equity capital. The
firm monitors this to raise capital and funds at the cheapest cost – in the shareholders
interests'
d) Benchmarking of performance. Although these are accounting measures, they do
contribute to the long-term performance on the company. Raising return on capital
employed means rewarding shareholders more each year for their investment

Significance of long-term owner focus
a) As maximising shareholder wealth is a long-term goal for a business, inevitably
managers must decide between what funds they want to disburse now and what
funds need to be maintained in the business to ensure the prospects of long-term
profitability.
b) Shareholders own the business, and so the directors of the company have a duty to
safeguard their interests.
c) What the shareholders require as a return is used to judge the validity of investment
projects.
d) Shareholders assess the quality of management by how well the business performs
financially.
e) Shareholders are the principal source of capital investment in a business. They
provide funds on share issues or permit managers to retain profits for investment.

What are shareholders interested in?
a) Current earnings
b) Future earnings
c) Dividend policy
d) The relative risk of the investments compared to other investments and the return
available
Page 740

Difficulties of incorporating shareholder concerns in performance
measurement for managers
a) Accounting. Shareholders are interested in future returns whereas accounts
generally provide historic information. Accounting measures such as ROCE do not
measure shareholder wealth.
b) Shareholders have an assessment of risk different from managers. Managers,
typically, worry about their careers, which concern shareholders not at all.
Shareholders are concerned about the security of their investment and the likelihood
of making a return.
c) At operating level, it is not easy to identify exactly how well a business is doing in
relation to other businesses.
d) Any other yardstick than shareholders' objectives effectively means that managers
may run an organisation in their own interests.

Why should managers bother to know who their shareholders are?
A company's senior management should remain aware of who its major shareholders are, and
it will often help to retain shareholders' support if the chairman or the managing director
meets occasionally with the major shareholders, to exchange views.
a) The company's management might learn about shareholders' preferences for either
high dividends or high retained earnings for profit growth and capital gain.
b) For public companies, changes in shareholdings might help to explain recent share
price movements.
c) The company's management should be able to learn about shareholders' attitudes to
both risk and gearing. If a company is planning a new investment, its management
might have to consider the relative merits of seeking equity finance or debt finance,
and shareholders' attitudes would be worth knowing about before the decision is
taken.
d) Management might need to know its shareholders in the event of an unwelcome
takeover bid from another company, to identify key shareholders whose views on the
takeover bid might be crucial to the final outcome.
Page 741

Aligning shareholder and managerial goals
One way of rewarding managers is share options.
a) This is regarded as a good thing as it means that managers have a direct financial
interest in increasing owner wealth, ensuring goal congruence.
b) Drawbacks are more subtle.
(i)

(ii)

Managers are rewarded for past performance, and the rewards are often
immediate. They may be incentivised to take short-term measures, and ignore
the long term.
There may be a general rise in share prices which is not performance related.

Case Study
From The Times, London, Tues 13 November 2007
'Bonnie Brown was not in a position to haggle. She was recently divorced and living with her
sister so when a small technology start-up offered her a job in 1999 as a part-time masseuse
she took it. The post paid RWF450 a week, plus a pile of what were then worthless stock
options.
Today, on the back of that package, Ms Brown, Google employee No 41, is a
multimillionaire.'
This story gives a flavour of some of the rewards possible from owning options. However,
no-one would suggest that Ms Brown, or indeed the other 1,000 or so Google employees who
are multimillionaires put in as much as they have reaped in rewards. The dramatic rise in
Google's share price is due to a range of factors from the hard work of its employees to
market fever for Google shares.

Internet businesses
Between 1995 and 1999, investors in Internet companies offered managers share options,
in return for a lower salary and long hours. The share options, potentially, could have made
Page 742

the managers into millionaires. For several years, managers worked long hours for reward
that correlated neatly with the rewards offered to shareholders.
In 2000, the market lost confidence in Internet companies. Hired managers saw a potential
RWF20m reward fall to $2m, and in many cases have left. Founders of the business have
stayed on. (Financial Times 6/2/01).
It may therefore be unrealistic to expect managers to take the same risks with their rewards as
investors, who are able to spread risks.
The growth in the internet has given shareholders an opportunity to organise and lobby the
companies in which they hold shares..

Page 743

BLANK

Page 744

SURVIVAL AND GROWTH
Achieving objectives of survival and growth ultimately depends on making profits.

Successful businesses might report expanding sales volumes, manufacture prestigious brands,
receive awards and recognition and be a good company to work for. These may be desirable
achievements and objectives, but they are not enough to guarantee the survival and growth of
an organisation.
The clearest measure of success for a business is continued existence and expansion. It is
widely accepted that growth requires profits and that growth can produce profits; growth
without profits can mean a company is taken over or goes into liquidation, that it does not
survive. So whatever else it aims to do, a business must make profits and make them in
perpetuity.
Despite the overriding importance of profits, growth can be measured in a number of ways.

Area of growth

Comment

Revenue

In the long term, growth in revenue is only really valuable to investors if
it means growth in profits.

Profitability

There are many measures of this (see next section). Growing
profitability is more useful if it is related to the level of investment.

Return of
investment

A growing return on investment suggests that capital is being used more
productively.

Market share

Growth in market share is generally regarded as a good thing, as it can
generate economies of scale.

Number of
employees

Shareholders are interested in productivity and profit per employee. An
increasing head count is a measure of success if people are needed to
deliver a service but people need to be employed productively.

Number of
products

Growth in the number of products is only useful if the products are
profitable.

Cash flow

This is one of the most important measures of growth as it ultimately
determines how much a business has to invest.
Page 745

Most of the time, growth is a sign of success, provided it is profitable. This is why it is
crucial. At other times, growth can be achieved in many different ways. Look at the case
example below, noting the strategies for growth and performance measures.

Case Study
From Times Online, 15 April 2008
Tesco, Britain's largest retailer, reported a 12 per cent jump in underlying pre-tax profit for
the 12 months to February 23, and announced plans for 30,000 new jobs across the group.
The company gave no further details about the type or location of its new jobs.
The £30 billion supermarket chain made an underlying pre-tax profit of £2.8 billion, giving
investors a 13 per cent increase in their annual dividend to 10.9p per share.
The underlying figures strip out costs such as pension payments and losses on financial
instruments, as well as boosts such as rent-free periods. When these are added into the
figures, Tesco's pre-tax profit was up 5.7 per cent.
Tesco reported a strong start to the year, with a like-for-like increase of 4 percent in UK
sales, excluding petrol, in the first five weeks of 2008.
Sir Terry Leahy, the chief executive, said: "We began the new financial year confidently,
with a good start in the UK, excellent progress in our established international markets and
promising early performance from our investments in future growth, particularly in the US,
China and Turkey."
Sir Terry's confidence is in contrast to the gloom on Britain's high streets. Last week the
British Retail Consortium urged the Bank of England to cut interest rates by half a percentage
point to avert a wave of job cuts as consumer spending slows down. Yesterday the BRC said
that the like-for-like value of takings at the country's tills fell by 1.6 per cent in March.
At Tesco, UK sales were up 6.7 per cent like-for-like to £37.9 billion last year despite what
the company described as "challenging market conditions". Without petrol sales, the
percentage increase in sales was almost 4 per cent per cent, which Tesco said was slightly
ahead of its planned performance.
The company said that attempts to lower prices had been hit by increased market prices for
commodities and some seasonal fresh foods. Unseasonal summer weather slowed growth in
Page 746

the first half, Tesco said, while tougher competition from competitors and a downturn in
customer demand for some of the company's non-food products cut sales in the second half.
The company said earlier this month that it would halt plans to sell its clothing lines online so
that it could "improve the offer". Tesco Direct, the store's online grocery deliver service,
racked up initial operating losses of £90 million.
Tesco's international division reported a 25 per cent increase in sales to £13.8 billion, with a
£702 million contribution from China, which was consolidated in the accounts for the first
time.
The company said last month that it would "pause for breath" in the US after opening 60
Fresh & Easy stores across Southern California and Arizona in five months. The stores are
styled on the Tesco Express outlets in the UK but analysts have claimed that the shops are
missing sales targets by as much as 70 per cent.
The company said today that it would separate out US sales and trading results in its interim
results in September. Until then, they are included in the UK figures. Tesco said: "We are
very encouraged by the start Fresh & Easy has made. ... Whilst it is still early days, the
response of customers to our offer has surpassed our expectations." It insisted that US sales
were ahead of budget and that plans to open about 150 new US stores this year were on track.
In its personal finance business, Tesco made a pre-tax profit of £64 million after a £11
million hit from household insurance claims from last summer's floods in Yorkshire and the
Midlands.
The company's £5 billion programme to release value from its property portfolio delivered
proceeds of £1.2 billion over the past year. The company admitted that yields had increased
only modestly in recent months but said that demand for its property remained strong. The
cash raised is used to fund a share buyback programme that has seen £1.1 billion worth of
shares repurchased so far.
Tesco opened 17 new superstores and 103 express stores last year, taking its total number of
outlets to 1,608 — which has raised concerns among campaigners who claim that the
supermarket giants are killing Britain's high streets.
Tesco said that it was continuing to work with the Competition Commission on its inquiry
into the grocery industry but issued a warning against red tape that would halt its expansion.
"This is a very competitive industry from which consumers benefit hugely," the company
said today. "We hope that the regulatory authorities will give due weight to this and to the
need to avoid costly and burdensome new regulation."

Page 747

BLANK

Page 748

PROFITABILITY
Measures relating to profit include sales margin, EBITDA and EPS. More sophisticated
measures (ROCE, ROI) take the size of investment into account. Later on in the chapter we
consider how measures of profitability are used for short-run or long-run performance
measurement. Bear this in mind particularly when you study the sections on RI, ROI and
NPV and go through the examples covering these.

Case Study
Pearson, the education and publishing group, listed the following as 'financial highlights' in
its 2007 annual accounts.
a) Sales
b) Operating profit (before goodwill, exceptional and non-operating items)
c) Adjusted earnings per share
d) EBITDA
e) Operating cash flow
f) Operating free cash flow
g) Return on invested capital
h) Net debt

Pearson actively targets sales growth, EBITDA and adjusted earnings per share.

You need to be able to discuss the appropriateness of the measures of 'profitability' covered
in this section.
As a general principle, these measures of performance, which we will be looking at, are
only meaningful if they are used for comparison.
a)
b)
c)
d)

Over time (equivalent time periods)
With other companies
With other measures of performance
With other industries

Page 749

Profitability
A company ought of course to be profitable, and there are obvious checks on profitability.
a) Whether the company has made a profit or a loss on its ordinary activities
b) By how much this year's profit or loss is bigger or smaller than last year's profit or
loss
It is probably better to consider separately the profits or losses on exceptional items if there
are any. Such gains or losses should not be expected to occur again, unlike profits or losses
on normal trading.

Question
A company has the following summarised income statements for two consecutive years.
Year 1
RWF
‘000
70,000
42,000
28,000
21,000
7,000

Revenue
Less cost of sales
Gross profit
Less expenses
Net profit

Year 2
RWF
‘000
100,000
55,000
45,000
35,000
10,000

Although the net profit margin is the same for both years at 10%, the gross profit margin is
not.
Year 1

28,000,000
= 40%
70,000,000

Year 2

Is this good or bad for the business?

Page 750

45,000,000
= 45%
100,000,000

Solution
An increased profit margin must be good because this indicates a wider gap between selling
price and cost of sales. Given that the net profit ratio has stayed the same in the second year,
however, expenses must be rising. In year 1 expenses were 30% of revenue, whereas in year
2 they were 35% of revenue. This indicates that administration, selling and distribution
expenses or interest costs require tighter control.

Percentage analysis of profit between year 1 and year 2
Year 1
%
60
40
100

Cost of sales as a % of sales
Gross profit as a % of sales

Expenses as a % of sales
Net profit as a % of sales
Gross profit as a % of sales

30
10
40

Year 2
%
55
45
100

35
10
45

Profit on ordinary activities before taxation is generally thought to be a better figure to
use than profit after taxation, because there might be unusual variations in the tax charge
from year to year which would not affect the underlying profitability of the company's
operations.
Another profit figure that should be calculated is PBIT: profit before interest and tax.
a) This is the amount of profit which the company earned before having to pay interest
to the providers of loan capital. By providers of loan capital, we usually mean
longer-term loan capital, such as debentures and medium-term bank loans, which will
be shown in the statement of financial position (balance sheet) as 'Suppliers: amounts
falling due after more than one year.' This figure is of particular importance to
bankers and lenders.
b) How is profit before interest and tax calculated?
The profit on ordinary activities before taxation
plus Interest charges on long-term loan capital
Page 751

c) To calculate PBIT, in theory, all we have to do is to look at the interest payments in
the relevant note to the accounts. Do not take the net interest figure in the income
statement itself, because this represents interest payments less interest received, and
PBIT is profit including interest received but before interest payments.

Sales margin
Sales margin is revenue less cost of sales.
Look at the following examples (both UK companies so stated in £ sterling).

a) Wyndeham Press, a printer

2000
£'000

Revenue

89,844

Cost of sales

(60,769)

Gross profit

29,075

Distribution expenses

(1,523)

Administrative expenses

(13,300)
(212)

Goodwill amortisation

14,040

Operating profit (15.6%)
(Interest etc)

Cost of sales comprises direct material cost, such as paper, and direct labour. Distribution
and administrative expenses include depreciation. Sales margin = 32%.
Sales margin at least shows the contribution that is being made, especially when direct
variable costs are very significant.

Page 752

b) Arriva plc, a bus company

1999
£m

Revenue
Cost of sales
Gross profit
Net operating expenses
Operating profit (7.6%)
(Interest etc)

1,534.3
1,282.6
251.7
133.8
117.9

Sales margin = 16%. Clearly a higher percentage of costs are operating costs.

c) Lessons to be learnt
(i)

Sales margin as a measure is not really any use in comparing different
industries.

(ii)

Sales margin is influenced by the level of fixed costs.

(iii) Trends in sales margin are of interest. A falling sales margin suggests an
organisation has not been able to pass on input price rises to customers.
(iv) Comparisons with similar companies are of interest. If an organisation has a lower
sales margin than a similar business, this suggests problems in controlling input
costs.

In short, the value of sales margin as a measure of performance depends on the
cost structure of the industry and the uses to which it is put.

Page 753

EBITDA
EBITDA is earnings before interest, tax, depreciation and amortisation.

Question
A multiple of EBITDA remains the most popular method for valuing media companies. Take
Pearson plc as an example.
Why do you think EBITDA is 'increasingly used as a basis for valuing media companies such as
Pearson'?

Solution
Pearson's 2007 sales were £4,162m.
Here is an extract from the company's 31 December 2007 statement of financial position
(balance sheet) (stated in £m).

£m

Non-current assets
Intangible

3,814

Tangible

355

Investments in joint ventures and associates
Other assets*

20
594
4,783

In other words, 80% of Pearson's non-current assets are intangible and are mainly
goodwill.
* Various classes of asset itemised in the statement of financial position (balance sheet) but
consolidated here for brevity.

Page 754

To see what EBITDA actually does, it is worth identifying what it omits.

Item

Comment

Earnings

In practice this equals profit after tax for the financial year with some
adjustments, as you should be aware from your financial accounting
studies.

Interest

Essentially this is a financing cost. Pearson's statement of financial
position (balance sheet) at 31/12/05 showed net assets of £3,733m.
Suppliers due over one year were £2,500m, most of which were bonds
and commercial paper.

Tax

The government's take is not relevant to the operating performance of the
business.

Depreciation
and
amortisation

This is the income statement charge for tangible and intangible assets.
Depreciation generally represents the writing off of expenditure incurred
several years ago, not in itself relevant to performance in any particular
financial year.

Advantages of EBITDA
a) It is a good proxy for cash flow from operations, and therefore is a measure of
underlying performance. It can be seen as the proportion of operating profits
converted to cash.
b) Tax and interest – while important – are effectively distributions to the government (tax)
and a finance charge (interest). They are not relevant to the underlying success of this
particular business.
c) EBITDA is easy to calculate and understand.
d) EBITDA can be used to assess the performance of a manager who has no control over
acquisition and financing policy as it excludes costs associated with assets
(depreciation) and debt (interest).

Page 755

Question
When might interest be relevant in a significant way to the operating performance of the
business?

Solution
•

It depends. Short term bank interest can be a significant operating expense.

•

Also, a bank itself earns money from an interest margin so interest is at the heart of
what it does.

Earnings per share (EPS)
EPS is a convenient measure as it shows how well the shareholder is doing.
EPS is widely used as a measure of a company's performance, especially in comparing
results over a period of several years. A company must be able to sustain its earnings in
order to pay dividends and re-invest in the business so as to achieve future growth. Investors
also look for growth in the EPS from one year to the next.

Earnings per share (EPS) is defined (in Financial Reporting Standard 3) as the profit in
cents attributable to each equity (ordinary) share. EPS is calculated as follows.
Profit of the period after tax, minority
interests and extraordina
items, and after deducting preference
vidends
di
Number of equity shares in issue andking
ranfor dividend

Extraordinary items are unusual, non-repeating items that affect profit but have effectively
been outlawed by FRS 3.

EPS on its own does not really tell us anything. It must be seen in context.
a) EPS is used for comparing the results of a company over time. Is its EPS growing?
What is the rate of growth? Is the rate of growth increasing or decreasing?
Page 756

b) Is there likely to be a significant dilution of EPS in the future, perhaps due to the exercise
of share options or warrants, or the conversion of convertible loan stock into equity?
c) EPS should not be used blindly to compare the earnings of one company with another.
For example, if A Co has an EPS of RWF120 for its 10,000,000 RWF100 shares and
B Co has an EPS of RWF240 for its 50,000,000 RWF250 shares, we must take
account of the numbers of shares. When earnings are used to compare one
company's shares with another, this is done using the P/E ratio or perhaps the
earnings yield.
d) If EPS is to be a reliable basis for comparing results, it must be calculated
consistently. The EPS of one company must be directly comparable with the EPS of
others, and the EPS of a company in one year must be directly comparable with its
published EPS figures for previous years. Changes in the share capital of a company
during the course of a year cause problems of comparability.

Note that EPS is a figure based on past data, and it is easily manipulated by changes in
accounting policies and by mergers or acquisitions. The use of the measure in calculating
management bonuses makes it particularly liable to manipulation. The attention given to
EPS as a performance measure by City analysts is arguably disproportionate to its true worth.
Investors should be more concerned with future earnings, but of course estimates of these
are more difficult to reach than the readily available figure.
A fully diluted EPS (FDEPS) can be measured where the company has issued securities that
might be converted into ordinary shares at some future date, such as convertible loan stock,
share warrants or share options. The FDEPS gives investors an appreciation of by how much
EPS might be affected if and when the options, warrants or conversion rights are exercised.

Profitability and return: the return on capital employed (ROCE)
It is impossible to assess profits or profit growth properly without relating them to the amount
of funds (the capital) employed in making the profits. An important profitability ratio is
therefore return on capital employed (ROCE), which states the profit as a percentage of
the amount of capital employed.
Profit is usually taken as PBIT, and capital employed is shareholders' capital plus 'suppliers:
amount falling due after more than one year' plus long-term provisions for liabilities and
charges. This is the same as total assets less current liabilities. The underlying principle is
Page 757

that we must compare like with like, and so if capital means share capital and reserves plus
long-term liabilities and debt capital, profit must mean the profit earned by all this capital
together. This is PBIT, since interest is the return for loan capital.
Return on capital employed (ROCE) indicates the productivity of capital employed. It is
calculated as:
Profit before interest and tax x 100
Average capital employed

The denominator is normally calculated as the average of the capital employed at the
beginning and end of the year. Problems of seasonality, new capital introduced or other
factors may necessitate taking the average of a number of periods within the year.

Evaluating the ROCE
What does a company's ROCE tell us? What should we be looking for? There are three
comparisons that can be made.
a) The change in ROCE from one year to the next
b) The ROCE being earned by other companies, if this information is available
c) A comparison of the ROCE with current market borrowing rates
(i)

(ii)

What would be the cost of extra borrowing to the company if it needed more
loans, and is it earning an ROCE that suggests it could make high enough
profits to make such borrowing worthwhile?
Is the company making an ROCE which suggests that it is making profitable use
of its current borrowing?

Analysing profitability and return in more detail: the secondary ratios
We may analyse the ROCE, to find out why it is high or low, or better or worse than last
year. There are two factors that contribute towards a return on capital employed, both related
to revenue.

Page 758

a) Profit margin. A company might make a high or a low profit margin on its sales. For
example, a company that makes a profit of RWF250 per RWF1,000 of sales is making
a bigger return on its revenue than another company making a profit of only RWF100
per RWF1,000 of sales.
b) Asset turnover. Asset turnover is a measure of how well the assets of a business are
being used to generate sales. For example, if two companies each have capital
employed of RWF100,000, and company A makes sales of RWF400,000 a year
whereas company B makes sales of only RWF200,000 a year, company A is making a
higher revenue from the same amount of assets and this will help company A to make
a higher return on capital employed than company B. Asset turnover is expressed as 'x
times' so that assets generate x times their value in annual revenue. Here, company
A's asset turnover is 4 times and company B's is 2 times.

Profit margin and asset turnover together explain the ROCE, and if the ROCE is the primary
profitability ratio, these other two are the secondary ratios. The relationship between the
three ratios is as follows.
Profit margin

×

asset turnover

=

PBIT
Sales

×

Sales
Capital employed

=

ROCE
PBIT
Capitalemployed

It is also worth commenting on the change in revenue from one year to the next. Strong sales
growth will usually indicate volume growth as well as revenue increases due to price rises,
and volume growth is one sign of a prosperous company.

Return on investment (ROI)
Return on investment (ROI) is a form of ROCE and is calculated as:

Profit before interest and tax x 100
Operations management capital employed

Page 759

The ROI compares income with the operational assets used to generate that income. Profit is
taken before tax and interest because tax is an appropriation of profit made from the use of
the investment, and the introduction of interest charges introduces the effect of financing
decisions into an appraisal of operating performance.
ROI is normally used to apply to investment centres or profit centres. These normally reflect
the existing organisation structure of the business.

Main reasons for the widespread use of ROI
a) Financial reporting. It ties in directly with the accounting process, and is identifiable
from the income statement and statement of financial position (balance sheet), the
firm's most important communications media with investors.
b) Aggregation. ROI is a very convenient method of measuring the performance for a
division or company as an entire unit.

Other advantages include its ability to permit comparisons to be drawn between investment
centres that differ in their absolute size.

Measurement problems: non-current assets
a) It is probably most common to use return on net assets.
(i)

If an investment centre maintains the same annual profit, and keeps the same
assets without a policy of regular non-current asset replacement, its ROI will
increase year by year as the assets get older. This can give a false impression
of improving 'real' performance over time.
(ii) It is not easy to compare fairly the performance of one investment centre with
another. Non-current assets may be of different ages or may be depreciated in
different ways.
(iii) Inflation and technological change alter the cost of non-current assets. If one
investment centre has non-current assets bought ten years ago with a gross cost
of RWF1,000 million, and another investment centre, in the same area of
Page 760

business operations, has non-current assets bought very recently for RWF1,000
million, the quantity and technological character of the non-current assets of the
two investment centres are likely to be very different.
(iv) Measuring ROI as return on gross assets ignores the age factor. Older noncurrent assets usually cost more to repair and maintain. An investment centre with
old assets may therefore have its profitability reduced by repair costs.
b) Measurement problems: what are 'assets' anyway?
Prudence and other accounting principles require that items such as research and
development should only be carried forward as an investment in special circumstances.
Many 'costs' do have the effect of enhancing the long-term revenue-earning capacity of
the business. A good example is brands: many firms have capitalised brands for this
reason. For decision-making and control purposes, the expenditure on brands might be
better treated as an investment.

The target return for a group of companies
If a group of companies sets a target return for the group as a whole, or if a company sets a
target return for each SBU, it might be company policy that no investment project should go
ahead in any subsidiary or investment centre unless the project promises to earn at least the
target return. Here is an example.
a) There should be no new investment by any subsidiary in the group unless it is
expected to earn at least a 15% return.
b) Similarly, no non-current asset should be disposed of if the asset is currently earning a
return in excess of 15% of its disposal value.
c) Investments which promise a return of 15% or more ought to be undertaken.

Problems with such a policy include:
a) Investments are appraised by DCF whereas actual performance will probably be
measured on the basis of ROI.
b) The target return makes no allowance for the different risk of each investment
centre.

Page 761

c) In a conglomerate, an identical target return may be unsuitable to many businesses
in a group.

Since managers will be judged on the basis of the ROI that their centre earns each year, they
are likely to be motivated into taking those decisions which increase their centre's shortterm ROI.
a) An investment might be desirable from the group's point of view, but would not be in
the individual investment centre's 'best interest' to undertake. Thus there is a lack of
goal congruence.
b) In the short term, a desire to increase ROI might lead to projects being taken on
without due regard to their risk.
c) Any decisions which benefit the company in the long term but which reduce the
ROI in the immediate short term would reflect badly on the manager's reported
performance.

Divisional performance: residual income (RI)
An alternative way of measuring the performance of an investment centre, instead of using
ROI, is residual income (RI).

Residual income is a measure of the centre's profits after deducting a notional or imputed
interest cost.

Its use highlights the finance charge associated with funding.
The imputed cost of capital might be the organisation's cost of borrowing or its weighted
average cost of capital. Alternatively, the cost of capital can be adjusted to allow for the risk
characteristics of each investment centre, with a higher imputed interest rate being applied to
higher risk centres.

Page 762

Example: calculation of ROI and RI
Division M is a division of MR Co. The following data relate to Division M.
Capital employed (net assets)
Annual profit
Cost of capital

RWF20m
RWF5m
15% per annum

MR Co is considering two proposals.
Proposal 1
Invest a further RWF2m in fixed assets to earn an annual profit of RWF0.30m.

Proposal 2
Dispose of fixed assets at their net book value of RWF5.5m. This would lead to profits falling
by RWF0.8m per annum. Proceeds from the disposal of these fixed assets would not be
credited to Division M (but to the Holding Company of MR Co instead).

Required
a) Calculate the current Return on Investment and Residual Income for Division M.
b) Consider each of the two proposals and show how the Return on Investment and
Residual Income would change if these proposals were adopted.

Solution
a) Current Return on Investment
Profit before interest and tax

Return on Investment =

× 100%

Operations management capital employed

=

Rwf 5m
× 100%
Rwf20m

= 25%
Residual Income

= Annual profit – imputed interest charge on net assets
=

RWF5m – (15% × RWF20m)
Page 763

=

RWF5m – RWF3m

=

RWF2m

The Return on Investment (25%) exceeds the cost of capital (15%) and the residual
income is positive (+RWF2m) and therefore Division M is performing well.

b) Let us now look at the situations that would arise if proposals 1 and 2 were to be
adopted.

Proposal 1
New profit

= RWF5m + RWF0.3m
= RWF5.3m

∴ New capital employed

= RWF20m + RWF2m
= RWF22m

∴ New Return on Investment

=

Rwf5.3m
× 100%
Rwf22m

= 24.1%
∴ New Residual Income

= RWF5.3m – (15% × RWF22m)
= RWF5.3m – RWF3.3m
= RWF2m

Proposal 2
New profit

= RWF5m – RWF0.8m
= RWF4.2m

New capital employed

= RWF20m − RWF5.5m
= RWF14.5m

Page 764

∴ New Return on Investment

=

Rwf4.2m
× 100%
Rwf14.5m

= 29%
∴ New Residual Income

= RWF4.2m – (15% × RWF14.5m)
= RWF4.2m – RWF2.18m
= RWF2.02m

Summary
Current

Proposal 1

Proposal 2

Return on Investment (%)

25

24.1

29

Residual Income (RWFm)

2

2

2.02

On first inspection it appears that proposal 2 should be adopted as the ROI increases
from 25% to 29% and the RI also increases slightly from RWF2m to RWF2.02m.
However, divisional managers should also consider the asset rate of return relevant to
Proposal 2.

Asset rate of return =

=

Change in profit
Change in investment

Rwf0.8m
× 100%
Rwf5.5m

= 14.5%
Since MR Co's current rate of return is 25%, any asset which has a rate of return less
than this should be disposed of. It is important to remember, therefore, that whichever
proposal is accepted, it should lead to goal congruence.

Page 765

The advantages and weaknesses of RI compared with ROI
Advantages of RI
a) Residual income increases in the following circumstances.
(i)
(ii)

Investments earning above the cost of capital are undertaken
Investments earning below the cost of capital are eliminated

b) Residual income is more flexible since a different cost of capital can be applied to
investments with different risk characteristics.

Weaknesses of RI
The first is that it does not facilitate comparisons between investment centres nor does it
relate the size of a centre's income to the size of the investment, other than indirectly through
the interest charge. The second is that it can be difficult to decide on an appropriate and
accurate measure of the capital employed upon which to base the imputed interest charge
(see comments on ROI).

Cash flows: NPV and IRR
The Study Guide mentions NPV and IRR as measures of 'profitability' to be considered in
this context.

The advantages and weaknesses of NPV compared with ROI and RI
Advantages include:
a) Cash flows are less subject to manipulation and subjective decisions than accounting
profits.
b) It considers the opportunity cost of not holding money.
c) Risk can be allowed for by adjusting the cost of capital.
d) Shareholders are interested in cash flows (both in the short term and long term).
Page 766

The disadvantages of the NPV approach are centred on the assumptions underlying the
values of critical variables within the model. For example:
a) The duration of the cash flows
b) The timing of the cash flows
c) The appropriate cost of capital
NPV and IRR are typically used to evaluate capital investment or other discrete items of
expenditure, or to compare investment projects.

Cash flows and NPVs for strategic control: shareholder wealth
Control and performance measures at a strategic level do need to pay some attention to
wealth. Shareholders are interested in cash flow as the safest indicator of business success.
According to one model of share valuations, the market value of the shares is based on the
expected future dividend.
Control at a strategic level should be based on measurements of cash flows (actual cash
flows for the period just ended and revised forecasts of future cash flows). Since the objective
of a company might be to maximise the wealth of its shareholders, a control technique based
on the measurement of cash flows and their NPV could be a very useful technique to apply. A
numerical example might help to illustrate this point.

Suppose that ABC Co agrees to a strategic plan from 1 January 20X1 as follows.
Year
Planned net
(RWFm)

20X1
cash

inflow 200

NPV at cost of capital 15%

174

20X2

20X3

20X4

20X5

Total

300

300

400

500

1,700

227

197

229

249

1,076

Now suppose that ABC Co reviews its position one year later.
a) It can measure its actual total cash flow in 20X1 – roughly speaking, this will be the
funds generated from operations minus tax paid and minus expenditure on noncurrent assets and plus/minus changes in working capital.
b) It can revise its forecast for the next few years.
Page 767

We will assume that there has been no change in the cost of capital. Control information at
the end of 20X1 might be as follows.
Year

20X1

20X2

20X3

20X4

20X5

Total

(actual)

(forecast)

Net cash inflow (RWF m)

180

260

280

400

540

1,660

NPV at cost of capital 15%

180

226

212

263

309

1,190

A control summary comparing the situation at the start of 20X1 and the situation one year
later would now be as follows.
RWF m
Expected NPV as at 1.1.20X1

1,076

Uplift by cost of capital 15% *

161

What NPV should have been at 31.12.20X1 **

1,237

Expected NPV as at 31.12.20X1

1,190

Variance

47 (A)

* You might wonder why we are doing this. Each cash flow in the original calculation was ×
by discount factor of 1/(1.15)N, where N = number of years between 20X1 and the cash
flow. If we were to calculate the NPV starting at a point a year later the discount factor for
each of the cash flows would be 1/(1.15)N – 1 (i.e. a cash flow at year 2 (31 December
20X2) from 1 January 20X1 would have a discount factor of 1/1.152, but when NPV is
recalculated at 31 December 20X1 discount factor for 31 December 20X2 cash flow =
1/1.15. So each discount factor for recalculating is multiplied by 1.15 (changing 1/1.15N to
1/1.15N – 1). We can therefore × total NPV at 1 January 20X1 by 1.15 to get what NPV
should have been at 31 December 20X1.
** The uplifting shows by how much the expected NPV would change if we were doing the
calculation 12 months later.

Page 768

The control information shows that by the end of 20X1, ABC Co shows signs of not
achieving the strategic targets it set itself at the start of 20X1. This is partly because actual
cash flows in 20X1 fell short of target by (200-180) RWF2 m, but also because the revised
forecast for the future is not as good now either. In total, the company has a lower NPV by
RWF47,000,000.
The reasons for the failure to achieve target should be investigated. Here are some
possibilities.
a) A higher-than-expected pay award to employees, which will have repercussions for
the future as well as in 20X1
b) An increase in the rate of tax on profit.
c) A serious delay in the implementation of some major new projects
d) The slower-than-expected growth of an important new market

Strategic progress can therefore be measured by reconciling successive net present values
and the intervening cash flows. The arithmetic is straightforward and can be summed up as
follows.

Step 1

The previous NPV is uplifted by the cost of capital applicable to the current
period.

Step 2

The result is the 'benchmark NPV' indicating what the new NPV needs to be if
long-term health has been maintained.

Step 3

Comparison of the new NPV with the benchmark produces a variance which can
be analysed by cause and by time frame.

Attempt your own solution to the following question.

Internal rate of return
IRR is another way of reviewing investments. The IRR of a project can be compared to the
cost of capital. It should be possible in theory to assess an IRR for a company, but other
models or measures may be simpler.
Page 769

BLANK

Page 770

GEARING
As well as profitability, liquidity and gearing are key measures of performance.

Capital structure
The assets of a business must be financed somehow, and when a business is growing, the
additional assets must be financed by additional capital. Capital structure refers to the way
in which an organisation is financed, by a combination of long-term capital (ordinary
shares and reserves, preference shares, debentures, bank loans, convertible loan stock and so
on) and short-term liabilities, such as a bank overdraft and trade suppliers.

Debts and financial risk
There are two main reasons why companies should keep their debt burden under control.
a) When a company is heavily in debt, and seems to be getting even more heavily into
debt, banks and other would-be lenders are very soon likely to refuse further
borrowing and the company might well find itself in trouble.
b) When a company is earning only a modest profit before interest and tax, and has a
heavy debt burden, there will be very little profit left over for shareholders after the
interest charges have been paid. And so if interest rates were to go up or the company
were to borrow even more, it might soon be incurring interest charges in excess of
PBIT. This might eventually lead to the liquidation of the company.

A high level of debt creates financial risk. Financial risk can be seen from different points of
view.
a) The company as a whole. If a company builds up debts that it cannot repay when
they fall due, it will be forced into liquidation.
b) Suppliers. If a company cannot pay its debts, the company will go into liquidation
owing suppliers money that they are unlikely to recover in full.

Page 771

c) Ordinary shareholders. A company will not make any distributable profits unless it
is able to earn enough profit before interest and tax to pay all its interest charges, and
then tax. The lower the profits or the higher the interest-bearing debts, the less there
will be, if there is anything at all, for shareholders.

When a company has preference shares in its capital structure, ordinary shareholders will not
get anything until the preference dividend has been paid.

The appraisal of capital structures
One way in which the financial risk of a company's capital structure can be measured is by a
gearing ratio. A gearing ratio should not be given without stating how it has been defined.

Gearing ratios
Gearing ratios measure the financial risk of a company's capital structure. Business risk can
be measured by calculating a company's operational gearing.

Financial gearing/leverage is the use of debt finance to increase the return on equity by
using borrowed funds in such a way that the return generated is greater than the cost of
servicing the debt. If the return on borrowed funds is less than the cost of servicing the debt,
the effect of gearing is to reduce the return on equity.

Gearing measures the relationships between shareholders' capital plus reserves, and
either prior charge capital or borrowings or both.

Prior charge capital is capital which has a right to the receipt of interest or preference
dividends before any claim is made by ordinary shareholders on distributable earnings. On
winding up, the claims of holders of prior charge capital rank before those of ordinary
shareholders.
Page 772

Prior charge capital is:
a) Any preference share capital
b) Interest-bearing long-term capital
c) Interest-bearing short-term debt capital with less than 12 months to maturity,
including any bank overdraft

However, (c) might be excluded.
Here are some commonly used measures of financial gearing, which are based on the
statement of financial position (balance sheet) values (book values) of the fixed interest
and equity capital.

Prior charge capital
Equity capital (including reserv

Prior charge capital
Total capital employe

With the first definition above, a company is low geared if the gearing ratio is less than
100%, highly geared if the ratio is over 100% and neutrally geared if it is exactly 100%.

Example
From the following statement of financial position (balance sheet), compute the company's
financial gearing ratio.
RWF m

RWF m

RWF m

ASSETS
Non-current assets

12,400

Current assets

1,000
13,400

Page 773

RWF m

RWF m

RWF m

EQUITY AND LIABILITIES
Equity
Called up share capital
Ordinary shares

1,500

Preference shares

500

Share premium account

760

Revaluation reserve

1,200

Retained earnings

2,810

Non-current liabilities
Debentures

4,700

Bank loans

500
5,200

Deferred tax

300

Deferred income

250

Current liabilities
Loans

120

Bank overdraft

260

Trade suppliers

430

Bills of exchange

70
880
13,400

Page 774

Solution
Prior charge capital

RWF m

Preference shares

500

Debentures

4,700

Long-term bank loans

500

Prior charge capital, ignoring short-term debt

5,700

Short-term loans

120

Overdraft

260

Prior charge capital, including short-term interest bearing debt

6,080

Either figure, RWF6,080 m or RWF 5,700 m, could be used. If gearing is calculated with
capital employed in the denominator, and capital employed is net non-current assets plus net
current assets, it would seem more reasonable to exclude short-term interest bearing debt
from prior charge capital. This is because short-term debt is set off against current assets in
arriving at the figure for net current assets.

Equity = 1,500 + 760 + 1,200 + 2,810 = RWF6,270,000,000
The gearing ratio can be calculated in one of the following ways.
a)

Priorchargecapital
x
Equity

b)

Priorchargecapital
x
Totalcapitalemployed

100% =

6,080
6,270

100% =

x 100% = 97%

5,700
12,520

x 100% = 45.5%

There is no absolute limit to what a gearing ratio ought to be. Many companies are highly
geared, but if a highly geared company is increasing its gearing, it is likely to have difficulty
in the future when it wants to borrow even more, unless it can also boost its shareholders'
capital, either with retained profits or with a new share issue.

Page 775

The effect of gearing on earnings
The level of gearing has a considerable effect on the earnings attributable to the ordinary
shareholders. A highly geared company must earn enough profits to cover its interest
charges before anything is available for equity. On the other hand, if borrowed funds are
invested in projects which provide returns in excess of the cost of debt capital, then
shareholders will enjoy increased returns on their equity.
Gearing, however, also increases the probability of financial failure occurring through a
company's inability to meet interest payments in poor trading circumstances.

Example: gearing
Suppose that two companies are identical in every respect except for their gearing. Both have
assets of RWF20 m and both make the same operating profits (profit before interest and tax:
PBIT). The only difference between the two companies is that Nonlever Co is all-equity
financed and Lever Co is partly financed by debt capital, as follows.

Assets

Nonlever Co

Lever Co

RWF ‘000

RWF ‘000

20,000

10% Loan stock

0

Ordinary shares of RWF 1,000

20,000
(10,000)

20,000

10,000

20,000

10,000

Because Lever Co has RWF10,000,000 of 10% loan stock it must make a profit before
interest of at least RWF1,000,000 in order to pay the interest charges. Nonlever Co, on the
other hand, does not have any minimum PBIT requirement because it has no debt capital. A
company, which is lower geared, is considered less risky than a higher geared company
because of the greater likelihood that its PBIT will be high enough to cover interest charges
and make a profit for equity shareholders.
Page 776

Operating gearing
Financial risk, as we have seen, can be measured by financial gearing. Business risk refers to
the risk of making only low profits, or even losses, due to the nature of the business that
the company is involved in. One way of measuring business risk is by calculating a
company's operating gearing or 'operational gearing'.

Operating gearing or leverage =

Contribution
Profit before interest and tax (PB

If contribution is high but PBIT is low, fixed costs will be high, and only just covered by
contribution. Business risk, as measured by operating gearing, will be high. If contribution is
not much bigger than PBIT, fixed costs will be low, and fairly easily covered. Business risk,
as measured by operating gearing, will be low.

Page 777

BLANK

Page 778

LIQUIDITY
A company can be profitable but at the same time get into cash flow problems. Liquidity
ratios (current and quick) and working capital turnover ratios give some idea of a
company's liquidity.

Profitability is of course an important aspect of a company's performance, and debt or gearing
is another. Neither, however, addresses directly the key issue of liquidity. A company needs
liquid assets so that it can meet its debts when they fall due.

Liquidity is the amount of cash a company can obtain quickly to settle its debts (and possibly
to meet other unforeseen demands for cash payments too).

Liquid assets
Liquid funds include:
a) Cash
b) Short-term investments for which there is a ready market, such as investments in
shares of other companies (NB not subsidiaries or associates)
c) Fixed-term deposits with a bank or building society, for example six month deposits
with a bank
d) Trade customers
e) Bills of exchange receivable

Some assets are more liquid than others. Inventories of goods are fairly liquid in some
businesses. Inventories of finished production goods might be sold quickly, and a
supermarket will hold consumer goods for resale that could well be sold for cash very soon.
Raw materials and components in a manufacturing company have to be used to make a
finished product before they can be sold to realise cash, and so they are less liquid than
Page 779

finished goods. Just how liquid they are depends on the speed of inventory turnover and the
length of the production cycle.
Non-current assets are not liquid assets. A company can sell off non-current assets, but
unless they are no longer needed, or are worn out and about to be replaced, they are necessary
to continue the company's operations. Selling non-current assets is certainly not a solution to
a company's cash needs, and so although there may be an occasional non-current asset item
which is about to be sold off, probably because it is going to be replaced, it is safe to
disregard non-current assets when measuring a company's liquidity.
In summary, liquid assets are current asset items that will or could soon be converted
into cash, and cash itself. Two common definitions of liquid assets are all current assets or
all current assets with the exception of inventories.
The main source of liquid assets for a trading company is sales. A company can obtain
cash from sources other than sales, such as the issue of shares for cash, a new loan or the sale
of non-current assets. But a company cannot rely on these at all times, and in general,
obtaining liquid funds depends on making sales and profits.

Why does profit not provide an indication of liquidity?
If a company makes profits, it should earn money, and if it earns money, it might seem that it
should receive more cash than it pays out. In fact, profits are not always a good guide to
liquidity. Two examples will show why this is so.
a) Suppose that company X makes all its sales for cash, and pays all its running costs in
cash without taking any credit. Its profit for the year just ended was as follows.
b)
RWF
Revenue
Less costs:

RWF
400,000

running costs

200,000

Depreciation

50,000
250,000
150,000

Profit
Less dividends (all paid)

80,000

Retained profits

70,000
Page 780

During the year, the company purchased a non-current asset for RWF180,000 and paid for it
in full.
Depreciation is not a cash outlay, and so the company's 'cash profits' less dividends were
sales less running costs less dividends = RWF120,000. However, the non-current asset
purchase required RWF180,000, and so the company's cash position worsened in the year by
RWF60,000, in spite of the profit.

a) Suppose that company Y buys three items for cash, each costing RWF5,000, and
resells them for RWF7,000 each. The buyers of the units take credit, and by the end of
the company's accounting year, they were all still customers.
(i)

The profit on the transactions is RWF2,000 per unit and RWF6,000 in total.

(ii)

The company has paid RWF15,000 to buy the goods, but so far it has received
no cash back from selling them, and so its cash position is so far RWF15,000
worse off from the transactions.

(iii) The effect so far of the transactions is:
Reduction in cash

RWF15,000

Increase in customers

RWF21,000

Increase in profit

RWF6,000

The increase in assets is RWF6,000 in total, to match the RWF6,000 increase in
profit, but the increase in assets is the net change in cash (reduced balance) and
customers (increased balance).
Both of these examples show ways in which a company can be profitable but at the same
time get into cash flow problems. If an analysis of a company's published accounts is to give
us some idea of the company's liquidity, profitability ratios are not going to be appropriate for
doing this. Instead, we look at liquidity ratios and working capital turnover ratios.

Page 781

Liquidity ratios
Current ratio
The standard test of liquidity is the current ratio. It can be obtained from the statement of
financial position (balance sheet), and is current assets/current liabilities.
A company should have enough current assets that give a promise of 'cash to come' to meet
its commitments to pay its current liabilities. Obviously, a ratio in excess of 1 should be
expected. Otherwise, there would be the prospect that the company might be unable to pay
its debts on time. In practice, a ratio comfortably in excess of 1 should be expected, but what
is 'comfortable' varies between different types of businesses.
Companies are not able to convert all their current assets into cash very quickly. In
particular, some manufacturing companies might hold large quantities of raw material
inventories, which must be used in production to create finished goods. Finished goods might
be warehoused for a long time, or sold on lengthy credit. In such businesses, where inventory
turnover is slow, most inventories are not very liquid assets, because the cash cycle is so
long. For these reasons, we calculate an additional liquidity ratio, known as the quick ratio or
acid test ratio.

Quick ratio
The quick ratio, or acid test ratio, is (current assets less inventories)/current liabilities.
This ratio should ideally be at least 1 for companies with a slow inventory turnover. For
companies with a fast inventory turnover, a quick ratio can be less than 1 without
suggesting that the company is in cash flow difficulties.
Do not forget the other side of the coin. The current ratio and the quick ratio can be bigger
than they should be. A company that has large volumes of inventories and customers might
be over-investing in working capital, and so tying up more funds in the business than it needs
to. This would suggest poor management of customers or inventories by the company.

Page 782

Turnover periods
We can calculate turnover periods for inventory, customers and suppliers (the question
below revises these calculations). The time taken to collect amounts due from customers is
known as the accounts receivable collection period. Credit from suppliers is known as the
accounts payable payment period. If we add together the inventory days and the days taken
to collect accounts owed from customers, this should give us an indication of how soon
inventory is convertible into cash. This gives us a further indication of the company's
liquidity.

Example
What are the liquidity and working capital ratios from the accounts of a manufacturer of
products for the construction industry, and comment on the ratios.

20X8
RWFm

20X7
RWFm

Revenue

2,065.0

1,788.7

Cost of sales

1,478.6

1,304.0

Gross profit

586.4

484.7

ASSETS
Current assets
Inventories

119.0

109.0

Customers (note 1)

400.9

347.4

4.2
48.2

18.8
48.0

572.3

523.2

EQUITY AND LIABILITIES
Non-current liabilities
Loans and overdrafts

49.1

35.3

Taxes

62.0

46.7

Dividend

19.2

14.3

370.7

324.0

501.0

420.3

RWFm
71.3

RWFm
102.9

Short-term investments
Cash at bank and in hand

Suppliers (note 2)

Net current assets
Page 783

20X8
Notes
1
Trade customers
2
Trade suppliers

20X7

329.8
236.2

285.4
210.8

Solution
20X8

20X7

Current ratio

572.3/501.0 = 1.14

523.2/420.3 = 1.24

Quick ratio

453.3/501.0 = 0.90

414.2/420.3 = 0.99

Accounts receivable collection
period

329.8/2,065.0 × 365 = 58
days

285.4/1,788.7 × 365 = 58
days

Inventory turnover period

119.0/1,478.6 × 365 = 29
days

109.0/1,304.0 × 365 = 31
days

Accounts payable payment
period

236.2/1,478.6 × 365 = 58
days

210.8/1,304.0 × 365 = 59
days

As a manufacturing group serving the construction industry, the company would be expected
to have a comparatively lengthy accounts receivable collection period, because of the
relatively poor cash flow in the construction industry. It is clear that the company
compensates for this by ensuring that they do not pay for raw materials and other costs before
they have sold their inventories of finished goods (hence the similarity of accounts receivable
and accounts payable turnover periods).
The company's current ratio is a little lower than average but its quick ratio is better than
average and very little less than the current ratio. This suggests that inventory levels are
strictly controlled, which is reinforced by the low inventory turnover period. It would seem
that working capital is tightly managed, to avoid the poor liquidity which could be caused by
a high accounts receivable collection period and comparatively high suppliers.
The accounts payable payment period is ideally calculated by the formula (trade accounts
payable/purchases) × 365.
However, it is rare to find purchases disclosed in published accounts and so cost of sales
serves as an approximation. The ratio often helps to assess a company's liquidity; an increase
is often a sign of lack of long-term finance or poor management of current assets, resulting in
the use of extended credit from suppliers, increased bank overdraft and so on
Page 784

SHORT-RUN AND LONG-RUN FINANCIAL
PERFORMANCE
Short-termism is often due to the fact that managers' performance is measured on short-term
results.

In the previous chapter we saw how organisations often have to make a trade-off between
short-term and long-term objectives which can, of course, be focused on financial
performance. Advertising expenditure may be cut to increase short-term profit, but this is
likely to be at the expense of long-term financial results.
Earlier on in this chapter we looked at how RI, ROI and NPV are used to measure profitability.
Exam questions may test how useful these measures are for long-run and short-run
performance measurement.

Case Study
In April 2001, the Financial Times reported on how efforts to cut costs to boost short-term
profits at Marks & Spencer had long-term implications.
To fulfil expectations during the 1990s, staff numbers were limited or reduced, store
enhancements were restricted, and relationships with suppliers squeezed. As a result, earnings
matched market expectations for a while but eventually 'customers started to notice that value
for money was not quite as good as it could have been. That you had to wait to get the
attention of a sales assistant. That the shops were dowdy and so was some of the
merchandise. These impressions accumulated. Gradually the positive Marks & Spencer
anecdotes were replaced by negative ones. Suddenly the company's reputation fell off a cliff.
And so did its profits.'

Page 785

Using ROI
Suppose that an investment in a non-current asset would cost RWF100,000,000 and make a
profit of RWF11,000,000 p.a. after depreciation. The asset would be depreciated by
RWF25,000,000 pa for four years. It is group policy that investments must show a minimum
return of 15%. The DCF net present value of this investment would just about be positive,
and so the investment ought to be approved if group policy is adhered to.
Year

Cash flow (profit before dep'n)
RWF ‘000

Discount factor

Present value

15%

RWF ‘000

0

(100,000)

1.000

(100,000)

1

36,000

0.870

31,320

2

36,000

0.756

27,216

3

36,000

0.658

23,688

4

36,000

0.572

20,592

NPV

2,816

If the investment is measured year by year according to the accounting ROI it has earned, its
return is less than 15% in year 1, but more than 15% in years 2, 3 and 4.

Year

Profit

Net book value of equipment (mid-year ROI
value)

RWF ‘000

RWF ‘000

1

11,000

87,500

12.6%

2

11,000

62,500

17.6%

3

11,000

37,500

29.3%

4

11,000

12,500

88.0%

Page 786

In view of the low accounting ROI in year 1, should the investment be undertaken or not?
a) Strictly speaking, investment decisions should be based on DCF yield, and should
not be guided by short-term accounting ROI.
b) Even if accounting ROI is used as a guideline for investment decisions, ROI should
be looked at over the full life of the investment, not just in the short term. In the short
term (in the first year or so of a project's life) the accounting ROI is likely to be low
because the net book value of the asset will still be high.

DCF v ROI
In spite of the superiority of DCF yield over accounting ROI as a means of evaluating
investments, and in spite of the wisdom of taking a longer-term view rather than a short-term
view with investments, it is nevertheless an uncomfortable fact that the consideration of
short-run accounting ROI does often influence investment decisions.
In our example, it is conceivable that the group's management might disapprove of the project
because of its low accounting ROI in year 1. This approach is short-sighted, but it
nevertheless can make some sense to a company or group of companies which has to show a
satisfactory profit and ROI in its published accounts each year, to keep its shareholders
satisfied with performance.
A similar misguided decision would occur where a divisional manager is worried about the
low ROI of his division, and decides to reduce his investment by scrapping some machinery
which is not currently in use. The reduction in both depreciation charges and assets would
immediately improve the ROI. When the machinery is eventually required the manager
would then be obliged to buy new equipment. Such a situation may seem bizarre, but it does
occur in real life.
Short-term ROI should not be used to guide management decisions but there is a difficult
motivational problem. If management performance is measured in terms of ROI, any
decisions which benefit the company in the long term but which reduce the ROI in the
immediate short term would reflect badly on the manager's reported performance. In other
words, good investment decisions would make a manager's performance seem worse than if
the wrong investment decision were taken instead.

Page 787

BLANK

Page 788

PROFITS AND SHARE VALUE
The value of the P/E ratio reflects the market's appraisal of the shares' future prospects.

Shareholders value shares on the basis not of past performance but of expectations of
future performance.
Note that past performance is useful, however, in that it gives information about the quality
of the management team, and the business' success at devising and executing strategies to
maximise shareholders’ wealth, to date.
Shareholders may have a view towards a particular industry sector as well as an
individual business. No matter how well a business is run, it may operate in an unattractive
or mature industry sector.
Investors may have a genuinely different view of the prospects of a sector from managers, so
even well-run companies in an industry may feel starved of capital at an appropriate rate.
This is because they are always compared with other companies.

The management issues are contradictory.
a) Managers have a personal interest in the long-term survival of the business.
b) Shareholders want a long-term increase in their wealth from investment in a business
or other companies in the sector.

Short-termism often occurs, however.
a) Managers' performance is measured on short-term results (for example quarterly
reporting in the US).
b) Even investors are under pressure to maximise the growth in value of their portfolios
in a particular period.

Page 789

The price/earnings (P/E) ratio: profits and share value
The P/E ratio is the most important yardstick for assessing the relative worth of a share. It is:
Market price in ce
EPS in cents

Total market value of eq
Total earnings

which is the same as

The value of the P/E ratio reflects the market's appraisal of the shares' future prospects.
In other words, if one company has a higher P/E ratio than another it is because investors
either expect its earnings to increase faster than the other's or consider that it is a less risky
company or in a more 'secure' industry. The P/E ratio is, simply, a measure of the
relationship between the market value of a company's shares and the earnings from
those shares.

Example: price earnings ratio
A US company has recently declared a dividend of 12c per share. The share price is $3.72
cum div and earnings for the most recent year were 30c per share. Calculate the P/E ratio.
Solution
P/E ratio =

MV ex div
EPS

=

$3.60
30c

=

12

Changes in EPS: the P/E ratio and the share price
The dividend valuation model or fundamental theory of share values is the theory that share
prices are related to expected future dividends on the shares.
A common sense approach to assessing what share prices ought to be, which is often used in
practice, is a P/E ratio approach.
a) The relationship between the EPS and the share price is measured by the P/E ratio
b) There is no reason to suppose, in normal circumstances, that the P/E ratio will vary
much over time
Page 790

c) So, if the EPS goes up or down, the share price should be expected to move up or
down too, and the new share price will be the new EPS multiplied by the constant P/E
ratio
For example, if a company had an EPS last year of 300 francs and a share price of
RWF3.600, its P/E ratio would have been 12. If the current year's EPS is 330 francs, we
might expect that the P/E ratio would remain the same, 12, and so the share price ought to go
up to 12 × 33c0= RWF3,960.

Internet companies
In 1999/2000 share prices in the US and Europe rose to unprecedented heights. The drivers
for this were the rise of technology stocks, particularly those relating to internet companies.
There were a number of causes of this.
a) The internet appeared to offer unrivalled opportunities for growth. Everybody wanted
to jump on the bandwagon.
b) There were influential proponents of the 'new economy' who felt that some economic
laws had been re-written.
c) Internet firms offered increasing returns to scale thanks to network effects. In other
words, the more people using the Internet, the more useful it becomes for others to
use.
d) However, despite exciting websites and huge marketing expenditure, internet
companies (such as Boo.com) were made or broken on issues of logistics and
distribution.

Many internet firms used up large amounts of cash before attaining any profits, and so have
collapsed.
a) B2C (business-to-consumer companies) such as Boo.com lost money. Other
retailing sites kept going, however. Even so, amazon.com laid off staff. One of the
most successful Internet retailers in the UK is 'old economy' Tesco.
b) B2B (business-to-business internet companies) have had more success, if they offer
something of value.
Page 791

In fact, a recent study of ‘tech’ companies by Merrill Lynch reported that their earnings were
overstated by an average of 25% compared with what they would be if determined on the
basis of generally accepted accounting principles.
Despite the heady days of 2000, it is a fallacy that Internet companies can avoid the need for
profit and positive cash inflows.
'From peak to trough, Amazon.com's market value sank by $35 billion as Jeff Bezos
(Time magazine's 'person of the year' in 1999) claimed that his company was profitable
on a 'proforma basis'. But let's get real: its proforma profits were found by ignoring
interest payments on nearly $2 billion of debt. That's like saying my holiday home
doesn't cost me anything – as long as I ignore the mortgage payments.'(Ted
Stone,
'Trade Secrets', CIMA Insider, June 2002)
The Internet share market has learnt its lesson. Here's a recent example of how Google can
generate accounting profits and still excite the stock market into huge valuations.

Case Study
'In the interests of decorum, professionalism, etc., analysts will no doubt offer careful
assessments of Google's Q3 results. And the wires will bustle with stories about how
ridiculous it is that a stock that went public at US$85 Aug 2004 is now trading at $350ish,
etc.
The real story? These results are absolutely staggering.
Google's stock price – shocking though it is – is much less amazing than Google's
fundamental performance, which is simply not to be believed. A 7 year old company with a
revenue run-rate of $6 billion, an annual growth rate near 100%, 43% EBITDA margins,
100% plus return-on-invested-capital, a dominant global franchise, and already about half the
cash flow of Time Warner (a 100 year-old company with 85,000 employees). '
Source: Internet Outsider, October 2005
[In Jan 2102 Google is trading between $500- $600s - editor’s note]

Page 792

COMPARISONS OF ACCOUNTING FIGURES
Comparisons might be made between a company's results and the results of the most recent
year/previous years, other companies in the same industry and other companies in other
industries.

Results of the same company over successive accounting periods
Although a company might present useful information in its five-year or ten-year summary, it
is quite likely that the only detailed comparison you will be able to make is between the
current year's and the previous year's results. The comparison should give you some idea of
whether the company's situation has improved, worsened or stayed much the same between
one year and the next.

Useful comparisons over time include:
a) Percentage growth in profit (before and after tax) and percentage growth in revenue
b) Increases or decreases in the debt ratio and the gearing ratio
c) Changes in the current ratio, the inventory turnover period and the accounts
receivable collection period
d) Increases in the EPS, the dividend per share, and the market price

The principal advantage of making comparisons over time is that they give some indication
of progress: are things getting better or worse? However, there are some weaknesses in such
comparisons.
a) The effect of inflation should not be forgotten.
b) The progress a company has made, needs to be set in the context of what other
companies have done, and whether there have been any special environmental or
economic influences on the company's performance.

Page 793

Putting a company's results into context
The financial and accounting ratios of one company should be looked at in the context of
what other companies have been achieving, and also any special influences on the industry
or the economy as a whole. Here are two examples.
a) If a company achieves a 10% increase in profits, this performance taken in isolation
might seem commendable, but if it is then compared with the results of rival
companies, which might have been achieving profit growth of 30% the performance
might in comparison seem very disappointing.
b) An improvement in ROCE and profits might be attributable to a temporary economic
boom, and an increase in profits after tax might be attributable to a cut in the rate of
corporation tax. When improved results are attributable to factors outside the control
of the company's management, such as changes in the economic climate and tax rates
other companies might be expected to benefit in the same way.

Comparisons between different companies in the same industry
Making comparisons between the results of different companies in the same industry is a way
of assessing which companies are outperforming others.
a) Even if two companies are in the same broad industry (for example retailing)
they might not be direct competitors. For example, in the UK, the Kingfisher group
(hardware and garden supplies) does not compete directly with the Arcadia group
(clothes). Even so, they might still be expected to show broadly similar
performance, in terms of growth, because a boom or a depression in retail markets
will affect all retailers. The results of two such companies can be compared, and the
company with the better growth and accounting ratios might be considered more
successful than the other.
b) If two companies are direct competitors, a comparison between them would be
particularly interesting. Which has achieved the better ROCE, sales growth, or
profit growth? Does one have a better debt or gearing position, a better liquidity
position or better working capital ratios? How do their P/E ratios, dividend cover and
dividend yields compare? And so on.

Page 794

Comparisons between companies in the same industry can help investors to rank them in
order of desirability as investments, and to judge relative share prices or future prospects. It is
important, however, to make comparisons with caution: a large company and a small
company in the same industry might be expected to show different results, not just in terms of
size, but in terms of:
a) Percentage rates of growth in sales and profits
b) Percentages of profits re-invested (Dividend cover will be higher in a company that
needs to retain profits to finance investment and growth.)
c) Non-current assets (Large companies are more likely to have freehold property in
their statement of financial position (balance sheet) than small companies.)

Comparisons between companies in different industries
Useful information can also be obtained by comparing the financial and accounting ratios of
companies in different industries. An investor ought to be aware of how companies in one
industrial sector are performing in comparison with companies in other sectors. For example,
it is important to know:
a) Whether sales growth and profit growth is higher in some industries than in others
(For example, how does growth in the financial services industry compare with
growth in heavy engineering, electronics or leisure?)
b) How the return on capital employed and return on shareholder capital compare
between different industries
c) How the P/E ratios and dividend yields vary between industries (For example, if a
publishing company has a P/E ratio of, say, 20, which is average for its industry,
whereas an electronics company has a P/E ratio of, say, 14, do the better growth
performance and prospects of the publishing company justify its higher P/E ratio?)

Page 795

CHAPTER ROUNDUP
•

The overriding purpose of a business is to increase owner wealth in the long-term.

•

Achieving objectives of survival and growth ultimately depends on making profits.

•

Measures relating to profit include sales margin, EBITDA and EPS. More
sophisticated measures (ROCE, ROI) take the size of investment into account. Later
on in the chapter we considered how measures of profitability are used for short-run
or long-run performance measurement. Bear this in mind particularly when you
study the sections on RI, ROI and NPV and go through the example covering these.

•

As well as profitability, liquidity and gearing are key measures of performance.

•

Gearing ratios measure the financial risk of a company's capital structure. Business
risk can be measured by calculating a company's operational gearing.

•

A company can be profitable but at the same time get into cash flow problems.
Liquidity ratios (current and quick) and working capital turnover ratios give some
idea of a company's liquidity.

•

Short-termism is often due to the fact that managers' performance is measured on
short-term results.

•

The value of the P/E ratio reflects the market's appraisal of the shares' future
prospects.

•

Comparisons might be made between a company's results and the results of the most
recent year/previous years, other companies in the same industry and other companies
in other industries.

Page 796

STUDY UNIT 24
Non-Financial Performance Indicators
Contents

Page

Disadvantages Of Financial Performance Indicators …………….……..

799

Growing Emphasis On NFPIs ……………………………..………..……..

801

The Value Of NFPIs ……………….…………………………..…………….

805

NFPIs In Relation To Employees ………………..…………….................…

807

NFPIs In Relation To Product / Service Quality …………………………

809

Qualitative Issues ………………….…………………………….……….…

813

Page 797

EXAM GUIDE
You need to think about how you would use the information here on NFPIs in a report to
advise management. Also you must think about the action words used in the study guide so
you need to 'discuss' and 'identify' in your exam answer.

Page 798

DISADVANTAGES OF FINANCIAL PERFORMANCE
INDICATORS
Concentration on financial indicators means that important goals and factors may be
ignored.

Concentration on too few variables
If performance measurement systems focus entirely on those items which can be expressed in
monetary terms, managers will concentrate on only those variables and ignore other
important variables that cannot be expressed in monetary terms.
For example, pressure from senior management to cut costs and raise productivity will
produce short-term benefits in cost control but, in the long term, managerial performance
and motivation is likely to be affected, labour turnover will increase and product quality will
fall.
Reductions in cost can easily be measured and recorded in performance reports, employee
morale cannot. Performance reports should therefore include not only costs and revenues
but other important variables, to give an indication of expected future results from present
activity.

Lack of information on quality
Traditional ‘responsibility’ accounting systems also fail to provide information on the
quality or importance of operations. Drury provides the following example.

'Consider a situation where a purchasing department regularly achieved the budget for all
expense items. The responsibility performance reporting system therefore suggests that the
department was well managed. However, the department provided a poor service to the
production departments. Low-cost suppliers were selected who provided poor quality
materials and frequently failed to meet delivery dates. This caused much wasted effort in
chasing up orders and prejudiced the company's ability to deliver to its customers on time.'

Page 799

Measuring success, not ensuring success
Financial performance indicators have been said simply to measure success. What
organisations also require, however, are performance indicators that ensure success. Such
indicators, linked to an organisation's critical success factors such as quality and flexibility,
will be non financial in nature.

Page 800

GROWING EMPHASIS ON NFPIs
Changes in cost structures, the competitive environment and the manufacturing environment
have lead to an increased use of NFPIs.

Impact of changes in cost structures and the manufacturing and
competitive environments
These have led to a shift from treating financial figures as the foundation of performance
measurement to treating them as one of a range of measures.

Changes in cost structures
Modern technology requires massive investment and product life cycles have got shorter. A
greater proportion of costs are sunk and a large proportion of costs are planned, engineered or
designed into a product/service before production/delivery. At the time the product/service
is produced/delivered, it is therefore too late to control costs.

Changes in competitive environment
Financial measures do not convey the full picture of a company's performance, especially
in a modern business environment.
'In today's worldwide competitive environment companies are competing in terms of
product quality, delivery, reliability, after-sales service and customer satisfaction. None
of these variables is directly measured by the traditional responsibility accounting
system, despite the fact that they represent the major goals of world-class
manufacturing companies.'

Changes in manufacturing environment
New manufacturing techniques and technologies focus on minimising throughput times,
inventory levels and set-up times. But managers can reduce the costs for which they are
responsible by increasing inventory levels through maximising output. If a performance
Page 801

measurement system focuses principally on costs, managers may concentrate on cost
reduction and ignore other important strategic manufacturing goals.

Introducing NFPIs
Many companies are therefore discovering the usefulness of quantitative and qualitative nonfinancial performance indicators (NFPIs). The following definition from CIMA's Official
Terminology is useful because of the examples it provides.

Non-financial performance measures are 'measures of performance based on non-financial
information which may originate in and be used by operating departments to monitor and
control their activities without any accounting input.
Non-financial performance measures may give a more timely indication of the levels of
performance achieved than do financial ratios, and may be less susceptible to distortion by
factors such as uncontrollable variations in the effect of market forces on operations.

Examples of non-financial performance measures:

Area assessed

Performance measure

Service quality

Number of complaints
Proportions of repeat bookings
Customer waiting time
On-time deliverie

Production performance

Set-up times
Number of suppliers
Days' inventory in hand
Output per employee
Material yield percentage
Schedule adherence
Proportion of output requiring rework
Manufacturing lead time
Page 802

Marketing effectiveness

Trend in market share
Sales volume growth
Customer visits per salesperson
Client contact hours per salesperson
Sales volume forecast v actual
Number of customers
Customer survey response information

Personnel

Number of complaints received
Staff turnover
Days lost through absenteeism
Days lost through accidents/sickness
Training time per employee.

Page 803

BLANK

Page 804

THE VALUE OF NFPIs
Ease of use
NFPIs do have advantages over financial indicators but a combination of both types of
indicator is likely to be most successful.
Unlike traditional variance reports, NFPIs can be provided quickly for managers, per shift,
daily or even hourly as required. They are likely to be easy to calculate, and easier for nonfinancial managers to understand and therefore to use effectively.
The beauty of non-financial indicators is that anything can be compared if it is meaningful
to do so. The measures should be tailored to the circumstances so that, for example, number
of coffee breaks per 20 pages of Study Text might indicate to you how hard you are studying!
Many suitable measures combine elements from the chart shown below. Use it to answer the
question below.

Errors/failure
Defects
Equipment failures
Warranty claims
Complaints
Returns

Time
Second
Minute
Hour
Shift
Cycle

Quantity
Range of products
Parts/components
Units produced
Units sold
Services performed

Stockouts
Lateness/waiting
Misinformation
Miscalculation
Absenteeism

Day
Month
Year

kg/litres/metres
m²/m³
Documents
Deliveries
Enquiries

People
Employees
Employee skills
Customers
Competitors
Suppliers

Here are five indicators, showing you how to use the chart, but there are many other
possibilities.
a) Services performed late v total services performed
b) Total units sold v total units sold by competitors (indicating market share)
c) Warranty claims per month
d) Documents processed per employee
e) Equipment failures per 1,000 units produced
Page 805

Now think some for yourself but don't forget to explain how the ones that you chose might be
useful.

NFPIs and financial measures
Arguably, NFPIs are less likely to be manipulated than traditional profit-related measures
and they should, therefore, offer a means of counteracting short-termism, since short-term
profit at any (non-monetary) expense is rarely an advisable goal. The ultimate goal of
commercial organisations in the long run is likely to remain the maximisation of profit,
however, and so the financial aspect cannot be ignored.
There is a danger that too many such measures could be reported, leading to information
overload for managers, providing information that is not truly useful, or that sends
conflicting signals. A further danger of NFPIs is that they might lead managers to pursue
detailed operational goals and become blind to the overall strategy in which those goals are
set.
A combination of financial and non-financial indicators is therefore likely to be most
successful.

The balanced scorecard
The need to link financial and non-financial measures of performance and to identify the
key performance measures provided the impetus for the development of the balanced
scorecard, which we looked at in Study Unit 22.

Page 806

NFPIs IN RELATION TO EMPLOYEES
NFPIs can usefully be applied to employees.

One of the many criticisms of traditional accounting performance measurement systems is
that they do not measure the skills, morale and training of the workforce, which can be as
valuable to an organisation as its tangible assets. For example if employees have not been
trained in the manufacturing practices required to achieve the objectives of the new
manufacturing environment, an organisation is unlikely to be successful. Indeed, in a service
industry such an accountant’s business, the people are the key assets
Employee attitudes and morale can be measured by surveying employees. Education and
skills levels, promotion and training, absenteeism and labour turnover for the employees for
which each manager is responsible can also be monitored.
The weighting attached to employee-oriented NFPIs when assessing managerial performance
should be high. High profitability or tight cost control should not be accompanied by 100%
labour revenue.

Page 807

BLANK

Page 808

NFPIs IN RELATION TO PRODUCT / SERVICE
QUALITY
NFPIs are extremely useful when assessing product/service quality.

Performance measurement in a TQM environment
TQM is a highly significant trend in modern business thinking. We look at it in more detail in
Study Unit 17 when we look at Japanese businesses practices and when considering the costs
of quality.
Because TQM embraces every activity of a business, performance measures cannot be
confined to the production process but must also cover the work of sales and distribution
departments and administration departments, the efforts of external suppliers, and the
reaction of external customers.
In many cases the measures used will be non-financial ones. They may be divided into three
types.

Measuring the quality of incoming supplies
The quality of output depends on the quality of input materials, and so quality control should
include procedures for acceptance and inspection of goods inwards and measurement of
rejects.
a) Inspection will normally be based on statistical sampling techniques and the concept
of an acceptance quality level (AQL).
b) Another approach that can be used is to give each supplier a 'rating' for the quality
of the goods they tend to supply, and give preference with purchase orders to wellrated suppliers.
c) Where a quality assurance scheme is in place, the supplier guarantees the quality of
goods supplied. This places the onus on the supplier to carry out the necessary quality
checks, or face cancellation of the contract.

Page 809

Monitoring work done as it proceeds
This will take place at various key stages in the production process. Inspection, based on
random sampling and other statistical techniques, will provide a continual check that the
production process is under control. The aim of inspection is not really to sort out the bad
products from the good ones after the work has been done. The aim is to satisfy
management that quality control in production is being maintained.
'In-process' controls include statistical process controls and random sampling, and measures
such as the amount of scrap and reworking in relation to good production. Measurements can
be made by product, by worker or work team, by machine or machine type, by department, or
whatever is appropriate.

Measuring customer satisfaction
Some sub-standard items will inevitably be produced. In-process checks will identify some
bad output, but other items will reach the customer who is the ultimate judge of quality.
'Complaints' may be monitored in the form of letters of complaint, returned goods, penalty
discounts, claims under guarantee, or requests for visits by service engineers.
Some companies adopt a more pro-active approach to monitoring customer satisfaction by
surveying their customers on a regular basis. They use the feedback to obtain an index of
customer satisfaction which is used to identify quality problems before they affect profits.

Quality of service
Service quality is measured principally by qualitative measures, as you might expect,
although some quantitative measures are used by some businesses.
a) If it were able to obtain the information, a retailer might use number of lost customers
in a period as an indicator of service quality.
b) Lawyers use the proportion of time spent with clients.

Fitzgerald et al identify 12 factors pertaining to service quality and the following table shows
the measures used and the means of obtaining the information by British Airports Authority,
a mass transport service:
Page 810

Service quality factors

Measures

Mechanisms

Access

Walking distances
Ease of finding way around

Customer survey and internal
operational data
Customer survey

Aesthetics/appearance

Staff appearance
Airport's appearance
Quantity, quality,
appearance of food

Customer survey
Customer survey
Management inspection

Availability

Equipment availability

Internal fault monitoring
system and customer survey
Customer survey and internal
operational data

Cleanliness/tidiness

Cleanliness of environment
and equipment

Customer survey and
management inspection

Comfort

Crowdedness of airport

Customer survey and
management inspection

Communication

Information clarity
Clarity of labelling and
pricing

Customer survey
Management inspection

Courtesy

Courtesy of staff

Customer survey and
management inspection

Friendliness

Staff attitude and
helpfulness

Customer survey and
management inspection

Reliability

Number of equipment
faults

Internal fault monitoring
systems

Responsiveness

Staff responsiveness

Customer survey

Security

Efficiency of security
checks
Number of urgent safety
reports

Customer survey
Internal operational data

Page 811

BLANK

Page 812

QUALITATIVE ISSUES
Qualitative factors are not easily measured and so, in management accountancy, they can be
'those factors which can be expressed in monetary terms only with much difficulty or
imprecision'.

There will often be no conclusion that you as the management accountant can draw from
qualitative information. Your job is to be aware of its existence and report it under the
heading of 'other matters to be considered'. In practice of course, many decisions are
finally swayed by the strength of the qualitative arguments rather than the cold facts
presented in the quantitative analysis, and rightly so.

Exam Focus Point
As a general guideline, if you are asked to comment on qualitative issues, you should
consider matters such as the following.
a) The impact on or of human behaviour. What will be the reaction on the factory floor?
How will managers feel? Will customers be attracted or deterred? Can suppliers be
trusted?
b) The impact on or of the environment ('surroundings'). Is the country in a recession? Is
government or legislation influential? Are there 'green' issues to be considered? What
is the social impact? What action will competing companies take? Is changing
technology a help or a hindrance?
c) The impact on or of ethics. Is the action in the public interest? Are we acting
professionally? Are there conflicts of interest to be considered? Will fair dealing help
to win business? Are we treating staff properly?

Page 813

Branding
Brand identity conveys a lot of information very quickly and concisely. This helps customers
to identify the goods or services and thus helps to create customer loyalty to the brand. It is
therefore a means of increasing or maintaining sales.
Where a brand image promotes an idea of quality, a customer will be disappointed if his
experience of a product fails to live up to his expectations. Quality control is therefore of
utmost importance. It is essentially a problem for service industries such as hotels, airlines
and retail stores, where there is less possibility than in the manufacturing sector of detecting
and rejecting the work of an operator before it reaches the customer. Bad behaviour by
an employee in a face-to-face encounter with a customer will reflect on the entire company
and possibly deter the customer from using any of the company's services again.
Brand awareness is an indicator of a product's/organisation's place in the market.
Recall tests can be used to assess the public's brand awareness.

Company profile
Company profile is how an organisation is perceived by a range of stakeholders. For
example, stakeholders may have a negative attitude towards an organisation, perhaps as a
result of an ethical issue or a crisis that has struck the organisation and perhaps of the
associated media comment. Market research can determine company profile and marketing
campaigns can improve it if necessary.

Page 814

CHAPTER ROUNDUP
•

Concentration on financial indicators means that important goals and factors may
be ignored.

•

Changes in cost structures, the competitive environment and the manufacturing
environment have led to an increased use of NFPIs.

•

NFPIs do have advantages over financial indicators but a combination of both types
of indicator is likely to be most successful.

•

NFPIs can usefully be applied to employees.

•

NFPIs are extremely useful when assessing product/service quality.

•

Qualitative factors are 'those that can be expressed in monetary terms only with
much difficulty or imprecision'.

Page 815

BLANK

Page 816

STUDY UNIT 25
Impact Of Developments In Information Technology & ECommerce
Contents

Page

Information Needs Of Manufacturing And Service Businesses …………

819

Instant Access To Data …………………..………………………….……..

833

Remote Input Of Data ………………………………………..…………….

841

Developing Management Accounting Systems ………………………...…

843

Game Theory ………………………………………………..……......……

849

Page 817

EXAM GUIDE
This chapter covers material that is integral to your understanding of management
accounting. You should understand from your workplace how information technology is used
to communicate information around the organisation. You should expect a written question
or part-question covering this material.

Page 818

INFORMATION NEEDS OF MANUFACTURING AND
SERVICE BUSINESSES
Information needs of manufacturing businesses
All manufacturing businesses follow a simple model.

The information required by even modern manufacturing organisations is still based on the
demands of this model.
A variety of performance indicators are used by manufacturing businesses, but there are
some over-riding considerations.
Consideration

Detail

Cost behaviour

Labour is generally considered a variable cost. Machinery is a fixed cost.
Modern technology requires more overheads. (With advanced
manufacturing technology, there is a higher proportion of fixed equipment
costs compared with variable labour costs.)
Important in terms of output adherence to production specification.
Production bottlenecks; delivery times; deadlines; machine speed
Required in products and processes
Despite the tendency towards low inventory and just in time delivery, many
businesses still have to give a value to inventories of raw materials or
finished goods, as a major element in their profit calculations. Whether
complicated tracking systems are needed is a different question.

Quality
Time
Innovation
Valuation

We look at the first four of these considerations in more detail in the following paragraphs.

Page 819

Cost behaviour, quality, time and innovation
a) Cost behaviour

Uses

Comment

Planning

Standard costs can be outlined, and actual costs compared with
them.

Decision
making

Estimates of future costs may be needed to assess the likely
profitability of a product.

Control

Total cost information can be monitored, to ensure the best rates
for supplies.

b) Quality information is used to ensure that 'customer satisfaction' is built into the
manufacturing system and its outputs.

Uses

Comment

Planning

Ensure that products are well designed and manufactured
according to specification.

Decision
making

Businesses have a choice as to what level of quality they 'build'
into a product. Quality need not be perfection, it is 'fitness for the
purpose for which intended'.

Control

Falling levels of quality are an alarm bell – if products are not
manufactured according to their design specification, there will
be more rejects, more waste and more dissatisfied customers.
This means higher costs and lower profits.

Page 820

c) Time

Uses

Comment

Planning

Manufacturing time has to be scheduled to ensure the most
efficient use of the system; if production can be smoothed over a
period, this ensures effective capacity utilisation. Throughput time
is thus important.

Decision
making

Time is relevant to decision making, as it indicates a firm's ability
to keep its promises to its customers for delivery and so on.

Control

• New
product
implementation)

development

(from

conception

to

• Speed of delivery
• Bottlenecks
• In just-in-time systems, where firms hold little material
inventories, time is a measure of a factory's ability to function
at all. Inventory levels will be measured not in units but in
day's supplies
• As a measure of efficiency (e.g. inventory revenue, asset
turnover)

d) Innovation
Uses

Comment

Planning

• New product development
• Speed to market
• New process

Control

This generally refers to the launch and design of new products.

The experience curve can be used in strategic control of costs and is relevant to 'time' and
'innovation'. It suggests that as output increases, the cost per unit of output falls, for these
reasons.

Page 821

a) Economies of scale – in other words an increased volume of production leads to
lower unit costs, as the firm approaches full capacity.
b) A genuine 'learning effect' as the workforce becomes familiar with the job and learns
to carry out the task more efficiently. As a process is repeated, it is likely that costs
will reduce due to efficiency, discounts and reduced waste.
c) Technological improvements.

This brings us on to target costing, covered in your study of Paper F5 Performance
Management or the previous syllabus Paper 2.4
a) In the short run, because of development costs and the learning time needed, costs are
likely to exceed price.
b) In the longer term, costs should come down (for example, because of the experience
curve) to their target level.

Strategic, tactical and operational information
The information requirements of manufacturing businesses can also be considered in terms of
three levels.

Information type

Examples

Strategic

Future demand estimates
New product development plans
Competitor analysis

Tactical

Variance analysis
Departmental accounts
Inventory turnover

Operational

Production reject rates
Materials and labour used
Inventory levels

Page 822

The information requirements of commercial organisations are influenced by the need to
make and monitor profit. Information that contributes to the following measures is important.
a)
b)
c)
d)

Changeover times
Number of common parts
Level of product diversity
Product and process quality

Service businesses
Unlike manufacturing companies, services are characterised by intangibility, inseparability,
variability, perishability and no transfer of ownership.
Before we delve into the detail, here's the big picture. According to a 2006 Office for
National Statistics ONS report, the service sector made up 73% of the UK economy. 32 of the
top UK companies were in the service sector and one third of those employed over one
million people in total.
Despite the apparent domination in the service sector as a whole by a few large companies, in
reality most service organisations are very small (as normal experience would suggest): for
instance: hair-dresser, restaurants, cafés, CPA businesses.. So we are talking of a very large
number of organisations, many of them quite small, but collectively accounting for a
powerful proportion of the workforce.

Types of service business
Mass services are standard services provided to large numbers of people, and are often
automated. Personal services vary on the circumstances of the service delivery, and are
generally one-to-one.
With this in mind, we can identify different types of service.

Page 823

Type

Comment

Mass service

The delivery of the same, very standardised service to many people, as
a transaction, for example cheque processing.

Personalised
service

This service is unique to the recipient, such as dentistry: every mouth is
different, even though standard procedures are adopted to ensure best
practice.

Examples of service businesses include:
a) Mass service e.g. the banking sector, transportation (bus, air), mass entertainment
b) Either/or e.g. fast food, teaching, hotels and holidays, psychotherapy
c) Personal service e.g. pensions and financial advice, car maintenance

Service activities therefore cut across all sectors of the economy. In the UK, healthcare is
provided by the public sector but also by the private sector (for-profit). The objectives may
differ even though the activities remain the same.

'Services are any activity of benefit that one party can offer to another that is essentially
intangible and does not result in the ownership of anything. Its production may or may not be
tied to a physical product.'
(P Kotler, Social Marketing)

There are five major characteristics that distinguish services from manufacturing.
a) Intangibility refers to the lack of substance, which is involved with service delivery.
Unlike goods (physical products such as confectionery), there is no substantial
material or physical aspects to a service: no taste, feel, visible presence and so on. For
example, if you go to the theatre, you cannot take the 'play' away with you.
b) Inseparability/simultaneity. Many services are created at the same time as they are
consumed. (Think of dental treatment.) No service exists until it is actually being
experienced/consumed by the person who is buying it.
c) Variability/heterogeneity. Many services face the problem of maintaining
consistency in the standard of output. It may be hard to attain precise
standardisation of the service offered, but customers expect it (such as with fast food).
Page 824

Here also is the paradox: Is fast food a service or not? – the customer walks out with
a burger or whatever (the tangible product), but cannot simply lift it off the shelf until
it has been prepared and cooked – the intangible element.
d) Perishability. Services are innately perishable. The services of a beautician are
purchased for a period of time.
e) No transfer of ownership. Services do not result in the transfer of property. The
purchase of a service only confers on the customer access to or a right to use a
facility.

Most 'offers' to the public contain a product and service element.

Exam Focus Point
You could be asked to discuss any of these characteristics in the exam and use examples from
your own experience.

Quantitative/qualitative information and services
Service businesses need the same aggregate information as manufacturing firms, but also
need performance data as to their cost and volume drivers. Operational information is likely
to be more qualitative.

A dental practice needs a mix of quantitative and non-quantitative information to price its
services properly, to optimise capacity utilisation and to monitor performance. Many small
service businesses have similar concerns, for example garages or beauty parlours.
a) They need to control the total cost of providing the service operation.
b) They need positive cash flow to finance activities.
c) They need operating information to identify how costs are incurred and on what
services.
Page 825

Arguably, small service businesses, whose expenses are mainly overheads, provide a model,
in miniature, of the requirements of activity based costing, mentioned in Study Unit 1.
Are 'mass services' any different?
a) Because mass services, such as cheque clearing, are largely automated, there may be a
large fixed cost base.
b) Even if a service is heavily automated, each time the service is performed is a
'moment of truth' for the customer. Ensuring consistency and quality is important but
this is true for small service businesses too.

Quantitative information is information that can be expressed in numbers. A sub-category
of quantitative information is financial information (also known as monetary information),
which is information that can be expressed in terms of money.
Qualitative information is information that cannot be expressed in numbers.
Non-financial information (or non-monetary information) is information that is not
expressed in terms of money, although this does not mean that it cannot be expressed in terms
of numbers.

Below are a few examples of monetary and non-monetary information for a monthly
report for a dentist practice. (Hint. Ask yourself what is the key resource of the practice).

a) Monthly receipts and payments
(i)

(ii)

Receipts include payments from the government for publicly-funded work, fees
for private work and so on. Dentists are measured on Units of Dental Activity
(UDA) and are given annual targets for UDAs that they must undertake.
Payments include operating costs such as wages for nursing staff, reception staff,
rent, insurance, electricity, telephone expenses, medical equipment, medicine and
so on.

b) Capacity utilisation. In other words, how busy has the practice been? Have all
available appointments been booked or were there times when the dentist and his/her
staff were kicking their heels? Just by looking at the appointments diary you can
make comparatives.
Page 826

c) Treatment costs. Simple treatments such as teeth cleaning can be performed by the
dental hygienist. Other treatments, such as root canal surgery, require the dentist and
perhaps a dental nurse in attendance.
The cost of providing these different treatments will vary depending on the level of
staff and complexity of the treatment.
The cost driver is time.
The mix of treatments offered is thus significant in the total profitability of the practice.
The practice will probably profit more from relatively expensive treatments, such as
'crowns', but these come at a cost. Patients can also have several treatments within a
price band and be charged a single fixed price. This may also have an effect on just
how much work the dentist is willing to do for a single fixed charge.
The new treatment charges introduced in the UK in April 2006 have also affected how
dental services are provided. Patients now pay one of three fixed charges based on the
type of care and treatment required.

However, this information, while useful to monitor the financial health of the practice, does
not give us a sufficiently detailed picture of the operating performance. The key resource is
time, the dentist's time, and staff time.
For the long-term health of the practice, matters such as customer satisfaction and repeat
business must be considered. (Does your dentist remind you to have a check up every six
months?)

Colin Drury (Management and Cost Accounting) describes qualitative factors as those 'that
cannot be expressed in monetary terms’. Thus the information 'German people are very fond
of bananas' could be expressed as 'The value of the German banana market is Rwf x million
pa', but the value of x is very questionable.

Page 827

Example: qualitative information
As a more elaborate example, consider a firm that is thinking of sacking many of its customer
service staff and replacing them by automated telephone answering systems. Now consider
how difficult it would be to obtain the following information in order to appraise a decision
whether or not to replace staff with an untested system.
a) The cost of being sure that the new system would do the job as well as people can
b) The cost of loss of morale amongst other workers if large numbers are made
redundant
c) The cost of compensating the redundant staff for the psychological and financial
impact of the decision on themselves and their families
d) The cost of relocating people
e) The cost of retraining staff made redundant to improve their job prospects
f) The cost to the community in social and financial terms of unemployment or
relocation

These are not just political points. The company's treatment of its staff may have a profound
impact upon its ability to recruit skilled employees in the future and on the way the company
is perceived by potential customers. Whether the costs can be established or not, the
questions need to be considered.
Service industries, perhaps more than manufacturing firms, rely on their staff. Front-line
staff are those who convey the 'service' – and the experience of the brand – to the consumer.
They convey the 'moment of truth' with the customer.

Case Study
In 2005 in the UK, BA (Brirish Airways) ground staff went on strike in support of sacked
catering workers. This cost BA up to £40m in refunds and loss of flight revenue as well as a
loss in reputation.

Page 828

Management information therefore has to include intangible factors such as how customers
feel about the service, whether they would use it again, and so on.
There are some demonstrable relationships between staff revenue and positive customer
experiences. High staff revenue not only means higher recruitment and training costs but it
may also have an adverse impact on the firm's ability to retain customers (which is cheaper
than finding new ones).
For service businesses, management accounting information should incorporate the key
drivers of service costs.
a)
b)
c)
d)
e)

Repeat business
Churn rate (for subscriptions)*
Customer satisfaction surveys, complaints
Opportunity costs of not providing a service
Avoidable / unavoidable costs

* For any given period of time, the number of participants who discontinue their use of a
service divided by the average number of total participants is the churn rate. Churn rate
provides insight into the growth or decline of the subscriber base as well as the average
length of participation in the service.

Page 829

Case Study
Banks
Banks typically have high fixed costs of running a network of branches. At the same time,
they are trying to promote Internet and other services, as they are competing with internetonly banks.
The traditional banks have responded in a number of ways
a) Set up their own Internet operations, which offer higher rates of interest for savers
b) Closing down branches, at a cost of adverse publicity at times

However, there have been some opposing trends
a) A bank might advertise that it will put a moratorium on closure programmes
b) Most of the internet banks are not making money; some see the benefit of opening
branches

The key issue is productivity. The expensive branch networks are useful for selling other
services.
A third alternative to branch and Internet banking is the telephone banking service. The
banking service is conducted by telephone and lately over the Internet. There is no formal
branch network although customers may be offered the use the branch network of the parent
company.
This service operates at a lower cost than the traditional branch network but offers some
branch facilities and an alternative service for users who want 24 hour access and do not
require face to face banking.

Page 830

Strategic, tactical and operational information
Just as we did for manufacturing businesses, we can consider the strategic, tactical and
operational information requirements of service businesses.

Information
type

Examples

Strategic

Forecast sales growth and market share
Profitability, capital structure

Tactical

Resource utilisation such as average staff time charged out, number of
customers per hairdresser, number of staff per account
Customer satisfaction rating

Operational

Staff timesheets
Customer waiting time
Individual customer feedback

Organisations have become more customer and results oriented over the last decade or so.
As a consequence, the differences between service organisations' and other
organisations' information requirements has decreased. Businesses have realised that
most of their activities can be measured, and many can be measured in similar ways
regardless of the business sector.

Page 831

BLANK

Page 832

INSTANT ACCESS TO DATA
Access to data has been facilitated by groupware, intranets, extranets, databases, data
warehousing and data mining.

Via distribution of data
Developments in IT have facilitated the distribution of data, making it instantly available to
those who require it. Such developments are known generally as office automation systems.
a)
b)
c)
d)
e)
f)

Word processing/spreadsheets
Electronic schedules
Desktop databases (see below)
Web publishing
Voice mail
E-mail

Via sharing of data
There have also been significant developments in the ways in which data can be shared.

Groupware
Groupware is a term used to describe software that provides functions that can be used by
collaborative work groups.

Typically, groups using groupware are small project-oriented teams that have important tasks
and tight deadlines.

Page 833

Features might include the following.
a) A scheduler allowing users to keep track of their schedules and plan meetings with
others
b) An address book
c) ‘To do' lists
d) A journal, used to record interactions with important contacts, items (such as e-mail
messages) and files that are significant to the user, and activities of all types and track
them all without having to remember where each one was saved
e) A jotter for jotting down notes as quick reminders of questions, ideas, and so on
f) File sharing and distribution utilities

There are clearly advantages in having information such as this available from the desktop at
the touch of a button, rather than relying on scraps of paper, address books, and corporate
telephone directories. It is when groupware is used to share information with colleagues that
it comes into its own. Here are some of the features that may be found.

a) Messaging, comprising an e-mail in-box which is used to send and receive messages
from the office/home/on the road and routing facilities, enabling users to send a
message to a single person, send it sequentially to a number of people (who may add
to it or comment on it before passing it on), or sending it to everyone at once.
b) Access to an information database, and customisable 'views' of the information held
on it, which can be used to standardise the way information is viewed in a workgroup.
c) Group scheduling, to keep track of colleagues' itineraries.
d) Public folders. These collect, organise, and share files with others on a team or across
the organisation.
e) Hyperlinks in mail messages. The recipient can click the hyperlink to go directly to a
Web page or file server.

Page 834

Intranets
Intra- means within so an intranet is an internal network used to share information within the
company or group. Intranets utilise Internet technology. A firewall surrounding an intranet
fends off unauthorised access.

The idea behind an 'intranet' is that companies set up their own mini version of the Internet.
Intranets use a combination of the organisation's own networked computers and Internet
technology. Each employee has a browser, used to access a server computer that holds
corporate information on a wide variety of topics, and in some cases also offers access to the
Internet.
Potential applications include company newspapers, induction material, online procedure and
policy manuals, employee web pages where individuals post details of their activities and
progress, and internal databases of the corporate information store.

The benefits of intranets are diverse.
a) Savings accrue from the elimination of storage, printing and distribution of
documents that can be made available to employees on-line.
b) Documents on-line are often more widely used than those that are kept filed away,
especially if the document is bulky (e.g. manuals) and needs to be searched. This
means that there are improvements in productivity and efficiency.
c) It is much easier to update information in electronic form.
d) Wider access to corporate information should open the way to more flexible working
patterns, as material available on-line may be accessed from remote locations.

Remote access to intranets can be available quickly and easily. This means that people
working at different parts of the organisation or away from the office can access data when
they need it. Developments in IT allow information from a data warehouse (see below) to be
displayed and Excel has facilities to post spreadsheets straight to the intranet and for users to
drill down to the detail from a summary level.

Page 835

Extranets
An extranet is an intranet that is accessible to authorised outsiders.

Whereas an intranet resides behind a firewall and is accessible only to people who are
members of the same company or organisation, an extranet provides various levels of
accessibility to outsiders.
Only those outsiders with a valid username and password can access an extranet, with
varying levels of access rights enabling control over what people can view. Extranets are
becoming a very popular means for business partners to exchange information.

Databases
A typical accounting application package processes only one sort of data. A payroll file
processes only payroll data and an inventory file only inventory data. An organisation might
end up with separate files and processing subsystems for each area of the business. However,
in many cases the underlying data used by each application might be the same. A major
consequence is that data items are duplicated in a number of files (data redundancy). They
are input more than once (leading to errors and inconsistencies) and held in several files
(wasting space). For example, data relating to the hours which an hourly-paid employee has
worked on a particular job is relevant both to the payroll system, as the employee's wages
will be based on the hours worked, and to the job costing system, as the cost of the
employee's time is part of the cost of the job.
The problem of data redundancy is overcome, partly at least, by an integrated system. An
integrated system is a system where one set of data is used for more than one application.
In a cost accounting context, it might be possible to integrate parts of the sales ledger,
purchase ledger, inventory control systems and nominal ledger systems, so that the data input
to the sales ledger updates the nominal ledger automatically.
The integrated systems approach, where different applications update each other, is a halfway house between a system based on separate application-specific files and a database
approach.

Broadly speaking, a database is a file of data organised in such a way that it can be accessed
by many applications and users.
Page 836

Using the example of hours worked given above, the following situations are possible.
a) The employee's hours are input twice, once to the payroll application, once to the job
costing system, in a non-integrated system of application-specific files.
b) In an integrated system, the data would have been input once, to the HR database
which is used by the payroll application and by the job costing application.
c) In a database system it would only be input once and would be immediately
available to both systems.

A database provides a comprehensive file of data for a number of different users. Each
user will have access to the same data, and so different departments need not keep their
own data files, containing duplicate information but where the information on one file
disagrees with the corresponding information on another department's file.

Database management systems
The database management system (DBMS) is a complex software system that organises the
storage of data in the database in the most appropriate way to facilitate its storage,
retrieval and use in different applications. It also provides the link between the user and the
data.

Data warehousing
A data warehouse contains data from a range of internal (e.g. sales order processing
system, nominal ledger) and external sources. One reason for including individual
transaction data in a data warehouse is that if necessary the user can drill-down to access
transaction-level detail. Data are increasingly obtained from newer channels such as customer
care systems, outside agencies or websites.
The warehouse provides a coherent set of information to be used across the organisation
for management analysis and decision-making. The reporting and query tools available
within the warehouse should facilitate management reporting and analysis.
The reporting and query tools used within the warehouse need to be flexible enough to allow
multidimensional data analysis, also known as on-line analytical processing (OLAP). Each
Page 837

aspect of information (e.g. product, region, price, budgeted sales, actual sales, time period
and so on) represents a different dimension. OLAP enables data to be viewed from each
dimension, allowing each aspect to be viewed in relation to the other aspects. So, for
example, information about a particular product sold in a particular region during a particular
period would be available on-line and instantly.
Organisations may build a single central data warehouse to serve the entire organisation or
may create a series of smaller data marts. A data mart holds a selection of the organisation's
data for a specific purpose.
A data mart can be constructed more quickly and cheaply than a data warehouse. However, if
too many individual data marts are built, organisations may find it is more efficient to have a
single data warehouse serving all areas.

Advantages of setting up a data warehouse system include:
a) Decision makers can access data without affecting the use of operational systems.
b) Having a wide range of data available to be queried easily encourages the taking of a
wide perspective on organisational activities.
c) Data warehouses have proved successful in a number of areas.
(i) Quantifying the effect of marketing initiatives
(ii) Improving knowledge of customers
(iii) Identifying and understanding an enterprise's most profitable revenue streams
d) Information can be made available to business partners. For example, if customer
sales order information is in the data warehouse, it could be made available to
customers and even suppliers. Internal information on products and services could
also be provided.

Data mining
Data mining software looks for hidden patterns and relationships in large pools of data.

True data mining software discovers previously unknown relationships. Data mining
provides insights that cannot be obtained through OLAP. The hidden patterns and
relationships the software identifies can be used to guide decision-making and to predict
future behaviour.
Page 838

Case Study
(1)

The American retailer Wal-Mart discovered an unexpected relationship between the
sale of nappies and beer! Wal-Mart found that both tended to sell at the same time, just
after working hours, and concluded that men with small children stopped off to buy
nappies on their way home, and bought beer at the same time. Logically therefore, if
the two items were put in the same shopping aisle, sales of both should increase. WalMart tried this and it worked.

(2)

Some credit card companies have used data mining to predict which customers are
likely to switch to a competitor in the next few months. Based on the data mining
results, the bank can take action to retain these customers.

Reports
To make use of data, a suitable reporting framework is needed. Enterprise resource
planning packages aim to integrate all of a company's applications to give a single point of
access. A problem is that accessing source data is difficult if it is held in different formats and
systems.

Case Study
Time and time again finance directors say that their key IT issue is lack of reporting
capabilities in the systems they are using. Reporting problems tend to fall into three
categories.
First, the inability to access the source data. This is either because it is in a format that
cannot be accessed by PC technology or it is held in so many places that its structure is
incomprehensible to a member of the finance team.
Second, the tools to make the enquiries or produce the reports are often difficult to use and
do not produce the reports in a 'user friendly' format with 'drill down' capabilities.
Third, there is the issue of consistency of information across systems. In order to get an
overall picture of your organisation's performance you will usually need to access data from
different operation applications. All too often the data is not the same across these systems.
Page 839

The argument for replacing what you have is well rehearsed. New systems promise the latest
technology for reporting and enquiries. Enterprise Resource Planning (ERP) packages
promise to integrate your different applications smoothly and give you a single point of
access to all data. Customer Relationship Management (CRM) software has been added to
this recipe to give this approach a better chance of happening.
“There are a myriad of reporting tools costing from a few pounds to hundreds of thousands of
pounds. One that is regularly overlooked is the spreadsheet. Excel is the product most
commonly used by accountants. With the advent of Microsoft Office 2000 there is a
bewildering array of features to present information on your desktop or paper. Pivot tables
are starting to be used more widely for multi-dimensional analysis and can be combined with
the increasingly powerful graphical capabilities of Excel. Spreadsheets are much underrated
and it is surprising how many organisations go out and buy expensive new knowledgemanagement tools when they already have a product on their computer that will deliver all
the reporting/enquiry performance they require.
So, see how far your spreadsheet will take you and see if you can avoid the cost of another
new IT tool.”
Adapted from an article by John Tate, Management Accounting, April 2000

Page 840

REMOTE INPUT OF DATA
Developments in IT have enabled the remote input of data.

It is no longer the case that data input requires someone to sit at a desk and to tap away at a
keyboard. There is a wide range of data capture techniques, a number of which allow staff to
input data into the organisation's system whether or not they are in the office.
a) Sales staff can communicate sales orders directly to head office using laptop
computers, ‘smart’ phones etc .
b) The use of hand-held computers, often with touch sensitive screens, means there is
no need for subsequent manual entry of data, speeding up processes and reducing the
chance of error, because there are no transcription errors and computerised data
validation techniques can be employed.
c) EPOS (Electronic Point of Sale) systems (barcode scanners and tills) are primarily
intended to speed up and avoid error in the check-out process in supermarkets, to
allow customers to complete transactions, and to manage inventories. In addition,
however, they collect precise and detailed information about how many of what
products are being bought at what times. If linked to a loyalty scheme, 'and by
whom' can be added since this allows the purchase data to be combined with
demographic data.
d) Items such as pressure mats that sound a buzzer in smaller shops or sliding doors in
larger ones have the practical purpose of either alerting staff to the fact that there is
someone in the shop or simply for letting customers in and out, but if linked to a
computer they also collect information about number and movements of customers.
The same applies to ticket scanners in car parks, stations, and leisure facilities like
sports venues.

Page 841

BLANK

Page 842

DEVELOPING MANAGEMENT ACCOUNTING
SYSTEMS
Developments in IT have revolutionised the potential for management accounting data,
increasing the volume and variety of possible reports.

Management information systems (MIS)
Most information is provided by an information system, or management information system
(MIS).

Management information system is 'a system to convert data from internal and external
sources into information and to communicate that information, in an appropriate form, to
managers at all levels in all functions to enable them to make timely and effective decisions
for planning, directing and controlling the activities for which they are responsible'.(Lucey)

A management information system is therefore a system of disseminating information
that will enable managers to do their job. It should provide managers with data that they can
use for benchmarking and control purposes.
Management information is by no means confined to accounting information, but until
relatively recently accounting information systems have been the most formally-constructed
and well-developed part of the overall information system of a business enterprise. This is
still the case in all but the most advanced organisations.
Most management information systems are not designed, but grow up informally, with each
manager making sure that he or she gets all the information considered necessary to do the
job. Much accounting information, for example, is easily obtained, and managers can often
get along with frequent face-to-face contact and co-operation with each other. Such an
informal system works best in small organisations.
However, some information systems are specially designed, often because the introduction
of computers has forced management to consider its information needs in detail. This is
especially the case in large companies.
Management should try to develop/implement a management information system for their
enterprise with care. If they allow the MIS to develop without any formal planning, it will
Page 843

almost certainly be inefficient because data will be obtained and processed in a random and
disorganised way and the communication of information will also be random and hit-andmiss.
a) Some managers will keep data in their heads and will not commit information to
paper. Stand-ins/successors will not know as much as they could or should because no
information has been recorded to help them.
b) The organisation will not collect and process all the information that it should.
c) Information may be available but not disseminated to the appropriate managers.
d) Information is communicated late because the need to communicate it earlier is not
understood and appreciated by the data processors.

The consequences of a poor MIS might be dissatisfaction amongst employees who believe
they should be told more, a lack of understanding about what the targets for achievement are
and a lack of information about how well the work is being done.
Whether a management information system is formally or informally constructed, it should
therefore have certain essential characteristics.
a) The functions of individuals and their areas of responsibility in achieving
company objectives should be defined.
b) Areas of control within the company (e.g. cost centres, investment centres) should
also be clearly defined.
c) Information required for an area of control should flow to the manager who is
responsible for it. (Management structure of the organisation should therefore be
considered.)

Page 844

Types of MIS
Three particular types of management information system deserve special mention.

Type of MIS

Detail

Decision support
systems (DSS)

Used by management to help make decisions on poorly defined
problems (with high levels of uncertainty). They provide access to
information with a wide range of information gathering and
analytical tools. Decision support systems allow the manager to
scan the environment, consider a number of alternatives and
evaluate them under a variety of potential conditions. There is a
major emphasis upon flexibility and user-friendliness.

Executive
information systems
(EIS)

Give executives a straightforward means of access to key internal
and external data. They provide summary-level data, captured
from the organisation's main systems (which might involve
integrating the executive's desk top PC with the organisation's
mainframe), data manipulation facilities (such as comparison with
budget or prior year data and trend analysis) and user-friendly
presentation of data.

Expert systems

Draw on a computerised knowledge base (such as details of the
workings of tax legislation) and can give factual answers to
specific queries, as well as indicating to the user what a decision
ought to be in a particular situation.

Setting up a management accounting system
Taking a broad view, the following factors should be considered when setting up a
management accounting system (which is just one part of an overall MIS).
a) The output required. This is just another way of saying that the management
accountant must identify the information needs of managers. If a particular
manager finds pie-charts most useful the system should be able to produce them. If
another manager needs to know what time of day machinery failures occur, this
information should be available. Levels of detail and accuracy of output and methods
of processing must be determined in each case.
Page 845

b) When the output is required. If information is needed within the hour the system
should be capable of producing it at this speed. If it is only ever needed once a year, at
the year end, the system should be designed to produce it on time, no matter how long
it takes to produce.
c) The sources of input information. It is too easy to state that the outputs required
should dictate the inputs made. The production manager may require a report
detailing the precise operations of his machines, second by second. However, the
management accounting system could only acquire this information if suitable
production technology had been installed.

The need for management accounting systems to develop
Globalisation and competition require an external, forward-looking focus, with greater
facilities for modelling.

In the Study Guide, the ACCA states that management accounting systems need to be defined
and developed 'in an increasingly competitive and global market'.
Environmental analysis is covered in detail in Paper P3 Strategic Business Analysis.
We shall now describe some pointers for issues of competition and globalisation. The key
development is the use of management accounting systems for strategic decision making.

a) Competition
Impact

Management accounting impact

• More competitors

Better competitor intelligence
Model competitor cost structures

• More competing products

Identify which features add most value; model
impact on cost

• Faster response

Management accounting information has to be
produced speedily and be up-to-date for
decision making
Page 846

b) Globalisation
Impact
• Increases competition

Management accounting impact
• Similar impact to (a)
• Attention to behavioural impact on
management accounting systems in different
markets

• Access to overseas capital

• The cost of operating in different local
markets
• Aggregating information

• Overseas activities

• Repaid for exchange differences

We consider management accounting systems later in the text in Parts C and F.

Page 847

BLANK

Page 848

GAME THEORY
Scenario planning and foresight both highlight the inherent uncertainty in trying to predict the
future. One particular aspect of this uncertainty comes from how competitors will react to any
new strategy an organisation introduces.
Game Theory is the study of the ways in which the strategic interactions among rational
players produce strategic outcomes which were not intended by any of the players.

Game theory approach to strategy
Game-based approaches to strategy treat strategy as an interaction between an organisation
and its competitors. To this end, an organisation cannot simply develop its strategy by
analysing its current position in the environment, and looking at its internal resources. Instead
it also needs to look at its competitors, identify their strengths and weaknesses and examine
how their responses to a strategy could affect the effectiveness of that strategy.

Anticipating competitors’ moves is a crucial part of strategic thinking: gauging competitors’
likely reactions to a strategy greatly improves an organisation’s ability to choose a strategy
that will be successful.

We can illustrate this with a simple example.

Example: soft drinks market
Firms A and B are the local market-leaders in the soft drinks market, and between them they
hold virtually 100% of the market share. Firm A is considering launching a major advertising
campaign, because its marketing director believes this will not only increase its own sales and
profit, but will also reduce those of its rival (B).

However, the marketing director has not considered B’s response. B has become aware of
A’s campaign, and is now considering launching a campaign of its own to restore its market
share.
Page 849

At this moment, both A and B make profits of RWF250m per year. A is thinking of spending
RWF25m on its campaign, because it wants a major campaign to generate a significant
increase in revenue. The anticipated increase of revenue from the campaign is RWF75m.

Because A and B essentially share the market, A’s revenue increase is expected to come from
customers who switch to it from B. therefore, alongside A’s revenue increase of RWF75m, B
will suffer a revenue reduction of RWF75m.

Consequently, at the end of A’s initial campaign, and in the short term, B will have suffered a
reduction in profit of RWF75m, while A will have enjoyed an increase in profit of RWF50m
(RWF75m revenue less RWF25m marketing cost). This is a ‘win-lose’ situation, because A
has ‘won’ while B has ‘lost’.

However, B then runs a rival campaign, also costing RWF25m, and which also generates
RWF75m additional revenue following the logic from before, this is now a ‘lose-lose’
situation because although A has ‘lost’ and B has ‘won’, the contest has cost each them
RWF25m to gain nothing..

Let us look at the impact these campaigns have on A and B’s profits, and the overall profits
earned by the soft drink industry.

Option

A’s profit

B’s profit

Industry profit

Currently (no advertising)

RWF250m

RWF250m

RWF500m

A advertises

RWF300m

RWF175m

RWF475m

B then launches counter advert

RWF225m

RWF225m

RWF450m

Interestingly, after the advertising campaigns both firms are worse off than they were
before, and the industry profit has reduced by the cumulative cost of the advertising
campaigns. So overall the advertising campaign, created a ‘lose –lose ‘ situation.
Page 850

The figures show that although one firm can gain in the short run from a competitive
strategy, in the long run both firms are likely to be better off by working together and not
advertising, rather than competing with each other.

One of the assumptions of game theory is that the firms do not have any collusive agreements
and do not know what the other is going to do. So A and B must select their strategies based
solely on the outcome which they think is best for them regardless of the decision made by
their rival.

Under these circumstances, both firms will choose to advertise, because their own campaign
increases their own profit by RWF50m. However, collectively this course them both to lose
RWF25m each.

In this way, game theory illustrates the key problem of interdependent decision-making
which organisations face.
Organisations need to consider the possible responses of their competitors when making
strategic decisions or introducing new strategies.

Moreover, game theory also suggests that it may benefit firms to co-operate and negotiate
with others in the search for optimal solutions rather than simply working alone and
competing with all the other players in a market place. In order to create a ‘win-win’
scenario, firms are likely to have to compromise and co-operate rather than always seeking to
compete with each other.

In this context of networks and co-operation being preferable to constant competition, game
theory can help explain the reasoning behind strategic alliances. Game theory also supports
the cartel arrangement which the OPEC nations have established to control the production
and price of oil.

Page 851

Case Study: GlaxoSmithKline’s strategy for the developing world:
“GlaxoSmithKline(GSK), the world’s second largest pharmaceutical company is radically to
shift its attitude to providing cheap drugs to millions of people in the developing world.
In a major change of strategy, Andrew Witty, the new head of GSK has announced [in 2009]
he will slash prices of all medicines in the poorest countries, give back profits to be spent on
hospitals and clinics and - most ground-breaking of all - share knowledge about potential
drugs that are currently protected by patents.
Witty says he believes drug companies have an obligation to help the poor get treatment, and
he has challenged other pharmaceutical giants to follow his lead.
Pressure on the industry has been growing over the past decade as drug companies have been
repeatedly criticised for failing to drop their prices for HIV drugs while millions died in
Africa and Asia. Campaigners have criticised the drug companies for defending the patents,
which allow them to maintain high prices.
Campaigners have also been critical of the way drug companies have attempted to crush
competition from generic manufacturers, who undercut them dramatically in countries where
patents do not apply.
However, the moves which Witty has announced go a long way to addressing these concerns,
and mark a significant change to the way GSK does business in the developing world.

He said that GSK will:
•

•

•

Cut its prices for all drugs in the 50 least developed countries to no more than 25% of
the levels in the UK and US – and less if possible – and make drugs more affordable
in middle-income countries such as Brazil and India.
Put any chemicals or processes over which it has intellectual property rights that are
relevant to finding drugs for neglected diseases into a “patent pool”, so they can be
explored by other researchers.
Reinvest 20% of any profits it makes in the least developed countries in hospitals,
clinics and staff.

The extent of these changes is likely to stun not only critics of drug companies but also other
pharmaceutical companies, who risk being left exposed.
Witty accepts that his stance may not win him friends in other drug companies, but he is
inviting them to joy him in an attempt to make a significant difference to the health of people
in poor countries.
Page 852

Witty explained that the changes reflect his desire that GSK finds solutions for developing
and developed countries alike. However he is aware that the move may raise concerns among
GSK’s shareholders.
“I think the shareholders understand {the need to help the developing countries as well as the
richer, developed countries) and it’s my job to make sure I can explain it. I think we can. I
think it’s absolutely the kind of thing large global companies need to be demonstrating, that
they’ve got a more balanced view of the world than short-term returns”.
The move on intellectual property, until now regarded as the sacred cow of the
pharmaceutical industry, will be seen as the most radical of his proposals.” I think it’s the
first time anybody’s really come out and said we’re prepared to start talking to people about
pooling our patens to try to facilitate innovation in areas where, so far, there hasn’t been
progress” he said.
However, a key question now is how the other major pharmaceutical companies will
respond.”
(based on an article by Sarah Boseley inguardian.co.uk, 13 February 2009. Drug giant Glaxo
Smith Kline pledges cheap medicine for world’s poor)

The value of game theory is that it highlights that both competition and co-operation can exist
in an industry. An important part of an organisation’s strategy is how it interacts with the
other players in an industry in this respect.
Although both scenario planning and foresight aim to assist an organisation in designing their
future strategies, their effectiveness will depend in part on the organisation’s current strategic
intelligence.
However, critics of game theory argue that its value to strategic management is limited
because it focuses on only a small fraction of the strategy process. For example, it does not
provide any insight into the development of the competitive resources or capabilities of an
organisation. Equally, it does not provide any useful guidance as to how to actually
implement whatever co-operative strategies may have been negotiated.

Page 853

Section summary
Game theory illustrates that an organisation cannot develop its strategy without considering
the possible reactions of its competitors. Competitor reaction may mean that the outcomes of
a strategy are very different to what was initially intended.

Page 854

CHAPTER ROUNDUP
•

Unlike manufacturing companies, services are characterised by intangibility,
inseparability, variability, perishability and no transfer of ownership.

•

Mass services are standard services provided to large numbers of people, and are
often automated. Personal services vary on the circumstances of the service delivery,
and are generally one-to-one.

•

Service businesses need the same aggregate information as manufacturing firms,
but also need performance data as to their cost and volume drivers. Operational
information is likely to be more qualitative.

•

Access to data has been facilitated by groupware, intranets, extranets, databases,
data warehousing and data mining.

•

Developments in IT have enabled the remote input of data.

•

Developments in IT have revolutionised the potential for management accounting
data, increasing the volume and variety of possible reports.

•

Globalisation and competition require an external, forward-looking focus, with
greater facilities for modelling.

•

Game theory illustrates that an organisation cannot develop its strategy without
considering the possible reactions of its competitors. Competitor reaction may mean
that the outcomes of a strategy are very different to what was initially intended.

Page 855

BLANK

Page 856

STUDY UNIT 26
Benchmarking
Contents

Page

Benchmarking Benefits And Difficulties …………………………..…….

Page 857

859

BLANK

Page 858

BENCHMARKING BENEFITS AND DIFFICULTIES
Traditionally, control involves the comparison of actual results with an internal standard or
target. The practice of setting targets using external information is known as
benchmarking.

The value of external data to management accounting systems is its contribution to
planning, decision-making and control.

Here are examples.
Management
function
Planning

Type of information
• Demand estimates

Accounting document/process
• Sales budget

• Market research
Decision making

• Demand estimates

• Breakeven analysis

• Market research

• Production costs of providing product
features

• Competitor research

• Competitor costs
Control

• Demand estimates

• Sales variance reports

• Price variances

• Benchmarking
below)

for

variances

(see

Clearly, some external information, such as 'technological' or 'political' developments, does
not feed into the management accounting system, even though it can be in a broader
category of management information.
External information of a quantitative nature is easier to feed into the management
accounting system. For example, forecasts of revenues, costs and profits derived from
market research and targets based on competitors' performance (the information having been
sourced from the Internet) are easier to incorporate than qualitative information.

Page 859

Benchmarking
Benchmarking schemes enable precise comparisons to be drawn between firms. The use to
which benchmarking information is put is the key to its value. Benchmarking is best for firms
which have to 'catch up' rather than innovate.
Benchmarking. 'The establishment, through data gathering, of targets and comparators,
through whose use relative levels of performance (and particularly areas of
underperformance) can be identified. By the adoption of identified best practices it is hoped
that performance will improve. Types of benchmarking include the following.
•

Internal benchmarking. A method of comparing one operating unit or function with
another within the same industry.

•

Functional benchmarking. Internal functions are compared with those of the best
external practitioners of those functions, regardless of the industry they are in (also
known as operational benchmarking or generic benchmarking).

•

Competitive benchmarking. Information is gathered about direct competitors,
through techniques such as reverse engineering.*

•

Strategic benchmarking. A type of competitive benchmarking aimed at strategic
action and organisational change.

* Reverse engineering is the process of buying a competitor's product and dismantling it, in
order to understand its content and configuration.

As you will see from the list of the types of benchmarking, a benchmarking exercise doesn’t
necessarily have to involve the comparison of operations with those of a competitor. In
fact, it might be difficult to persuade a direct competitor to part with any information which is
useful for comparison purposes. Functional benchmarking, for example, does not always
involve direct competitors. A railway company could be identified as the 'best' in terms of onboard catering, and an airline company that operates on different routes would seek
opportunities to improve by sharing information and comparing their own catering operations
with those of the railway company.

Page 860

Exam Focus Point
There is ample information here on benchmarking to allow you to write a good essay
including the stages of benchmarking and the pros and cons of using this method.

Stages of benchmarking
Organisations should begin by asking themselves the following questions.
a) Is it possible and easy to obtain reliable competitor information?
b) Is there any wide discrepancy between different internal divisions?
c) Can similar processes be identified in non-competing environments and are these
non-competing companies willing to co-operate?
d) Is best practice operating in a similar environmental setting?
e) Is there time to complete the study?
f) It is possible to benchmark companies with similar objectives and strategies

The benchmarking exercise can then be divided into stages.

Step 1

Set objectives and determine the areas to benchmark

Step 2

Establish key performance measures

Step 3

Select organisations to study

Step 4

Measure own and others' performance

Step 5

Compare performances

Step 6

Design and implement improvement programme

Step 7

Monitor improvements

Step 1 requires consideration of the levels of benchmarking.

Page 861

Level of
benchmarking

Through

Examples of measures

Resources

Resource audit

Quantity of resources
• Revenue/employee
• Capital intensity
Quality of resources
• Qualifications of employees
• Age of machinery
• Uniqueness (e.g. patents)

Competences in
separate activities

Analysing activities

Sales calls per salesperson
Output per employee
Materials wastage

Competences in
linked activities

Analysing overall
performances

Market share
Profitability
Productivity

Step 4 requires information. Financial information about competitors is easier to acquire
than non-financial information. Information about products can be obtained from reverse
engineering, product literature, media comment and trade associations. Information
about processes (how an organisation deals with customers or suppliers) is more difficult to
find.
Such information can be obtained from group companies or possibly non-competing
organisations in the same industry (such as the train and airline companies mentioned
above).

Why use benchmarking?
a) Position audit. Benchmarking can assess a firm's existing position, and provide a
basis for establishing standards of performance.
b) The sharing of information can be a spur to innovation.
c) Its flexibility means that it can be used in both the public and private sectors and by
people at different levels of responsibility.
d) Cross comparisons (as opposed to comparisons with similar organisations) are more
likely to expose radically different ways of doing things.
Page 862

e) It is an effective method of implementing change, people being involved in
identifying and seeking out different ways of doing things in their own areas.
f) It identifies the processes to improve.
g) It helps with cost reduction.
h) It improves the effectiveness of operations.
i) It delivers services to a defined standard.
j) It provides a focus on planning.
k) It can provide early warning of competitive disadvantage.
l) It should lead to a greater incidence of team working and cross-functional learning.

Disadvantages of benchmarking
a) It implies there is one best way of doing business – arguably this boils down to the
difference between efficiency and effectiveness. A process can be efficient but its
output may not be useful. Other measures (such as amending the value chain) may be
a better way of securing competitive advantage.
b) The benchmark may be yesterday's solution to tomorrow's problem. For example,
a coffee-bar/fast-food restaurant might benchmark its activities (e.g. the quality of the
coffee, cleanliness and ambience) against another coffee bar, whereas the real
competitor could be a grocery store selling really good coffee and ready-made meals
to cook easily and eat at home.
c) It is a catching-up exercise rather than the development of anything distinctive. After the
benchmarking exercise, the competitor might improve performance in a different way.
d) It depends on accurate information about comparator companies.
e) It can be difficult to decide which activities to benchmark.

Page 863

CHAPTER ROUNDUP
•

Benchmarking schemes enable precise comparisons to be drawn between firms. The
use to which benchmarking information is put is the key to its value. Benchmarking is
best for firms which have to’ catch up’ rather than innovate.

Page 864

STUDY UNIT 27
Business Process Re-Engineering
Contents

Page

Business Process Re-Engineering ……………………………………..….

Page 865

867

BLANK

Page 866

BUSINESS PROCESS RE-ENGINEERING
Read this section to understand what business process re-engineering is. Also think about
how business process re-engineering affects systems development and its influence on
organisational performance (see the Case study).

Business process re-engineering involves focusing attention inwards to consider how
business processes can be redesigned or reengineered to improve efficiency.

Business process re-engineering involves focusing attention inwards to consider how
business processes can be redesigned or re-engineered to improve efficiency. It can lead to
fundamental changes in the way an organisation functions. In particular, it has been realised
that processes, which were developed in a paper-intensive processing environment, may not
be suitable for an environment that is underpinned by IT.
The main writing on the subject is Hammer and Champy's Reengineering the Corporation
(1993), from which the following definition is taken.

Business Process Re-engineering (BPR) is the fundamental rethinking and radical redesign
of business processes to achieve dramatic improvements in critical contemporary measures of
performance, such as cost, quality, service and speed.

The key words here are ‘fundamental’, ‘radical’, ‘dramatic’ and ‘process’.
a) Fundamental and radical indicate that BPR is somewhat akin to zero based
budgeting: it starts by asking basic questions such as 'why do we do what we do',
without making any assumptions or looking back to what has always been done in the
past.
b) Dramatic means that BPR should achieve 'quantum leaps in performance', not just
marginal, incremental improvements.
c) Process. BPR recognises that there is a need to change functional hierarchies:
'existing hierarchies have evolved into functional departments that encourage
functional excellence but which do not work well together in meeting customers'
requirements' (Rupert Booth, Management Accounting, 1994).

Page 867

A process is a collection of activities that takes one or more kinds of input and creates an
output.

For example, order fulfilment is a process that takes an order as its input and results in the
delivery of the ordered goods. Part of this process is the manufacture of the goods, but under
BPR the aim of manufacturing is not merely to make the goods. Manufacturing should
aim to deliver the goods that were ordered, and any aspect of the manufacturing process
that hinders this aim should be re-engineered. The first question to ask might be 'Do they
need to be manufactured at all?'

A re-engineered process has certain characteristics.
a) Often several jobs are combined into one.
b) Workers often make decisions.
c) The steps in the process are performed in a logical order.
d) Work is performed where it makes most sense.
e) Checks and controls may be reduced, and quality 'built-in'.
f) One manager provides a single point of contact.
g) The advantages of centralised and decentralised operations are combined.

Case Study
Based on a problem at a major car manufacturer.
A company employs 25 staff to perform the standard accounting task of matching goods
received notes with orders and then with invoices. About 80% of their time is spent trying to
find out why 20% of the set of three documents do not agree.
One way of improving the situation would have been to computerise the existing process to
facilitate matching. This would have helped, but BPR went further: why accept any incorrect
orders at all? What if all the orders are entered onto a computerised database? When goods
arrive at the goods inwards department they either agree to goods that have been ordered or
they don't. It is as simple as that. Goods that agree to an order are accepted and paid for.
Goods that are not agreed are sent back to the supplier. There are no files of unmatched items
and time is not wasted trying to sort out these files.
Page 868

Exam Focus Point
You may well get a question on BPR requiring you to assess its impact on the organisation.

Hammer's principles of BPR
a) Processes should be designed to achieve a desired outcome rather than focusing on
existing tasks.
b) Personnel who use the output from a process should perform the process. For
example, a company could set up a database of approved suppliers; this would allow
personnel who actually require supplies to order them themselves, perhaps using online technology, thereby eliminating the need for a separate purchasing function.
c) Information processing should be included in the work, which produces the
information. This eliminates the differentiation between information gathering and
information processing.
d) Geographically dispersed resources should be treated as if they are centralised.
This allows the benefits of centralisation to be obtained, for example, economies of
scale through central negotiation of supply contracts, without losing the benefits of
decentralisation, such as flexibility and responsiveness.
e) Parallel activities should be linked rather than integrated. This would involve, for
example, co-ordination between teams working on different aspects of a single
process.
f) 'Doers' should be allowed to be self-managing. The traditional distinction between
workers and managers can be abolished: decision aids such as expert systems can
be provided where they are required.
g) Information should be captured once at source. Electronic distribution of
information makes this possible.

Page 869

Business processes and the technological interdependence between
departments
The value chain describes a series of activities from input of raw materials to output of
finished goods/services for the customers. These activities may be organised into departments
even though the actual process of adding value may cross departmental boundaries.
The links between different departments of a business can vary, however, and hence the
need to manage the relationships between them. Interdependence is the extent to which
different departments depend on each other to accomplish their tasks. It is possible to
identify three types of interdependence.

a) In pooled interdependence, each department/section works independently of the
others, subject to achieving the overall goals of the organisation.
b) Sequential interdependence is when there is a sequence (or a linked chain of
activities) with a start and end point. An example is an assembly line: raw materials
are taken, moulded to the right sizes and shapes and are assembled into a product. The
outputs of each stage sequence must be precisely tailored to the inputs of the next –
standardisation of outputs, might be one form of co-ordination used. The first activity
must be performed correctly before the second can be tackled. Management effort is
required to ensure that the transfer of resources between departments is smooth.
They therefore need information about the process as a whole.
c) Reciprocal interdependence exists when a number of departments acquire inputs
from and offer outputs to each other. In other words, while resources have to be
transferred, there is no pre-set sequence. The output of one department might be sent
to another for processing, and then returned to the original department.

You should now have some idea as to the complexities of business processes overlapping
different departments. Some organisations have redesigned their structures on the lines of
business processes, adopting BPR to avoid all the co-ordination problems caused by
reciprocal interdependence.

Page 870

Key characteristics of organisations, which have adopted BPR
a) Work units change from functional departments to process teams, which replace
the old functional structure.
(i)

For example, within a functional framework, a sales order may be handled by
many different people, in different departments or business functions. (One
person takes the order in the department, and one person delivers).
(ii) In process teams, the people are grouped together. A case team might combine
to do all the work on a process and this applies not only to one-off projects but
to recurring work.
Multi-skilling/Multi-tasking also means that one individual has several skills and does
many of the tasks in a process.
b) Jobs change. People do more, as team members are responsible for results. This ties
in with job enlargement and job enrichment.
c) People's roles change. They are empowered to make decisions relevant to the
process.
d) Performance measures concentrate on results rather than activities. Process teams
create 'value' which is measurable.
e) Organisation structures change from hierarchical to flat (i.e. delayered).
(i) When a process becomes the work of a whole team, managing the process is
the team's responsibility. Interdepartmental issues become matters the team
resolves itself, rather than matters requiring managerial intervention.
(ii) Companies require less managerial input. Managers have less to do; there are
fewer of them and so fewer layers.
(iii) Organisation structure determines lines of communication, and in many
organisations is a weighty issue. This is not the case in process organisations, as
lines of communication 'naturally' develop around business processes.

Page 871

Implications of BPR for accounting systems
Issue

Implication

Performance
measurement

Performance measures must be built around processes not departments: this
may affect the design of responsibility centres.

Reporting

There is a need to identify where value is being added.

Activity

ABC might be used to model the business processes.

Structure

The complexity of the reporting system will depend on the organisational
structure. Arguably the reports should be designed round the process
teams, if there are independent process teams.

Variances

New variances may have to be developed.

Case Study
The case of Taco Bell (Taco Bell is a chain of fast food restaurants based in California USA,
but now operates world-wide. They specialise in Mexican foods) is one of the examples
quoted in Hammer and Champy’s book. The emphasis is the editor's.
In the 1980s, the company was entrenched in a command and control hierarchy that claimed
to understand what customers wanted, but did not ask directly. But major re-engineering
efforts – automating, changing the organisational structure and management system, reducing
kitchen space, and increasing customer space – focusing on what customers really wanted,
greatly simplified their processes.
These changes have had a huge impact on the company. It went from a failing regional
Mexican-American fast food chain with $500 million in sales in 1982, to a $3 billion national
company 10 years later, with a goal to expand further to $20 billion.
One BPR initiative was the K-Minus program, or kitchenless restaurant. Based on the
belief that they were a service company, not a manufacturer, a large majority of the
restaurants’ food preparation is now “outsourced” and occurs at central commissaries rather
than in the restaurant, pushing 15 hours of work a day out of the restaurant, improving
quality control and employee morale, reducing employee accidents and injuries, and
resulting in substantial savings on utilities. The K-Minus program saves Taco Bell about $7
million a year.
Page 872

Examples of business process re-engineering
a) A move from a traditional functional plant layout to a JIT cellular product layout is a
simple example.
b) Elimination of non-value-added activities. Consider a materials handling process,
which incorporates scheduling production, storing materials, processing purchase
orders, inspecting materials and paying suppliers.
This process could be re-engineered by sending the production schedule direct to
nominated suppliers with whom contracts are set up to ensure that materials are
delivered in accordance with the production schedule and that their quality is
guaranteed (by supplier inspection before delivery).
Such re-engineering should result in the elimination or permanent reduction of the
non-value-added activities of storing, purchasing and inspection.

Exam Focus Point
Be prepared to apply your knowledge of BPR to a particular scenario or to examples that you
are aware of from your reading or own experience. Examiners have stated that good answers
often draw on the candidate’s own experience in the context of the question set.

Page 873

CHAPTER ROUNDUP
•

Business process re-engineering involves focusing attention inwards to consider
how business processes can be redesigned or reengineered to improve efficiency.

Page 874



Source Exif Data:
File Type                       : PDF
File Type Extension             : pdf
MIME Type                       : application/pdf
PDF Version                     : 1.6
Linearized                      : Yes
Encryption                      : Standard V4.4 (128-bit)
User Access                     : Print, Print high-res
Author                          : Paul O'Neill
Create Date                     : 2012:09:19 16:25:58+01:00
Modify Date                     : 2012:09:19 16:25:58+01:00
Title                           : 
Tagged PDF                      : Yes
XMP Toolkit                     : Adobe XMP Core 5.2-c001 63.139439, 2010/09/27-13:37:26
Metadata Date                   : 2012:09:19 16:25:58+01:00
Creator Tool                    : Acrobat PDFMaker 10.1 for Word
Format                          : application/pdf
Creator                         : Paul O'Neill
Document ID                     : uuid:4e6beffa-4cbb-4b11-b58c-419d78c6be41
Instance ID                     : uuid:58af5a85-3a72-45fc-b8f6-b9209bf73480
Producer                        : Adobe PDF Library 10.0
Page Count                      : 877
EXIF Metadata provided by EXIF.tools

Navigation menu